模块4
更新时间:2021-06-27 21:07:54
模块 年级 问题 答案 正确答案
4 1 2006 1. 消化道平滑肌细胞动作电位的主要离子基础是(4.0分) A.K+大量内流
B.Ca2+大量内流
C.Cl-大量外流
D.Na+大量内流
E.Ca2+ 大量外流
B.Ca2+大量内流
4 1 2006 2. 肝脏的生物转化反应不包括:(3.0分) A.水解反应
B.羧化反应
C.氧化反应
D.还原反应
E.结合反应
B.羧化反应
4 1 2006 3. 牙周组织的组成是(3.0分) A.牙周膜、牙槽骨、牙龈
B.牙(本)质、牙槽骨、牙周膜
C.牙龈、牙髓、牙骨质
D.牙槽骨、牙髓、牙龈
E.牙釉质、牙周膜、牙槽骨
A.牙周膜、牙槽骨、牙龈
4 1 2006 4. 下列哪类物质可能不引起缩胆囊素的释放?(3.0分) A.糖
B.脂肪
C.盐酸
D.蛋白质分解产物
E.脂肪酸
A.糖
4 1 2006 5. 淀粉在小肠内被吸收的主要形式是(3.0分) A.麦芽糖
B.果糖
C.蔗糖
D.葡萄糖
E.半乳糖
D.葡萄糖
4 1 2006 6. 生物转化过程最主要的作用是:(3.0分) A.使药物失效
B.使毒物的毒性降低
C.使生物活性物质灭活
D.使非营养性物质极性增加,利于从肾脏及胆道排泄
E.使致癌物失活
D.使非营养性物质极性增加,利于从肾脏及胆道排泄
4 1 2006 7. 肝右锁骨中线上界平: (3.0分) A.第6肋
B.第5肋
C.第4肋
D.第7肋
E.第8肋
B.第5肋
4 1 2006 8. 在生理条件下,胆色素经肾脏从尿中排出的主要形式是:(4.0分) A.胆素原
B.未结合胆红素
C.结合胆红素
D.胆素
E.胆绿素
A.胆素原
4 1 2006 9. 结合胆红素是指胆红素与下列哪种物质结合(3.0分) A.血清白蛋白
B.葡萄糖醛酸
C.磷脂
D.硫酸盐
E.胆汁酸
B.葡萄糖醛酸
4 1 2006 10. 消化器官不具备下列哪一功能?(4.0分) A.水平衡
B.免疫
C.内分泌
D.消化
E.吸收
A.水平衡
4 1 2006 11. 迷走神经兴奋引起的胰液分泌的特点是(3.0分) A.HCO3-多
B.酶多
C.H2O多
D.酶和HCO3-均多
E.酶、H2O和HCO3-均多
B.酶多
4 1 2006 12. 乳牙一般在出生后何时开始萌出? (3.0分) A.9岁
B.3个月
C.3岁
D.6个月
E.6岁
B.3个月
4 1 2006 13. 胃肠道内下列哪种物质对胃液分泌有促进作用?(3.0分) A.糖及消化产物
B.脂肪及消化产物
C.蛋白质消化产物
D.盐酸
E.高张溶液
C.蛋白质消化产物
4 1 2006 14. 关于胃肠激素生理作用的错误叙述是。(3.0分) A.营养消化道组织
B.调节消化腺分泌
C.调节小肠内营养物的吸收量
D.调节其它胃肠激素的释放
E.调节消化道运动
C.调节小肠内营养物的吸收量
4 1 2006 15. 下列哪种化学物质对胃液分泌有抑制作用?(3.0分) A.乙酰胆碱
B.胃泌素
C.组织胺
D.阿托品
E.胆囊收缩素
D.阿托品
4 1 2006 16. 关于消化管平滑肌基本电节律的正确叙述是(3.0分) A.其后一定伴随动作电位
B.是平滑肌收缩节律的控制波
C.在切断支配胃肠的神经后消失
D.是一种超极化波
E.以上说法均不对
B.是平滑肌收缩节律的控制波
4 1 2006 17. 关于内因子的正确叙述是(4.0分) A.促进VitB12的吸收
B.属肽类激素
C.促进胃酸分泌
D.胃腺的主细胞分泌
E.促进蛋白质的消化
A.促进VitB12的吸收
4 1 2006 18. 头期胃液分泌的调节是(3.0分) A.纯体液调节性的
B.纯条件反射性的
C.神经--体液调节性的
D.纯神经调节性的
E.自主调节
C.神经--体液调节性的
4 1 2006 19. 关于消化道平滑肌基本电节律的错误叙述是(3.0分) A.在消化道各段平滑肌的发生频率一致
B.与细胞膜生电性钠泵活动的周期性变化有关
C.在胃肠安静状态下可记录到
D.起源于纵行肌和环形肌之间的Cajal细胞
E.神经和激素可使其产生
A.在消化道各段平滑肌的发生频率一致
4 1 2006 20. 下列说法哪一个是错的? (3.0分) A.阑尾根部体表投影:脐与右髂前上棘连线的中、外1/3交点处
B.胆囊底体表投影:右腹直肌外侧缘(或右锁骨中线 )与右肋弓交点附近
C.骶曲凸向前
D.Oddi's括约肌由肝胰壶腹括约肌、胆总管括约肌、胰管括约肌组成
E.下颌下腺导管开囗于舌下阜
C.骶曲凸向前
4 1 2006 21. 胰蛋白酶原的激活物是(3.0分) A.胰淀粉酶
B.肠激酶
C.胰脂肪酶
D.糜蛋白酶
E.胃蛋白酶
B.肠激酶
4 1 2006 22. 糖、蛋白质和脂肪消化产物主要的吸收部位是在(3.0分) A.结肠
B.十二指肠、空肠
C.回肠
D.胃
E.B和C均对
B.十二指肠、空肠
4 1 2006 23. 大肠的主要功能是吸收(3.0分) A.葡萄糖
B.氨基酸
C.水
D.脂肪酸
E.胆固醇
C.水
4 1 2006 24. 排便反射的初级中枢位于:(4.0分) A.延髓
B.大脑皮层
C.脊髓腰骶部
D.脑桥
E.以上均不是
C.脊髓腰骶部
4 1 2006 25. 临床上治疗胃酸过少的胃病可用极稀的(4.0分) A.H2CO3
B.H2SO4
C.HCI
D.HN03
E.H3PO4
C.HCI
4 1 2006 26. 迷走神经兴奋时(3.0分) A.胃肠平滑肌活动减弱,消化腺分泌增加
B.胃肠平滑肌活动增强,消化腺分泌减少
C.胃肠平滑肌活动增强,消化腺分泌增加
D.胃肠平滑肌活动减弱,消化腺分泌减少
E.以上都不是
C.胃肠平滑肌活动增强,消化腺分泌增加
4 1 2006 27. 关于胃容受性舒张的正确叙述是(4.0分) A.使胃容量大增
B.切断双侧迷走神经后仍出现
C.反射性地胃窦部肌肉舒张
D.阿托品可抑制它的出现
E.是体液调节
A.使胃容量大增
4 1 2006 28. 消化腺所分泌的大量消化液不具备下列哪一项功能?(3.0分) A.排除体内过多的水和盐
B.保护消化道粘膜
C.水解复杂的食物成分
D.稀释食物
E.分解食物中的各种成分
A.排除体内过多的水和盐
4 1 2006 29. 下列哪种化学物质对胃液分泌有促进作用?(3.0分) A.阿托品
B.前列腺素
C.甲氰咪呱
D.组织胺
E.甲肾上腺素
D.组织胺
4 1 2006 30. 食管第三个狭窄在: (3.0分) A.第4胸椎水平
B.第6胸椎水平
C.第10胸椎水平
D.第8胸椎水平
E.第12胸椎水平
C.第10胸椎水平
4 1 2006 31. 关于胆汁生理作用的错误叙述是(3.0分) A.促进脂溶性维生素的吸收
B.中和十二指肠的一部分胃酸
C.作为乳化剂,减低脂肪表面张力
D.负反馈抑制胆汁自身的分泌
E.促进脂肪分解产物的吸收
D.负反馈抑制胆汁自身的分泌
4 1 2007 1. 下列哪一组氨基酸属于营养半必需氨基酸(2.0分) A.Gly和Asp
B.Tyr和Cys
C.Gln和His
D.Met和Thr
E.Ala和Arg
B.Tyr和Cys
4 1 2007 2. 胃肠道微生态失调相关性疾病不包括: (2.0分) A.肝病
B.大肠癌
C.炎症性肠病(IBD)
D.抗生素相关性腹泻
E.肠易激综合征
E.肠易激综合征
4 1 2007 3. 炎症性肠病的特征性皮肤表现有:(2.0分) A.红斑狼疮
B.结节性红斑
C.天胞疮
D.剥脱性皮炎
E.银屑病
B.结节性红斑
4 1 2007 4. 下列不属于常用的Hp检测方法的是:(2.0分) A.Hp-DNA的聚合酶链式反应
B.快速尿素酶试验
C.血清抗体检测
D.粘膜组织染色
E.Hp呼气试验
A.Hp-DNA的聚合酶链式反应
4 1 2007 5. 舌下阜(2.0分) A.有舌腺的开口
B.只有下颌下腺管的开口
C.有舌下腺小管的开口
D.有下颌下腺管和舌下腺大管的开口
E.其深面藏有舌下腺
D.有下颌下腺管和舌下腺大管的开口
4 1 2007 6. 乳糖不耐受症是由于下列哪种酶缺乏所引起(2.0分) A.蔗糖酶
B. -淀粉酶
C.半乳糖酶
D.麦芽糖酶
E.乳糖酶
E.乳糖酶
4 1 2007 7. 恶性病变的声像图特点中,应除外: (2.0分) A.边缘不光滑或断续
B.形态较规则
C.回声低弱或增强
D.周围组织呈浸润性改变
E.血流信号改变
B.形态较规则
4 1 2007 8. 诊断肝性脑病最有意义的实验室检查是(2.0分) A.血胆红素
B.血氨
C.血糖
D.血尿素氮
E.血转氨酶
B.血氨
4 1 2007 9. 正常人体软组织的内部回声强度排列顺序,下述哪项正常: (2.0分) A.肾窦>肝脏>胰腺>肾实质;
B.肾窦>胰腺>肝脏>肾实质;
C.胰腺>肾窦>肝脏>肾实质;
D.胰腺>肾窦>肾实质>肝脏;
E.肾窦>胰腺> 肾实质>肝脏
B.肾窦>胰腺>肝脏>肾实质;
4 1 2007 10. 血清淀粉酶测定正确的是(2.0分) A.淀粉酶的高低与疾病严重程度呈正比
B.起病后6-12小时开始升高
C.发病后立即升高
D.超过正常值2倍即可确诊
E.E持续一周以上
B.起病后6-12小时开始升高
4 1 2007 11. 下列疾病可表现为肝内囊性病灶,除外(2.0分) A.多囊肝
B.包囊肿病
C.肝囊肿
D.转移性肝癌
E.局灶性增生结节
D.转移性肝癌
4 1 2007 12. MRCP主要应用于(2.0分) A.胆囊病变
B.胆道病变
C.胃肠道病变
D.血管性病变
E.是基于T1WI游离水高信号的磁共振成像技术
B.胆道病变
4 1 2007 13. 下列不属于门脉高压综合征的是(2.0分) A.肝大
B.痔核形成
C.食管和胃底静脉曲张
D.脾大
E.腹腔积液
A.肝大
4 1 2007 14. 胃的位置和毗邻,下列说法哪一个是错的? (2.0分) A.小部分位于腹上区
B.大部分位于左季肋区
C.贲门位于第11胸椎体左侧
D.幽门位于第1腰椎体右侧
E.胃前壁与脾、腹前壁、膈毗邻
E.胃前壁与脾、腹前壁、膈毗邻
4 1 2007 15. 在小肠蛋白质的消化过程中, 首先被激活的蛋白酶原是(2.0分) A.弹性蛋白酶原
B.胰糜蛋白酶原
C.羧肽酶原
D.胰脂肪酶原
E.胰蛋白酶原
E.胰蛋白酶原
4 1 2007 16. 关于胃良性溃疡,下列说法不正确的是:(2.0分) A.胃肠道钡餐检查可见点状钡斑
B. 胃蠕动增强
C.粘膜纠集征象
D.半月综合征
E.窄颈征
D.半月综合征
4 1 2007 17. 下列有关胃蛋白酶和胃蛋白酶原的描述哪一项是错误的(2.0分) A.胃酶抑素(Pepstatin)可以强烈抑制胃蛋白酶
B.胃蛋白可以自身激活酶原
C.胃泌素可以刺激胃蛋白酶原的分泌
D.酶原激活时,其一级结构保持不变,但空间结构发生了改变
E.胃表面存在粘液层,可以防止自身被胃蛋白酶消化
D.酶原激活时,其一级结构保持不变,但空间结构发生了改变
4 1 2007 18. 甲状腺功能亢进时典型的摄131I率表现为: (2.0分) A.摄131I率不增高,高峰4小时出现
B.摄131I率不增高,高峰>24小时
C.摄131I率增高,高峰>24小时
D.摄131I率增高,高峰4小时出现
E.摄131I率减低。
D.摄131I率增高,高峰4小时出现
4 1 2007 19. 肝胆疾病引起胆汁酸合成和分泌不足,可以引起哪些维生素的缺乏(2.0分) A.叶酸和B6
B.生物素 和 烟酸
C.脂溶性维生素
D.维生素B1和B2
E.维生素A 和C
C.脂溶性维生素
4 1 2007 20. 肝脏增强扫描动脉期,对以下哪种病变诊断价值最高(2.0分) A.肝脏炎性假瘤
B.转移性肝癌
C.原发性肝癌
D.局灶性结节增生
E.肝囊肿
C.原发性肝癌
4 1 2007 21. 不属于胃癌基本X线表现的是:(2.0分) A. 粘膜皱襞破坏中断
B.半月综合征
C.胃壁僵硬、边缘不齐
D.项圈征及狭颈征
E.胃呈皮革状变形
D.项圈征及狭颈征
4 1 2007 22. 肝脏常规CT及MR动态增强扫描需要包括:(2.0分) A.动脉期、门脉期、静脉期
B.造影剂(钡剂)
C.静脉滴注造影剂
D.静脉期
A.动脉期、门脉期、静脉期
4 1 2007 23. 常用医用硫酸钡作为对比剂的造影部位是:(2.0分) A.胃肠道造影
B.椎管造影
C.胆囊造影
D.脑血管造影
E.泌尿系造影
A.胃肠道造影
4 1 2007 24. 首选依靠断面影像手段检查的是?(2.0分) A.胃肠道病变
B.骨髂病变
C.神经病变
D.肝、胆、胰病变
E.胸部
C.神经病变
4 1 2007 25. 乳牙一般在出生后何时开始萌出? (2.0分) A.9岁
B.6个月
C.3个月
D.3岁
E.6岁
C.3个月
4 1 2007 26. 胰腺分泌的消化酶多以前体形式存在,其中对其它胰酶的激活具有关键作用的胰酶是:(2.0分) A. 弹性蛋白酶
B.胰蛋白酶
C.羧肽酶
D.糜蛋白酶
B.胰蛋白酶
4 1 2007 27. 胰腺外分泌的功能是:(2.0分) A.参与消化作用
B.参与代谢调节
C.参与信号转导
D.参与免疫调节
A.参与消化作用
4 1 2007 28. 下面不属于代谢综合征的成分是 (2.0分) A.超重/肥胖
B.高血压
C.高血糖
D.血脂异常
E.冠心病
E.冠心病
4 1 2007 29. 胃蛋白酶属于酸性蛋白酶,其最适pH为: (2.0分) A. pH 2~3
B.pH 7~8
C.pH 5~6
D.pH 7.4
E. pH 9~10
A. pH 2~3
4 1 2007 30. 关于大肠,下述何者正确(2.0分) A.长约2m
B.阑尾不属于大肠
C.在右髂窝处从回肠而起
D.大肠都没有系膜
E.结肠表面有3条结肠带
E.结肠表面有3条结肠带
4 1 2007 31. 有关胰 -淀粉酶( -amylase )的描述哪一项是错误的(2.0分) A. 含1个Cl- 和 1个Na+
B.不能水解 -1,6 糖苷键
C.是唾液淀粉酶的同工酶
D.可以水解直链淀粉和支链淀粉
E.水解 -1,4 糖苷键
A. 含1个Cl- 和 1个Na+
4 1 2007 32. 下列哪种氨基酸不是营养必需氨基酸(2.0分) A.Val
B.Lys
C.Ser
D.Leu
E.Phe
C.Ser
4 1 2007 33. 消化器官不具备下列哪一功能?(2.0分) A.消化
B.免疫
C.水平衡
D.内分泌
E.吸收
C.水平衡
4 1 2007 34. 维生素B12在胃肠道的吸收部位是(2.0分) A.回肠
B.十二指肠
C.结肠
D.空肠
E.胃
A.回肠
4 1 2007 35. 胃黏膜细胞分泌的一种内因子(IF),与下列哪种维生素吸收有关(2.0分) A.维生素B6
B.生物素
C.维生素B12
D.叶酸
E.维生素K
C.维生素B12
4 1 2007 36. 胃肠出血显像时,常用的显像剂为:(2.0分) A.99mTc-RBC
B.99mTc-MIBI
C.131I
D.99mTcO4-
E.99mTc-MDP
A.99mTc-RBC
4 1 2007 37. 一黄疸病人超声发现肝内胆管扩张、胆囊肿大、主胰管不扩张,其阻塞部位可能在 (2.0分) A.肝总管
B.壶腹部
C.肝门部
D.胆囊管以下
E.胆总管
C.肝门部
4 1 2007 38. 下列有关葡萄糖的吸收与转运的描述哪一项目是正确的(2.0分) A.不同组织的细胞使用同一种葡萄糖转运体
B.葡萄糖转运体分为5型,不需要Na+参与
C.通过葡萄糖转运体吸收和转运, 通常需要消耗ATP
D.葡萄糖在小肠中可以被动吸收
E.葡萄糖转运体是有多个亚基组成的跨膜蛋白
C.通过葡萄糖转运体吸收和转运, 通常需要消耗ATP
4 1 2007 39. 哪些器官的检查应首选超声检查(USG): (2.0分) A.胃炎
B.骨折后愈合情况
C.肝脏弥散性病变
D.肺炎性假瘤
E.头颅出血
C.肝脏弥散性病变
4 1 2007 40. 舌骨后肿块,为排除异位甲状腺,应首选下列哪项检查: (2.0分) A.CT
B.B超
C.MRI
D.核素显像
E.X线平片
D.核素显像
4 1 2007 41. 桥本氏甲状腺炎与甲亢的鉴别最有意义的是(2.0分) A.超声引导下细针穿刺活检
B.放射性核素扫描
C.常规超声检查
D.甲状腺功能五项
E.血常规
D.甲状腺功能五项
4 1 2007 42. 位于口咽部的结构是(2.0分) A.腭扁桃体
B.舌扁桃体
C.咽扁桃体
D.咽隐窝
E.咽鼓管咽口
A.腭扁桃体
4 1 2007 43. 卓-艾综合征的发生与下列胃肠激素分泌过多有关(2.0分) A.胰高血糖素
B.胆囊收缩素
C.促胰酶素
D.胰岛素
E.胃泌素
E.胃泌素
4 1 2007 44. 胆汁酸合成的原料是:(2.0分) A.乙酰辅酶A
B.脂肪酸
C.胆固醇
D.UDPG
C.胆固醇
4 1 2007 45. 常规肝CT增强扫描,有关动脉期影像的描述,下列哪项不对:(2.0分) A.肝脏明显强化
B.肾门以下下腔静脉无明显强化
C.脾脏强化程度比肝脏高
D.肝静脉强化不明显
E.增强后一般需30S内扫描
A.肝脏明显强化
4 1 2007 46. 诊断肝性脑病最有意义的体征是(2.0分) A.肌张力增高
B.扑翼样震颤
C.踝阵挛阳性
D.腱反射亢进
E.Babinski征阳性
B.扑翼样震颤
4 1 2007 47. 下列何者是131I治疗甲亢的禁忌征(2.0分) A.抗甲状腺药物(ATD)治疗甲亢过程中出现白细胞、血小板减少而停止治疗者
B.严重肾功能不全者
C.有效半衰期短于三天者
D.甲亢术后复发
E.年龄小于25周岁者
B.严重肾功能不全者
4 1 2007 48. 胃肠道疾病首选的影像学检查方法为(2.0分) A.钡剂造影
B.平片检查
C. CT检查
D.MRI检查
E.胆系造影检查
A.钡剂造影
4 1 2007 49. 消化道穿孔的描述中,正确的有(2.0分) A.穿孔穿入腹腔内时,立位腹平片可见膈下游离气体
B.小肠及阑尾穿孔后无气腹出现
C.穿孔穿入腹腔内可出现肾周气肿
D.没有游离气腹征象就能排除胃肠穿孔
E.以上均正确
A.穿孔穿入腹腔内时,立位腹平片可见膈下游离气体
4 1 2007 50. 关于消化管平滑肌基本电节律的正确叙述是 (2.0分) A.是平滑肌收缩节律的控制波
B.其后一定伴随动作电位
C.是一种超极化波
D.在切断支配胃肠的神经后消失
E.以上说法均不对
A.是平滑肌收缩节律的控制波
4 1 2008 1. 上消化道是指(2.0分) A.从口腔到咽
B.从口腔到食管
C.从口腔到胃.
D.从口腔到十二指肠
E.从口腔到空肠
D.从口腔到十二指肠
4 1 2008 2. 不含味蕾的结构是(2.0分) A.菌状乳头
B.轮廓乳头
C.丝状乳头
D.叶状乳头
E.软腭黏膜
C.丝状乳头
4 1 2008 3. 食管第三个狭窄在:(2.0分) A.第4胸椎水平
B.第6胸椎水平
C.第8胸椎水平
D.第10胸椎水平
E.第12胸椎水平
D.第10胸椎水平
4 1 2008 4. 关于阑尾,下述何者正确(2.0分) A.附于结肠起始部
B.位于左髂窝内
C.根部是3条结肠带集中之处
D.属腹膜间位器官
E.一般长度在l0cm以上
C.根部是3条结肠带集中之处
4 1 2008 5. 肝右锁骨中线上界平(2.0分) A.第4肋
B.第5肋
C.第6肋
D.第7肋
E.第8肋
B.第5肋
4 1 2008 6. 不属于直接引入法造影的是? (2.0分) A.胃肠钡餐造影
B.钡灌肠造影
C.逆行肾盂造影
D.子宫输卵管造影
E.静脉肾盂造影
E.静脉肾盂造影
4 1 2008 7. 关于胃良性溃疡,下列说法不正确的是: (2.0分) A.半月综合征
B.粘膜纠集征象
C.胃肠道钡餐检查可见点状钡斑
D.胃蠕动增强
E.窄颈征
A.半月综合征
4 1 2008 8. 胃肠道疾病首选的断面影像学检查方法为(2.0分) A.钡剂造影
B.平片检查
C.MRI检查
D.CT检查
E.胆系造影检查
D.CT检查
4 1 2008 9. 常规肝CT增强扫描,有关动脉期影像的描述,下列哪项不对: (2.0分) A.肝静脉强化不明显
B.肾门以下下腔静脉无明显强化
C.脾脏强化程度比肝脏高
D.肝脏明显强化
E.增强后一般需30S内扫描
D.肝脏明显强化
4 1 2008 10. 55岁女性,B超发现肝右叶4cm强回声团块,CT平扫为边缘锐利的略低密度灶,增强扫描呈填充式强化,最可能的诊断是: (2.0分) A.转移性肝癌
B.原发性肝癌
C.肝血管瘤
D.肝脓肿
E.肝囊肿
C.肝血管瘤
4 1 2008 11. 消化道穿孔的描述中,不正确的有(2.0分) A.没有游离气腹征象就能排除胃肠穿孔
B.穿孔穿入腹腔内时,立位腹平片可见膈下游离气体
C.阑尾穿孔后可无气腹出现
D.穿孔穿入后腹腔可出现肾周气肿
E.以上均正确
A.没有游离气腹征象就能排除胃肠穿孔
4 1 2008 12. 胃肠道异常的基本X线征象包括: (2.0分) A.胃肠道轮廓改变
B.粘膜皱襞的改变
C.功能性改变
D.位置及移动度改变
E.以上都是
E.以上都是
4 1 2008 13. 胰头肿瘤的诊断主要依赖于(2.0分) A.胃肠造影观察十二指肠环
B.血管造影观察血供情况
C.同位素检查
D.磁共振首选
E.CT平扫及增强扫描
E.CT平扫及增强扫描
4 1 2008 14. 甲状腺功能亢进时典型的摄131I率表现为: (2.0分) A.摄131I率增高,高峰>24小时
B.摄131I率增高,高峰4小时出现
C.摄131I率不增高,高峰>24小时
D.摄131I率不增高,高峰4小时出现
E.摄131I率减低。
B.摄131I率增高,高峰4小时出现
4 1 2008 15. 甲状腺癌在甲状腺显像时的典型表现为:(2.0分) A.甲状腺弥漫性浓聚影
B.甲状腺弥漫性稀疏影
C.热结节
D.温结节
E.冷结节
E.冷结节
4 1 2008 16. 胃肠出血显像时,常用的显像剂为:(2.0分) A.99mTcO4-
B.99mTc-MIBI
C.131I
D.99mTc-RBC
E.99mTc-MDP
D.99mTc-RBC
4 1 2008 17. 关于消化管平滑肌基本电节律的正确叙述是(2.0分) A.是一种超极化波
B.其后一定伴随动作电位
C.是平滑肌收缩节律的控制波
D.在切断支配胃肠的神经后消失
E.以上说法均不对
C.是平滑肌收缩节律的控制波
4 1 2008 18. 迷走神经兴奋时(2.0分) A.胃肠平滑肌活动增强,消化腺分泌减少
B.胃肠平滑肌活动减弱,消化腺分泌增加
C.胃肠平滑肌活动增强,消化腺分泌增加
D.胃肠平滑肌活动减弱,消化腺分泌减少
E.以上都不是
C.胃肠平滑肌活动增强,消化腺分泌增加
4 1 2008 19. 关于消化道平滑肌生理特性的错误叙述是(2.0分) A.有自动节律性
B.对电刺激敏感
C.富有伸展性
D.兴奋性较低
E.收缩速度较慢
B.对电刺激敏感
4 1 2008 20. 以下属于胆碱能受体的是(2.0分) A.M、N和α
B.M、N和β
C.M、N1和N2
D.M、α和β
E.M、β1和β2
C.M、N1和N2
4 1 2008 21. 可被扑尔敏阻断的受体是(2.0分) A.H1受体
B.H2受体
C.N型受体
D.M型受体
E.H1和H2型受体
A.H1受体
4 1 2008 22. 胰腺外分泌的功能是:(2.0分) A.参与代谢调节
B.参与消化作用
C.参与信号转导
D. 参与免疫调节
B.参与消化作用
4 1 2008 23. 胆汁酸合成的原料是:(2.0分) A.胆固醇
B.脂肪酸
C. 乙酰辅酶A
D.UDPG
A.胆固醇
4 1 2008 24. 次级胆汁酸的生成部位是:(2.0分) A.肝脏
B.胆囊
C. 肠道
D.胰腺
C. 肠道
4 1 2008 25. 肝脏的双重输入通道是指:(2.0分) A.肝静脉和门静脉
B.肝静脉和胆道
C.门静脉和胆道
D.以上答案均不是
D.以上答案均不是
4 1 2008 26. 起生理性调节和激素样作用的胃肠肽不包括(2.0分) A.促胃液素
B.缩胆囊素
C.促胰液素
D.抑胃肽
E.生长抑素
E.生长抑素
4 1 2008 27. 多种胃肠肽也分布于中枢神经系统中,这种双重分布的肽,被称为(2.0分) A.脑肠肽
B.糖皮质激素
C.内因子
D.胃泌素
E.胆囊收缩素
A.脑肠肽
4 1 2008 28. 下面不属于代谢综合征的成分是(2.0分) A.超重/肥胖
B.高血压
C.血脂异常
D.高血糖
E.冠心病
E.冠心病
4 1 2008 29. 炎症性肠病的特征性皮肤表现有:(2.0分) A.结节性红斑
B.红斑狼疮
C.天胞疮
D.剥脱性皮炎
E.银屑病
A.结节性红斑
4 1 2008 30. PCOS和脂肪肝的最重要共同发病机制:(2.0分) A.脂质代谢紊乱
B.糖代谢紊乱
C.胰岛素抵抗
D.基因多态性
E.蛋白质代谢紊乱
C.胰岛素抵抗
4 1 2008 31. 食管癌最典型的临床表现是(2.0分) A.进行性吞咽困难
B.咽下疼痛
C.咽部异物感
D.食物反流
E.左锁骨上淋巴结肿大
A.进行性吞咽困难
4 1 2008 32. 卓-艾综合征的发生与下列胃肠激素分泌过多有关(2.0分) A.胰岛素
B.胰高血糖素
C.胆囊收缩素
D.促胰酶素
E.胃泌素
E.胃泌素
4 1 2008 33. 诊断肝性脑病最有意义的实验室检查是(2.0分) A.血糖
B.血氨
C.血胆红素
D.血尿素氮
E.血转氨酶
B.血氨
4 1 2008 34. 血清淀粉酶测定正确的是(2.0分) A.发病后立即升高
B.起病后6-12小时开始升高
C.淀粉酶的高低与疾病严重程度呈正比
D.超过正常值2倍即可确诊
E.E持续一周以上
B.起病后6-12小时开始升高
4 1 2008 35. 上消化道出血是指出血的部位是(2.0分) A.食管至幽门
B.十二指肠屈氏韧带以上的消化器官
C.十二指肠以上的消化器官
D.胃以上的消化器官
E.食管至空肠
B.十二指肠屈氏韧带以上的消化器官
4 1 2008 36. 正常人体软组织的内部回声强度排列顺序,下述哪项正常: (2.0分) A.肾窦>胰腺>肝脏>肾实质;
B.肾窦>肝脏>胰腺>肾实质;
C.胰腺>肾窦>肝脏>肾实质;
D.胰腺>肾窦>肾实质>肝脏;
E.肾窦>胰腺>肾实质>肝脏
A.肾窦>胰腺>肝脏>肾实质;
4 1 2008 37. 恶性病变的声像图特点中,应除外:(2.0分) A.形态较规则
B. 边缘不光滑或断续
C.回声低弱或增强
D.周围组织呈浸润性改变
E.血流信号改变
A.形态较规则
4 1 2008 38. 下列疾病可表现为肝内囊性病灶,除外(2.0分) A.肝囊肿
B.多囊肝
C.转移性肝癌
D.包囊肿病
E.局灶性增生结节
C.转移性肝癌
4 1 2008 39. 女,33岁,发现颈部肿块1月余,无压痛,无发热,肿块可随吞咽上下移动,首选的检查是(2.0分) A.甲状腺功能五项
B.常规超声检查
C.超声引导下细针穿刺活检
D.放射性核素扫描
E.血常规
B.常规超声检查
4 1 2008 40. 胰腺癌最好发于胰腺的哪个部位(2.0分) A.胰尾
B.胰体
C.胰头
D.胰颈
E.胰管
C.胰头
4 1 2008 41. 超声表现为子宫形体不规则,表面有球状或结节状突出,呈弱或中等回声的肌瘤类型是(2.0分) A.粘膜下肌瘤
B.肌壁间肌瘤
C.浆膜下肌瘤
D.阔韧带肌瘤
E.宫颈肌瘤
C.浆膜下肌瘤
4 1 2008 42. 子宫肌瘤的液化或囊性变由哪种变性演变而来? (2.0分) A.玻璃样变
B.脂肪变
C.钙化
D.红色样变
E.肉瘤样变
A.玻璃样变
4 1 2008 43. 消化器官不具备下列哪一功能?(2.0分) A.消化
B.内分泌
C.水平衡
D.免疫
E.吸收
C.水平衡
4 1 2008 44. 胰蛋白酶原活化的最主要物质是(2.0分) A.肠激酶
B.胃蛋白酶
C.组胺
D.糜蛋白酶
E.盐酸
A.肠激酶
4 1 2008 45. 所有消化液中最重要的是(2.0分) A.唾液
B.胃液
C.胰液
D.小肠液
E.胆汁
C.胰液
4 1 2008 46. 大肠的主要功能是吸收(2.0分) A.葡萄糖
B.氨基酸
C.脂肪酸
D.水
E.胆固醇
D.水
4 1 2008 47. 胃蛋白酶原的激活物是(2.0分) A.内因子
B.HCl
C.Na十
D.K十
E.蛋白质
B.HCl
4 1 2008 48. 下列哪类物质可能不引起缩胆囊素的释放?(2.0分) A.蛋白质分解产物
B.脂肪
C.盐酸
D.糖
E.脂肪酸
D.糖
4 1 2008 49. 下列有关胃蛋白酶和胃蛋白酶原的描述哪一项是错误的(2.0分) A.胃泌素可以刺激胃蛋白酶原的分泌
B. 胃蛋白可以自身激活酶原
C. 胃酶抑素(pepstatin)可以强烈抑制胃蛋白酶
D.酶原激活时,其一级结构保持不变,但空间结构发生了改变
E.胃表面存在粘液层,可以防止自身被胃蛋白酶消化
D.酶原激活时,其一级结构保持不变,但空间结构发生了改变
4 1 2008 50. 下列哪种酶活性中心具有Zn2+,属于金属蛋白酶家族(2.0分) A. 胃蛋白酶 B.
B.羧肽酶A
C.胰蛋白酶
D.弹性蛋白酶
E.胰糜蛋白酶
B.羧肽酶A
4 1 2009 1. 上消化道是指 (2.0分) A.从口腔到咽
B.从口腔到食管
C.从口腔到胃.
D.从口腔到十二指肠
E.从口腔到空肠
D.从口腔到十二指肠
4 1 2009 2. 腭扁桃体位于(2.0分) A.扁桃体上窝内
B.扁桃体小窝内
C.咽隐窝内
D.腭咽弓后方的窝内
E.腭舌弓与腭咽弓之间
E.腭舌弓与腭咽弓之间
4 1 2009 3. 阑尾根部的体表投影在(2.0分) A.脐与左髂前上棘连线的中、内1/3交点处
B.脐与左髂前上棘连线的中、外1/3交点处
C.脐与右髂前上棘连线的中、内1/3交点处
D.脐与右髂前上棘连线的中、外l/3交点处
E.左、右髂前上棘连线的中点
D.脐与右髂前上棘连线的中、外l/3交点处
4 1 2009 4. 肝胰壶腹开口于(2.0分) A.十二指肠上部
B.十二指肠降部
C.十二指肠水平部
D.十二指肠升部
E.十二指肠空肠曲
B.十二指肠降部
4 1 2009 5. 手术打开腹膜腔后,寻找阑尾的最可靠标志是 (2.0分) A.右髂窝内
B.盲肠末端
C.三条结肠带汇合处
D.回盲部
E.沿阑尾动脉
C.三条结肠带汇合处
4 1 2009 6. 起生理性调节和激素样作用的胃肠肽不包括(2.0分) A.促胃液素
B.缩胆囊素
C.促胰液素
D.抑胃肽
E.生长抑素
E.生长抑素
4 1 2009 7. 多种胃肠肽也分布于中枢神经系统中,这种双重分布的肽,被称为(2.0分) A.脑肠肽
B.糖皮质激素
C.内因子
D.胃泌素
E.胆囊收缩素
A.脑肠肽
4 1 2009 8. 下面不属于代谢综合征的成分是(2.0分) A.超重/肥胖
B.高血压
C.血脂异常
D.高血糖
E.冠心病
E.冠心病
4 1 2009 9. 炎症性肠病的特征性皮肤表现有:(2.0分) A.结节性红斑
B.红斑狼疮
C.天胞疮
D.剥脱性皮炎
E.银屑病
A.结节性红斑
4 1 2009 10. PCOS和脂肪肝的最重要共同发病机制:(2.0分) A.脂质代谢紊乱
B.糖代谢紊乱
C.胰岛素抵抗
D.基因多态性
E.蛋白质代谢紊乱
C.胰岛素抵抗
4 1 2009 11. 食管癌最典型的临床表现是(2.0分) A.进行性吞咽困难
B.咽下疼痛
C.咽部异物感
D.食物反流
E.左锁骨上淋巴结肿大
A.进行性吞咽困难
4 1 2009 12. 卓-艾综合征的发生与下列胃肠激素分泌过多有关(2.0分) A.胰岛素
B.胰高血糖素
C.胆囊收缩素
D.促胰酶素
E.胃泌素
E.胃泌素
4 1 2009 13. 诊断肝性脑病最有意义的实验室检查是(2.0分) A.血糖
B.血氨
C.血胆红素
D.血尿素氮
E.血转氨酶
B.血氨
4 1 2009 14. 血清淀粉酶测定正确的是(2.0分) A.发病后立即升高
B.起病后6-12小时开始升高
C.淀粉酶的高低与疾病严重程度呈正比
D.超过正常值2倍即可确诊
E.持续一周以上
B.起病后6-12小时开始升高
4 1 2009 15. 上消化道出血是指出血的部位是(2.0分) A.食管至幽门
B.十二指肠屈氏韧带以上的消化器官
C.十二指肠以上的消化器官
D.胃以上的消化器官
E.食管至空肠
B.十二指肠屈氏韧带以上的消化器官
4 1 2009 16. 下列哪项不是胶囊内镜的适应症(2.0分) A.不明原因消化道出血
B.溃疡性结肠炎
C.疑似小肠肿瘤
D.腹痛待查
E.不明原因吸收不良综合征
B.溃疡性结肠炎
4 1 2009 17. 甲状腺功能亢进时典型的摄131I率表现为: (2.0分) A.摄131I率增高,高峰>24小时
B.摄131I率增高,高峰4小时出现
C.摄131I率不增高,高峰>24小时
D.摄131I率不增高,高峰4小时出现
E.摄131I率减低。
B.摄131I率增高,高峰4小时出现
4 1 2009 18. 甲状腺癌在甲状腺显像时的典型表现为:(2.0分) A.甲状腺弥漫性浓聚影
B.甲状腺弥漫性稀疏影
C.热结节
D.温结节
E.冷结节
E.冷结节
4 1 2009 19. 胃肠出血显像时,常用的显像剂为:(2.0分) A.99mTcO4-
B.99mTc-MIBI
C.131I
D.99mTc-RBC
E.99mTc-MDP
D.99mTc-RBC
4 1 2009 20. 一黄疸病人超声发现肝内胆管扩张、胆囊肿大、主胰管不扩张,其阻塞部位可能在(2.0分) A.肝门部
B.肝总管
C.胆囊管以下
D.壶腹部
E.胆总管
A.肝门部
4 1 2009 21. 哪些器官的检查应首选超声检查(USG): (2.0分) A.肝脏弥散性病变
B.骨折后愈合情况
C.胃炎
D.肺炎性假瘤
E.头颅出血
A.肝脏弥散性病变
4 1 2009 22. 桥本氏甲状腺炎与甲亢的鉴别最有意义的是(2.0分) A.甲状腺功能五项
B.常规超声检查
C.超声引导下细针穿刺活检
D.放射性核素扫描
E.血常规
A.甲状腺功能五项
4 1 2009 23. 女,33岁,发现颈部肿块1月余,无压痛,无发热,肿块可随吞咽上下移动,最有意义的检查是(2.0分) A.甲状腺功能五项
B.常规超声检查
C.超声引导下细针穿刺活检
D.放射性核素扫描
E.血常规
C.超声引导下细针穿刺活检
4 1 2009 24. 男,32岁,酗酒后1天出现上腹痛、恶心、呕吐、发热,化验示血、尿淀粉酶均升高,超声检查最可能出现的声像图表现(2.0分) A.胰腺体积缩小
B.主胰管重度扩张
C.胰腺囊肿
D.胰腺假性囊肿
E.胰腺体积增大,形态饱满
E.胰腺体积增大,形态饱满
4 1 2009 25. 慢性胰腺炎超声声像图表现对其诊断最有意义的是(2.0分) A.胰管扩张伴结石
B.胰腺体积增大
C.胰腺呈低回声
D.胰腺与周围组织分界不清
E.胰腺体积缩小
A.胰管扩张伴结石
4 1 2009 26. "宫腔分离征"见于一下那种肌瘤类型? (2.0分) A.粘膜下肌瘤
B.肌壁间肌瘤
C.浆膜下肌瘤
D.阔韧带肌瘤
E.宫颈肌瘤
A.粘膜下肌瘤
4 1 2009 27. 粘膜下肌瘤需以下那种疾病鉴别(2.0分) A.卵巢肿瘤
B.子宫内膜息肉
C.子宫内膜异位
D.输卵管异位妊娠
E.黄体囊肿
B.子宫内膜息肉
4 1 2009 28. 下列哪一组氨基酸属于营养半必需氨基酸 (2.0分) A.Met和Thr
B.Glu和Arg
C. Gln和Arg
D.Tyr和Cys
E. Gly和Asp
D.Tyr和Cys
4 1 2009 29. 胃蛋白酶属于酸性蛋白酶,其最适pH为: (2.0分) A.pH 5~6
B.pH 7.4
C.pH 7~8
D.pH 2~3
E.pH 9~10
D.pH 2~3
4 1 2009 30. 下列哪种酶活性中心具有Zn2+,属于金属蛋白酶家族 (2.0分) A.胃蛋白酶
B.羧肽酶A
C.胰蛋白酶
D.弹性蛋白酶
E.胰糜蛋白酶
B.羧肽酶A
4 1 2009 31. 下列有关氨基酸吸收和转运的描述哪一项是错误的 (2.0分) A.小肠上皮细胞膜上有氨基酸的转运体(transporter),可以将氨基酸吸收并转运进入血液
B.氨基酸可以通过被动扩散过程而吸收
C.氨基酸的转运体包括需Na+和不需Na+两种
D.氨基酸的吸收过程通常需要消耗ATP
E.某些氨基酸既可以通过需Na+转运体也可以通过不需Na+转运体吸收
B.氨基酸可以通过被动扩散过程而吸收
4 1 2009 32. 营养学上价值最高的脂肪酸为 (2.0分) A.长链脂肪酸
B.短链脂肪酸
C.顺式脂肪酸
D.反式脂肪酸
E.ω-3和ω-6系列不饱和脂肪酸
E.ω-3和ω-6系列不饱和脂肪酸
4 1 2009 33. 下列关于脂类吸收的描述哪一项是错误的 (2.0分) A.脂肪酶水解甘油三酯成为游离脂肪酸和甘油一酯
B.甘油三酯的消化产物,可以搀入到微团成为混合微团(mixed micelle),容易穿过小肠黏膜细胞的细胞膜而被吸收
C.短链和中链脂肪酸可由小肠细胞吸收并通过淋巴入血
D.长链脂肪酸吸收后,在黏膜细胞内重新合成甘油三脂,然后与其他类脂和蛋白形成乳糜微粒(CM),通过淋巴进入血循环
E.食物中的游离胆固醇可直接被吸收,而胆固醇脂则先被酶水解成游离的胆固醇,再被吸收。
C.短链和中链脂肪酸可由小肠细胞吸收并通过淋巴入血
4 1 2009 34. 碳水化合物的消化开始于 (2.0分) A.口腔
B.胃
C.小肠
D.大肠
E.食管
A.口腔
4 1 2009 35. 下列有关葡萄糖的吸收与转运的描述哪一项目是正确的(2.0分) A.通过葡萄糖转运体吸收和转运, 通常需要消耗ATP
B.葡萄糖转运体分为5型,不需要Na+参与
C.不同组织的细胞使用同一种葡萄糖转运体
D.葡萄糖在小肠中可以被动吸收
E.葡萄糖转运体是有多个亚基组成的跨膜蛋白
A.通过葡萄糖转运体吸收和转运, 通常需要消耗ATP
4 1 2009 36. 肝胆疾病引起胆汁酸合成和分泌不足,可以引起哪些维生素的缺乏(2.0分) A.脂溶性维生素
B.生物素和烟酸
C.叶酸和B6
D. 维生素B1和B2
E.维生素A 和C
A.脂溶性维生素
4 1 2009 37. 下列哪种矿物质的缺少可以引起心肌病(克山病)和大关节病(2.0分) A.硒
B.锌
C.碘
D.钙
E.铁
A.硒
4 1 2009 38. 不属于直接引入法造影的是? (2.0分) A.胃肠钡餐造影
B.钡灌肠造
C.静脉肾盂造影
D.子宫输卵管造影
E.逆行肾盂造影
C.静脉肾盂造影
4 1 2009 39. 食管前缘可见三个压迹,自上倒下依次为: (2.0分) A.主动脉弓、左心房和左主支气管
B.主动脉弓、左主支气管和左心房
C.左主支气管、主动脉弓和左心房
D.主动脉弓、右主支气管和右心房
E.主动脉弓、右主支气管和右心室
B.主动脉弓、左主支气管和左心房
4 1 2009 40. 常用医用硫酸钡作为对比剂的造影部位是(2.0分) A.胃肠道造影
B.脑血管造影
C.胆囊造影
D.椎管造影
E.泌尿系造影
A.胃肠道造影
4 1 2009 41. 膈下游离气体的原因是(2.0分) A.小肠、结肠穿孔
B.手术后腹腔残留气体
C.产气菌引起的急性腹膜炎
D.胃穿孔
E.以上都是
E.以上都是
4 1 2009 42. 常规肝CT动态增强扫描,三期扫描依次为(2.0分) A.动脉期、门脉期、平衡期
B.动脉期、门脉期、排泄期
C.平衡期、动脉期、门脉期
D.门脉期、动脉期、排泄期
E.门脉期、平衡期、动脉期
A.动脉期、门脉期、平衡期
4 1 2009 43. 55岁女性,慢性肝病病史,B超发现肝右叶4cm低回声团块,CT平扫为边缘欠清楚的低密度灶,增强扫描呈快进快出,最可能的诊断是:(2.0分) A.转移性肝癌
B.原发性肝癌
C.肝血管瘤
D.肝脓肿
E.肝囊肿
B.原发性肝癌
4 1 2009 44. 消化道穿孔的描述中,正确的有(2.0分) A.没有游离气腹征象就能排除胃肠穿孔
B.穿孔穿入腹腔内时,立位腹平片可见膈下游离气体
C.小肠及阑尾穿孔后无气腹出现
D.穿孔穿入腹腔内可出现肾周气肿
E.以上均正确
B.穿孔穿入腹腔内时,立位腹平片可见膈下游离气体
4 1 2009 45. 首选依靠断面影像手段检查的是?(2.0分) A.胃肠道病变
B.骨髂病变
C.肝、胆、胰病变
D.神经病变
E.胸部
D.神经病变
4 1 2009 46. 单纯性小肠梗阻不包括以下那个征象: (2.0分) A.阶梯状排列液气平面
B.肠腔扩大
C.环状黏膜
D.结肠袋变浅消失
D.结肠袋变浅消失
4 1 2009 47. 在MR扫描T2WI出现'灯泡征',最可能的诊断: (2.0分) A.转移性肝癌
B.原发性肝癌
C.肝血管瘤
D.肝脓肿
E.肝囊肿
C.肝血管瘤
4 1 2009 48. 关于肝药酶的描述,错误的是:(2.0分) A.属P-450酶系统
B.其活性有限
C.易发生竞争性抑制
D.个体差异大
E.只代谢20余种药物
E.只代谢20余种药物
4 1 2009 49. 药物的作用强度,主要取决于:(2.0分) A.药物在血液中的浓度
B.在靶器官的浓度大小
C.药物排泄的速率大小
D.药物与血浆蛋白结合率之高低
E.以上都对
B.在靶器官的浓度大小
4 1 2009 50. 可被扑尔敏阻断的受体是(2.0分) A.H1受体
B.H2受体
C.N型受体
D.M型受体
E.H1和H2型受体
A.H1受体
4 1 2010 1. 关于消化管平滑肌基本电节律的正确叙述是 (2.5分) A.是一种超极化波
B.其后一定伴随动作电位
C.是平滑肌收缩节律的控制波
D.在切断支配胃肠的神经后消失
E.以上说法均不对
C.是平滑肌收缩节律的控制波
4 1 2010 2. 迷走神经兴奋时(2.5分) A.胃肠平滑肌活动增强,消化腺分泌减少
B.胃肠平滑肌活动减弱,消化腺分泌增加
C.胃肠平滑肌活动增强,消化腺分泌增加
D.胃肠平滑肌活动减弱,消化腺分泌减少
E.以上都不是
C.胃肠平滑肌活动增强,消化腺分泌增加
4 1 2010 3. 关于消化道平滑肌生理特性的错误叙述是 (2.5分) A.有自动节律性
B.对电刺激敏感
C.富有伸展性
D.兴奋性较低
E.收缩速度较慢
B.对电刺激敏感
4 1 2010 4. 以下属于胆碱能受体的是 (2.5分) A.M、N和α
B.M、N和β
C.M、N1和N2
D.M、α和β
E.M、β1和β2
C.M、N1和N2
4 1 2010 5. 可被扑尔敏阻断的受体是 (2.5分) A.H1受体
B.H2受体
C.N型受体
D.M型受体
E.H1和H2型受体
A.H1受体
4 1 2010 6. 甲状腺功能亢进时典型的摄131I率表现为: (2.5分) A.摄131I率增高,高峰>24小时
B.摄131I率增高,高峰4小时出现
C.摄131I率不增高,高峰>24小时
D.摄131I率不增高,高峰4小时出现
E.摄131I率减低。
B.摄131I率增高,高峰4小时出现
4 1 2010 7. 甲状腺癌在甲状腺显像时的典型表现为:(2.5分) A.甲状腺弥漫性浓聚影
B.甲状腺弥漫性稀疏影
C.热结节
D.温结节
E.冷结节
E.冷结节
4 1 2010 8. 下列何者是131I治疗甲亢的禁忌征(2.5分) A.甲亢术后复发
B.抗甲状腺药物(ATD)治疗甲亢过程中出现白细胞、血小板减少而停止治疗者
C.孕妇
D.有效半衰期短于三天者
E.年龄小于25周岁者
C.孕妇
4 1 2010 9. 胃排空显像发现胃排空加快,可见于:(2.5分) A.甲亢
B.十二指肠溃疡
C.胃窦癌
D.胃大部切除伴倾倒综合征
E.糖尿病
A.甲亢
4 1 2010 10. 胰腺内分泌的功能是调控:(2.5分) A.血糖稳态
B.电解质稳态
C. 氮库稳态
D.脂代谢稳态
E. 氧化还原稳态
A.血糖稳态
4 1 2010 11. 胰腺分泌的消化酶多以前体形式存在,其中对其它胰酶的激活具有关键作用的胰酶是:(2.5分) A.肠激酶
B. 羧肽酶
C.胰蛋白酶
D.糜蛋白酶
E.弹性蛋白酶
C.胰蛋白酶
4 1 2010 12. 以下属于次级胆汁酸的是:(2.5分) A.鹅脱氧胆酸
B. 胆酸
C.脱氧胆酸
D.硫磺胆酸
E.甘氨胆酸
C.脱氧胆酸
4 1 2010 13. 次级胆汁酸的生成部位是:(2.5分) A.肝脏
B.胆囊
C.肠道
D.胰腺
E.胃
C.肠道
4 1 2010 14. 肝脏的双重输入通道是指:(2.5分) A.肝静脉和门静脉
B.肝静脉和胆道
C.门静脉和胆道
D.门静脉和肝动脉
E.以上答案均不是
D.门静脉和肝动脉
4 1 2010 15. 胰蛋白酶原活化的最主要物质是(2.5分) A.肠激酶
B.胃蛋白酶
C.组胺
D.糜蛋白酶
E.盐酸
A.肠激酶
4 1 2010 16. 胆汁中与消化有关的成分是(2.5分) A.胆盐
B.胆固醇
C.胆色素
D.水和无机盐
E.各种消化酶
A.胆盐
4 1 2010 17. 头期胃液分泌的调节是(2.5分) A.纯条件反射性的
B.纯神经调节性的
C.纯体液调节性的
D.神经--体液调节性的
E.自主调节
D.神经--体液调节性的
4 1 2010 18. 下列哪类食物成分是胃泌素释放的强刺激物?(2.5分) A.糖
B.蛋白质
C.脂肪
D.维生素
E.无机盐
B.蛋白质
4 1 2010 19. 关于胃容受性舒张的正确叙述是(2.5分) A.反射性地胃窦部肌肉舒张
B.切断双侧迷走神经后仍出现
C.使胃容量大增
D.阿托品可抑制它的出现
E.是体液调节
C.使胃容量大增
4 1 2010 20. 所有消化液中最重要的是(2.5分) A.唾液
B.胃液
C.胰液
D.小肠液
E.胆汁
C.胰液
4 1 2010 21. 迷走神经兴奋引起的胰液分泌的特点是(2.5分) A.酶多
B.HCO3-多
C.H2O多
D.酶和HCO3-均多
E.酶、H2O和HCO3-均多
A.酶多
4 1 2010 22. 胆汁对脂肪的消化和吸收有促进作用,主要是由于它含有(2.5分) A.脂肪酶
B.胆红素
C.胆盐
D.胆绿质
E.胆固醇
C.胆盐
4 1 2010 23. 下列哪种消化液中没有消化酶?(2.5分) A.唾液
B.胃液
C.胆汁
D.胰液
E.小肠液
C.胆汁
4 1 2010 24. 促胰液素引起胆汁分泌的特点是(2.5分) A.量多,HCO3一量很少
B.量多,HCO3一量也多
C.量少,HCO3一量多
D.量少,HCO3一量多
E.量少,胆盐量多
B.量多,HCO3一量也多
4 1 2010 25. 大肠的主要功能是吸收(2.5分) A.葡萄糖
B.氨基酸
C.脂肪酸
D.水
E.胆固醇
D.水
4 1 2010 26. 水在小肠内的主要吸收机制是(2.5分) A.渗透
B.单纯扩散
C.易化扩散
D.主动转运
E.以上均不是
A.渗透
4 1 2010 27. 盐酸可促进小肠吸收(2.5分) A.NaCl
B.铁和钙
C.维生素B12
D.葡萄糖
E.氨基酸
B.铁和钙
4 1 2010 28. 淀粉在小肠内被吸收的主要形式是(2.5分) A.麦芽糖
B.果糖
C.葡萄糖
D.蔗糖
E.半乳糖
C.葡萄糖
4 1 2010 29. 切断支配小肠的迷走神经,可以导致小肠(2.5分) A.紧张性消失,蠕动减弱
B.紧张性减弱,蠕动消失
C.紧张性消失,蠕动消失
D.紧张性和蠕动性均减弱
E.以上都不对
D.紧张性和蠕动性均减弱
4 1 2010 30. 胃蛋白酶原的激活物是(2.5分) A.内因子
B.HCl
C.Na十
D.K十
E.蛋白质
B.HCl
4 1 2010 31. 下列哪类物质可能不引起缩胆囊素的释放?(2.5分) A.蛋白质分解产物
B.脂肪
C.盐酸
D.糖
E.脂肪酸
D.糖
4 1 2010 32. 诊断消化性溃疡主要依靠下列哪项方法? (2.5分) A.病史分析,如典型的周期性和节律性上腹部疼痛
B.X线钡餐检查
C.内镜检查
D. H.pylori检查
E.诊断性治疗
C.内镜检查
4 1 2010 33. 目前认为下列哪项为慢性胃炎及消化性溃疡的最常见病因: (2.5分) A.吸烟
B.饮酒
C.自身免疫
D.幽门螺杆菌感染
E.十二指肠液反流
D.幽门螺杆菌感染
4 1 2010 34. 下列哪项消化性溃疡的并发症一般不发生在十二指肠溃疡患者中? (2.5分) A.出血
B.幽门梗阻
C.穿孔
D.癌变
E.溃疡复发
D.癌变
4 1 2010 35. 哪些检查是H. pylori根除治疗后首选的检查 (2.5分) A.血清H. pylori IgG
B.C-14或C-13呼气试验
C.血清H. pylori IgM
D.粪便H. pylori 抗原 培养
E.H. pylori培养
B.C-14或C-13呼气试验
4 1 2010 36. 下面有关消化性溃疡的流行病学情况的描述哪项是错误的? (2.5分) A.近年来消化性溃疡的发病率有所下降
B.男性抽烟较多,所以GU好发于女性而DU好发于男性
C.DU多见于青壮年而GU多见于老年
D.总的发病率来看DU比GU多见
E.消化性溃疡与H. pylori密切相关
B.男性抽烟较多,所以GU好发于女性而DU好发于男性
4 1 2010 37. 有关根除H. pylori的治疗哪项是错误的? (2.5分) A.目前常用的新三联治疗中不包括铋剂
B.确定H. pylori是否被根除的试验应在治疗后2周内进行
C.复查H. pylori是否被根除的试验前1周应停制酸药1周
D.H. pylori根除治疗的疗程一般为7天
E.确定H. pylori是否被根除的试验可用非侵入性方法复查
B.确定H. pylori是否被根除的试验应在治疗后2周内进行
4 1 2010 38. A 38-year-old woman with rheumatoid arthritis presents to her physician because of increased joint pain. On physical examination, both passive and active range of motion of the hips are decreased. Her physician increases her dose of nonsteroidal son inflammatory drugs (NSAIDs). Which of the following should be prescribed as well to prevent peptic ulcer disease?(2.5分) A.Cimetidine
B.Clarithromydn
C.Misoprostol
D.Omeprazole
E.Sucralfate
C.Misoprostol
4 1 2010 39. A 28-year-old Caucasian male presents to your office with periodic abdominal pain. The pain starts several hours after a meal and is relieved by antacids. Physical examination is unremarkable. You schedule a work-up in this patient. A mucosal ulcer is detected on endoscopy. In which of the following structures would you most expect this defect to be located? (2.5分) A.Angle of His
B.Lesser curvature
C.Pylorus
D.Ampulla of Vater
E.Ligament of Treitz
C.Pylorus
4 1 2010 40. Partial gastrectomy is performed on a 32-year-old Caucasian male because of refractory peptic ulcer disease. The pathologist receives the tissue and notes significant enlargement of the gastric rugal folds on gross examination. Microscopy of the gastric mucosa reveals parietal cell hyperplasia. Which of the following stimuli is the most likely cause of parietal cell proliferation in this patient? (2.5分) A.Acetylcholine
B.Serotonin
C.Somatostatin
D.Gastrin
E.Secretin
D.Gastrin
4 1 2011 1. 胰腺分泌的消化酶多以前体形式存在,其中对其它胰酶的激活具有关键作用的胰酶是:(2.5分) A.肠激酶
B.羧肽酶
C.胰蛋白酶
D.糜蛋白酶
E.弹性蛋白酶
C.胰蛋白酶
4 1 2011 2. 胆汁酸合成的起始原料是(2.5分) A.胆固醇
B.脂肪酸
C.乙酰辅酶A
D.UDPG
E.胆红素
A.胆固醇
4 1 2011 3. 出入肝门的结构是(2.5分) A.肝静脉、肝外胆道、肝固有动脉
B.肝门静脉、肝固有动脉、肝静脉
C.肝外胆道、肝总动脉、肝门静脉
D.胆总管、肝门静脉、肝固有动脉
E.肝外胆道、肝固有动脉、肝门静脉
E.肝外胆道、肝固有动脉、肝门静脉
4 1 2011 4. 关于腮腺管,下述哪项正确(2.5分) A.由腺体深部的前缘发出
B.开口于平对上颌第2前磨牙所对的颊黏膜上
C.开口于平对上颌第2磨牙所对的颊黏膜上
D.开口于平对下颌第2前磨牙所对的颊黏膜上
E.开口于平对下颌第2磨牙所对的颊黏膜上
B.开口于平对上颌第2前磨牙所对的颊黏膜上
4 1 2011 5. 甲状腺功能亢进时典型的摄131I率表现为 (2.5分) A.摄131I率增高,高峰>24小时
B.摄131I率增高,高峰4小时出现
C.摄131I率不增高,高峰>24小时
D.摄131I率不增高,高峰4小时出现
E.摄131I率减低。
B.摄131I率增高,高峰4小时出现
4 1 2011 6. 药物的生物转化和排泄速度决定其(2.5分) A.副作用的多少
B.最大效应的高低
C.作用持续时间的长短
D.起效的快慢
E.后遗效应的大小
C.作用持续时间的长短
4 1 2011 7. 医用硫酸钡作为对比剂的造影部位是:(2.5分) A.胃肠道造影
B.肠系膜上动脉血管造影
C.胆道造影
D.椎管造影
E.泌尿系造影
A.胃肠道造影
4 1 2011 8. 胃粘膜皱襞改变,下列哪项是错误描述: (2.5分) A.粘膜肥厚多代表慢性炎症。
B.粘膜破坏中断多为良性溃疡所致
C.良性溃疡可引起粘膜纠集。
D.恶性肿瘤可引起粘膜皱襞破坏消失
E.脑回状外观多为慢性炎症所致
B.粘膜破坏中断多为良性溃疡所致
4 1 2011 9. 人体不同组织和体液回声强度是不同的,下列哪一项是错误的(2.5分) A.液体均是无回声的,固体均是有回声的
B.实质性组织如肝脾是中等水平回声
C.脂肪组织是低水平回声
D.纤维化和钙化引起组织回声增强
E.胃壁-胃内容物(含气)界面产生很强的反射
A.液体均是无回声的,固体均是有回声的
4 1 2011 10. 关于小肝癌的描述,下列哪项是错误的(2.5分) A.圆形低回声结节
B.远侧回声可轻度增强
C.可见侧方声影
D.边界清楚
E.周围见较宽的晕环,中心液化呈"牛眼征"
E.周围见较宽的晕环,中心液化呈"牛眼征"
4 1 2011 11. 下列哪条血管是寻找胰腺体尾部的标志: (2.5分) A.门静脉
B.胃左动脉
C.脾静脉
D.左肾动脉
E.腹主动脉
C.脾静脉
4 1 2011 12. 实性肿瘤声像图特征下列哪项是错误的(2.5分) A.边界回声可光滑、整齐或不规则
B.外形常呈圆形、椭圆形或不规则
C.内部有回声
D.后方回声均有衰减
E.侧方声影可有可无
D.后方回声均有衰减
4 1 2011 13. 患者男性,45岁,上腹不适2 月伴体重下降,声像图显示胰头区不规则低回声实性肿块,边界不清,呈蟹足状,肠系膜上静脉轻度抬高,肝内外胆管扩张,主胰管内径4mm,最可能诊断是:(2.5分) A.胰岛细胞瘤
B.胰腺癌
C.腹膜后肿瘤
D.胆管癌
E.壶腹癌
B.胰腺癌
4 1 2011 14. 乳糖不耐受症是由于下列哪种酶缺乏所引起(2.5分) A.半乳糖酶
B.蔗糖酶
C.麦芽糖酶
D. -淀粉酶
E.乳糖酶
E.乳糖酶
4 1 2011 15. 肝胆疾病引起胆汁酸合成和分泌不足,可以引起哪些维生素的缺乏(2.5分) A.脂溶性维生素
B.生物素 和 烟酸
C.叶酸和B6
D.维生素B1和B2
E.维生素A 和C
A.脂溶性维生素
4 1 2011 16. 下列有关氨基酸吸收和转运的描述哪一项是错误的(2.5分) A.小肠上皮细胞膜上有氨基酸的转运体(transporter),可以将氨基酸吸收并转运进入血液
B.氨基酸可以通过被动扩散过程而吸收
C.氨基酸的转运体包括需Na+ 和 不需Na+ 两种
D.氨基酸的吸收过程通常需要消耗ATP
E.某些氨基酸既可以通过需Na+ 转运体也可以通过不需Na+ 转运体吸收
B.氨基酸可以通过被动扩散过程而吸收
4 1 2011 17. 下列哪种氨基酸不是营养必需氨基酸(2.5分) A.Val
B.Ser
C.Trp
D.Lys
E.Ile
B.Ser
4 1 2011 18. 胃蛋白酶属于酸性蛋白酶,其最适pH为: (2.5分) A.pH 5~6
B.pH 7.4
C.pH 7~8
D.pH 2~3
E.pH 9~10
D.pH 2~3
4 1 2011 19. 诊断消化性溃疡主要依靠下列哪项方法? (2.5分) A.病史分析,如典型的周期性和节律性上腹部疼痛
B.X线钡餐检查
C.内镜检查
D.H.pylori检查
E.诊断性治疗
C.内镜检查
4 1 2011 20. 导致消化性溃疡最主要的侵袭因子为 (2.5分) A.胃酸
B.胆盐
C.胰酶
D.NSAID
E.胃蛋白酶
A.胃酸
4 1 2011 21. 哪些检查是H. pylori根除治疗后首选的检查 (2.5分) A.血清H. pylori IgG
B.C-14或C-13呼气试验
C.血清H. pylori IgM
D.粪便H. pylori 抗原 培养
E.H. pylori培养
B.C-14或C-13呼气试验
4 1 2011 22. 除下列哪项以外都是胃粘膜的防御修复因素? (2.5分) A.生长抑素
B.前列腺素
C.表皮生长因子
D.胃粘液屏障
E.丰富的粘膜血流
A.生长抑素
4 1 2011 23. A 38-year-old woman with rheumatoid arthritis presents to her physician because of increased joint pain. On physical examination, both passive and active range of motion of the hips are decreased. Her physician increases her dose of nonsteroidal son inflammatory drugs (NSAIDs). Which of the following should be prescribed as well to prevent peptic ulcer disease? (2.5分) A.Cimetidine
B.Clarithromydn
C.Misoprostol
D.Omeprazole
E.Sucralfate
C.Misoprostol
4 1 2011 24. A 34-year-old Caucasian male presents to your office with periodic abdominal pain. Upper gastrointestinal endoscopy shows an ulcer in the distal duodenum. His basal gastric acid secretion is high with the ratio of basal to maximal acid output of 0.7. This patient's condition is most likely caused by which of the following?(2.5分) A.Stress-related mucosal erosions
B.Chronic gastritis with antral sparing
C.Chronic antral-predominant gastritis
D.Peptic ulcer disease
E.Non-gastric neoplasm
E.Non-gastric neoplasm
4 1 2011 25. A 45-year-old Caucasian male presents to your office with abdominal discomfort and recent weight loss. Stool guaiac testis positive. If detected on further work-up, an ulcer in which of the following locations is most likely to be benign? (2.5分) A.Esophagus
B.Stomach
C.Duodenum
D.Sigmoid colon
E.Rectum
C.Duodenum
4 1 2011 26. 胃溃疡的腹痛规律是(2.5分) A.无明显节律性
B.疼痛-排便-加重
C.进食-疼痛-缓解
D.疼痛-进食-缓解
E.疼痛-排便-缓解
C.进食-疼痛-缓解
4 1 2011 27. 对球后溃疡的正确描述是(2.5分) A.多发生在距离幽门2-3cm以内
B.指十二指肠球部后壁溃疡
C.易并发出血
D.临床症状多不明显
E.内科治疗效果好
C.易并发出血
4 1 2011 28. 对降低消化性溃疡复发最有效的治疗措施是(2.5分) A.抗生素治疗
B.根除幽门螺杆菌治疗
C.高选择性迷走神经切除术
D.抑酸剂治疗
E.胃黏你保护剂治疗
B.根除幽门螺杆菌治疗
4 1 2011 29. 为判断幽门螺杆菌是否被根除,正确的检查时间应在治疗结束后至少(2.5分) A.2周
B.1周
C.4周
D.3周
E.3天
C.4周
4 1 2011 30. 关于营养物质的吸收,叙述错误的是(2.5分) A.营养物质透过消化管粘膜进入血液的过程,称吸收
B.有主动吸收和被动吸收两种形式
C.一般形成小分子物质后才能被吸收
D.脂肪分解产物均经淋巴进入体循环
E.白质分解产物均经血液途径吸收
D.脂肪分解产物均经淋巴进入体循环
4 1 2011 31. 有关大肠功能的叙述,错误的是(2.5分) A.贮存食物残渣并形成粪便
B.大肠液中的消化酶分解食物残渣
C.大肠液保护润滑粪便
D.肠内细菌可利用简单物质合成维生素B族和维生素K
E.吸收水分
B.大肠液中的消化酶分解食物残渣
4 1 2011 32. 关于迷走神经兴奋效应的叙述,错误的是(2.5分) A.胆囊收缩
B.肠管蠕动加强
C.胃肠管括约肌收缩
D.消化腺分泌
E.胃运动增强
C.胃肠管括约肌收缩
4 1 2011 33. 下列哪种物质的吸收不需钠泵参与?(2.5分) A.NaCl
B.葡萄糖
C.氨基酸
D.水溶性维生素
E.半乳糖
D.水溶性维生素
4 1 2011 34. 关于胰液中碳酸氢盐的错误叙述是(2.5分) A.由胰腺内小导管细胞分泌
B.是胰液无机物成分中含量最低者
C.中和进入十二脂肠的胃酸
D.提供小肠内多种消化酶的最适pH环境
E.以上均正确
B.是胰液无机物成分中含量最低者
4 1 2011 35. 下列哪项不是胰液中的消化酶 ?(2.5分) A.胰淀粉酶
B.胰脂肪酶
C.胰蛋白酶
D.糜蛋白酶
E.胰岛素
E.胰岛素
4 1 2011 36. 关于内因子的正确叙述是(2.5分) A.胃腺的主细胞分泌
B.属肽类激素
C.促进胃酸分泌
D.促进VitB12的吸收
E.促进蛋白质的消化
D.促进VitB12的吸收
4 1 2011 37. 支配胃肠道的交感节后纤维释放的递质是(2.5分) A.乙酰胆碱
B.多巴胺
C.肾上腺素
D.去甲肾上腺素
E.织胺
D.去甲肾上腺素
4 1 2011 38. 人体内最大、最复杂的内分泌器官是(2.5分) A.消化道
B.下丘脑
C.腺垂体
D.心脏
E.胰脏
A.消化道
4 1 2011 39. 关于消化管平滑肌基本电节律的正确叙述是(2.5分) A.是一种超极化波
B.其后一定伴随动作电位
C.是平滑肌收缩节律的控制波
D.在切断支配胃肠的神经后消失
E.以上说法均不对
C.是平滑肌收缩节律的控制波
4 1 2011 40. 支配胃肠道的交感节后纤维释放的递质是 (2.5分) A.乙酰胆碱
B.多巴胺
C.肾上腺素
D.去甲肾上腺素
E.织胺
D.去甲肾上腺素
4 1 2012 1. 下列选项中关于胆汁酸正确的是(2.5分) A.其生物合成的初始底物是脂肪酸
B.只有疏水性,没有亲水性
C.结合胆汁酸比游离胆汁酸水溶性更高,易于乳化脂肪
D.胆汁酸能直接降解脂肪
E.释放后绝大部分直接排出体外
C.结合胆汁酸比游离胆汁酸水溶性更高,易于乳化脂肪
4 1 2012 2. 关于胰酶分泌的说法中不正确的是 (2.5分) A.未消化的糖类刺激释放
B.饮食中如脂肪和蛋白质含量低则释放也少
C.食物到达胃部以前已能够刺激胰酶释放
D.肠道的酸碱值对胰酶分泌影响很大
E.胰酶分泌调控的头期属神经性调控
A.未消化的糖类刺激释放
4 1 2012 3. 关于盲肠,下列叙述哪项正确(2.5分) A.长约10cm
B.为腹膜外位器官
C.3条结肠带在阑尾根部汇聚
D.小儿盲肠较低,常达盆腔
E.内侧为右结肠旁沟
C.3条结肠带在阑尾根部汇聚
4 1 2012 4. 出入肝门的结构是(2.5分) A.肝静脉、肝外胆道、肝固有动脉
B.肝门静脉、肝固有动脉、肝静脉
C.肝外胆道、肝总动脉、肝门静脉
D.胆总管、肝门静脉、肝固有动脉
E.肝外胆道、肝固有动脉、肝门静脉
E.肝外胆道、肝固有动脉、肝门静脉
4 1 2012 5. 甲状腺功能亢进时典型的摄131I率表现为: (2.5分) A.摄131I率增高,高峰>24小时
B.摄131I率增高,高峰4小时出现
C.摄131I率不增高,高峰>24小时
D.摄131I率不增高,高峰4小时出现
E.摄131I率减低
B.摄131I率增高,高峰4小时出现
4 1 2012 6. 胃肠出血显像时,常用的显像剂为(2.5分) A.99mTcO4-
B.99mTc-MIBI
C.131I
D.99mTc-RBC
E.99mTc-MDP
D.99mTc-RBC
4 1 2012 7. 迷走神经兴奋时(2.5分) A.胃肠平滑肌活动增强,消化腺分泌减少
B.胃肠平滑肌活动减弱,消化腺分泌增加
C.胃肠平滑肌活动增强,消化腺分泌增加
D.胃肠平滑肌活动减弱,消化腺分泌减少
E.以上都不是
C.胃肠平滑肌活动增强,消化腺分泌增加
4 1 2012 8. 可被阿托品阻断的受体是(2.5分) A.α受体
B.β受体
C.N型受体
D.M型受体
E.N型和M型受体
D.M型受体
4 1 2012 9. 药物的生物转化和排泄速度决定其(2.5分) A.副作用的多少
B.最大效应的高低
C.作用持续时间的长短
D.起效的快慢
E.后遗效应的大小
C.作用持续时间的长短
4 1 2012 10. 肝损显著影响异丙酚药效的原因是(2.5分) A.异丙酚脂溶性高
B.异丙酚在体内迅速分布
C.异丙酚药效维持时间短
D.异丙酚起效迅速
E.异丙酚与血浆蛋白结合率高
A.异丙酚脂溶性高
4 1 2012 11. 人体不同组织和体液回声强度是不同的,下列哪一项是错误的(2.5分) A.液体均是无回声的,固体均是有回声的
B.实质性组织如肝脾是中等水平回声
C.脂肪组织是低水平回声
D.纤维化和钙化引起组织回声增强
E.胃壁-胃内容物(含气)界面产生很强的反射
A.液体均是无回声的,固体均是有回声的
4 1 2012 12. 超声扫查可用的声像图基本断面是(2.5分) A.纵断面(矢状断面)
B.横断面
C.冠状断面
D.斜断面
E.纵断面、横断面、冠状断面、斜断面
E.纵断面、横断面、冠状断面、斜断面
4 1 2012 13. 肝转移性肿瘤,以"牛眼征"为典型超声表现的多见于()转移而来(2.5分) A.肉瘤
B.乳腺癌
C.胃肠道腺癌
D.肺癌
E.胰腺癌
C.胃肠道腺癌
4 1 2012 14. 一黄疸患者肝内胆管扩张,肝外胆管未显示,胆囊未充盈,其梗阻部位最可能为(2.5分) A.胆总管十二指肠后段
B.肝门部
C.胆总管胰腺段
D.胆总管壶腹段
E.胆总管十二指肠上段
B.肝门部
4 1 2012 15. 常用的诊断超声频率是(2.5分) A.小于1.5MHz
B.在 2MHz-10MHz 之间
C.在 20MHz-40MHz 之间
D.在 40MHz-80MHz 之间
E.大于80MHz
B.在 2MHz-10MHz 之间
4 1 2012 16. 以下营养成分是无机物的是(2.5分) A.蛋白质
B.脂肪
C.糖
D.矿物质
E.维生素
D.矿物质
4 1 2012 17. 下列矿物质尚未发现直接参与人体内酶催化反应的矿物质是(2.5分) A.硼
B.铁
C.铜
D.锌
E.硒
A.硼
4 1 2012 18. 下列选项中关于氨基酸吸收正确的是(2.5分) A.只能以单个氨基酸形式被吸收
B.氨基酸可以直接通过细胞膜
C.部分氨基酸借助钠离子梯度被吸收
D.吸收位置在大肠
E.钠钾泵直接参与氨基酸转运
C.部分氨基酸借助钠离子梯度被吸收
4 1 2012 19. 主要的催化脂肪降解反应的酶来自(2.5分) A.胃
B.唾液腺
C.胰脏
D.肝脏
E.小肠
C.胰脏
4 1 2012 20. 下列关于维生素的说法中正确的视(2.5分) A.维生素A缺乏导致败血症
B.普通人需要经常口服维生素D
C.叶酸缺乏导致巨幼红细胞性贫血
D.维生素B具有抗氧化作用
E.正常人体内生物素主要来自食物摄入
C.叶酸缺乏导致巨幼红细胞性贫血
4 1 2012 21. 诊断消化性溃疡主要依靠下列哪项方法? (2.5分) A.病史分析,如典型的周期性和节律性上腹部疼痛
B.X线钡餐检查
C.内镜检查
D.H.pylori检查
E.诊断性治疗
C.内镜检查
4 1 2012 22. 下列哪项消化性溃疡的并发症一般不发生在十二指肠溃疡患者中? (2.5分) A.出血
B.幽门梗阻
C.穿孔
D.癌变
E.溃疡复发
D.癌变
4 1 2012 23. 以下哪项关于十二指肠球后溃疡的描述是错误的(2.5分) A.夜间疼痛较为明显
B.患者常有背部的放射痛
C.多发生于十二指肠的后壁
D.对药物治疗的反应较差,易出血
E.多发生在十二指肠乳头的近端
C.多发生于十二指肠的后壁
4 1 2012 24. Partial gastrectomy is performed on a 32-year-old Caucasian male because of refractory peptic ulcer disease. The pathologist receives the tissue and notes significant enlargement of the gastric rugal folds on gross examination. Microscopy of the gastric mucosa reveals parietal cell hyperplasia. Which of the following stimuli is the most likely cause of parietal cell proliferation in this patient?(2.5分) A.Acetylcholine
B.Serotonin
C.Somatostatin
D.Gastrin
E.Secretin
F.Transforming growth factor - α
D.Gastrin
4 1 2012 25. 胃溃疡的腹痛规律是(2.5分) A.无明显节律性
B.疼痛-排便-加重
C.进食-疼痛-缓解
D.疼痛-进食-缓解
E.疼痛-排便-缓解
C.进食-疼痛-缓解
4 1 2012 26. 男性,45岁。间断上腹痛3年,加重2月,胃镜检查发现胃角切迹溃疡,幽门螺杆菌阳性。其治疗方案首选: (2.5分) A.H2受体拮抗剂治疗,6周复查胃镜
B.胃粘膜保护剂治疗,6周复查胃镜
C.抗幽门螺杆菌治疗,2周复查胃镜
D.质子泵抑制剂+胃粘膜保护剂,4周复查胃镜
E.抗幽门螺杆菌治疗+质子泵抑制剂治疗,6周复查胃镜
E.抗幽门螺杆菌治疗+质子泵抑制剂治疗,6周复查胃镜
4 1 2012 27. 消化器官不具备下列哪一功能? (2.5分) A.消化
B.内分泌
C.水平衡
D.免疫
E.吸收
C.水平衡
4 1 2012 28. 人体内最大、最复杂的内分泌器官是(2.5分) A.消化道
B.下丘脑
C.腺垂体
D.心脏
E.胰脏
A.消化道
4 1 2012 29. 胰蛋白酶原活化的最主要物质是(2.5分) A.肠激酶
B.胃蛋白酶
C.组胺
D.糜蛋白酶
E.盐酸
A.肠激酶
4 1 2012 30. 头期胃液分泌的调节是(2.5分) A.纯条件反射性的
B.纯神经调节性的
C.纯体液调节性的
D.神经--体液调节性的自主调节
D.神经--体液调节性的自主调节
4 1 2012 31. 所有消化液中最重要的是(2.5分) A.唾液
B.胃液
C.胰液
D.小肠液
E.胆汁
C.胰液
4 1 2012 32. 下列哪种形式的小肠运动使食糜与消化液充分混合,便于进行化学消化? (2.5分) A.紧张性收缩
B.分节运动
C.蠕动
D.蠕动冲
E.容受性舒张
B.分节运动
4 1 2012 33. 下列物质哪种存在于唾液中(2.5分) A.凝乳酶
B.寡糖酶
C.肽酶
D.溶菌酶
E.蛋白酶
D.溶菌酶
4 1 2012 34. 关于胰液中碳酸氢盐的错误叙述是(2.5分) A.由胰腺内小导管细胞分泌
B.是胰液无机物成分中含量最低者
C.中和进入十二脂肠的胃酸
D.提供小肠内多种消化酶的最适pH环境
E.以上均正确
B.是胰液无机物成分中含量最低者
4 1 2012 35. 有关大肠功能的叙述,错误的是(2.5分) A.贮存食物残渣并形成粪便
B.大肠液中的消化酶分解食物残渣
C.大肠液保护润滑粪便
D.肠内细菌可利用简单物质合成维生素B族和维生素K
E.吸收水分
B.大肠液中的消化酶分解食物残渣
4 1 2012 36. 对怀疑有鱼刺卡在食管壁上,合理的检查方法(2.5分) A.胸部平片
B.食管钡餐造影
C.食道CT
D.血管造影
E.以上均正确
C.食道CT
4 1 2012 37. 膈下游离气体的原因是(2.5分) A.人工气腹
B.腹腔术后残留气体
C.产气菌引起的急性腹膜炎
D.胃肠道穿孔
E.以上都是
E.以上都是
4 1 2012 38. 肝脏增强扫描,早出晚归的增强模式对以下哪种病变诊断价值最高(2.5分) A.原发性肝癌
B.肝脏炎性假瘤
C.肝海绵状血管瘤
D.转移性肝癌
E.肝囊肿
C.肝海绵状血管瘤
4 1 2012 39. 某女性患者,56岁,B超发现肝右叶4cm强回声团块,CT平扫显示为边缘锐利的低密度灶,增强扫描呈充填式强化,最有可能的诊断是: (2.5分) A.转移性肝癌
B.原发性肝癌
C.血管瘤
D.肝脓肿
E.肝囊肿
C.血管瘤
4 1 2012 40. 不属于胃癌基本X线表现的是(2.5分) A.半月综合征
B.胃壁僵硬、边缘不齐
C.狭颈征
D.粘膜皱襞破坏中断
E.胃呈皮革状变形
C.狭颈征
4 1 2013 1. 胰腺内分泌的功能是调控:(2.5分) A.血糖稳态
B.电解质稳态
C.氮库稳态
D.脂代谢稳态
E.氧化还原稳态
A.血糖稳态
4 1 2013 2. 胰腺分泌的消化酶多以前体形式存在,其中对其它胰酶的激活具有关键作用的胰酶是:(2.5分) A.肠激酶
B.羧肽酶
C.胰蛋白酶
D.糜蛋白酶
E.弹性蛋白酶
C.胰蛋白酶
4 1 2013 3. 以下属于次级胆汁酸的是(2.5分) A.鹅脱氧胆酸
B.胆酸
C.脱氧胆酸
D.硫磺胆酸
E.甘氨胆酸
C.脱氧胆酸
4 1 2013 4. 胆汁酸合成的不受下列那种因素调节(2.5分) A.胆汁酸
B.不饱和脂肪酸
C.糖皮质激素
D.胆固醇
E.生长激素
B.不饱和脂肪酸
4 1 2013 5. 人体不同组织和体液回声强度是不同的,下列哪一项是错误的(2.5分) A.液体均是无回声的,固体均是有回声的
B.实质性组织如肝脾是中等水平回声
C.脂肪组织是低水平回声
D.纤维化和钙化引起组织回声增强
E.胃壁-胃内容物(含气)界面产生很强的反射
A.液体均是无回声的,固体均是有回声的
4 1 2013 6. 关于小肝癌的描述,下列哪项是错误的(2.5分) A.圆形低回声结节
B.远侧回声可轻度增强
C.可见侧方声影
D.边界清楚
E.周围见较宽的晕环,中心液化呈"牛眼征"
E.周围见较宽的晕环,中心液化呈"牛眼征"
4 1 2013 7. 超声扫查可用的声像图基本断面是(2.5分) A.纵断面(矢状断面)
B.横断面
C.冠状断面
D.斜断面
E.纵断面、横断面、冠状断面、斜断面
E.纵断面、横断面、冠状断面、斜断面
4 1 2013 8. 下列哪条血管是寻找胰腺体尾部的标志(2.5分) A.门静脉
B.胃左动脉
C.脾静脉
D.左肾动脉
E.腹主动脉
C.脾静脉
4 1 2013 9. 当超声波经过声阻抗相差较大的介质形成界面时(2.5分) A.穿透力增强
B.分辨率增强
C.被反射的声能增多
D.被吸收的声能增多
E.混响增强
C.被反射的声能增多
4 1 2013 10. 下列关于囊性肿物的描述,错误的是(2.5分) A.无回声性肿物
B.壁薄而光滑
C.后壁回声清晰而锐利
D.具有后方声影
E.内部无多普勒血流信号
D.具有后方声影
4 1 2013 11. 甲状腺功能亢进时典型的摄131I率表现为(2.5分) A.摄131I率增高,高峰>24小时
B.摄131I率增高,高峰4小时出现
C.摄131I率不增高,高峰>24小时
D.摄131I率不增高,高峰4小时出现
E.摄131I率减低。
B.摄131I率增高,高峰4小时出现
4 1 2013 12. 舌骨后肿块,为排除异位甲状腺,应首选下列哪项检查(2.5分) A.MRI
B.CT
C.核素显像
D.B超
E.X线平片
C.核素显像
4 1 2013 13. 药物的生物转化和排泄速度决定其(2.5分) A.副作用的多少
B.最大效应的高低
C.作用持续时间的长短
D.起效的快慢
E.后遗效应的大小
C.作用持续时间的长短
4 1 2013 14. 肝损显著影响异丙酚药效的原因是 (2.5分) A.异丙酚脂溶性高
B.异丙酚在体内迅速分布
C.异丙酚药效维持时间短
D.异丙酚起效迅速
E.异丙酚与血浆蛋白结合率高
A.异丙酚脂溶性高
4 1 2013 15. 对怀疑有鱼刺卡在食管壁上,合理的检查方法(2.5分) A.胸部平片
B.食管钡餐造影
C.食道CT
D.血管造影
E.以上均正确
C.食道CT
4 1 2013 16. 医用硫酸钡作为对比剂的造影部位是(2.5分) A.胃肠道造影
B.肠系膜上动脉血管造影
C.胆道造影
D.椎管造影
E.泌尿系造影
A.胃肠道造影
4 1 2013 17. 膈下游离气体的原因是(2.5分) A.人工气腹
B.腹腔术后残留气体
C.产气菌引起的急性腹膜炎
D.胃肠道穿孔
E.以上都是
E.以上都是
4 1 2013 18. 关于颏舌肌,下列描述何者正确(2.5分) A.属于舌内肌
B.单侧收缩使舌尖伸向前下
C.单侧收缩使舌尖伸向同侧
D.双侧收缩可拉舌向前下
E.双侧收缩可使舌尖伸向前上
D.双侧收缩可拉舌向前下
4 1 2013 19. 肝外胆道不包括(2.5分) A.肝左管
B.肝右管
C.胆囊管
D.胰管
E.肝总管
D.胰管
4 1 2013 20. 关于胰,下列叙述哪项正确(2.5分) A.位于右季肋区
B.为腹膜间位器官
C.有韧带连于胃底
D.仅有内分泌功能
E.分头、颈、体、尾4部分
E.分头、颈、体、尾4部分
4 1 2013 21. 不与肝右叶毗邻的是(2.5分) A.结肠右曲
B.右肾
C.胃小弯
D.十二指肠上曲
E.胆囊
C.胃小弯
4 1 2013 22. 鼻咽癌好发生于(2.5分) A.梨状隐窝
B.咽鼓管圆枕
C.咽隐窝
D.口咽部
E.喉咽部
C.咽隐窝
4 1 2013 23. 下列哪种氨基酸不是营养必需氨基酸(2.5分) A.Val
B.Ser
C.Trp
D.Lys
E.Ile
B.Ser
4 1 2013 24. 胃蛋白酶属于酸性蛋白酶,其最适pH为:(2.5分) A.pH 5~6
B.pH 7.4
C.pH 7~8
D.pH 2~3
E.pH 9~10
D.pH 2~3
4 1 2013 25. 胰蛋白酶对食物蛋白质肽键的水解作用具有选择性, 可以水解肽链的左侧氨基酸R1为: (2.5分) A.碱性氨基酸 Lys和Arg
B.芳香族氨基酸
C.侧链较小的氨基酸
D.疏水氨基酸
E.酸性氨基酸
A.碱性氨基酸 Lys和Arg
4 1 2013 26. 下列哪一种脂肪酸是营养必需脂肪酸(2.5分) A.亚油酸
B.棕榈酸
C.硬脂酸
D.油酸
E.花生酸
A.亚油酸
4 1 2013 27. 营养学上价值最高的脂肪酸为(2.5分) A.长链脂肪酸
B.短链脂肪酸
C.顺式脂肪酸
D.反式脂肪酸
E.ω-3和ω-6系列不饱和脂肪酸
E.ω-3和ω-6系列不饱和脂肪酸
4 1 2013 28. 乳糖不耐受症是由于下列哪种酶缺乏所引起(2.5分) A.半乳糖酶
B.蔗糖酶
C.麦芽糖酶
D.a-淀粉酶
E.乳糖酶
E.乳糖酶
4 1 2013 29. 消化道平滑肌细胞动作电位的主要离子基础是(2.5分) A.Na+大量内流
B.K+大量内流
C.Ca2+大量内流
D.Cl-大量外流
E.Ca2+ 大量外流
C.Ca2+大量内流
4 1 2013 30. 支配胃肠道的交感节后纤维释放的递质是(2.5分) A.乙酰胆碱
B.多巴胺
C.肾上腺素
D.去甲肾上腺素
E.织胺
D.去甲肾上腺素
4 1 2013 31. 诊断消化性溃疡主要依靠下列哪项方法? (2.5分) A.病史分析,如典型的周期性和节律性上腹部疼痛
B.X线钡餐检查
C.内镜检查
D.H.pylori检查
E.诊断性治疗
C.内镜检查
4 1 2013 32. 下列哪项消化性溃疡的并发症一般不发生在十二指肠溃疡患者中? (2.5分) A.出血
B.幽门梗阻
C.穿孔
D.癌变
E.溃疡复发
D.癌变
4 1 2013 33. 下面有关消化性溃疡的流行病学情况的描述哪项是错误的? (2.5分) A.近年来消化性溃疡的发病率有所下降
B.男性抽烟较多,所以GU好发于女性而DU好发于男性
C.DU多见于青壮年而GU多见于老年
D.总的发病率来看DU比GU多见
E.消化性溃疡与H. pylori密切相关
B.男性抽烟较多,所以GU好发于女性而DU好发于男性
4 1 2013 34. A 38-year-old woman with rheumatoid arthritis presents to her physician because of increased joint pain. On physical examination, both passive and active range of motion of the hips are decreased. Her physician increases her dose of nonsteroidal son inflammatory drugs (NSAIDs). Which of the following should be prescribed as well to prevent peptic ulcer disease? (2.5分) A.Cimetidine
B.Clarithromydn
C.Misoprostol
D.Omeprazole
E.Sucralfate
C.Misoprostol
4 1 2013 35. Partial gastrectomy is performed on a 32-year-old Caucasian male because of refractory peptic ulcer disease. The pathologist receives the tissue and notes significant enlargement of the gastric rugal folds on gross examination. Microscopy of the gastric mucosa reveals parietal cell hyperplasia. Which of the following stimuli is the most likely cause of parietal cell proliferation in this patient? (2.5分) A.Acetylcholine
B.Serotonin
C.Somatostatin
D.Gastrin
E.Secretin
D.Gastrin
4 1 2013 36. 男,53岁。上腹胀痛10余年,多于饭后30分钟加重。半年来上腹痛加重,伴反酸,间断呕吐胃内容物。吸烟15年,饮白酒10年,每日约半斤。患者的病变最可能位于:(2.5分) A.十二指肠球部
B.胃窦
C.胃体
D.贲门
E.胃底
B.胃窦
4 1 2013 37. 关于幽门管溃疡的描述,正确的是(2.5分) A.消化性溃疡的特殊类型
B.有典型的节律性症状
C.不少为穿透性
D.甲腈米胍治疗效果良好
E.病变发展较慢
A.消化性溃疡的特殊类型
4 1 2013 38. 消化性溃疡的最常见并发症是(2.5分) A.穿孔
B.出血
C.幽门梗阻
D.癌变
E.瘘管形成
B.出血
4 1 2013 39. 为判断幽门螺杆菌是否被根除,正确的检查时间应在治疗结束后至少(2.5分) A.2周
B.1周
C.4周
D.3周
E.3天
C.4周
4 1 2013 40. 降低胃酸最有效的药物是(2.5分) A.H2受体拮抗剂
B.含铝抗酸剂
C.抗胆碱能药物
D.质子泵抑制剂
E.促胃液素受体拮抗剂
D.质子泵抑制剂
4 1 2014 1. 胰腺内分泌的功能是调控(2.5分) A.血糖稳态
B.电解质稳态
C.氮库稳态
D.脂代谢稳态
E.氧化还原稳态
A.血糖稳态
4 1 2014 2. 胆汁酸合成的起始原料是(2.5分) A.胆固醇
B.脂肪酸
C.乙酰辅酶A
D.UDPG
E.胆红素
A.胆固醇
4 1 2014 3. 胰液的主要成份不包括)(2.5分) A.水
B.电解质
C.蛋白酶
D.多糖
E.脂酶
D.多糖
4 1 2014 4. 舌骨后肿块,为排除异位甲状腺,应首选下列哪项检查:(2.5分) A.MRI
B.CT
C.核素显像
D.B超
E.X线平片
C.核素显像
4 1 2014 5. 正常成人甲状腺的重量为(2.5分) A.5-10g
B.10-20g
C.20-30g
D.30-40g
E.40-50g
C.20-30g
4 1 2014 6. 胃肠出血显像时,常用的显像剂为(2.5分) A.99mTcO4-
B.99mTc-MIBI
C.131I
D.99mTc-RBC
E.99mTc-MDP
D.99mTc-RBC
4 1 2014 7. 药物的生物转化和排泄速度决定其(2.5分) A.副作用的多少
B.最大效应的高低
C.作用持续时间的长短
D.起效的快慢
E.后遗效应的大小
C.作用持续时间的长短
4 1 2014 8. 关于肝药酶的描述,错误的是(2.5分) A.属P450酶系统
B.其活性有限
C.易发生竞争性抑制
D.个体差异大
E.只代谢20余种药物
E.只代谢20余种药物
4 1 2014 9. 关于消化管平滑肌基本电节律的正确叙述是(2.5分) A.是一种超极化波
B.其后一定伴随动作电位
C.是平滑肌收缩节律的控制波
D.在切断支配胃肠的神经后消失
E.以上说法均不对
C.是平滑肌收缩节律的控制波
4 1 2014 10. 支配胃肠道的交感节后纤维释放的递质(2.5分) A.乙酰胆碱
B.多巴胺
C.肾上腺素
D.去甲肾上腺素
E.织胺
D.去甲肾上腺素
4 1 2014 11. 关于颏舌肌,下列描述何者正确(2.5分) A.属于舌内肌
B.单侧收缩使舌尖伸向前下
C.单侧收缩使舌尖伸向同侧
D.双侧收缩可拉舌向前下
E.双侧收缩可使舌尖伸向前上
D.双侧收缩可拉舌向前下
4 1 2014 12. 下列大肠中,最短的是(2.5分) A.直肠
B.肛管
C.升结肠
D.横结肠
E.降结肠
B.肛管
4 1 2014 13. 关于胰,下列叙述哪项正确(2.5分) A.位于右季肋区
B.为腹膜间位器官
C.有韧带连于胃底
D.仅有内分泌功能
E.分头、颈、体、尾4部分
E.分头、颈、体、尾4部分
4 1 2014 14. 不与肝右叶毗邻的是(2.5分) A.结肠右曲
B.右肾
C.胃小弯
D.十二指肠上曲
E.胆囊
C.胃小弯
4 1 2014 15. 鼻咽癌好发生于(2.5分) A.梨状隐窝
B.咽鼓管圆枕
C.咽隐窝
D.口咽部
E.喉咽部
C.咽隐窝
4 1 2014 16. 胃蛋白酶属于酸性蛋白酶,其最适pH为: (2.5分) A.pH 5~6
B.pH 7.4
C.pH 7~8
D.pH 2~3
E.pH 9~10
D.pH 2~3
4 1 2014 17. 下列哪种酶活性中心具有Zn2+,属于金属蛋白酶家族(2.5分) A.胃蛋白酶
B.羧肽酶A
C.胰蛋白酶
D.弹性蛋白酶
E.胰糜蛋白酶
B.羧肽酶A
4 1 2014 18. 下列哪一种脂肪酸是营养必需脂肪酸(2.5分) A.亚油酸
B.棕榈酸
C.硬脂酸
D.油酸
E.花生酸
A.亚油酸
4 1 2014 19. 下列哪一组碳水化合物属于双糖(2.5分) A.葡萄糖和果糖
B.蔗糖和乳糖
C.水苏糖和棉子糖
D.半乳糖和阿拉伯糖
E.以上都不是
B.蔗糖和乳糖
4 1 2014 20. 乳糖不耐受症是由于下列哪种酶缺乏所引起(2.5分) A.半乳糖酶
B.蔗糖酶
C.麦芽糖酶
D.a-淀粉酶
E.乳糖酶
E.乳糖酶
4 1 2014 21. 胃肠道穿孔的最典型X线征象(2.5分) A.咖啡豆征
B.阶梯状气液平面
C.膈下游离气体
D.结肠积气
E.肠管扩张
C.膈下游离气体
4 1 2014 22. 不属于直接引入法造影的是? (2.5分) A.胃肠钡餐造影
B.钡灌肠造影
C.肝脏增强扫描
D.子宫输卵管造影
E.逆行肾盂造影
C.肝脏增强扫描
4 1 2014 23. 关于胃恶性溃疡,下列说法不正确的是(2.5分) A.半月综合征
B.粘膜破坏
C.胃肠道钡餐检查可见充盈缺损
D.胃蠕动消失
E.窄颈征
E.窄颈征
4 1 2014 24. 常用医用硫酸钡作为对比剂的造影部位是(2.5分) A.胃肠道造影
B.脑血管造影
C.胆囊造影
D.椎管造影
E.泌尿系造影
A.胃肠道造影
4 1 2014 25. 胃肠道钡餐造影检查,以下描述哪项错误(2.5分) A.内腔的改变
B.胃形态改变、
C.胃蠕动功能改变
D.粘膜与粘膜皱襞的改变
E.胃腔周围结构改变
E.胃腔周围结构改变
4 1 2014 26. 诊断消化性溃疡主要依靠下列哪项方法? (2.5分) A.病史分析,如典型的周期性和节律性上腹部疼痛
B.X线钡餐检查
C.内镜检查
D.H.pylori检查
E.诊断性治疗
C.内镜检查
4 1 2014 27. 导致消化性溃疡最主要的侵袭因子为(2.5分) A.胃酸
B.胆盐
C.胰酶
D.NSAID
E.胃蛋白酶
A.胃酸
4 1 2014 28. 以下哪项关于十二指肠球后溃疡的描述是错误的(2.5分) A.夜间疼痛较为明显
B.患者常有背部的放射痛
C.多发生于十二指肠的后壁
D.对药物治疗的反应较差,易出血
E.多发生在十二指肠乳头的近端
C.多发生于十二指肠的后壁
4 1 2014 29. 胃溃疡最常见的位置是(2.5分) A.胃前壁
B.胃后壁
C.胃大弯及胃底
D.胃小弯及贲门处
E.胃窦小弯侧
A.胃前壁
4 1 2014 30. 胃溃疡的腹痛规律是(2.5分) A.无明显节律性
B.疼痛-排便-加重
C.进食-疼痛-缓解
D.疼痛-进食-缓解
E.疼痛-排便-缓解
C.进食-疼痛-缓解
4 1 2014 31. 对球后溃疡的正确描述是(2.5分) A.多发生在距离幽门2-3cm以内
B.指十二指肠球部后壁溃疡
C.易并发出血
D.临床症状多不明显
E.内科治疗效果好
C.易并发出血
4 1 2014 32. 确诊胃溃疡首选的检查方法是(2.5分) A.胃镜及胃粘膜组织检查
B.24小时食管pH监测
C.X线钡餐检查
D.幽门螺杆菌监测
E.胃液分析
A.胃镜及胃粘膜组织检查
4 1 2014 33. 了解胃溃疡患者胃酸分泌情况依据(2.5分) A.胃组织学检查
B.快速尿素酶试验
C.幽门螺杆菌培养
D.14C尿素呼气试验
E.血清学检查
B.快速尿素酶试验
4 1 2014 34. 消化性溃疡的最常见并发症是(2.5分) A.穿孔
B.出血
C.幽门梗阻
D.癌变
E.瘘管形成
B.出血
4 1 2014 35. 不属于十二指肠球部溃疡的并发症的是(2.5分) A.急性穿孔
B.幽门梗阻
C.癌变
D.出血
E.慢性穿孔
C.癌变
4 1 2014 36. 对诊断消化性溃疡穿孔最有价值的临床表现是(2.5分) A.突发上腹部剧痛
B.腹式呼吸消失
C.上腹压痛明显
D.上腹部有反跳痛
E.肝浊音界消失
E.肝浊音界消失
4 1 2014 37. 为判断幽门螺杆菌是否被根除,正确的检查时间应在治疗结束后至少(2.5分) A.2周
B.1周
C.4周
D.3周
E.3天
C.4周
4 1 2014 38. 初次诊断活动期十二指肠溃疡,下列治疗中最合适的是(2.5分) A.联合应用两种粘膜保护剂
B.促动力剂+H2受体拮抗剂
C.质子泵抑制剂+粘膜保护剂
D.质子泵抑制剂+两种抗生素
E.抑酸剂
D.质子泵抑制剂+两种抗生素
4 1 2014 39. 降低胃酸最有效的药物是(2.5分) A.H2受体拮抗剂
B.含铝抗酸剂
C.抗胆碱能药物
D.质子泵抑制剂
E.促胃液素受体拮抗剂
D.质子泵抑制剂
4 1 2014 40. 上消化道出血可表现为呕血或黑便,最重要取决于(2.5分) A.出血的部位
B.出血的量和速度
C.病变的性质
D.凝血机制
E.胃肠蠕动情况
B.出血的量和速度
4 1 2015 1. 胰腺内分泌的功能是调控(2.5分) A.血糖稳态
B.电解质稳态
C.氮库稳态
D.脂代谢稳态
E.氧化还原稳态
A.血糖稳态
4 1 2015 2. 胆汁酸合成的起始原料是(2.5分) A.胆固醇
B.脂肪酸
C.乙酰辅酶A
D.UDPG
E.胆红素
A.胆固醇
4 1 2015 3. 对怀疑有鱼刺卡在食管壁上,合理的检查方法(2.5分) A.胸部平片
B.食管钡餐造影
C.食道CT
D.血管造影
E.以上均正确
C.食道CT
4 1 2015 4. 医用硫酸钡作为对比剂的造影部位是(2.5分) A.胃肠道造影
B.肠系膜上动脉血管造影
C.胆道造影
D.椎管造影
E.泌尿系造影
A.胃肠道造影
4 1 2015 5. 膈下游离气体的原因是(2.5分) A.人工气腹
B.腹腔术后残留气体
C.产气菌引起的急性腹膜炎
D.胃肠道穿孔
E.以上都是
E.以上都是
4 1 2015 6. 肝细胞肝癌,CT动态增强的典型表现:(2.5分) A.慢进快出
B.快进慢出
C.快进快出
D.慢进慢出
E.强化不明显
C.快进快出
4 1 2015 7. 不属于胃癌基本X线表现的是(2.5分) A.半月综合征
B.胃壁僵硬、边缘不齐
C.狭颈征
D.粘膜皱襞破坏中断
E.胃呈皮革状变形
C.狭颈征
4 1 2015 8. 关于胰腺癌的CT表现,不正确的是(2.5分) A.多合并出血和脓肿
B.胰腺肿块
C.肿瘤侵及血管使其变形狭窄
D.胰腺周围脂肪消失
E.胰管和胆总管扩张
A.多合并出血和脓肿
4 1 2015 9. 舌骨后肿块,为排除异位甲状腺,应首选下列哪项检查(2.5分) A.MRI
B.CT
C.核素显像
D.B超
E.X线平片
C.核素显像
4 1 2015 10. 核医学胃肠道出血显像的优点不包括(2.5分) A.可用于急性出血定位诊断
B.能作出病因诊断
C.灵敏度高
D.无创
E.简单方便
B.能作出病因诊断
4 1 2015 11. 诊断消化性溃疡主要依靠下列哪项方法?(2.5分) A.病史分析,如典型的周期性和节律性上腹部疼痛
B.X线钡餐检查
C.内镜检查
D.H.pylori检查
E.诊断性治疗
C.内镜检查
4 1 2015 12. 导致消化性溃疡最主要的侵袭因子为(2.5分) A.胃酸
B.胆盐
C.胰酶
D.NSAID
E.胃蛋白酶
A.胃酸
4 1 2015 13. 哪些检查是H. pylori根除治疗后首选的检查(2.5分) A.血清H. pylori IgG
B.C-14或C-13呼气试验
C.血清H. pylori IgM
D.粪便H. pylori 抗原培养
E.H. pylori培养
B.C-14或C-13呼气试验
4 1 2015 14. 除下列哪项以外都是胃粘膜的防御修复因素? (2.5分) A.生长抑素
B.前列腺素
C.表皮生长因子
D.胃粘液屏障
E.丰富的粘膜血流
A.生长抑素
4 1 2015 15. 对球后溃疡的正确描述是(2.5分) A.多发生在距离幽门2-3cm以内
B.指十二指肠球部后壁溃疡
C.易并发出血
D.临床症状多不明显
E.内科治疗效果好
C.易并发出血
4 1 2015 16. 女性,32岁。阵发性上腹痛2年,夜间加重,疼痛有季节性,冬季明显,有反酸。为进一步确诊,首选的检查方法是(2.5分) A.胃镜检查
B.CT检查
C.X线钡餐检查
D.胃液分析
E.B超
A.胃镜检查
4 1 2015 17. 降低胃酸最有效的药物是(2.5分) A.H2受体拮抗剂
B.含铝抗酸剂
C.抗胆碱能药物
D.质子泵抑制剂
E.促胃液素受体拮抗剂
D.质子泵抑制剂
4 1 2015 18. 肝损显著影响异丙酚药效的原因是(2.5分) A.异丙酚脂溶性高
B.异丙酚在体内迅速分布
C.异丙酚药效维持时间短
D.异丙酚起效迅速
E.异丙酚与血浆蛋白结合率高
A.异丙酚脂溶性高
4 1 2015 19. 关于颏舌肌,下列描述何者正确(2.5分) A.属于舌内肌
B.单侧收缩使舌尖伸向前下
C.单侧收缩使舌尖伸向同侧
D.双侧收缩可拉舌向前下
E.双侧收缩可使舌尖伸向前上
D.双侧收缩可拉舌向前下
4 1 2015 20. 关于盲肠,下列叙述哪项正确(2.5分) A.长约10cm
B.为腹膜外位器官
C.3条结肠带在阑尾根部汇聚
D.小儿盲肠较低,常达盆腔
E.内侧为右结肠旁沟
C.3条结肠带在阑尾根部汇聚
4 1 2015 21. 关于胆总管,下列叙述哪项正确(2.5分) A.位于小网膜左侧的游离缘内
B.由肝左、右管汇合而成
C.居肝门静脉的左前方
D.管径通常在1cm以上
E.由肝总管和胆囊管汇合而成
E.由肝总管和胆囊管汇合而成
4 1 2015 22. 关于腮腺管,下述哪项正确(2.5分) A.由腺体深部的前缘发出
B.开口于平对上颌第2前磨牙所对的颊黏膜上
C.开口于平对上颌第2磨牙所对的颊黏膜上
D.开口于平对下颌第2前磨牙所对的颊黏膜上
E.开口于平对下颌第2磨牙所对的颊黏膜上
B.开口于平对上颌第2前磨牙所对的颊黏膜上
4 1 2015 23. 在小肠蛋白质的消化过程中, 首先被激活的蛋白酶原是(2.5分) A.弹性蛋白酶原
B.胰脂肪酶原
C.羧肽酶原
D.胰糜蛋白酶原
E.胰蛋白酶原
E.胰蛋白酶原
4 1 2015 24. 下列哪一组碳水化合物属于双糖(2.5分) A.葡萄糖和果糖
B.蔗糖和乳糖
C.水苏糖和棉子糖
D.半乳糖和阿拉伯糖
B.蔗糖和乳糖
4 1 2015 25. 有关胰a-淀粉酶(a-amylase )的描述哪一项是错误的(2.5分) A.含1个Cl- 和 1个Na+
B.是唾液淀粉酶的同工酶
C.不能水解a-1,6 糖苷键
D.可以水解直链淀粉和支链淀粉
E.水解a-1,4 糖苷键
A.含1个Cl- 和 1个Na+
4 1 2015 26. 胃黏膜细胞分泌的一种内因子(IF),与下列哪种维生素吸收有关(2.5分) A.维生素B6
B.生物素
C.叶酸
D.维生素B12
E.维生素K
D.维生素B12
4 1 2015 27. 下列哪种矿物质的缺少可以引起心肌病(克山病)和大关节病(2.5分) A.硒
B.锌
C.碘
D.钙
E.铁
A.硒
4 1 2015 28. 鉴别囊肿与实性肿瘤最可靠的依据是(2.5分) A.边界光滑,整齐与否
B.外形是否圆形或椭圆形
C.内部无回声或有回声
D.后方回声增强与否
E.边界回声、外形、内部回声、后方回声综合分析
E.边界回声、外形、内部回声、后方回声综合分析
4 1 2015 29. 下列哪个器官不位于腹膜后间隙内(2.5分) A.胰腺
B.肾上腺
C.肾
D.输尿管
E.脾脏
E.脾脏
4 1 2015 30. 肝转移性肿瘤,以"牛眼征"为典型超声表现的多见于()转移而来(2.5分) A.肉瘤
B.乳腺癌
C.胃肠道腺癌
D.肺癌
E.胰腺癌
C.胃肠道腺癌
4 1 2015 31. 一黄疸患者肝内胆管扩张,肝外胆管未显示,胆囊未充盈,其梗阻部位最可能为(2.5分) A.胆总管十二指肠后段
B.肝门部
C.胆总管胰腺段
D.胆总管壶腹段
E.胆总管十二指肠上段
B.肝门部
4 1 2015 32. 常用的诊断超声频率是(2.5分) A.小于1.5MHz
B.在 2MHz-10MHz 之间
C.在 20MHz-40MHz 之间
D.在 40MHz-80MHz 之间
E.大于80MHz
B.在 2MHz-10MHz 之间
4 1 2015 33. 关于消化管平滑肌基本电节律的正确叙述是(2.5分) A.是一种超极化波
B.其后一定伴随动作电位
C.是平滑肌收缩节律的控制波
D.在切断支配胃肠的神经后消失
E.以上说法均不对
C.是平滑肌收缩节律的控制波
4 1 2015 34. 支配胃肠道的交感节后纤维释放的递质是(2.5分) A.乙酰胆碱
B.多巴胺
C.肾上腺素
D.去甲肾上腺素
E.织胺
D.去甲肾上腺素
4 1 2015 35. 下列哪种化学物质对胃液分泌有抑制作用?(2.5分) A.阿托品
B.乙酰胆碱
C.胃泌素
D.组织胺
E.胆囊收缩素
A.阿托品
4 1 2015 36. 胆汁中与消化有关的成分是(2.5分) A.胆盐
B.胆固醇
C.胆色素
D.水和无机盐
E.各种消化酶
A.胆盐
4 1 2015 37. 胃液中胃蛋白酶含量最高的时期是(2.5分) A.非消化期
B.消化期的头期
C.消化期的胃期
D.消化期的肠期
B.消化期的头期
4 1 2015 38. 胃肠道内下列哪种物质对胃液分泌有促进作用?(2.5分) A.盐酸
B.脂肪及消化产物
C.蛋白质消化产物
D.糖及消化产物
E.高张溶液
C.蛋白质消化产物
4 1 2015 39. 胰蛋白酶原的激活物是(2.5分) A.糜蛋白酶
B.胰淀粉酶
C.肠激酶
D.胰脂肪酶
E.胃蛋白酶
C.肠激酶
4 1 2015 40. 胆汁对脂肪的消化和吸收有促进作用,主要是由于它含有(2.5分) A.脂肪酶
B.胆红素
C.胆盐
D.胆绿质
E.胆固醇
C.胆盐
4 2 2006 1. 碳水化合物的消化开始于(2.0分) A.大肠
B.胃
C.小肠
D.口腔
E.食管
D.口腔
4 2 2006 2. 非糜烂性反流病(NERD)诊断的金标准是:(2.0分) A.胸片
B.胃镜
C.PPI试验
D.心电图
E.24小时PH 监测
E.24小时PH 监测
4 2 2006 3. Pirenzepine是一种(2.0分) A.M1受体阻断药
B.H2受体阻断药
C.H1受体阻断药
D.D2受体阻断药
E.胃壁细胞H+泵抑制药
A.M1受体阻断药
4 2 2006 4. Domperidone(多潘立酮)的止吐作用是通过阻断(2.0分) A.5-HT3受体
B.M1受体
C.多巴胺受体
D.α1受体
E.H2受体
C.多巴胺受体
4 2 2006 5. 不影响胃蛋白酶分泌的药物是(2.0分) A.Pirenzepine
B.Omeprazole
C.Cimetidine
D.Proglumide(丙谷胺)
E.Misoprostol
B.Omeprazole
4 2 2006 6. A 37-year-old woman presents to the emergency department with emesis. She states that she has fibromyalgia syndrome and uses a number of "pain-killers" to control her pain. On waking this morning, she promptly vomited "coffee grounds." On examination, she(2.0分) A.Gastric ulcers
B.Esophageal varices
C.Gastric neoplasm
D.Esophagitis
E.Mallory-Weiss tears
A.Gastric ulcers
4 2 2006 7. 制备小鼠肝损伤模型,carbon tetrachloride采用的给药途径是(2.0分) A.肝脏注射
B.腹腔注射
C.静脉注射
D.口服
E.皮下注射
E.皮下注射
4 2 2006 8. 胃蛋白酶属于酸性蛋白酶,其最适pH为: (2.0分) A.pH 2~3
B.pH 7.4
C.pH 7~8
D.pH 5~6
E.pH 9~10
A.pH 2~3
4 2 2006 9. 胃穿孔具有特征性的X线影像是哪项: (2.0分) A.subdiaphragmatic free air
B.air fluid level
C.distention of colon
D.enlargement of gastric vacuole
E.distention of intestine
A.subdiaphragmatic free air
4 2 2006 10. 以下哪项关于十二指肠球后溃疡的描述是错误的(2.0分) A.患者常有背部的放射痛
B.夜间疼痛较为明显
C.多发生于十二指肠的后壁
D.对药物治疗的反应较差,易出血
C.多发生于十二指肠的后壁
4 2 2006 11. 胰蛋白酶对食物蛋白质肽键的水解作用具有选择性, 可以水解肽链的左侧氨基酸R1为(2.0分) A.芳香族氨基酸
B.侧链较小的氨基酸
C.疏水氨基酸
D.碱性氨基酸 Lys和Arg
E.酸性氨基酸
D.碱性氨基酸 Lys和Arg
4 2 2006 12. 胃壁细胞H+泵抑制药有(1.0分) A.Cimetidine
B.Pirenzepine
C.Famotidine
D.Omeprazole
E.Proglumide(丙谷胺)
D.Omeprazole
4 2 2006 13. 下食管括约肌(LES)主要作用是(2.0分) A.帮助食管蠕动,使其顺利进入胃内
B.防止十二指肠内容物反流入胃
C.防止咽部食物呛入气管
D.防止胃内容物反流入食管
E.使食物由胃体进入胃窦
D.防止胃内容物反流入食管
4 2 2006 14. 十二指肠溃疡宜选择下列哪项手术方法:(2.0分) A.胃肠吻合术
B.局部切除术
C.胃次全切除术加/或不加迷走神经切断术
D.全胃切除术
E.迷走神经干切断术
C.胃次全切除术加/或不加迷走神经切断术
4 2 2006 15. 关于十二指肠溃疡的手术适应证,下列哪项错误:(2.0分) A.急性穿孔
B.一个疗程内科治疗无效
C.大出血
D.瘢痕性幽门梗阻
E.溃疡较大、病史较长和症状严重者,手术适应证可放宽
B.一个疗程内科治疗无效
4 2 2006 16. 起效快、作用强、作用短暂的抗酸药是(2.0分) A.Aluminum hydroxide(氢氧化铝)
B.Magnesium hydroxide(氢氧化镁)
C.Calcium sulfate(硫酸钙)
D.Magnesium trisilicate(三硅酸镁)
E.Sodium bicarbonate(碳酸氢钠)
E.Sodium bicarbonate(碳酸氢钠)
4 2 2006 17. 有关根除H. pylori的治疗哪项是错误的? (2.0分) A.目前常用的新三联治疗中不包括铋剂
B.复查H. pylori是否被根除的试验前1周应停制酸药1周
C.确定H. pylori是否被根除的试验应在治疗后2周内进行
D.H. pylori根除治疗的疗程一般为7天
C.确定H. pylori是否被根除的试验应在治疗后2周内进行
4 2 2006 18. 除了以下哪项以外,都是HP感染引起慢性胃炎的依据是:(1.0分) A.HP与胃炎的活动密切相关
B.慢性胃炎患者HP检出率高
C.慢性胃炎病变的分布与HP存在一致
D.HP阳性者组织学证实胃炎与血清中HP抗体相关
E.无HP感染者不会发生慢性胃炎
E.无HP感染者不会发生慢性胃炎
4 2 2006 19. 诊断反流性食管炎最准确的方法是(2.0分) A.胃镜检查
B.食管测压
C.24小时食管PH监测
D.食管吞钡X线
E.食管滴酸试验
A.胃镜检查
4 2 2006 20. 瘢痕性幽门梗阻的老年患者,应选用:(2.0分) A.胃空肠吻合术
B.高位选择性胃迷走神经切断术
C.迷走神经干切断术+胃窦部切除术
D.毕Ⅰ式胃大部切除术
E.毕Ⅱ式胃大部切除术
A.胃空肠吻合术
4 2 2006 21. Ondansrtron主要用于治疗 (2.0分) A.化疗、放疗引起的呕吐
B.去水吗啡引起的呕吐
C.晕动病引起的呕吐
D.十二指肠溃疡
E.胃溃疡
A.化疗、放疗引起的呕吐
4 2 2006 22. 下列哪种情况不适宜用超声影像诊断 (2.0分) A.胆囊炎的明确诊断
B.肝脏肿瘤良恶性判断
C.直肠肿瘤分期
D.急性阑尾炎明确诊断
D.急性阑尾炎明确诊断
4 2 2006 23. 慢性浅表性胃炎胃镜检查可见:(2.0分) A.粘膜覆盖较多灰白色苔
B.粘膜高低不平
C.粘膜充血、水肿、糜烂和出血
D.透见粘膜下血管
E.粘膜中断现象
C.粘膜充血、水肿、糜烂和出血
4 2 2006 24. 下列有关氨基酸吸收和转运的描述哪一项是错误的(2.0分) A.氨基酸的吸收过程通常需要消耗ATP
B.氨基酸可以通过被动扩散过程而吸收
C.氨基酸的转运体包括需Na+ 和 不需Na+ 两种
D.小肠上皮细胞膜上有氨基酸的转运体(transporter),可以将氨基酸吸收并转运进入血液
E.某些氨基酸既可以通过需Na+ 转运体也可以通过不需Na+ 转运体吸收
B.氨基酸可以通过被动扩散过程而吸收
4 2 2006 25. 在小肠蛋白质的消化过程中, 首先被激活的蛋白酶原是(2.0分) A.胰脂肪酶原
B.弹性蛋白酶原
C.羧肽酶原
D.胰糜蛋白酶原
E.胰蛋白酶原
E.胰蛋白酶原
4 2 2006 26. 下列哪种酶活性中心具有Zn2+,属于金属蛋白酶家族(2.0分) A.胃蛋白酶
B.弹性蛋白酶
C.胰蛋白酶
D.羧肽酶A
E.胰糜蛋白酶
D.羧肽酶A
4 2 2006 27. 诊断消化性溃疡主要依靠下列哪项方法? (2.0分) A.X线钡餐检查
B.病史分析,如典型的周期性和节律性上腹部疼痛
C.内镜检查
D.H.pylori检查
E.诊断性治疗
C.内镜检查
4 2 2006 28. 关于十二指肠溃疡伴幽门梗阻时,下列哪项正确:(2.0分) A.在胃切除时,关闭十二指肠残端要特别小心
B.术前胃管吸引约48小时,以减少组织水肿
C.手术前要特别注意纠正电解质平衡失调
D.由于胃较大,需切除较多的胃
E.上述都是
E.上述都是
4 2 2006 29. 清除食管内容物的主要动力是(2.0分) A.吞咽动作
B.对食管内物体推进性蠕动
C.食物的重力
D.LES(下食管括约肌)舒张
E.食管张力
B.对食管内物体推进性蠕动
4 2 2006 30. 下列哪一组碳水化合物属于双糖(2.0分) A.葡萄糖和果糖
B.水苏糖和棉子糖
C.蔗糖和乳糖
D.半乳糖和阿拉伯糖
C.蔗糖和乳糖
4 2 2006 31. 治疗重症胃食管反流病的首选药物是:(2.0分) A.西沙必利
B.氢氧化铝
C.雷尼替丁
D.奥美拉唑
E.丙谷胺
D.奥美拉唑
4 2 2006 32. 胃食管反流病最主要的发病机制是什么? (2.0分) A.胃酸分泌增加
B.LES静息压降低
C.食管裂孔疝
D.一过性下食管括约肌松弛
E.食管酸清除能力下降
D.一过性下食管括约肌松弛
4 2 2006 33. 下列哪种氨基酸不是营养必需氨基酸(2.0分) A.Leu
B.Val
C.Ser
D.Lys
E.Phe
C.Ser
4 2 2006 34. A 38-year-old woman with rheumatoid arthritis presents to her physician because of increased joint pain. On physical examination, both passive and active range of motion of the hips are decreased. Her physician increases her dose of nonsteroidal son infla(2.0分) A.Clarithromydn
B.Cimetidine
C.Misoprostol
D.Omeprazole
E.Sucralfate
C.Misoprostol
4 2 2006 35. 肝功能不全病人用药中,不正确的是(2.0分) A.定期检查肝功能,及时调整治疗方案
B.避免或减少使用对肝脏毒性大的药物
C.肝功能损害较重者应减少用药剂量
D.选用从肾脏排泄的药物
E.联合用药时,特别应避免合用肝毒性的药物
D.选用从肾脏排泄的药物
4 2 2006 36. 关于消化管平滑肌基本电节律的正确叙述是(2.0分) A.是平滑肌收缩节律的控制波
B.其后一定伴随动作电位
C.是一种超极化波
D.在切断支配胃肠的神经后消失
E.以上说法均不对
A.是平滑肌收缩节律的控制波
4 2 2006 37. 超声图像中"靶环征"最常见于以下哪类疾病(1.0分) A.原发性肝细胞肝癌
B.肝脏血管瘤
C.肠道肿瘤肝转移
D.胆囊充满型结石
C.肠道肿瘤肝转移
4 2 2006 38. 导致消化性溃疡最主要的侵袭因子为: (1.0分) A.胃酸
B.NSAID
C.胰酶
D.胆盐
E.胃蛋白酶
A.胃酸
4 2 2006 39. 男性,25岁,上腹部刀割样疼痛2小时,并迅速转移至右下腹。体检:全腹压痛,伴反跳痛及肌紧张,肠鸣音消失,肝浊音界缩小。最可能的诊断是: (2.0分) A.急性胰腺炎
B.急性阑尾炎穿孔
C.急性胆囊炎
D.消化性溃疡穿孔
E.急性肠梗阻
D.消化性溃疡穿孔
4 2 2006 40. 侵入性试验诊断H. pylori 的首选方法是(2.0分) A.粘膜涂片染色
B.PCR检测
C.微需氧培养
D.RUT
E.组织学检查
D.RUT
4 2 2006 41. 下列哪种矿物质的缺少可以引起心肌病(克山病)和大关节病(2.0分) A.碘
B.硒
C.钙
D.锌
E.铁
B.硒
4 2 2006 42. 胃黏膜细胞分泌的一种内因子(IF),与下列哪种维生素吸收有关(1.0分) A.叶酸
B.生物素
C.维生素B6
D.维生素B12
E.维生素K
D.维生素B12
4 2 2006 43. 支配胃肠道的交感节后纤维释放的递质是(2.0分) A.去甲肾上腺素
B.多巴胺
C.肾上腺素
D.乙酰胆碱
E.织胺
A.去甲肾上腺素
4 2 2006 44. 患者,男性,34岁,反复胸骨后不适6周,伴反酸、咽痛,进餐后尤其是饱食后症状加重,夜间也有发作,否认吞咽困难,否认吞咽痛,否认消瘦。该患者在近2年里体重增加了20公斤。首要的治疗是什么? (2.0分) A.胸部CT检查
B.PPI试验
C.食管测压
D.24小时PH 监测
E.上消化道造影
B.PPI试验
4 2 2006 45. 男性,32岁,因十二指肠溃疡,瘢痕粘连严重,行毕罗氏II式胃次全切除术后第4天, 突然右上腹部持续剧痛,伴呕吐气短,查体:P120次/分,BP80/60mmHg, 右上腹有压痛和反跳痛,肌紧张,最可能是:(2.0分) A.右隔下脓肿
B.吻合口溃疡穿孔
C.十二指肠残端破裂
D.急性出血性胰腺炎
E.急性胆囊炎
C.十二指肠残端破裂
4 2 2006 46. 关于消化道平滑肌基本电节律的错误叙述是(2.0分) A.在胃肠安静状态下可记录到
B.起源于纵行肌和环形肌之间的Cajal细胞
C.在消化道各段平滑肌的发生频率一致
D.与细胞膜生电性钠泵活动的周期性变化有关
E.神经和激素可使其产生
C.在消化道各段平滑肌的发生频率一致
4 2 2006 47. 以下属于胆碱能受体的是 (2.0分) A.M、α和β
B.M、N和β
C.M、N1和N2
D.M、N和α
E.M、β1和β2
C.M、N1和N2
4 2 2006 48. 下列关于脂类吸收的描述哪一项是错误的(1.0分) A.短链和中链脂肪酸可由小肠细胞吸收并通过淋巴入血
B.甘油三酯的消化产物,可以搀入到微团成为混合微团(mixed micelle),容易穿过小肠黏膜细胞的细胞膜而被吸收
C.脂肪酶水解甘油三酯成为游离脂肪酸和甘油一酯
D.长链脂肪酸吸收后,在黏膜细胞内重新合成甘油三脂,然后与其他类脂和蛋白形成乳糜微粒(CM),通过淋巴进入血循环
E.食物中的游离胆固醇可直接被吸收,而胆固醇脂则先被酶水解成游离的胆固醇,再被吸收。
A.短链和中链脂肪酸可由小肠细胞吸收并通过淋巴入血
4 2 2006 49. 慢性胃炎临床表现下列哪些说法是错误的: (2.0分) A.常伴有诊断意义上的消化不良症状
B.患者可有上腹压痛
C.可发生恶性贫血
D.抗HP治疗可彻底治愈一切慢性胃炎
E.病程迁延,以上腹部不适或疼痛为主要症状
D.抗HP治疗可彻底治愈一切慢性胃炎
4 2 2006 50. B型胃炎主要是由下列哪个原因引起:(2.0分) A.吸烟
B.消炎药物
C.胆汁返流
D.幽门螺杆菌感染
E.酒癖
D.幽门螺杆菌感染
4 2 2006 1. Carbon tetrachloride致肝损伤的特点中,不正确的是(3.0分) A.损伤主要由其体内代谢产物所致
B.染毒后24小时损伤最为明显
C.染毒后90小时损伤可恢复
D.损伤主要由其对肝细胞的直接作用所致
E.Vitamin E对损伤可能具有保护作用
B.染毒后24小时损伤最为明显
D.损伤主要由其对肝细胞的直接作用所致
4 2 2006 2. Thiopental sodium具有以下药代动力学特征(3.0分) A.主要由肝脏混合功能酶代谢
B.主要以原型经肾脏排泄
C.二次分布速度快
D.脂溶性高,血浆蛋白结合率低
E.易透过血脑屏障
A.主要由肝脏混合功能酶代谢
C.二次分布速度快
E.易透过血脑屏障
4 2 2007 1. 关于A型胃炎,下列哪项是正确的: (2.0分) A.病变主要累及胃体和胃底
B.大多数由幽门螺杆菌感染引起
C.较常见
D.发病与遗传素质无关
E.做种不易导致恶性贫血
A.病变主要累及胃体和胃底
4 2 2007 2. 诊断反流性食管炎最准确的方法是(2.0分) A.24小时食管PH监测
B.食管测压
C.食管吞钡X线
D.胃镜检查
E.食管滴酸试验
D.胃镜检查
4 2 2007 3. 除下列哪项以外都是胃粘膜的防御修复因素? (2.0分) A.表皮生长因子
B.前列腺素
C.生长抑素
D.胃粘液屏障
E.丰富的粘膜血流
C.生长抑素
4 2 2007 4. Domperidone(多潘立酮)的止吐作用是通过阻断 (2.0分) A.多巴胺受体
B.M1受体
C.α1受体
D.5-HT3受体
E.H2受体
A.多巴胺受体
4 2 2007 5. Aluminum hydroxide(氢氧化铝)常见不良反应是 (2.0分) A.产气
B.便秘
C.腹泻
D.收敛溃疡表面
E.起效快
B.便秘
4 2 2007 6. 下食管括约肌(LES)主要作用是(2.0分) A.防止胃内容物反流入食管
B.帮助食管蠕动,使其顺利进入胃内
C.防止咽部食物呛入气管
D.防止十二指肠内容物反流入胃
E.使食物由胃体进入胃窦
A.防止胃内容物反流入食管
4 2 2007 7. 关于肝药酶的描述,错误的是:(2.0分) A.其活性有限
B.属P-450酶系统
C.易发生竞争性抑制
D.个体差异大
E.只代谢20余种药物
E.只代谢20余种药物
4 2 2007 8. Which of the following is NOT a characteristic of tumor suppressor genes?(2.0分) A.Act dominantly in tumor cells.
B.Associated with "two-hit" model of carcinogenesis.
C.May be associated with loss of heterozygosity in tumors.
D.Transmitted as dominant traits in families.
E.Loss of function
A.Act dominantly in tumor cells.
4 2 2007 9. Six weeks after surgery, the patient returns complaining of postprandial weakness, sweating, light-headedness, crampy abdominal pain, and diarrhea. The best management would be (2.0分) A.Workup for neuroendocrine tumor (e.g., carcinoid)
B.Dietary advice and counseling that symptoms will probably abate within 3 mo of surgery
C.Dietary advice and counseling that symptoms will probably not abate but are not dangerous
D.Antispasmodic medications (e.g., Lomotil)
E.Preparation for revision to Roux-en-Y gastrojejunostomy
B.Dietary advice and counseling that symptoms will probably abate within 3 mo of surgery
4 2 2007 10. Pirenzepine是一种(2.0分) A.H2受体阻断药
B.H1受体阻断药
C.M1受体阻断药
D.D2受体阻断药
C.M1受体阻断药
4 2 2007 11. 溃疡病应用某些抗菌药的目的是 (2.0分) A.抑制胃酸分泌
B.抗幽门螺杆菌
C.清除肠道寄生菌
D.减轻溃疡病的症状
E.保护胃粘膜
B.抗幽门螺杆菌
4 2 2007 12. A routine ophthalmologic examination in a 20-year-old woman demonstrates several well-defined oval, darkly pigmented lesions, one surrounded by a pale halo, on the retina. Identified as congenital hypertrophy of the retinal epithelium (CHEPE), this condit(2.0分) A.BRCA1
B.MLH1
C.BRCA2
D.APC
E.TP53
D.APC
4 2 2007 13. Ondansrtron主要用于治疗 (2.0分) A.去水吗啡引起的呕吐
B.晕动病引起的呕吐
C. 化疗、放疗引起的呕吐
D.十二指肠溃疡
E.胃溃疡
C. 化疗、放疗引起的呕吐
4 2 2007 14. 治疗重症胃食管反流病的首选药物是: (2.0分) A.西沙必利
B.雷尼替丁
C.奥美拉唑
D.氢氧化铝
E.丙谷胺
C.奥美拉唑
4 2 2007 15. 男性,60岁,中上腹胀痛10余年,做胃镜检查与活检,下列哪项病理变化对萎缩性胃炎诊断最有价值 (2.0分) A.淋巴细胞浸润
B.中性粒细胞浸润
C.不典型增生
D.肠上皮化生
E.假幽门腺化生
D.肠上皮化生
4 2 2007 16. 清除食管内容物的主要动力是(2.0分) A.食物的重力
B.吞咽动作
C.对食管内物体推进性蠕动
D.LES(下食管括约肌)舒张
E.食管张力
C.对食管内物体推进性蠕动
4 2 2007 17. 对于肾功能低下者,用药时主要考虑:(2.0分) A.胃肠对药物的吸收
B.药物在肝脏的转化
C.药物自肾脏的转运
D.药物与血浆蛋白的结合率
E.个体差异
C.药物自肾脏的转运
4 2 2007 18. 下列用于胃食管反流病维持治疗的药物中,效果最好的是: (2.0分) A.西沙必利
B.氢氧化铝
C.多潘立酮(吗丁啉)
D.西咪替丁
E.奥美拉唑
E.奥美拉唑
4 2 2007 19. 患者,女性,34岁,反复烧心和胸骨后痛6个月,伴反流。心电图提示非特异性ST-T改变。经过7天的奥美拉唑哦20mg,bid治疗,症状显著改善。最有可能的诊断是什么? (2.0分) A.肺炎
B.心绞痛
C.肋软骨炎
D.胃食管反流病
E.慢性胃炎
D.胃食管反流病
4 2 2007 20. 瘢痕性幽门梗阻的年老体弱患者,应选用:(2.0分) A.毕Ⅰ式胃大部切除术
B.高位选择性胃迷走神经切断术
C.胃空肠吻合术
D.迷走神经干切断术+胃窦部切除术
E.毕Ⅱ式胃大部切除术
C.胃空肠吻合术
4 2 2007 21. 非糜烂性反流病(NERD)诊断的金标准是:(2.0分) A.胃镜
B.胸片
C.心电图
D.PPI试验
E.24小时PH 监测
E.24小时PH 监测
4 2 2007 22. 胃镜检查见胃窦部粘膜苍白,皱襞变细,可见粘膜下紫蓝色的血管纹,最可能的诊断是: (2.0分) A.慢性浅表性胃炎
B.急性糜烂性胃炎
C.急性单纯性胃炎
D.慢性萎缩性胃炎
E.功能消化不良
D.慢性萎缩性胃炎
4 2 2007 23. 慢性胃窦胃炎与下列哪种细菌感染有关: (2.0分) A.厌氧杆菌
B.链球菌
C.幽门螺杆菌
D.大肠杆菌
E.嗜盐杆菌
C.幽门螺杆菌
4 2 2007 24. 患者,男性,34岁,反复胸骨后不适6周,伴反酸、咽痛,进餐后尤其是饱食后症状加重,夜间也有发作,否认吞咽困难,否认吞咽痛,否认消瘦。该患者在近2年里体重增加了20公斤。首要的治疗是什么? (2.0分) A.食管测压
B.24小时PH 监测
C.胸部CT检查
D.PPI试验
E.上消化道造影
D.PPI试验
4 2 2007 25. 消化道平滑肌细胞动作电位的主要离子基础是(2.0分) A.Cl-大量外流
B.K+大量内流
C.Ca2+大量内流
D.Na+大量内流
E.Ca2+ 大量外流
C.Ca2+大量内流
4 2 2007 26. 下述哪个不符合浅表性胃炎胃镜所见: (2.0分) A.胃粘膜浅小溃疡
B.胃粘膜片状出血
C.病变多见于胃底及胃体
D.胃粘膜多发性糜烂
E.病理检查有大量淋巴细胞浸润
E.病理检查有大量淋巴细胞浸润
4 2 2007 27. 诊断消化性溃疡主要依靠下列哪项方法? (2.0分) A.X线钡餐检查
B.病史分析,如典型的周期性和节律性上腹部疼痛
C.内镜检查
D.H.pylori检查
E.诊断性治疗
C.内镜检查
4 2 2007 28. 下面有关消化性溃疡的流行病学情况的描述哪项是错误的? (2.0分) A.男性抽烟较多,所以GU好发于女性而DU好发于男性
B.近年来消化性溃疡的发病率有所下降
C.DU多见于青壮年而GU多见于老年
D.总的发病率来看DU比GU多见
E.消化性溃疡与H. pylori密切相关
A.男性抽烟较多,所以GU好发于女性而DU好发于男性
4 2 2007 29. 下列哪项消化性溃疡的并发症一般不发生在十二指肠溃疡患者中? (2.0分) A.出血
B.癌变
C.穿孔
D.幽门梗阻
E.溃疡复发
B.癌变
4 2 2007 30. 哪些检查是H. pylori根除治疗后首选的检查 (2.0分) A.粪便H. pylori 抗原 培养
B.C-14或C-13呼气试验
C.血清H. pylori IgM
D.血清H. pylori IgG
E.H. pylori培养
B.C-14或C-13呼气试验
4 2 2007 31. 切断支配小肠的迷走神经,可以导致小肠 (2.0分) A.紧张性消失,蠕动消失
B.紧张性和蠕动性均减弱
C.紧张性减弱,蠕动消失
D.紧张性消失,蠕动减弱
E.以上都不对
B.紧张性和蠕动性均减弱
4 2 2007 32. 胃穿孔的特征性X线表现是(2.0分) A.气液平面
B.胃泡增大
C.肠管膨胀
D.膈肌升高
E.膈下游离气体
E.膈下游离气体
4 2 2007 33. B型胃炎主要是由下列哪个原因引起: (2.0分) A.幽门螺杆菌感染
B.胆汁返流
C.消炎药物
D.吸烟
E.酒癖
A.幽门螺杆菌感染
4 2 2007 34. 抑酸作用最强的药物是 (2.0分) A.Omeprazole
B.Pirenzepine
C.Proglumide(丙谷胺)
D.Cimetidine
E.Sodium bicarbonate(碳酸氢钠)
A.Omeprazole
4 2 2007 35. 外科治疗消化性溃疡的目的是(2.0分) A.防止溃疡发生恶变
B.去除溃疡病灶
C.防止消化道出血
D.彻底治愈溃疡
E.治愈溃疡,消除症状,防止复发
E.治愈溃疡,消除症状,防止复发
4 2 2007 36. 关于消化道平滑肌生理特性的错误叙述是 (2.0分) A.富有伸展性
B.对电刺激敏感
C.有自动节律性
D.兴奋性较低
E.收缩速度较慢
B.对电刺激敏感
4 2 2007 37. 十二指肠溃疡宜选择下列哪项手术方法:(2.0分) A.全胃切除术
B.胃肠吻合术
C.胃次全切除术加/或不加迷走神经切断术
D.局部切除术
E.迷走神经干切断术
C.胃次全切除术加/或不加迷走神经切断术
4 2 2007 38. 女性,43岁,12小时以来呕吐出咖啡样物约1200ml。查体:脉搏120次/分,血压80/45mmHg,目前首要的处理是: (2.0分) A.上消化道造影
B.胃镜检查
C.抗休克治疗
D.气管插管术
E.腹部B超
C.抗休克治疗
4 2 2007 39. A 55-year-old man complains of chronic intermittent epigastric pain, and gastroscopy demonstrates a 2-cm ulcer of the distal lesser curvature. Endoscopic biopsy yields no malignant tissue. After a 6-wk trial of H 2 blockade and antacid therapy, the ulcer (2.0分) A.Billroth II subtotal gastrectomy with vagotomy
B.Local excision of the ulcer
C.Billroth I partial gastrectomy
D.Repeat trial of medical therapy
E.Vagotomy and pyloroplasty
C.Billroth I partial gastrectomy
4 2 2007 40. 药物的生物转化和排泄速度决定其:(2.0分) A.副作用的多少
B.最大效应的高低
C.起效的快慢
D.作用持续时间的长短
E.后遗效应的大小
D.作用持续时间的长短
4 2 2007 41. Sylvia has recently learned she has cancer. She has a family history of this same type of cancer. Her doctor orders microsatellite instability testing on tumor tissue after Sylvia's surgery to remove her cancer. What inherited cancer syndrome does her doc(2.0分) A.Hereditary nonpolyposis colon cancer
B.Hereditary breast and ovarian cancer
C.Familial adenomatous polyposis (FAP)
D.Li-Fraumeni syndrome
E.Neurofibromatosis
A.Hereditary nonpolyposis colon cancer
4 2 2007 42. 引起上消化道最常见的原因是(2.0分) A.胃癌
B.出血性胃炎
C.胃十二指肠溃疡
D.门脉高压症
E.胆道感染出血
C.胃十二指肠溃疡
4 2 2007 43. 胃食管反流病最主要的发病机制是什么? (2.0分) A.LES静息压降低
B.一过性下食管括约肌松弛
C.食管裂孔疝
D.胃酸分泌增加
E.食管酸清除能力下降
B.一过性下食管括约肌松弛
4 2 2007 44. Which of the following combinations of mutations could be associated with a tumor in the colon?(2.0分) A.A somatic deletion of APC and hypermethylation of the APC promoter in the tumor
B.Inherited APC mutation and loss of heterozygosity for APC in the tumor
C.Inherited APC mutation and hypermethylation of APC promoter in the tumor
D.A somatic nonsense mutation in APC and a somatic deletion of APC
E.Any of the above
E.Any of the above
4 2 2007 45. A 38-year-old woman with rheumatoid arthritis presents to her physician because of increased joint pain. On physical examination, both passive and active range of motion of the hips are decreased. Her physician increases her dose of nonsteroidal son infla(2.0分) A.Cimetidine
B.Misoprostol
C.Clarithromydn
D.Omeprazole
E.Sucralfate
B.Misoprostol
4 2 2007 46. A 37-year-old woman presents to the emergency department with emesis. She states that she has fibromyalgia syndrome and uses a number of "pain-killers" to control her pain. On waking this morning, she promptly vomited "coffee grounds." On examination, she(2.0分) A.Esophagitis
B.Gastric ulcers
C.Gastric neoplasm
D.Esophageal varices
E.Mallory-Weiss tears
B.Gastric ulcers
4 2 2007 47. 导致消化性溃疡最主要的侵袭因子为: (2.0分) A.胰酶
B.胃酸
C.NSAID
D.胆盐
E.胃蛋白酶
B.胃酸
4 2 2007 48. 可被扑尔敏阻断的受体是(2.0分) A.H2受体
B.H1受体
C.N型受体
D.M型受体
E.H1和H2型受体
B.H1受体
4 2 2007 49. A couple request genetic counseling because the wife has contracted early-onset breast cancer at age 23. The husband has a benign family history, but the wife has several relatives who developed cancers at relative early ages. Affective relatives include (2.0分) A.Germline mutations in a tumor suppressor gene, with neoplasia from chemical exposure
B.Possible autosomal dominant inheritance or multifactorial inheritance of cancer predisposition
C.Germline mutations in an oncogene, with somatic mutations that suppress the oncogene
D.No genetic predisposition to cancer since most individuals have different types of cancer
E.Mitochondrial inheritance of tumor predisposition evidenced by the affected maternal relatives
B.Possible autosomal dominant inheritance or multifactorial inheritance of cancer predisposition
4 2 2007 50. 除了以下哪项以外,都是HP感染引起慢性胃炎的依据是:(2.0分) A.HP阳性者组织学证实胃炎与血清中HP抗体相关
B.慢性胃炎病变的分布与HP存在一致
C.慢性胃炎患者HP检出率高
D.HP与胃炎的活动密切相关
E.无HP感染者不会发生慢性胃炎
E.无HP感染者不会发生慢性胃炎
4 2 2008 1. 假性神经递质引起肝性脑病的机制是: (2.0分) A.干扰脑的能量代谢
B.使脑细胞产生抑制性突触后电位
C.干扰脑细胞膜的功能
D.与正常递质竞争受体,但其效应远较正常递质为弱
E.引起血浆氨基酸失衡
D.与正常递质竞争受体,但其效应远较正常递质为弱
4 2 2008 2. 肝性脑病时,患者氨生成过多的最常见原因是: (2.0分) A.肠道产氨增多
B.肌肉产氨增多
C.脑产氨增多
D.血液NH4+ 向NH3化增多
E.肾产氨产多、并向血液弥散增多
A.肠道产氨增多
4 2 2008 3. 血浆氨基酸失衡学说中所说的芳香族氨基酸包括:(2.0分) A.亮氨酸、异亮氨酸和缬氨酸
B.苯丙氨酸、酪氨酸和色氨酸
C.亮氨酸、缬氨酸和色氨酸
D.谷氨酸和乙酰胆碱
E.苯丙氨酸、酪氨酸
B.苯丙氨酸、酪氨酸和色氨酸
4 2 2008 4. 关于肝性脑病的表现,下列哪一个是错的? (2.0分) A.患者都有昏迷
B.可以是急性.亚急性和慢性
C.可表现为性格和行为异常
D.可出现理解能力减退
E.可出现神经体征
A.患者都有昏迷
4 2 2008 5. 下列哪个不是肝功能衰竭的临床表现? (2.0分) A..出血
B.黄疸
C.PaO2↓
D.神经精神症状
E.氮质血症
C.PaO2↓
4 2 2008 6. 下列哪一项不是胆盐排泄障碍引起的? (2.0分) A.血中胆碱酯酶活性↓
B.VitK吸收障碍
C.心动过缓
D.肠腔内毒素吸收↑
E.血清胆固醇↑
E.血清胆固醇↑
4 2 2008 7. 早期胃癌是指癌组织(2.0分) A.尚未侵犯黏膜下层
B.未突破基底膜
C.未侵犯到浆膜层
D.尚未侵犯肌层
E.尚未转移到引流淋巴结
D.尚未侵犯肌层
4 2 2008 8. 50-year-old man has had persistent nausea for 5 years with occasional vomiting. On physical examination there are no abnormal findings. He undergoes an upper GI endoscopy, and a small area of gastric fundal mucosa has loss of rugal folds. Biopsies are taken and microscopically reveal well-differentiated adenocarcinoma confined to the mucosa. An upper GI endoscopy performed 5 years previously showed a pattern of gastritis and microscopically there was chronic inflammation with the presence of Helicobacter pylori. Which of the following interpretations is most applicable to his neoplasm? (2.0分) A.Metastases limited to regional lymph nodes
B.A 5-year survival following resection of >90%
C.A signet ring cell pattern by light microscopy
D.The gross appearance of linitisplastica
E. A high prevalence in the United States
C.A signet ring cell pattern by light microscopy
4 2 2008 9. 主要与过量饮酒和服用非固醇类抗炎药有关的急性胃炎是(2.0分) A.急性刺激性胃炎
B.急性出血性胃炎
C.腐蚀性胃炎
D.急性感染性胃炎
E.急性卡他性胃炎
B.急性出血性胃炎
4 2 2008 10. 肠上皮化生常见于(2.0分) A.急性胃炎
B.十二指肠溃疡
C.慢性肠血吸虫病
D.慢性萎缩性胃炎
E.溃疡性结肠炎
D.慢性萎缩性胃炎
4 2 2008 11. 慢性胃溃疡肉眼形态通常是(2.0分) A.直径2cm以上火山口状或不规则溃疡
B.直径2cm以内圆形溃疡,贲门侧边缘呈阶梯状
C.直径2cm以内圆形溃疡,幽门侧边缘呈阶梯状
D.直径2cm以内圆形溃疡,幽门侧边缘呈潜掘状
E.直径2cm以内圆形溃疡,周围粘膜皱壁排列紊乱
C.直径2cm以内圆形溃疡,幽门侧边缘呈阶梯状
4 2 2008 12. 胃溃疡病的合并症最常见的是 (2.0分) A.幽门梗阻
B.穿孔
C.大出血
D.癌变
E.粘连
C.大出血
4 2 2008 13. 诊断功能性消化不良需满足条件:症状持续超过__ ___个月(2.0分) A.1
B.3
C.6
D.12
C.6
4 2 2008 14. 关于功能性消化不良症状的解释错误的是 (2.0分) A.餐后饱胀是指食物长时间存留于胃内引起的不适感
B.早饱感是指进食少许食物即感胃部饱满
C.上腹烧灼感是指位于胸骨后的烧灼样疼痛或不适感
D.上腹痛是指位于胸骨剑突下与脐水平以上、两侧锁骨中线之间区域的疼痛感
C.上腹烧灼感是指位于胸骨后的烧灼样疼痛或不适感
4 2 2008 15. 功能性消化不良依据哪项检查确诊 (2.0分) A.内镜检查
B. 排除性诊断
C.腹部CT检查
D. 胃肠功能检查
B. 排除性诊断
4 2 2008 16. 功能性消化不良出现症状 时需警惕器质性疾病发生(2.0分) A.上腹痛、恶心、嗳气
B.早饱、胸痛、腹鸣
C.消瘦、贫血、黑便
D.腹泻、反酸、咽痛
C.消瘦、贫血、黑便
4 2 2008 17. 下食管括约肌(LES)主要作用是(2.0分) A.防止十二指肠内容物反流入胃
B.防止咽部食物呛入气管
C.防止胃内容物反流入食管
D.帮助食管蠕动,使其顺利进入胃内
E.使食物由胃体进入胃窦
C.防止胃内容物反流入食管
4 2 2008 18. 治疗重症胃食管反流病的首选药物是: (2.0分) A.雷尼替丁
B.西沙必利
C.奥美拉唑
D.氢氧化铝
E.丙谷胺
C.奥美拉唑
4 2 2008 19. 胃食管反流病的发病机制不包括:(2.0分) A.下食管括约肌(LES)压力降低
B.一过性下食管括约肌松弛(TLESR)
C.胃排空加快
D.食管酸清除能力下降
E.食管粘膜防御能力下降
C.胃排空加快
4 2 2008 20. 患者,男性,34岁,反复胸骨后不适6周,伴反酸、咽痛,进餐后尤其是饱食后症状加重,夜间也有发作,否认吞咽困难,否认吞咽痛,否认消瘦。该患者在近2年里体重增加了20公斤。首要的治疗是什么? (2.0分) A.PPI试验
B.食管测压
C.24小时PH 监测
D.胸部CT检查
E.上消化道造影
A.PPI试验
4 2 2008 21. 非糜烂性反流病(NERD)诊断的金标准是: (2.0分) A.胃镜
B.心电图
C.胸片
D.PPI试验
E.24小时PH 监测
E.24小时PH 监测
4 2 2008 22. 诊断慢性胃炎最可靠的依据是: (2.0分) A.X线钡餐检查
B.胃液分析
C.慢性上腹部疼痛
D.上腹部轻压痛
E.以上都不是
E.以上都不是
4 2 2008 23. 慢性胃窦胃炎与下列哪种细菌感染有关: (2.0分) A.大肠杆菌
B.链球菌
C.幽门螺杆菌
D.厌氧杆菌
E.嗜盐杆菌
C.幽门螺杆菌
4 2 2008 24. 除了以下哪项以外,都是HP感染引起慢性胃炎的依据是: (2.0分) A.慢性胃炎患者HP检出率高
B.HP阳性者组织学证实胃炎与血清中HP抗体相关
C.HP与胃炎的活动密切相关
D.慢性胃炎病变的分布与HP存在一致
E.无HP感染者不会发生慢性胃炎
E.无HP感染者不会发生慢性胃炎
4 2 2008 25. B型胃炎主要是由下列哪个原因引起:(2.0分) A.胆汁返流
B.幽门螺杆菌感染
C.消炎药物
D.吸烟
E.酒癖
B.幽门螺杆菌感染
4 2 2008 26. 在慢性胃炎的发病机制中,与幽门螺杆菌感染无关的因素是: (2.0分) A.产生胃壁细胞抗体
B.分泌空泡毒素A
C.释放尿素酶分解尿素产生NH3
D.产生细胞毒素相关基因蛋白
E.菌体胞壁作为抗原诱导免疫反应
A.产生胃壁细胞抗体
4 2 2008 27. Aluminum hydroxide(氢氧化铝)常见不良反应是 (2.0分) A.产气
B.腹泻
C.便秘
D.收敛溃疡表面
E.起效快
C.便秘
4 2 2008 28. Omeprazole用于治疗 (2.0分) A.消化不良
B.慢性腹泻
C.慢性便秘
D.胃肠道平滑肌痉挛
E.十二指肠溃疡
E.十二指肠溃疡
4 2 2008 29. 对非甾体抗炎药引起的消化性溃疡有特效的药物是 (2.0分) A.Ondansetron
B.Magnesium trisilicate(三硅酸镁)
C.Misoprostol
D.Sucralfate(硫糖铝)
E.Domperidone(多潘立酮)
C.Misoprostol
4 2 2008 30. Domperidone(多潘立酮)的止吐作用是通过阻断 (2.0分) A.5-HT3受体
B.M1受体
C.α1受体
D.多巴胺受体
E.H2受体
D.多巴胺受体
4 2 2008 31. 关于胰腺癌黄疸不正确的是(2.0分) A.黄疸出现早晚与癌肿部位有关
B.大部分病人出现黄疸时已属中晚期
C.黄疸呈进行性加重,病人均有瘙痒
D.黄疸时间长者可有出血倾向
E.深度黄疸时大便可被染成浅黄色
C.黄疸呈进行性加重,病人均有瘙痒
4 2 2008 32. 胰腺癌最常见的组织类型是(2.0分) A.腺泡细胞癌
B.多形性腺癌
C.纤毛细胞癌
D.导管细胞癌
E.黏液癌
D.导管细胞癌
4 2 2008 33. 胰头癌的主要临床特点是(2.0分) A.厌食、消瘦、乏力
B.上腹部隐痛
C.肝脏肿大
D.胆囊肿大
E.黄疸
E.黄疸
4 2 2008 34. 关于胰岛外分泌物质,下列哪项是错误的(2.0分) A.碳酸氢盐
B.淀粉酶
C.胃泌素
D.脂肪酶
E.淀粉酶
C.胃泌素
4 2 2008 35. 有关胃泌素瘤的陈述下列哪项是不正确的(2.0分) A.超过半数病人的肿瘤是恶性的
B.1/4以上的病人同时合并其他内分泌肿瘤
C.部分病人的肿瘤发生于胰腺外的器官
D.主要表现为消化性溃疡的症状和腹泻
E.全胃切除可以治愈此病
E.全胃切除可以治愈此病
4 2 2008 36. 诊断消化性溃疡主要依靠下列哪项方法? (2.0分) A.病史分析,如典型的周期性和节律性上腹部疼痛
B. X线钡餐检查
C.内镜检查
D.H.pylori检查
E.诊断性治疗
C.内镜检查
4 2 2008 37. 目前认为下列哪项为慢性胃炎及消化性溃疡的最常见病因: (2.0分) A.吸烟
B.饮酒
C.自身免疫
D.幽门螺杆菌感染
E.十二指肠液反流
D.幽门螺杆菌感染
4 2 2008 38. 下列哪项消化性溃疡的并发症一般不发生在十二指肠溃疡患者中? (2.0分) A.出血
B.幽门梗阻
C.穿孔
D.癌变
E. 溃疡复发
D.癌变
4 2 2008 39. 哪些检查是H. pylori根除治疗后首选的检查 (2.0分) A.血清H. pyloriIgG
B.C-14或C-13呼气试验
C.血清H. pyloriIgM
D.粪便H. pylori 抗原培养
E.H. pylori培养
B.C-14或C-13呼气试验
4 2 2008 40. 下面有关消化性溃疡的流行病学情况的描述哪项是错误的?(2.0分) A.近年来消化性溃疡的发病率有所下降
B.男性抽烟较多,所以GU好发于女性而DU好发于男性
C.DU多见于青壮年而GU多见于老年
D.总的发病率来看DU比GU多见
E.消化性溃疡与H. pylori密切相关
B.男性抽烟较多,所以GU好发于女性而DU好发于男性
4 2 2008 41. 有关根除H. pylori的治疗哪项是错误的? (2.0分) A.目前常用的新三联治疗中不包括铋剂
B.确定H. pylori是否被根除的试验应在治疗后2周内进行
C.复查H. pylori是否被根除的试验前1周应停制酸药1周
D.H. pylori根除治疗的疗程一般为7天
E.确定H. pylori是否被根除的试验可用非侵入性方法复查
B.确定H. pylori是否被根除的试验应在治疗后2周内进行
4 2 2008 42. A 37-year-old woman presents to the emergency department with emesis. She states that she has fibromyalgia syndrome and uses a number of "pain-killers" to control her pain. On waking this morning, she promptly vomited "coffee grounds." On examination, she is cool but well perfused. Her blood pressure is 120/70 mm Hg,.and her pulse is 110/min, with no orthostasis. The remainder of her physical examination is unremarkable. A nasogastric tube is passed, which returns 200 ml, of coffee ground material that eventually dears with normal saline lavage. The patient is sent for endoscopy. Which of the following is the most likely diagnosis?(2.0分) A.Esophagitis
B.Esophageal varices
C.Gastric neoplasm
D.Gastric ulcers
E.Mallory-Weiss tears
D.Gastric ulcers
4 2 2008 43. 引起上消化道出血最常见的原因是(2.0分) A.胃癌
B.出血性胃炎
C.门脉高压症
D.胃十二指肠溃疡
E.胆道感染出血
D.胃十二指肠溃疡
4 2 2008 44. 胃穿孔的特征性X线表现是(2.0分) A.胃泡增大
B.肠管膨胀
C.膈肌升高
D.气液平面
E.膈下游离气体
E.膈下游离气体
4 2 2008 45. 女性,40岁,十二指肠球部溃疡多年,近两月,食后上腹部胀满,呕吐,吐宿食。查体:消瘦,脱水貌,上腹部稍膨隆,偶见胃型,有振水音。治疗应首选(2.0分) A.使用抗酸、解痉治疗
B.急症胃大部切除术
C.选择性迷走神经切除术加幽门成形术
D.补液、洗胃继续观察
E.胃肠减压,补液,温盐水洗胃后行胃大部切除术
C.选择性迷走神经切除术加幽门成形术
4 2 2008 46. 瘢痕性幽门梗阻的年老体弱患者,应选用:(2.0分) A.迷走神经干切断术+胃窦部切除术
B.高位选择性胃迷走神经切断术
C.胃空肠吻合术
D.毕Ⅰ式胃大部切除术
E.毕Ⅱ式胃大部切除术
C.胃空肠吻合术
4 2 2008 47. A 45-year-old woman is explored for a perforated duodenal ulcer 6 h after onset of symptoms. She has a history of chronic peptic ulcer disease treated medically with minimal symptoms.The procedure of choice is(2.0分) A.Simple closure with omental patch
B.Truncalvagotomy and pyloroplasty
C.Antrectomy and truncalvagotomy
D.Highly selective vagotomy
E.Hemigastrectomy
C.Antrectomy and truncalvagotomy
4 2 2008 48. Which of the following surgical procedures for peptic ulcer disease is the most effective in preventing the dumping syndrome ?(2.0分) A.Truncalvagotomy with pyloroplasty
B.Highly selective vagotomy
C.Antrectomy with vagotomy
D.Subtotal gastrectomy
E.Selective vagotomy
B.Highly selective vagotomy
4 2 2008 49. Which of the following combinations of mutations could be associated with a tumor in the colon?(2.0分) A.Inherited APC mutation and hypermethylation of APC promoter in the tumor
B.Inherited APC mutation and loss of heterozygosity for APC in the tumor
C.A somatic deletion of APC and hypermethylation of the APC promoter in the tumor
D.A somatic nonsense mutation in APC and a somatic deletion of APC
E.Any of the above
E.Any of the above
4 2 2008 50. A couple request genetic counseling because the wife has contracted early-onset breast cancer at age 23. The husband has a benign family history, but the wife has several relatives who developed cancers at relative early ages. Affective relatives include a sister (colon cancer, age 42), a brother (colon cancer, age 46), mother (breast cancer, age 56), maternal aunt (leukemia, age 45), maternal uncle (muscle sarcoma, age 49), and a nephew through the brother with colon cancer ((leukemia, age 8). Which of the following is the correct conclusion from the family history?(2.0分) A.No genetic predisposition to cancer since most individuals have different types of cancer
B.Possible autosomal dominant inheritance or multifactorial inheritance of cancer predisposition
C.Germline mutations in an oncogene, with somatic mutations that suppress the oncogene
D.Germline mutations in a tumor suppressor gene, with neoplasia from chemical exposure
E.Mitochondrial inheritance of tumor predisposition evidenced by the affected maternal relatives
B.Possible autosomal dominant inheritance or multifactorial inheritance of cancer predisposition
4 2 2009 1. 与食管癌发生无关的因素是:(2.0分) A.霉变食物
B.食物中含亚硝酸盐
C.返流性食管炎
D.食管痉挛
E.喜食过热饮食
D.食管痉挛
4 2 2009 2. 早期胃癌最多见的类型是 (2.0分) A.隆起型
B.表浅型
C.表浅凹陷型
D.表浅平坦型
E.凹陷型
E.凹陷型
4 2 2009 3. 胃癌最主要的转移途径是(2.0分) A.直接转移
B.淋巴道转移
C.血道转移
D.腹腔内种植转移
E.神经转移
B.淋巴道转移
4 2 2009 4. 在胃癌HE染色切片中,癌细胞弥漫浸润于粘膜下层和肌层,有大量淡蓝色无结构的物质,其中漂浮小堆癌细胞,部分癌细胞核偏于一侧,胞浆内亦有上述物质,该切片应诊断为: (2.0分) A. 未分化癌
B.髓样癌
C.硬癌
D.粘液腺癌
E.低分化腺癌
D.粘液腺癌
4 2 2009 5. 胃消化性溃疡不易愈合的局部因素主要是(2.0分) A.局部溃疡过深
B.溃疡周边组织水肿
C.局部溃疡过大
D.溃疡底部神经纤维变性、断裂
E.溃疡底部小动脉增生性内膜炎
E.溃疡底部小动脉增生性内膜炎
4 2 2009 6. 肠上皮化生常见于(2.0分) A.急性胃炎
B.十二指肠溃疡
C.慢性肠血吸虫病
D.慢性萎缩性胃炎
E.溃疡性结肠炎
D.慢性萎缩性胃炎
4 2 2009 7. 胃溃疡病变最多见的部位是(2.0分) A.大弯近幽门部
B.小弯近幽门部
C.小弯区近贲门部
D.小弯角切迹部
E.贲门
B.小弯近幽门部
4 2 2009 8. 慢性胃溃疡肉眼形态通常是(2.0分) A.直径2cm以上火山口状或不规则溃疡
B.直径2cm以内圆形溃疡,贲门侧边缘呈阶梯状
C.直径2cm以内圆形溃疡,幽门侧边缘呈阶梯状
D.直径2cm以内圆形溃疡,幽门侧边缘呈潜掘状
E.直径2cm以内圆形溃疡,周围粘膜皱壁排列紊乱
C.直径2cm以内圆形溃疡,幽门侧边缘呈阶梯状
4 2 2009 9. 下列哪种情况与胃消化性溃疡无关: (2.0分) A.幽门螺杆菌感染
B.溃疡底部正常胃壁结构破坏
C.常反复发作
D.溃疡直径通常在2cm以下
E.患者胃酸分泌减少
E.患者胃酸分泌减少
4 2 2009 10. 慢性浅表性胃炎的主要肉眼所见是:(2.0分) A.胃粘膜皱襞变浅,几乎消失
B.透过胃粘膜可见粘膜下血管
C.胃粘膜面平滑
D.胃粘膜失去正常光泽而发灰
E.以上都不是
D.胃粘膜失去正常光泽而发灰
4 2 2009 11. 胃溃疡底部镜检所见为:(2.0分) A.渗出层
B.坏死层
C.肉芽组织层
D.疤痕组织层
E.以上都是
E.以上都是
4 2 2009 12. 慢性消化性溃疡的发生与下列哪些因素无关?(2.0分) A.遗传因素
B.胃泌素分泌亢进
C.胆汁返流增多
D.迷走神经兴奋性降低
E.胃酸分泌增多
D.迷走神经兴奋性降低
4 2 2009 13. 功能性消化不良的特异性症状不包括 (2.0分) A.餐后饱胀、早饱
B.上腹痛
C.上腹烧灼感
D.频繁呕吐
D.频繁呕吐
4 2 2009 14. 功能性消化不良的主要病理生理机制不包括(2.0分) A.胃容受性舒张受损
B. 胃排空过快
C.内脏高敏感
D.精神心理因素
B. 胃排空过快
4 2 2009 15. 功能性消化不良可以与以下疾病同时存在,除了 (2.0分) A.消化性溃疡
B. 胃食管反流病
C.肠易激综合征
D.功能性腹胀
A.消化性溃疡
4 2 2009 16. 功能性消化不良的治疗不包括 (2.0分) A.抑酸药
B. 促动力药
C.解痉药
D.精神心理治疗
C.解痉药
4 2 2009 17. Aluminum hydroxide(氢氧化铝)常见不良反应是(2.0分) A.产气
B.腹泻
C.便秘
D.收敛溃疡表面
E.起效快
C.便秘
4 2 2009 18. 奥美拉唑的作用机制是阻断 (2.0分) A.H2受体
B.M受体
C.质子泵(H+-K+-ATP酶、H+泵)
D.胃泌素受体
E.多巴胺受体
C.质子泵(H+-K+-ATP酶、H+泵)
4 2 2009 19. 对非甾体抗炎药引起的消化性溃疡最有效的药物是(2.0分) A.Ondansetron(昂丹司琼)
B.Magnesium trisilicate(三硅酸镁)
C.Misoprostol(米索前列醇)
D.Sucralfate(硫糖铝)
E.Domperidone(多潘立酮)
C.Misoprostol(米索前列醇)
4 2 2009 20. Domperidone(多潘立酮)的止吐作用是通过阻断(2.0分) A.5-HT3受体
B.M1受体
C.α1受体
D.多巴胺受体
E.H2受体
D.多巴胺受体
4 2 2009 21. 女性,43岁,12小时以来呕吐出咖啡样物约1200ml。查体:脉搏120次/分,血压80/45mmHg,目前首要的处理是: (2.0分) A.胃镜检查
B.抗休克治疗
C.气管插管术
D.上消化道造影
E.腹部B超
B.抗休克治疗
4 2 2009 22. 外科治疗消化性溃疡的目的是(2.0分) A.防止消化道出血
B.去除溃疡病灶
C.防止溃疡发生恶变
D.彻底治愈溃疡
E.治愈溃疡,消除症状,防止复发
E.治愈溃疡,消除症状,防止复发
4 2 2009 23. 胃大部切除术后并发症中,哪项切忌再次手术?(2.0分) A.十二指肠残端破裂
B.胃肠吻合口破裂
C.胃肠吻合口排空障碍
D. 急性完全性输入袢梗阻
E.输出段梗阻
C.胃肠吻合口排空障碍
4 2 2009 24. 十二指肠溃疡宜选择下列哪项手术方法:(2.0分) A.胃肠吻合术
B.全胃切除术
C.胃次全切除术加/或不加迷走神经切断术
D.局部切除术
E.迷走神经干切断术
C.胃次全切除术加/或不加迷走神经切断术
4 2 2009 25. A 55-year-old man complains of chronic intermittent epigastric pain, and gastroscopy demonstrates a 2-cm ulcer of the distal lesser curvature. Endoscopic biopsy yields no malignant tissue. After a 6-wk trial of H 2 blockade and antacid therapy, the ulcer is unchanged. Proper therapy at this point is (2.0分) A.Repeat trial of medical therapy
B.Local excision of the ulcer
C.Billroth I partial gastrectomy
D.Billroth II subtotal gastrectomy with vagotomy
E.Vagotomy and pyloroplasty
C.Billroth I partial gastrectomy
4 2 2009 26. Six weeks after surgery, the patient returns complaining of postprandial weakness, sweating, light-headedness, crampy abdominal pain, and diarrhea. The best management would be (2.0分) A.Antispasmodic medications (e.g., Lomotil)
B.Dietary advice and counseling that symptoms will probably abate within 3 mo of surgery
C.Dietary advice and counseling that symptoms will probably not abate but are not dangerous
D.Workup for neuroendocrine tumor (e.g., carcinoid)
E.Preparation for revision to Roux-en-Y gastrojejunostomy
B.Dietary advice and counseling that symptoms will probably abate within 3 mo of surgery
4 2 2009 27. 氨对脑的毒性作用不包括: (2.0分) A.干扰脑的能量代谢
B.使脑内兴奋性递质产生减少
C.使脑内抑制性递质产生增多
D.使脑的敏感性增高
E.抑制脑细胞膜的功能
C.使脑内抑制性递质产生增多
4 2 2009 28. 肝性脑病患者血浆支链氨基酸减少的原因是:(2.0分) A.血浆胰高血糖素浓度升高所致
B.高胰岛素血症所致
C.肝对支链氨基酸灭活减少
D.支链氨基酸合成来源减少
E.血浆芳香族氨基酸增多引起
B.高胰岛素血症所致
4 2 2009 29. 假性神经递质的毒性作用是(2.0分) A.干扰去甲肾上腺素
B.使γ-氨基丁酸蓄积
C.阻碍三羧酸循环
D.使乙酰辅酶A减少
E.使乙酰胆碱减少
A.干扰去甲肾上腺素
4 2 2009 30. 肝性脑病的发病主因: (2.0分) A.脑组织发生特异性形态学改变
B.肝功能衰竭
C.脑细胞的代谢和功能障碍
D.体内产生毒性物作用
E.抑制性神经递质增加
C.脑细胞的代谢和功能障碍
4 2 2009 31. 真性肝肾综合征是指: (2.0分) A.肝脏病引起的器质性肾衰
B.肝脏病引起的功能性肾衰
C.肝脏病引起的功能性肾衰或急性肾小管坏死
D.肝和肾受同一病因损害引起的综合征
E.肾脏病引起的肝功能不全
C.肝脏病引起的功能性肾衰或急性肾小管坏死
4 2 2009 32. 肝功能衰竭时不会直接引起: (2.0分) A.肾功能障碍
B.心力衰竭
C.单核吞噬细胞系统功能障碍
D.凝血功能障碍
E.物质代谢障碍
B.心力衰竭
4 2 2009 33. 诊断反流性食管炎最准确的方法是(2.0分) A.24小时食管PH监测
B.食管测压
C.胃镜检查
D.食管吞钡X线
E.食管滴酸试验
C.胃镜检查
4 2 2009 34. 清除食管内容物的主要动力是(2.0分) A.食物的重力
B.对食管内物体推进性蠕动
C.吞咽动作
D.LES(下食管括约肌)舒张
E.食管张力
B.对食管内物体推进性蠕动
4 2 2009 35. 下列用于胃食管反流病维持治疗的药物中,效果最好的是: (2.0分) A.西沙必利
B.多潘立酮(吗丁啉)
C.氢氧化铝
D.西咪替丁
E.奥美拉唑
E.奥美拉唑
4 2 2009 36. 患者,女性,34岁,反复烧心和胸骨后痛6个月,伴反流。心电图提示非特异性ST-T改变。经过7天的奥美拉唑哦20mg,bid治疗,症状显著改善。最有可能的诊断是什么? (2.0分) A.心绞痛
B.肺炎
C.肋软骨炎
D.胃食管反流病
E.慢性胃炎
D.胃食管反流病
4 2 2009 37. 胃食管反流病最主要的发病机制是什么? (2.0分) A.一过性下食管括约肌松弛
B.LES静息压降低
C.食管裂孔疝
D.胃酸分泌增加
E.食管酸清除能力下降
A.一过性下食管括约肌松弛
4 2 2009 38. 男性,60岁,中上腹胀痛10余年,做胃镜检查与活检,下列哪项病理变化对萎缩性胃炎诊断最有价值 (2.0分) A.肠上皮化生
B.不典型增生
C.淋巴细胞浸润
D.中性粒细胞浸润
E.假幽门腺化生
A.肠上皮化生
4 2 2009 39. 胃镜检查见胃窦部粘膜苍白,皱襞变细,可见粘膜下紫蓝色的血管纹,最可能的诊断是: (2.0分) A.急性单纯性胃炎
B.急性糜烂性胃炎
C.慢性浅表性胃炎
D.慢性萎缩性胃炎
E.功能消化不良
D.慢性萎缩性胃炎
4 2 2009 40. 慢性浅表性胃炎胃镜检查可见:(2.0分) A.粘膜充血、水肿、糜烂和出血
B.粘膜高低不平
C.粘膜覆盖较多灰白色苔
D.透见粘膜下血管
E.粘膜中断现象
A.粘膜充血、水肿、糜烂和出血
4 2 2009 41. 慢性胃炎临床表现下列哪些说法是错误的: (2.0分) A.抗HP治疗可彻底治愈一切慢性胃炎
B.患者可有上腹压痛
C.可发生恶性贫血
D.常伴有诊断意义上的消化不良症状
E.病程迁延,以上腹部不适或疼痛为主要症状
A.抗HP治疗可彻底治愈一切慢性胃炎
4 2 2009 42. 关于A型胃炎,下列哪项是正确的: (2.0分) A.较常见
B.大多数由幽门螺杆菌感染引起
C.病变主要累及胃体和胃底
D.发病与遗传素质无关
E.做种不易导致恶性贫血
C.病变主要累及胃体和胃底
4 2 2009 43. 下述哪个不符合浅表性胃炎胃镜所见: (2.0分) A.病变多见于胃底及胃体
B.胃粘膜片状出血
C.胃粘膜浅小溃疡
D.胃粘膜多发性糜烂
E.病理检查有大量淋巴细胞浸润
E.病理检查有大量淋巴细胞浸润
4 2 2009 44. Sylvia has recently learned she has cancer. She has a family history of this same type of cancer. Her doctor orders microsatellite instability testing on tumor tissue after Sylvia's surgery to remove her cancer. What inherited cancer syndrome does her doctor suspect in Sylvia's family?(2.0分) A.Hereditary breast and ovarian cancer
B.Hereditary nonpolyposis colon cancer
C.Familial adenomatous polyposis (FAP)
D.Li-Fraumeni syndrome
E.Neurofibromatosis
B.Hereditary nonpolyposis colon cancer
4 2 2009 45. Which of the following is the largest contributor to cancer risk in the developed world?(2.0分) A.Genetic variation
B.Diet
C.Infectious agents
D.UV exposure
E.Smoking
E.Smoking
4 2 2009 46. Genes responsible for hereditary nonpolyposis colon cancer encode proteins with which of the following function?(2.0分) A.Cell membrane receptor
B.Transcription factor
C.DNA mismatch repair
D.Cell cycle control
E.apoptosis inhibitor
C.DNA mismatch repair
4 2 2009 47. A routine ophthalmologic examination in a 20-year-old woman demonstrates several well-defined oval, darkly pigmented lesions, one surrounded by a pale halo, on the retina. Identified as congenital hypertrophy of the retinal epithelium (CHEPE), this condition is most often associated with mutations in which gene?(2.0分) A.MLH1
B.BRCA1
C.BRCA2
D.APC
E.TP53
D.APC
4 2 2009 48. 诊断消化性溃疡主要依靠下列哪项方法? (2.0分) A.病史分析,如典型的周期性和节律性上腹部疼痛
B.X线钡餐检查
C.内镜检查
D.H.pylori检查
E.诊断性治疗
C.内镜检查
4 2 2009 49. 消化性溃疡与胃癌的鉴别诊断错误的是: (2.0分) A.GU与胃癌很难从症状上作出诊断,必须依靠胃镜检查及病理学诊断。
B.对于胃镜怀疑恶性溃疡而一次活检阴性者,必须短期内复查内镜并再次活检。
C.CT检查能发现胃壁的增厚及强化,可以鉴别良恶性溃疡。
D.强力抗酸治疗后,溃疡明显缩小但不是判断良恶性溃疡的可靠依据。
E.对治疗后愈合不良的难治性GU,需内镜复查随访,直至证实溃疡愈合
C.CT检查能发现胃壁的增厚及强化,可以鉴别良恶性溃疡。
4 2 2009 50. A 38-year-old woman with rheumatoid arthritis presents to her physician because of increased joint pain. On physical examination, both passive and active range of motion of the hips are decreased. Her physician increases her dose of nonsteroidal son inflammatory drugs (NSAIDs). Which of the following should be prescribed as well to prevent peptic ulcer disease?(2.0分) A.Cimetidine
B.Clarithromydn
C.Misoprostol
D. Omeprazole
E. Sucralfate
C.Misoprostol
4 2 2010 1. 胃消化性溃疡不易愈合的局部因素主要是(2.0分) A.局部溃疡过深
B.溃疡周边组织水肿
C.局部溃疡过大
D.溃疡底部神经纤维变性、断裂
E.溃疡底部小动脉增生性内膜炎
E.溃疡底部小动脉增生性内膜炎
4 2 2010 2. 肠上皮化生常见于(2.0分) A.急性胃炎
B.十二指肠溃疡
C.慢性肠血吸虫病
D.慢性萎缩性胃炎
E.溃疡性结肠炎
D.慢性萎缩性胃炎
4 2 2010 3. 胃溃疡病变最多见的部位是(2.0分) A.大弯近幽门部
B.小弯近幽门部
C.小弯区近贲门部
D.小弯角切迹部
E.贲门
B.小弯近幽门部
4 2 2010 4. 慢性胃溃疡肉眼形态通常是(2.0分) A.直径2cm以上火山口状或不规则溃疡
B.直径2cm以内圆形溃疡,贲门侧边缘呈阶梯状
C.直径2cm以内圆形溃疡,幽门侧边缘呈阶梯状
D.直径2cm以内圆形溃疡,幽门侧边缘呈潜掘状
E.直径2cm以内圆形溃疡,周围粘膜皱壁排列紊乱
C.直径2cm以内圆形溃疡,幽门侧边缘呈阶梯状
4 2 2010 5. 下列哪种情况与胃消化性溃疡无关: (2.0分) A.幽门螺杆菌感染
B.溃疡底部正常胃壁结构破坏
C.常反复发作
D.溃疡直径通常在2cm以下
E.患者胃酸分泌减少
E.患者胃酸分泌减少
4 2 2010 6. 慢性浅表性胃炎的主要肉眼所见是(2.0分) A.胃粘膜皱襞变浅,几乎消失
B.透过胃粘膜可见粘膜下血管
C.胃粘膜面平滑
D.胃粘膜失去正常光泽而发灰
E.以上都不是
D.胃粘膜失去正常光泽而发灰
4 2 2010 7. 胃溃疡底部镜检所见为(2.0分) A.渗出层
B.坏死层
C.肉芽组织层
D.疤痕组织层
E.以上都是
E.以上都是
4 2 2010 8. 慢性消化性溃疡的发生与下列哪些因素无关?(2.0分) A.遗传因素
B.胃泌素分泌亢进
C.胆汁返流增多
D.迷走神经兴奋性降低
E.胃酸分泌增多
D.迷走神经兴奋性降低
4 2 2010 9. 与食管癌发生无关的因素是(2.0分) A.霉变食物
B.食物中含亚硝酸盐
C.返流性食管炎
D.食管痉挛
E.喜食过热饮食
D.食管痉挛
4 2 2010 10. 早期胃癌最多见的类型是 (2.0分) A.隆起型
B.表浅型
C.表浅凹陷型
D.表浅平坦型
E.凹陷型
E.凹陷型
4 2 2010 11. 胃癌最主要的转移途径是(2.0分) A.直接转移
B.淋巴道转移
C.血道转移
D.腹腔内种植转移
E.神经转移
B.淋巴道转移
4 2 2010 12. 在胃癌HE染色切片中,癌细胞弥漫浸润于粘膜下层和肌层,有大量淡蓝色无结构的物质,其中漂浮小堆癌细胞,部分癌细胞核偏于一侧,胞浆内亦有上述物质,该切片应诊断为: 未分化癌(2.0分) A.髓样癌
B.硬癌
C.粘液腺癌
D.低分化腺癌
D.低分化腺癌
4 2 2010 13. 抗休克治疗(2.0分) A.胃镜检查
B.气管插管术
C.上消化道造影
D.腹部B超
A.胃镜检查
4 2 2010 14. 外科治疗消化性溃疡的目的是(2.0分) A.防止消化道出血
B.去除溃疡病灶
C.防止溃疡发生恶变
D.彻底治愈溃疡
E.治愈溃疡,消除症状,防止复发
E.治愈溃疡,消除症状,防止复发
4 2 2010 15. 胃大部切除术后并发症中,哪项切忌再次手术?(2.0分) A.十二指肠残端破裂
B.胃肠吻合口破裂
C.胃肠吻合口排空障碍
D. 急性完全性输入袢梗阻
E.输出段梗阻
C.胃肠吻合口排空障碍
4 2 2010 16. 十二指肠溃疡宜选择下列哪项手术方法:(2.0分) A.胃肠吻合术
B.全胃切除术
C.胃次全切除术加/或不加迷走神经切断术
D.局部切除术
E.迷走神经干切断术
C.胃次全切除术加/或不加迷走神经切断术
4 2 2010 17. A 55-year-old man complains of chronic intermittent epigastric pain, and gastroscopy demonstrates a 2-cm ulcer of the distal lesser curvature. Endoscopic biopsy yields no malignant tissue. After a 6-wk trial of H 2 blockade and antacid therapy, the ulcer is unchanged. Proper therapy at this point is (2.0分) A.Repeat trial of medical therapy
B.Local excision of the ulcer
C.Billroth I partial gastrectomy
D.Billroth II subtotal gastrectomy with vagotomy
E.Vagotomy and pyloroplasty
C.Billroth I partial gastrectomy
4 2 2010 18. Six weeks after surgery, the patient returns complaining of postprandial weakness, sweating, light-headedness, crampy abdominal pain, and diarrhea. The best management would be (2.0分) A.Antispasmodic medications (e.g., Lomotil)
B.Dietary advice and counseling that symptoms will probably abate within 3 mo of surgery
C.Dietary advice and counseling that symptoms will probably not abate but are not dangerous
D.Workup for neuroendocrine tumor (e.g., carcinoid)
E.Preparation for revision to Roux-en-Y gastrojejunostomy
B.Dietary advice and counseling that symptoms will probably abate within 3 mo of surgery
4 2 2010 19. 关于消化管平滑肌基本电节律的正确叙述是(2.0分) A.是一种超极化波
B.其后一定伴随动作电位
C.是平滑肌收缩节律的控制波
D.在切断支配胃肠的神经后消失
E.以上说法均不对
C.是平滑肌收缩节律的控制波
4 2 2010 20. 迷走神经兴奋时 (2.0分) A.胃肠平滑肌活动增强,消化腺分泌减少
B.胃肠平滑肌活动减弱,消化腺分泌增加
C.胃肠平滑肌活动增强,消化腺分泌增加
D.胃肠平滑肌活动减弱,消化腺分泌减少
E.以上都不是
C.胃肠平滑肌活动增强,消化腺分泌增加
4 2 2010 21. 关于消化道平滑肌生理特性的错误叙述是(2.0分) A.有自动节律性
B.对电刺激敏感
C.富有伸展性
D.兴奋性较低
E.收缩速度较慢
B.对电刺激敏感
4 2 2010 22. 以下属于胆碱能受体的是(2.0分) A.M、N和α
B.M、N和β
C.M、N1和N2
D.M、α和β
E.M、β1和β2
C.M、N1和N2
4 2 2010 23. 可被扑尔敏阻断的受体是(2.0分) A.H1受体
B.H2受体
C.N型受体
D.M型受体
E.H1和H2型受体
A.H1受体
4 2 2010 24. 诊断功能性消化不良需满足条件:症状持续超过____个月 (2.0分) A.1
B.3
C. 6
D.12
C. 6
4 2 2010 25. 关于功能性消化不良症状的解释错误的是 (2.0分) A.餐后饱胀是指食物长时间存留于胃内引起的不适感
B.早饱感是指进食少许食物即感胃部饱满
C.上腹烧灼感是指位于胸骨后的烧灼样疼痛或不适感
D.上腹痛是指位于胸骨剑突下与脐水平以上、两侧锁骨中线之间区域的疼痛感
C.上腹烧灼感是指位于胸骨后的烧灼样疼痛或不适感
4 2 2010 26. 功能性消化不良依据哪项检查确诊 (2.0分) A.内镜检查
B.排除性诊断
C.腹部CT检查
D.胃肠功能检查
B.排除性诊断
4 2 2010 27. 功能性消化不良出现症状 时需警惕器质性疾病发生(2.0分) A.上腹痛、恶心、嗳气
B.早饱、胸痛、腹鸣
C.消瘦、贫血、黑便
D. 腹泻、反酸、咽痛
C.消瘦、贫血、黑便
4 2 2010 28. Sylvia has recently learned she has cancer. She has a family history of this same type of cancer. Her doctor orders microsatellite instability(微卫星不稳定性) testing on tumor tissue after Sylvia's surgery to remove her cancer. What inherited cancer syndrome does her doctor suspect in Sylvia's family?(2.0分) A.Hereditary breast and ovarian cancer
B.Hereditary nonpolyposis colon cancer(HNPCC)
C.Familial adenomatous polyposis (FAP)
D.Li-Fraumeni syndrome
E.Neurofibromatosis
B.Hereditary nonpolyposis colon cancer(HNPCC)
4 2 2010 29. Which of the following is the largest contributor to cancer risk in the developed world?(2.0分) A.Genetic variation
B.Diet
C.Infectious agents
D.UV exposure
E.Smoking
E.Smoking
4 2 2010 30. Genes responsible for hereditary nonpolyposis colon cancer(HNPCC) encode proteins with which of the following function?(2.0分) A.Cell membrane receptor
B.Transcription factor
C.DNA mismatch repair
D.Cell cycle control
E.apoptosis inhibitor
C.DNA mismatch repair
4 2 2010 31. A routine ophthalmologic examination in a 20-year-old woman demonstrates several well-defined oval, darkly pigmented lesions, one surrounded by a pale halo, on the retina. Identified as congenital hypertrophy of the retinal epithelium (CHEPE), this condition is most often associated with mutations in which gene?(2.0分) A.MLH1
B.BRCA1
C.BRCA2
D.APC
E.TP53
D.APC
4 2 2010 32. 起效快、作用强、作用短暂的抗酸药是 (2.0分) A.硫酸钙
B.氢氧化镁
C.氢氧化铝
D.三硅酸镁
E.碳酸氢钠
E.碳酸氢钠
4 2 2010 33. 胃壁细胞H+泵抑制药有 (2.0分) A.Pirenzepine(哌仑西平)
B.Cimetidine(西咪替丁)
C.Famotidine(法莫西汀)
D.Omeprazole(奥美拉唑)
E.Proglumide(丙谷胺)
D.Omeprazole(奥美拉唑)
4 2 2010 34. 硫糖铝(sucralfate)用于抗溃疡病治疗的主要作用是 (2.0分) A.中和胃酸
B.抑制胃酸分泌
C.拮抗胆碱受体
D.抑制前列腺素E2合成
E.在溃疡面表面形成保护屏障
E.在溃疡面表面形成保护屏障
4 2 2010 35. 哪一药物治疗非甾体抗炎药引起的消化性溃疡最有效? (2.0分) A.Ondansetron(昂丹司琼)
B.Magnesium trisilicate(三硅酸镁)
C.Misoprostol(米索前列醇)
D.Sucralfate(硫糖铝)
E.Domperidone(多潘立酮)
C.Misoprostol(米索前列醇)
4 2 2010 36. Domperidone(多潘立酮)通过阻断哪种受体而产生止吐作用? (2.0分) A.5-HT3受体
B.M1受体
C.α1受体
D.多巴胺受体
E.H2受体
D.多巴胺受体
4 2 2010 37. 假性神经递质引起肝性脑病的机制是: (2.0分) A.干扰脑的能量代谢
B.使脑细胞产生抑制性突触后电位
C.干扰脑细胞膜的功能
D.与正常递质竞争受体,但其效应远较正常递质为弱
E.引起血浆氨基酸失衡
D.与正常递质竞争受体,但其效应远较正常递质为弱
4 2 2010 38. 肝性脑病时,患者氨生成过多的最常见原因是: (2.0分) A.肠道产氨增多
B.肌肉产氨增多
C.脑产氨增多
D.血液NH4+ 向NH3化增多
E.肾产氨产多、并向血液弥散增多
A.肠道产氨增多
4 2 2010 39. 血浆氨基酸失衡学说中所说的芳香族氨基酸包括:(2.0分) A.亮氨酸、异亮氨酸和缬氨酸
B.苯丙氨酸、酪氨酸和色氨酸
C.亮氨酸、缬氨酸和色氨酸
D.谷氨酸和乙酰胆碱
E.苯丙氨酸、酪氨酸
B.苯丙氨酸、酪氨酸和色氨酸
4 2 2010 40. 关于肝性脑病的表现,下列哪一个是错的? (2.0分) A.患者都有昏迷
B.可以是急性.亚急性和慢性
C.可表现为性格和行为异常
D.可出现理解能力减退
E.可出现神经体征
A.患者都有昏迷
4 2 2010 41. 下列哪个不是肝功能衰竭的临床表现? (2.0分) A.出血
B.黄疸
C.PaO2↓
D.神经精神症状
E.氮质血症
C.PaO2↓
4 2 2010 42. 下列哪一项不是胆盐排泄障碍引起的? (2.0分) A.血中胆碱酯酶活性↓
B.VitK吸收障碍
C.心动过缓
D.肠腔内毒素吸收↑
E.血清胆固醇↑
E.血清胆固醇↑
4 2 2010 43. 胰腺假性囊肿实际上是胰腺周围的包裹性积液,囊壁由纤维组织和肉芽组织构成,囊液内含有组织碎片和大量胰酶,多发生于(2.0分) A.急性胰腺炎基础上
B.慢性胰腺炎基础上
C.急性胰腺炎好转基础上
D.慢性胰腺炎好转基础上
E.重症急性胰腺炎胰腺坏死基础上
E.重症急性胰腺炎胰腺坏死基础上
4 2 2010 44. 急性胰腺炎患者饮食应该(2.0分) A.少量多餐
B.低盐饮食
C.高维生素饮食
D.急性期禁食
E.忌辛辣食物
D.急性期禁食
4 2 2010 45. 我国急性胰腺炎最常见的病因是(2.0分) A..暴饮暴食
B.酒精性
C.胆道疾病
D.高脂血症
E.病毒感染
C.胆道疾病
4 2 2010 46. CRP 检测以下哪项提示胰腺广泛坏死(2.0分) A.CRP大于100mg/L
B.CRP大于130mg/L
C.CRP大于150mg/L
D.CRP大于170mg/L
E.CRP大于200mg/L
C.CRP大于150mg/L
4 2 2010 47. 胰腺假性囊肿多在急性胰腺炎起病2周后发生,多发生于(2.0分) A.胰头
B.胰头、胰体
C.胰体
D.胰体、胰尾
E.胰尾
D.胰体、胰尾
4 2 2010 48. 功能消化不良最常见的临床亚型为(2.0分) A.上腹痛综合征
B.餐后不适综合征
C.功能性腹痛综合征
D.嗳气病
E.成人反刍综合征
A.上腹痛综合征
4 2 2010 49. 根据2007年中国庐山共识意见,除了哪一项外,Hp阳性患者必须根除?(2.0分) A.消化性溃疡
B.胃MALT淋巴瘤
C.慢性胃炎伴胃粘膜萎缩、糜烂
D.胃癌家族史
E.早期胃癌术后
D.胃癌家族史
4 2 2010 50. 依据罗马III诊断标准,诊断功能性消化不良,要求症状至少出现的时间为(2.0分) A.2个月
B.4个月
C.6个月
D.10个月
E.1年
C.6个月
4 2 2011 1. Sylvia has recently learned she has cancer. She has a family history of this same type of cancer. Her doctor orders microsatellite instability(微卫星不稳定性) testing on tumor tissue after Sylvia's surgery to remove her cancer. What inherited cancer syndrome does her doctor suspect in Sylvia's family?(2.5分) A.Hereditary breast and ovarian cancer
B.Hereditary nonpolyposis colon cancer (HNPCC)
C.Familial adenomatous polyposis (FAP)
D.Li-Fraumeni syndrome
E.Neurofibromatosis
B.Hereditary nonpolyposis colon cancer (HNPCC)
4 2 2011 2. Which of the following is the largest contributor to cancer risk in the developed world?(2.5分) A.Genetic variation
B.Diet
C.Infectious agents
D.UV exposure
E.Smoking
E.Smoking
4 2 2011 3. Genes responsible for hereditary nonpolyposis colon cancer(HNPCC) encode proteins with which of the following function?(2.5分) A.Cell membrane receptor
B.Transcription factor
C.DNA mismatch repair
D.Cell cycle control
E.apoptosis inhibitor
C.DNA mismatch repair
4 2 2011 4. A routine ophthalmologic examination(眼科检查) in a 20-year-old woman demonstrates several well-defined oval, darkly pigmented lesions, one surrounded by a pale halo, on the retina. Identified as congenital hypertrophy of the retinal epithelium (CHEPE), this condition is most often associated with mutations in which gene?(2.5分) A.MLH1
B.BRCA1
C.BRCA2
D.APC
E.TP53
D.APC
4 2 2011 5. 起效快、作用强、作用短暂的抗酸药是 (2.5分) A.Calcium sulfate(硫酸钙)
B.Magnesium hydroxide(氢氧化镁)
C.Aluminum hydroxide(氢氧化铝)
D.Magnesium trisilicate(三硅酸镁)
E.Sodium bicarbonate(碳酸氢钠)
E.Sodium bicarbonate(碳酸氢钠)
4 2 2011 6. 抑酸作用最强的药物是 (2.5分) A.Cimetidine
B.Pirenzepine
C.Proglumide(丙谷胺)
D.Omeprazole
E.Sodium bicarbonate(碳酸氢钠)
D.Omeprazole
4 2 2011 7. Omeprazole的作用机制是阻断 (2.5分) A.H2受体
B.M受体
C.质子泵(H+-K+-ATP酶、H+泵)
D.胃泌素受体
E.多巴胺受体
C.质子泵(H+-K+-ATP酶、H+泵)
4 2 2011 8. 对非甾体抗炎药引起的消化性溃疡有特效的药物是 (2.5分) A.Ondansetron
B.Magnesium trisilicate(三硅酸镁)
C.Misoprostol
D.Sucralfate(硫糖铝)
E.Domperidone(多潘立酮)
C.Misoprostol
4 2 2011 9. Domperidone(多潘立酮)的止吐作用是通过阻断 (2.5分) A.5-HT3受体
B.M1受体
C.α1受体
D.多巴胺受体
E.H2受体
D.多巴胺受体
4 2 2011 10. 假性神经递质引起肝性脑病的机制是(2.5分) A.干扰脑的能量代谢
B.使脑细胞产生抑制性突出后电位
C.干扰脑细胞膜的功能
D.与正常递质竞争受体,但其效应远较正常递质弱
E.促进血浆氨基酸失衡
D.与正常递质竞争受体,但其效应远较正常递质弱
4 2 2011 11. 肝功能障碍患者常伴的电解质代谢紊乱主要是(2.5分) A.由放腹水引起
B.由肾功能衰竭引起
C.由胃肠功能衰竭引起
D.由醛固酮.ADH引起
E.应用利尿药引起
E.应用利尿药引起
4 2 2011 12. 肝性脑病时脑组织乙酰胆碱的变化是(2.5分) A.由于肝脏合成胆碱脂酶减少,乙酰胆碱因分解减少而增加
B.血氨增高抑制乙酰胆碱合成而使其减少
C.分解减少与合成减少共同作用,其含量正常
D.血氨使乙酰胆碱分解加速,其含量减少
E.以上都不对
B.血氨增高抑制乙酰胆碱合成而使其减少
4 2 2011 13. 肝性脑病时,血氨升高的原因是(2.5分) A.肠道细菌繁殖分解蛋白质和尿素增加
B.肠内氨经侧支循环直接进入体内
C.体内氨生成增加,清除减少
D.肾脏产生的氨吸收入血增多
E.肝合成尿素减少
C.体内氨生成增加,清除减少
4 2 2011 14. 肝性脑病出现扑翼样震颤的机制是(2.5分) A.氨对脑组织的毒性作用
B.GABA减少
C.乙酰胆碱减少
D.谷氨酸.天门冬氨酸减少
E.假性神经递质取代多巴胺
E.假性神经递质取代多巴胺
4 2 2011 15. 治疗肝性脑病病人可用(2.5分) A.含高浓度酪氨酸的营养液
B.去甲,肾上腺素和多巴胺
C.含有高浓度支链氨基酸的营养液
D.偏碱性的生理溶液
E.含有低浓度蛋氨酸的混合溶液
C.含有高浓度支链氨基酸的营养液
4 2 2011 16. 氨对脑组织的毒性作用是(2.5分) A.干扰脑组织的能量代谢
B.使兴奋性神经递质增多
C.使抑制性神经递质减少
D.刺激神经细胞膜兴奋
E.使NADH增加
A.干扰脑组织的能量代谢
4 2 2011 17. 引起肝性水肿的主要机制是(2.5分) A.血浆胶体渗透压↓
B.组织液胶体渗透压↑
C.门静脉压↑
D.肝静脉回流受阻
E.毛细血管内压↑
C.门静脉压↑
4 2 2011 18. 蜘蛛痣与肝功能障碍时何种激素的灭活障碍有关(2.5分) A.胰岛素
B.胰高血糖素
C.雌激素
D.皮质醇
E.雄激素
C.雌激素
4 2 2011 19. 以下哪项不是慢性胃炎的病因(2.5分) A.幽门螺杆菌(H.pylori)
B.自身免疫
C.十二指肠液反流
D.化脓性
E.胃粘膜损伤因子
D.化脓性
4 2 2011 20. GERD 发病率增加的原因除以下哪一项以外?(2.5分) A.年龄
B.肥胖
C.Hp被根治
D.暴饮暴食
E.睡眠障碍
D.暴饮暴食
4 2 2011 21. 不同反流物对食管黏膜的损伤作用,哪句不正确(2.5分) A.肠液的反流可中和胃酸,减少食管黏膜的损伤
B.食管酸暴露的时间与损伤呈正相关
C.pH<2时损伤作用最强
D.胃蛋白酶的活性在pH<3时最强,pH>4时减弱
A.肠液的反流可中和胃酸,减少食管黏膜的损伤
4 2 2011 22. 关于慢性萎缩性胃炎,下列叙述哪项是错误的(2.5分) A.光镜下病变深达胃壁全层
B.胃黏膜变薄、腺体萎缩、肠上皮化生
C.常见肠上皮化生
D.分A、B型
E.可见假幽门腺化生
A.光镜下病变深达胃壁全层
4 2 2011 23. 下列哪项不是慢性胃炎常见的病因(2.5分) A.滥用非固醇类抗炎药
B.自身免疫性损伤
C.十二指肠液返流
D.EB病毒感染
E.喜食刺激性食物
D.EB病毒感染
4 2 2011 24. 关于慢性浅表性胃炎,下列叙述那项是正确的(2.5分) A.常位于胃前壁,较广泛
B.胃镜下可见充血、出血、糜烂
C.大多转为慢性萎缩性胃炎
D.发病率比慢性萎缩性胃炎低
E.病变局部可有少量腺体萎缩
B.胃镜下可见充血、出血、糜烂
4 2 2011 25. 有关肠上皮化生的一些正确说法是(2.5分) A.发生在肠黏膜的增生上皮
B.慢性浅表性胃炎的组织病理学特点之一
C.胃主细胞被类似肠黏膜上皮的黏液分泌细胞所替代的过程
D.整个胃黏膜上皮被肠黏膜上皮取代的过程
E.与胃癌发生可能有关的一种病变
E.与胃癌发生可能有关的一种病变
4 2 2011 26. 关于IIb型肠上皮化生说法正确的是(2.5分) A.属于完全型化生
B.是胃型化生
C.柱状细胞分泌硫酸黏液
D.杯状细胞分梁羽生 中性黏液
E.刷状缘PAS阳性
C.柱状细胞分泌硫酸黏液
4 2 2011 27. 下述哪句话是正确的(2.5分) A.疣状胃炎的特征是胃黏膜出现多发性糜烂、凹陷,周围黏膜隆起
B.淋巴细胞性胃炎可以大量T淋巴细胞浸润胃黏膜
C.肉芽肿性胃炎黏膜内可以大量肉芽组织形成
D.嗜酸性胃炎时大量嗜酸性粒细胞浸润胃壁全层
E.肥厚性胃炎时黏膜固有层炎症细胞浸润不明显
C.肉芽肿性胃炎黏膜内可以大量肉芽组织形成
4 2 2011 28. 慢性消化性溃疡最常见的部位是(2.5分) A.胃小弯近幽门部
B.十二指肠球部
C.胃与十二指肠复合
D.胃体部
E.十二指肠下段
B.十二指肠球部
4 2 2011 29. 慢性胃溃疡底部的小动脉常可出现何种变化(2.5分) A.细动脉玻璃样变性
B.小动脉硬化
C.增生性动脉内膜炎
D.动脉内血栓形成
E.C+D
E.C+D
4 2 2011 30. 关于十二指肠溃疡,下列哪项是错误的(2.5分) A.比胃溃疡小
B.比胃溃疡浅
C.易癌变
D.易引起腹膜炎、
E.比胃溃疡易穿孔
C.易癌变
4 2 2011 31. 慢性溃疡病发生幽门梗阻的原因,哪项有错误 (2.5分) A.瘢痕收缩
B.因出血形成血块而阻塞
C.因幽门水肿而引起
D.因幽门平滑肌痉挛引起
E.因幽门充血而引起
B.因出血形成血块而阻塞
4 2 2011 32. 十二指肠溃疡的并发症不应有(2.5分) A.出血
B.癌变
C.穿孔
D.幽门梗阻
E.粘连
B.癌变
4 2 2011 33. 早期胃癌是指(2.5分) A.局限于黏膜内的癌
B.无淋巴结转移的癌
C.直径在2cm以内的癌
D.原位癌
E.未侵及肌层的癌
E.未侵及肌层的癌
4 2 2011 34. 关于溃疡型胃癌的描述哪项是错误的(2.5分) A.溃疡形状不规则
B.溃疡通常较浅
C.溃疡周围黏膜皱襞中断
D.属于早期胃癌
E.溃疡直径通常>2厘米
D.属于早期胃癌
4 2 2011 35. 关于胃癌的扩散哪种说法是错误的(2.5分) A.可经腹膜后淋巴管转移形成Krukenberg瘤
B.Krukenberg瘤是胃癌种植转移至卵巢形成
C.不存在"跳跃式"淋巴结转移
D.可不经血道,直接扩散至肝
E.血道转移是最常见的转移方式
E.血道转移是最常见的转移方式
4 2 2011 36. 女性,43岁,12小时以来呕吐出咖啡样物约1200ml。查体:脉搏120次/分,血压80/45mmHg,目前首要的处理是: (2.5分) A.抗休克治疗
B.胃镜检查
C.气管插管术
D.上消化道造影
E.腹部B超
A.抗休克治疗
4 2 2011 37. 外科治疗消化性溃疡的目的是(2.5分) A.防止消化道出血
B.去除溃疡病灶
C.防止溃疡发生恶变
D.彻底治愈溃疡
E.治愈溃疡,消除症状,防止复发
E.治愈溃疡,消除症状,防止复发
4 2 2011 38. 胃大部切除术后并发症中,哪项切忌再次手术?(2.5分) A.十二指肠残端破裂
B.胃肠吻合口破裂
C.胃肠吻合口排空障碍
D.急性完全性输入袢梗阻
E.输出段梗阻
C.胃肠吻合口排空障碍
4 2 2011 39. 十二指肠溃疡宜选择下列哪项手术方法:(2.5分) A.胃肠吻合术
B.全胃切除术
C.胃次全切除术加/或不加迷走神经切断术
D.局部切除术
E.迷走神经干切断术
C.胃次全切除术加/或不加迷走神经切断术
4 2 2011 40. A 55-year-old man complains of chronic intermittent epigastric pain, and gastroscopy demonstrates a 2-cm ulcer of the distal lesser curvature. Endoscopic biopsy yields no malignant tissue. After a 6-wk trial of H 2 blockade and antacid therapy, the ulcer is unchanged. Proper therapy at this point is (2.5分) A.Repeat trial of medical therapy
B.Local excision of the ulcer
C.Billroth I partial gastrectomy
D.Billroth II subtotal gastrectomy with vagotomy
E.Vagotomy and pyloroplasty
C.Billroth I partial gastrectomy
4 2 2012 1. 关于消化管平滑肌基本电节律的正确叙述是 (2.5分) A.是一种超极化波
B.其后一定伴随动作电位
C.是平滑肌收缩节律的控制波
D.在切断支配胃肠的神经后消失
E.以上说法均不对
C.是平滑肌收缩节律的控制波
4 2 2012 2. 迷走神经兴奋时(2.5分) A.胃肠平滑肌活动增强,消化腺分泌减少
B.胃肠平滑肌活动减弱,消化腺分泌增加
C.胃肠平滑肌活动增强,消化腺分泌增加
D.胃肠平滑肌活动减弱,消化腺分泌减少
E.以上都不是
C.胃肠平滑肌活动增强,消化腺分泌增加
4 2 2012 3. 药物的生物转化和排泄速度决定其(2.5分) A.副作用的多少
B.最大效应的高低
C.作用持续时间的长短
D.起效的快慢
E.后遗效应的大小
C.作用持续时间的长短
4 2 2012 4. 对于肾功能低下者,用药时主要考虑(2.5分) A.药物自肾脏的转运
B.药物在肝脏的转化
C.胃肠对药物的吸收
D.药物与血浆蛋白的结合率
E.个体差异
A.药物自肾脏的转运
4 2 2012 5. Which of the following surgical procedures for peptic ulcer disease is the most effective in preventing the dumping syndrome ? (2.5分) A.Truncal vagotomy with pyloroplasty
B.Highly selective vagotomy
C.Antrectomy with vagotomy
D.Subtotal gastrectomy
E.Selective vagotomy
B.Highly selective vagotomy
4 2 2012 6. 瘢痕性幽门梗阻的年老体弱患者,应选用(2.5分) A.迷走神经干切断术+胃窦部切除术
B.高位选择性胃迷走神经切断术
C.胃空肠吻合术
D.毕Ⅰ式胃大部切除术
E.毕Ⅱ式胃大部切除术
C.胃空肠吻合术
4 2 2012 7. 引起上消化道出血最常见的原因是(2.5分) A.胃癌
B.出血性胃炎
C.胃十二指肠溃疡
D.门脉高压症
E.胆道感染出血
C.胃十二指肠溃疡
4 2 2012 8. Sylvia has recently learned she has cancer. She has a family history of this same type of cancer. Her doctor orders microsatellite instability(微卫星不稳定性) testing on tumor tissue after Sylvia's surgery to remove her cancer. What inherited cancer syndrome does her doctor suspect in Sylvia's family? (2.5分) A.Hereditary breast and ovarian cancer
B.Hereditary nonpolyposis colon cancer (HNPCC)
C.Familial adenomatous polyposis (FAP)
D.Li-Fraumeni syndrome
E.Neurofibromatosis
B.Hereditary nonpolyposis colon cancer (HNPCC)
4 2 2012 9. A routine ophthalmologic examination(眼科检查) in a 20-year-old woman demonstrates several well-defined oval, darkly pigmented lesions, one surrounded by a pale halo(晕), on the retina(视网膜). Identified as congenital hypertrophy of the retinal epithelium (CHEPE), this condition is most often associated with mutations in which gene? (2.5分) A.MLH1
B.BRCA1
C.BRCA2
D.APC
E.TP53
D.APC
4 2 2012 10. Genes responsible for hereditary nonpolyposis colon cancer(HNPCC) encode proteins with which of the following function? (2.5分) A.Cell membrane receptor
B.Transcription factor
C.DNA mismatch repair
D.Cell cycle control
E.apoptosis inhibitor
C.DNA mismatch repair
4 2 2012 11. Ondansrtron主要用于治疗 (2.5分) A.化疗、放疗引起的呕吐
B.晕动病引起的呕吐
C.去水吗啡引起的呕吐
D.十二指肠溃疡
E.胃溃疡
A.化疗、放疗引起的呕吐
4 2 2012 12. 对非甾体抗炎药引起的消化性溃疡有特效的药物是 (2.5分) A.Ondansetron
B.Magnesium trisilicate(三硅酸镁)
C.Misoprostol
D.Sucralfate(硫糖铝)
E.Domperidone(多潘立酮)
C.Misoprostol
4 2 2012 13. 抑酸作用最强的药物是 (2.5分) A.Cimetidine
B.Pirenzepine(哌仑西平)
C.Proglumide(丙谷胺)
D.Omeprazole
E.Sodium bicarbonate(碳酸氢钠)
D.Omeprazole
4 2 2012 14. Omeprazole的作用机制是阻断 (2.5分) A.H2受体
B.M受体
C.质子泵(H+-K+-ATP酶、H+泵)
D.胃泌素受体
E.多巴胺受体
C.质子泵(H+-K+-ATP酶、H+泵)
4 2 2012 15. 假性神经递质引起肝性脑病的机制是(2.5分) A.干扰脑的能量代谢
B.使脑细胞产生抑制性突出后电位
C.干扰脑细胞膜的功能
D.与正常递质竞争受体,但其效应远较正常递质弱
E.促进血浆氨基酸失衡
D.与正常递质竞争受体,但其效应远较正常递质弱
4 2 2012 16. 上消化道出血诱发肝性脑病的主要机制是(2.5分) A.引起失血性休克
B.脑组织缺血缺氧
C.经肠道细菌作用而产生氨
D.血液中苯乙胺和酪胺增加
E.破坏血脑屏障,假性神经递质入脑
C.经肠道细菌作用而产生氨
4 2 2012 17. 肝性脑病时,血氨升高的原因是(2.5分) A.肠道细菌繁殖分解蛋白质和尿素增加
B.肠内氨经侧支循环直接进入体内
C.体内氨生成增加,清除减少
D.肾脏产生的氨吸收入血增多
E.肝合成尿素减少
C.体内氨生成增加,清除减少
4 2 2012 18. 下列哪项不是肝性脑病的临床特征(2.5分) A.可以是急性或慢性
B.患者都有昏迷
C.可以是复发性
D.可表现为性格和行为异常
E.有的患者发病可有明显的诱因
B.患者都有昏迷
4 2 2012 19. 氨对脑组织的毒性作用是(2.5分) A.干扰脑组织的能量代谢
B.使兴奋性神经递质增多
C.使抑制性神经递质减少
D.刺激神经细胞膜兴奋
E.使NADH增加
A.干扰脑组织的能量代谢
4 2 2012 20. 碱中毒诱发肝性脑病的原因是(2.5分) A.NH3↓+,NH4+↑
B.NH3↑,NH4+↓
C.NH3↓,NH4+↓
D.NH3↑,NH4+↑
E.NH3不变,NH4+不变
B.NH3↑,NH4+↓
4 2 2012 21. 下列关于Barrett食管的叙述是错误的(2.5分) A.内镜下可表现为齿状线上方数厘米处食管黏膜呈橘红色
B.Barrett食管可发生鳞状细胞癌
C.胃食管反流是Barrett食管形成的主要原因
D.Barrett食管发生食管腺癌的危险性与病灶大小有关
E.组织学上可见鳞状上皮被柱状上皮所取代,若出现杯状细胞便可以确诊
B.Barrett食管可发生鳞状细胞癌
4 2 2012 22. 男,3岁,被推入急诊科,急性病容。表现为胸骨后疼痛,吞咽困难,流涎。生命体征平稳。患儿母亲叙述该男孩曾喝下用于洁厕的清洁液。这位男孩的食管最可能出现下列哪项病理改变(2.5分) A.凋亡
B.凝固性坏死
C.脂肪坏死
D.玻璃样变
E.液化性坏死
B.凝固性坏死
4 2 2012 23. 女,49岁,长期风湿性关节炎,觉乏力,虚弱。在服用非甾体抗炎药(NSAID)后,最近出现黑便,胃镜显示黏膜多处浅表出血。下列哪项是最可能的诊断(2.5分) A.急性糜烂性胃炎
B.早期胃癌
C.幽门螺杆菌胃炎
D.Menetrier病
E.消化性溃疡
A.急性糜烂性胃炎
4 2 2012 24. 下列哪项因素一般与慢性胃炎无关(2.5分) A.自身免疫损伤
B.长期慢性刺激
C.幽门螺杆菌感染
D.十二指肠液反流
E.胃食管反流
E.胃食管反流
4 2 2012 25. 下列哪一种细菌与B型慢性胃炎的发病有关(2.5分) A.葡萄球菌
B.幽门螺杆菌
C.大肠杆菌
D.链球菌
E.青霉菌
B.幽门螺杆菌
4 2 2012 26. 胃溃疡病的病变部位最常见于(2.5分) A.胃后壁
B.胃小弯近贲门处
C.胃前壁
D.胃小弯近幽门处
E.胃大弯近幽门处
D.胃小弯近幽门处
4 2 2012 27. 关于消化性溃疡的结局和合并症的描述,下列哪项是错误的(2.5分) A.再生愈合
B.幽门梗阻
C.穿孔
D.出血
E.癌变
A.再生愈合
4 2 2012 28. 下列关于食管癌的病因正确的是(2.5分) A.亚硝酸类化合物含量高
B.环境中钼、硒、锌等较低
C.长期反流性食管炎
D.人类乳头瘤状病毒感染
E.以上均是
E.以上均是
4 2 2012 29. 关于慢性胃炎的病理描述错误的是(2.5分) A.组织学上表现为炎症、萎缩和化生
B.胃黏膜可表现黏膜肌层增厚、淋巴滤泡形成、纤维组织增生
C.开始时呈灶性分布,随病程发展逐渐融合
D.病理活检诊断通常报告4种变量:炎症、活动性、化生、H.pylori
E.一般胃窦重于胃体,小弯侧重于大弯
D.病理活检诊断通常报告4种变量:炎症、活动性、化生、H.pylori
4 2 2012 30. 胃癌最主要的转移途径是(2.5分) A.直接转移
B.淋巴道转移
C.血道转移
D.腹腔内种植转移
E.神经转移
B.淋巴道转移
4 2 2012 31. 以下哪项不是慢性胃炎的病因(2.5分) A.幽门螺杆菌(H.pylori)
B.自身免疫
C.十二指肠液反流
D.化脓性
E.胃粘膜损伤因子
D.化脓性
4 2 2012 32. 哪项不是侵入性hp的检测方法(2.5分) A.血清抗体检测
B.快速尿素酶(RUT)
C.Giemsa或嗜银染色
D.细菌培养
E.PCR
A.血清抗体检测
4 2 2012 33. TLESR主要通过什么机制引起GERD?(2.5分) A.食管感觉异常
B.抗反流屏障功能下降
C.食管清除能力下降
D.食管粘膜防御作用减弱
E.食管括约肌松弛
B.抗反流屏障功能下降
4 2 2012 34. 常见的食管外症状,除外? (2.5分) A.咳嗽,
B.牙痛
C.哮喘,
D.声音嘶哑,
E.憋气
B.牙痛
4 2 2012 35. 哪项不是临床上可用于GERD诊断的方法有(2.5分) A.食管测压
B.24小时胆汁检测
C.食管内阻抗测定
D.胸部CT
E.PPI 试验
D.胸部CT
4 2 2012 36. which is not the advantages of empirical PPI trial for diagnosing GERD(2.5分) A.the test is office based,
B.easily done,
C.relatively expensive,
D.available to all physicians,
E.avoids many needless procedures
C.relatively expensive,
4 2 2012 37. which is not the lifestyle Modifications for GERD(2.5分) A.Losing weight
B.Elevating the head of the bed,
C.Restricting alcohol and smoking
D.Refraining from lying down after meals,
E.Avoiding bedtime snacks
A.Losing weight
4 2 2012 38. 在我国哪些常用的抗生素对幽门螺杆菌耐药率较高(2.5分) A.四环素
B.甲硝唑
C.呋喃唑酮
D.替硝唑
E.阿莫西林
B.甲硝唑
4 2 2012 39. 胃酸分泌主要是来自__的粘膜上皮?(2.5分) A.胃底
B.胃体
C.胃角
D.幽门
E.胃窦
B.胃体
4 2 2012 40. 对于幽门螺杆菌初次治疗失败的患者再次治疗,一般选用抗菌药物(2.5分) A.三联2周
B.四联10天
C.三联4周
D.四联2周
E.三联10天
B.四联10天
4 2 2013 1. 关于消化管平滑肌基本电节律的正确叙述是(2.5分) A.是一种超极化波
B.其后一定伴随动作电位
C.是平滑肌收缩节律的控制波
D.在切断支配胃肠的神经后消失
E.以上说法均不对
C.是平滑肌收缩节律的控制波
4 2 2013 2. 迷走神经兴奋时(2.5分) A.胃肠平滑肌活动增强,消化腺分泌减少
B.胃肠平滑肌活动减弱,消化腺分泌增加
C.胃肠平滑肌活动增强,消化腺分泌增加
D.胃肠平滑肌活动减弱,消化腺分泌减少
E.以上都不是
C.胃肠平滑肌活动增强,消化腺分泌增加
4 2 2013 3. 药物的生物转化和排泄速度决定其(2.5分) A.副作用的多少
B.最大效应的高低
C.作用持续时间的长短
D.起效的快慢
E.后遗效应的大小
C.作用持续时间的长短
4 2 2013 4. 对于肾功能低下者,用药时主要考虑(2.5分) A.药物自肾脏的转运
B.药物在肝脏的转化
C.胃肠对药物的吸收
D.药物与血浆蛋白的结合率
E.个体差异
A.药物自肾脏的转运
4 2 2013 5. Which of the following surgical procedures for peptic ulcer disease is the most effective in preventing the dumping syndrome ? (2.5分) A.Truncal vagotomy with pyloroplasty
B.Highly selective vagotomy
C.Antrectomy with vagotomy
D.Subtotal gastrectomy
E.Selective vagotomy
B.Highly selective vagotomy
4 2 2013 6. 瘢痕性幽门梗阻的年老体弱患者,应选用(2.5分) A.迷走神经干切断术+胃窦部切除术
B.高位选择性胃迷走神经切断术
C.胃空肠吻合术
D.毕Ⅰ式胃大部切除术
E.毕Ⅱ式胃大部切除术
C.胃空肠吻合术
4 2 2013 7. 引起上消化道出血最常见的原因是(2.5分) A.胃癌
B.出血性胃炎
C.胃十二指肠溃疡
D.门脉高压症
E.胆道感染出血
C.胃十二指肠溃疡
4 2 2013 8. Sylvia has recently learned she has cancer. She has a family history of this same type of cancer. Her doctor orders microsatellite instability(微卫星不稳定性) testing on tumor tissue after Sylvia's surgery to remove her cancer. What inherited cancer syndrome does her doctor suspect in Sylvia's family? (2.5分) A.Hereditary breast and ovarian cancer
B.Hereditary nonpolyposis colon cancer (HNPCC)
C.Familial adenomatous polyposis (FAP)
D.Li-Fraumeni syndrome
E.Neurofibromatosis
B.Hereditary nonpolyposis colon cancer (HNPCC)
4 2 2013 9. A routine ophthalmologic examination(眼科检查) in a 20-year-old woman demonstrates several well-defined oval, darkly pigmented lesions, one surrounded by a pale halo(晕), on the retina(视网膜). Identified as congenital hypertrophy of the retinal epithelium (CHEPE), this condition is most often associated with mutations in which gene? (2.5分) A.MLH1
B.BRCA1
C.BRCA2
D.APC
E.TP53
D.APC
4 2 2013 10. Genes responsible for hereditary nonpolyposis colon cancer(HNPCC) encode proteins with which of the following function? (2.5分) A.Cell membrane receptor
B.Transcription factor
C.DNA mismatch repair
D.Cell cycle control
E.apoptosis inhibitor
C.DNA mismatch repair
4 2 2013 11. Ondansrtron主要用于治疗 (2.5分) A.化疗、放疗引起的呕吐
B.晕动病引起的呕吐
C.去水吗啡引起的呕吐
D.十二指肠溃疡
E.胃溃疡
A.化疗、放疗引起的呕吐
4 2 2013 12. 对非甾体抗炎药引起的消化性溃疡有特效的药物是 (2.5分) A.Ondansetron
B.Magnesium trisilicate(三硅酸镁)
C.Misoprostol
D.Sucralfate(硫糖铝)
E.Domperidone(多潘立酮)
C.Misoprostol
4 2 2013 13. 抑酸作用最强的药物是 (2.5分) A.Cimetidine
B.Pirenzepine(哌仑西平)
C.Proglumide(丙谷胺)
D.Omeprazole
E.Sodium bicarbonate(碳酸氢钠)
D.Omeprazole
4 2 2013 14. Omeprazole的作用机制是阻断 (2.5分) A.H2受体
B.M受体
C.质子泵(H+-K+-ATP酶、H+泵)
D.胃泌素受体
E.多巴胺受体
C.质子泵(H+-K+-ATP酶、H+泵)
4 2 2013 15. 假性神经递质引起肝性脑病的机制是(2.5分) A.干扰脑的能量代谢
B.使脑细胞产生抑制性突出后电位
C.干扰脑细胞膜的功能
D.与正常递质竞争受体,但其效应远较正常递质弱
E.促进血浆氨基酸失衡
D.与正常递质竞争受体,但其效应远较正常递质弱
4 2 2013 16. 上消化道出血诱发肝性脑病的主要机制是(2.5分) A.引起失血性休克
B.脑组织缺血缺氧
C.经肠道细菌作用而产生氨
D.血液中苯乙胺和酪胺增加
E.破坏血脑屏障,假性神经递质入脑
C.经肠道细菌作用而产生氨
4 2 2013 17. 肝性脑病时,血氨升高的原因是(2.5分) A.肠道细菌繁殖分解蛋白质和尿素增加
B.肠内氨经侧支循环直接进入体内
C.体内氨生成增加,清除减少
D.肾脏产生的氨吸收入血增多
E.肝合成尿素减少
C.体内氨生成增加,清除减少
4 2 2013 18. 下列哪项不是肝性脑病的临床特征(2.5分) A.可以是急性或慢性
B.患者都有昏迷
C.可以是复发性
D.可表现为性格和行为异常
E.有的患者发病可有明显的诱因
B.患者都有昏迷
4 2 2013 19. 氨对脑组织的毒性作用是(2.5分) A.干扰脑组织的能量代谢
B.使兴奋性神经递质增多
C.使抑制性神经递质减少
D.刺激神经细胞膜兴奋
E.使NADH增加
A.干扰脑组织的能量代谢
4 2 2013 20. 碱中毒诱发肝性脑病的原因是(2.5分) A.NH3↓+,NH4+↑
B.NH3↑,NH4+↓
C.NH3↓,NH4+↓
D.NH3↑,NH4+↑
E.NH3不变,NH4+不变
B.NH3↑,NH4+↓
4 2 2013 21. 下列关于Barrett食管的叙述是错误的(2.5分) A.内镜下可表现为齿状线上方数厘米处食管黏膜呈橘红色
B.Barrett食管可发生鳞状细胞癌
C.胃食管反流是Barrett食管形成的主要原因
D.Barrett食管发生食管腺癌的危险性与病灶大小有关
E.组织学上可见鳞状上皮被柱状上皮所取代,若出现杯状细胞便可以确诊
B.Barrett食管可发生鳞状细胞癌
4 2 2013 22. 男,3岁,被推入急诊科,急性病容。表现为胸骨后疼痛,吞咽困难,流涎。生命体征平稳。患儿母亲叙述该男孩曾喝下用于洁厕的清洁液。这位男孩的食管最可能出现下列哪项病理改变(2.5分) A.凋亡
B.凝固性坏死
C.脂肪坏死
D.玻璃样变
E.液化性坏死
B.凝固性坏死
4 2 2013 23. 女,49岁,长期风湿性关节炎,觉乏力,虚弱。在服用非甾体抗炎药(NSAID)后,最近出现黑便,胃镜显示黏膜多处浅表出血。下列哪项是最可能的诊断(2.5分) A.急性糜烂性胃炎
B.早期胃癌
C.幽门螺杆菌胃炎
D.Menetrier病
E.消化性溃疡
A.急性糜烂性胃炎
4 2 2013 24. 下列哪项因素一般与慢性胃炎无关(2.5分) A.自身免疫损伤
B.长期慢性刺激
C.幽门螺杆菌感染
D.十二指肠液反流
E.胃食管反流
E.胃食管反流
4 2 2013 25. 下列哪一种细菌与B型慢性胃炎的发病有关(2.5分) A.葡萄球菌
B.幽门螺杆菌
C.大肠杆菌
D.链球菌
E.青霉菌
B.幽门螺杆菌
4 2 2013 26. 胃溃疡病的病变部位最常见于(2.5分) A.胃后壁
B.胃小弯近贲门处
C.胃前壁
D.胃小弯近幽门处
E.胃大弯近幽门处
D.胃小弯近幽门处
4 2 2013 27. 关于消化性溃疡的结局和合并症的描述,下列哪项是错误的(2.5分) A.再生愈合
B.幽门梗阻
C.穿孔
D.出血
E.癌变
A.再生愈合
4 2 2013 28. 下列关于食管癌的病因正确的是(2.5分) A.亚硝酸类化合物含量高
B.环境中钼、硒、锌等较低
C.长期反流性食管炎
D.人类乳头瘤状病毒感染
E.以上均是
E.以上均是
4 2 2013 29. 关于慢性胃炎的病理描述错误的是(2.5分) A.组织学上表现为炎症、萎缩和化生
B.胃黏膜可表现黏膜肌层增厚、淋巴滤泡形成、纤维组织增生
C.开始时呈灶性分布,随病程发展逐渐融合
D.病理活检诊断通常报告4种变量:炎症、活动性、化生、H.pylori
E.一般胃窦重于胃体,小弯侧重于大弯
D.病理活检诊断通常报告4种变量:炎症、活动性、化生、H.pylori
4 2 2013 30. 胃癌最主要的转移途径是(2.5分) A.直接转移
B.淋巴道转移
C.血道转移
D.腹腔内种植转移
E.神经转移
B.淋巴道转移
4 2 2013 31. 以下哪项不是慢性胃炎的病因(2.5分) A.幽门螺杆菌(H.pylori)
B.自身免疫
C.十二指肠液反流
D.化脓性
E.胃粘膜损伤因子
D.化脓性
4 2 2013 32. 哪项不是侵入性hp的检测方法(2.5分) A.血清抗体检测
B.快速尿素酶(RUT)
C.Giemsa或嗜银染色
D.细菌培养
E.PCR
A.血清抗体检测
4 2 2013 33. TLESR主要通过什么机制引起GERD?(2.5分) A.食管感觉异常
B.抗反流屏障功能下降
C.食管清除能力下降
D.食管粘膜防御作用减弱
E.食管括约肌松弛
B.抗反流屏障功能下降
4 2 2013 34. 常见的食管外症状,除外? (2.5分) A.咳嗽,
B.牙痛
C.哮喘,
D.声音嘶哑,
E.憋气
B.牙痛
4 2 2013 35. 哪项不是临床上可用于GERD诊断的方法有(2.5分) A.食管测压
B.24小时胆汁检测
C.食管内阻抗测定
D.胸部CT
E.PPI 试验
D.胸部CT
4 2 2013 36. which is not the advantages of empirical PPI trial for diagnosing GERD: (2.5分) A.the test is office based,
B.easily done,
C.relatively expensive,
D.available to all physicians,
E.avoids many needless procedures
C.relatively expensive,
4 2 2013 37. which is not the lifestyle Modifications for GERD(2.5分) A.Losing weight
B.Elevating the head of the bed,
C.Restricting alcohol and smoking
D.Refraining from lying down after meals,
E.Avoiding bedtime snacks
A.Losing weight
4 2 2013 38. 在我国哪些常用的抗生素对幽门螺杆菌耐药率较高(2.5分) A.四环素
B.甲硝唑
C.呋喃唑酮
D.替硝唑
E.阿莫西林
B.甲硝唑
4 2 2013 39. 胃酸分泌主要是来自__的粘膜上皮?(2.5分) A.胃底
B.胃体
C.胃角
D.幽门
E.胃窦
B.胃体
4 2 2013 40. 对于幽门螺杆菌初次治疗失败的患者再次治疗,一般选用抗菌药物(2.5分) A.三联2周
B.四联10天
C.三联4周
D.四联2周
E.三联10天
B.四联10天
4 2 2014 1. 胃癌发生血行转移的最常见器官是(2.5分) A.肺
B.脑
C.肝
D.肾
E.骨
C.肝
4 2 2014 2. B型慢性萎缩性胃炎好发于(2.5分) A.贲门部
B.胃底
C.胃体
D.胃窦部
E.幽门部
D.胃窦部
4 2 2014 3. 胃癌最主要的转移途径是(2.5分) A.直接扩散
B.淋巴道转移
C.血道转移
D.腹腔内种植
E.盆腔内种植
B.淋巴道转移
4 2 2014 4. Krukenberg瘤是指(2.5分) A.胃黏液癌种植转移至膀胱上
B.肺癌种植转移至胸膜上
C.胃黏液癌种植转移至腹膜上
D.胃黏液癌种植转移至肠壁上
E.胃黏液癌种植转移至卵巢上
E.胃黏液癌种植转移至卵巢上
4 2 2014 5. 下列哪项不是溃疡型胃癌的大体特点? (2.5分) A.边缘隆起,不整齐
B.呈火山口状
C.溃疡直径一般>2cm
D.有出血坏死
E.周围粘膜向溃疡集中呈放射状
E.周围粘膜向溃疡集中呈放射状
4 2 2014 6. 溃疡病的镜下结构不包括(2.5分) A.渗出层
B.坏死层
C.肉芽组织
D.瘢痕组织
E.出血层
E.出血层
4 2 2014 7. 下列哪项是不正确的(2.5分) A.胃溃疡为进食后痛
B.十二指肠溃疡为进食后痛
C.十二指肠溃疡为饥饿痛
D.十二指肠溃疡为夜间痛
E.十二指肠溃疡为空腹痛
B.十二指肠溃疡为进食后痛
4 2 2014 8. 食管癌的类型不包括(2.5分) A.蕈伞型
B.髓质型
C.溃疡型
D.缩窄型
E.弥漫型
E.弥漫型
4 2 2014 9. 食管癌的最常见的好发部位是(2.5分) A.上段
B.中段
C.下段
D.贲门口
E.喉部
B.中段
4 2 2014 10. 革囊胃是指(2.5分) A.慢性胃炎
B.浸润型胃癌
C.溃疡型胃癌
D.结节性型胃癌
E.胃溃疡
B.浸润型胃癌
4 2 2014 11. 下列关于慢性胃炎的病因,错误的是(2.5分) A.幽门螺旋杆菌感染
B.长期慢性刺激
C.胆汁反流
D.长期饥饿
E.自身免疫性损伤
D.长期饥饿
4 2 2014 12. 关于B型胃炎的特点,下列哪项正确? (2.5分) A.与幽门螺旋杆菌感染无关
B.常无恶性贫血
C.多见于胃体部
D.十二指肠溃疡发生率较高
E.可以胃变
B.常无恶性贫血
4 2 2014 13. 引起上消化道出血最常见的原因是(2.5分) A.胃癌
B.胃十二指肠溃疡
C.出血性胃炎
D.门脉高压症
E.胆道感染出血
B.胃十二指肠溃疡
4 2 2014 14. 外科治疗消化性溃疡的目的是(2.5分) A.防止消化道出血
B.去除溃疡病灶
C.防止溃疡发生恶变
D.彻底治愈溃疡
E.治愈溃疡,消除症状,防止复发
E.治愈溃疡,消除症状,防止复发
4 2 2014 15. 胃大部切除术后并发症中,哪项切忌再次手术?(2.5分) A.十二指肠残端破裂
B.胃肠吻合口破裂
C.胃肠吻合口排空障碍
D.急性完全性输入袢梗阻
E.输出段梗阻
C.胃肠吻合口排空障碍
4 2 2014 16. 瘢痕性幽门梗阻的年老体弱患者,应选用(2.5分) A.迷走神经干切断术+胃窦部切除术
B.高位选择性胃迷走神经切断术
C.胃空肠吻合术
D.毕Ⅰ式胃大部切除术
E.毕Ⅱ式胃大部切除术
C.胃空肠吻合术
4 2 2014 17. 以下哪项不是慢性胃炎的病因(2.5分) A.幽门螺杆菌(H.pylori)
B.自身免疫
C.十二指肠液反流
D.化脓性
E.胃粘膜损伤因子
D.化脓性
4 2 2014 18. 哪些不是A型胃炎特点? (2.5分) A.好发于胃体部,
B.壁细胞抗体 PCA阳性
C.内因子抗体 IFA阳性,
D.血胃泌素水平升高
E.胃癌危险性降低
E.胃癌危险性降低
4 2 2014 19. 以下哪项不是萎缩性胃炎癌变监测可以使用的内镜技术? (2.5分) A.超声内镜
B.定标活检技术
C.NBI放大内镜
D.染色胃镜
E.荧光内镜
A.超声内镜
4 2 2014 20. 哪种药物不会引起GERD(2.5分) A.Ca2+拮抗剂
B.硝酸钠
C.利尿剂
D.抗胆碱药
E.b阻滞剂
C.利尿剂
4 2 2014 21. 哪项不是临床上可用于GERD诊断的方法有(2.5分) A.食管测压
B.24小时胆汁检测
C.食管内阻抗测定
D.胸部CT
E.PPI 试验
D.胸部CT
4 2 2014 22. 治疗幽门螺杆菌可能有哪些负面影响(2.5分) A.GERD
B.肥胖
C.类风湿性关节炎
D.哮喘
E.菌群紊乱
C.类风湿性关节炎
4 2 2014 23. 在我国哪些常用的抗生素对幽门螺杆菌耐药率较高(2.5分) A.四环素
B.甲硝唑
C.呋喃唑酮
D.替硝唑
E.阿莫西林
B.甲硝唑
4 2 2014 24. 萎缩性胃炎发生癌变的几率一般认为在(2.5分) A.0.01-0.1%
B.0.1-1%
C.1-2%
D.2-5%
E.>5%
B.0.1-1%
4 2 2014 25. 新悉尼系统提出病理学观察胃粘膜炎症分为几个等级? (2.5分) A.5
B.4
C.3
D.2
E.1
B.4
4 2 2014 26. 统一规范的悉尼胃炎诊断病理学取材要求活检几块? (2.5分) A.5
B.8
C.6
D.2
E.3
A.5
4 2 2014 27. 对于幽门螺杆菌初次治疗失败的患者再次治疗,一般选用抗菌药物(2.5分) A.三联2周
B.四联10天
C.三联4周
D.四联2周
E.三联10天
B.四联10天
4 2 2014 28. 胃食管反流病的治疗的疗程一般是(2.5分) A.2周
B.4周
C.6周
D.8周
E.10周
D.8周
4 2 2014 29. Action mechanism of omeprazole is(2.5分) A.Neutralizing gastric acid
B.Antagonizing H2 receptor
C.Antagonizing M receptor
D.Inhibiting proton pump
E.Protecting gastric mucosa
D.Inhibiting proton pump
4 2 2014 30. Inhibition of basal acid secretion(2.5分) A.Cimetidine (Tagamet)
B.Histamine
C.Sucralfate (Carafate)
D.Colloidal bismuth
E.Ranitidine
A.Cimetidine (Tagamet)
4 2 2014 31. 西咪替丁抑制胃酸分泌的机制是(2.5分) A.阻断M受体
B.保护胃粘膜
C.阻断H1 受体
D.促进PGE2合成
E.阻断H2受体
E.阻断H2受体
4 2 2014 32. 不属于抗消化性溃疡药的是(2.5分) A.乳酶生
B.三硅酸镁
C.西咪替丁
D.哌仑西平
E.雷尼替丁
A.乳酶生
4 2 2014 33. 下列哪一个基因不是抑癌基因? (2.5分) A.APC
B.TP53
C.RB1
D.BRCA1
E.MYC
E.MYC
4 2 2014 34. APC基因位于第几号染色体上? (2.5分) A.5号染色体
B.6号染色体
C.9号染色体
D.11号染色体
E.22号染色体
A.5号染色体
4 2 2014 35. 下列哪一个不是抑癌基因的特征? (2.5分) A.在家系中作为显性性状传递。
B.某些抑癌基因在肿瘤组织中可发生杂合性丢失(loss of heterozygosity,LOH)。
C.致癌方式为显性作用。
D.与致癌作用的二次突变假说("two-hit"model)有关。
E.一般为功能失去(loss of function)突变
C.致癌方式为显性作用。
4 2 2014 36. 对某20岁男性的肠镜检查(colonoscopy)发现,在其大肠上皮表面有数百个息肉。组织活检显示轻度到中度细胞发育异常(cellular dysplasia)。DNA分析最有可能揭示是哪一个基因的突变? (2.5分) A.APC
B.DCC
C.MLH1
D.RAS
E.TP53
A.APC
4 2 2014 37. 严重梗阻性黄疸患者发生低血压倾向的机制是(2.5分) A.交感神经兴奋
B.交感神经抑制
C.迷走神经兴奋
D.迷走神经抑制
E.心血管反应性改变
E.心血管反应性改变
4 2 2014 38. 引起肠源性内毒素血症最主要的因素是(2.5分) A.肝窦血流过少
B.枯否细胞功能受抑制
C.肠壁水肿
D.胆盐排泌障碍
E.枯否细胞功能激活
B.枯否细胞功能受抑制
4 2 2014 39. 引起肝性水肿的主要机制是(2.5分) A.血浆胶体渗透压↓
B.组织液胶体渗透压↑
C.门静脉压↑
D.肝静脉回流受阻
E.毛细血管内压↑
C.门静脉压↑
4 2 2014 40. 治疗肝性脑病病人可用(2.5分) A.含高浓度酪氨酸的营养液
B.去甲,肾上腺素和多巴胺
C.含有高浓度支链氨基酸的营养液
D.偏碱性的生理溶液
E.含有低浓度蛋氨酸的混合溶液
C.含有高浓度支链氨基酸的营养液
4 2 2015 1. 关于消化管平滑肌基本电节律的正确叙述是(2.5分) A.是一种超极化波
B.其后一定伴随动作电位
C.是平滑肌收缩节律的控制波
D.在切断支配胃肠的神经后消失
E.以上说法均不对
C.是平滑肌收缩节律的控制波
4 2 2015 2. 切断支配小肠的迷走神经,可以导致小肠(2.5分) A.紧张性消失,蠕动减弱
B.紧张性减弱,蠕动消失
C.紧张性消失,蠕动消失
D.紧张性和蠕动性均减弱
E.以上都不对
D.紧张性和蠕动性均减弱
4 2 2015 3. 可被阿托品阻断的受体是(2.5分) A.α受体
B.β受体
C.N型受体
D.M型受体
E.N型和M型受体
D.M型受体
4 2 2015 4. 某女性被怀疑罹患了癌症。主管医师在询问了病史后发现,其家族史为阳性。主管医师安排了对术后肿瘤组织进行微卫星不稳定性(microsatellite unstability)的分子病理检测。医师推断该女性患者的家系罹患有哪一种遗传性癌症综合征?(2.5分) A.Hereditary nonpolyposis colon cancer(HNPCC)
B.Hereditary breast and ovarian cancer
C.Familial adenomatous polyposis(FAP)
D.Li-Fraumeni syndrome
E.Neurofibromatosis(神经纤维瘤)
A.Hereditary nonpolyposis colon cancer(HNPCC)
4 2 2015 5. APC基因位于人第几号染色体上? (2.5分) A.第5号染色体
B.第6号染色体
C.第9号染色体
D.第11号染色体
E.第22号染色体
A.第5号染色体
4 2 2015 6. Lynch综合征又称遗传性非息肉性结直肠癌(HNPCC),其致病基因编码的蛋白具有下列哪一种功能?(2.5分) A.细胞膜受体
B.DNA错配修复
C.转录因子
D.细胞周期控制
E.凋亡抑制因子
B.DNA错配修复
4 2 2015 7. 对某20岁男性的肠镜检查(colonoscopy)发现,在其大肠上皮表面有数百个息肉。组织活检显示轻度到中度细胞发育异常(cellular dysplasia)。DNA分析最有可能揭示是哪一个基因的突变?(2.5分) A.APC
B.DCC
C.MLH1
D.RAS
E.TP53
A.APC
4 2 2015 8. Action mechanism of omeprazole is(2.5分) A.Neutralizing gastric acid
B.Antagonizing H2 receptor
C.Antagonizing M receptor
D.Inhibiting proton pump
E.Protecting gastric mucosa
D.Inhibiting proton pump
4 2 2015 9. Which one of the following drugs binds to the mucosa and forms physical barrier to against gastric acid and pepsin? (2.5分) A.Aluminum hydroxide
B.Misoprostol
C.Sucralfate
D.Cimitidine
E.Omeprazole
C.Sucralfate
4 2 2015 10. 不属于抗消化性溃疡药的是:(2.5分) A.乳酶生
B.三硅酸镁
C.西咪替丁
D.哌仑西平
E.雷尼替丁
A.乳酶生
4 2 2015 11. 血氨升高引起肝性脑病的机制是(2.5分) A.影响大脑皮质的兴奋传导过程
B.使脑内形成谷氨酰胺减少
C.抑制大脑边缘系统
D.干扰脑细胞能量代谢
E.使去甲肾上腺素作用减弱
D.干扰脑细胞能量代谢
4 2 2015 12. 假性神经递质引起肝性脑病的机制是(2.5分) A.干扰脑的能量代谢
B.使脑细胞产生抑制性突触后电位
C.干扰脑细胞膜的功能
D.与正常递质竞争受体,但其效应远较正常递质为弱
E.引起血浆氨基酸失衡
D.与正常递质竞争受体,但其效应远较正常递质为弱
4 2 2015 13. 女性,43岁,12小时以来呕吐出咖啡样物约1200ml。查体:脉搏120次/分,血压80/45mmHg,目前首要的处理是:(2.5分) A.抗休克治疗
B.胃镜检查
C.气管插管术
D.上消化道造影
E.腹部B超
A.抗休克治疗
4 2 2015 14. 外科治疗消化性溃疡的目的是(2.5分) A.防止消化道出血
B.去除溃疡病灶
C.防止溃疡发生恶变
D.彻底治愈溃疡
E.治愈溃疡,消除症状,防止复发
E.治愈溃疡,消除症状,防止复发
4 2 2015 15. 胃穿孔的特征性X线表现是(2.5分) A.胃泡增大
B.肠管膨胀
C.膈肌升高
D.膈下游离气体
E.气液平面
D.膈下游离气体
4 2 2015 16. 十二指肠溃疡宜选择下列哪项手术方法(2.5分) A.胃肠吻合术
B.全胃切除术
C.胃次全切除术加/或不加迷走神经切断术
D.局部切除术
E.迷走神经干切断术
C.胃次全切除术加/或不加迷走神经切断术
4 2 2015 17. 瘢痕性幽门梗阻的年老体弱患者,应选用: (2.5分) A.迷走神经干切断术+胃窦部切除术
B.高位选择性胃迷走神经切断术
C.胃空肠吻合术
D.毕Ⅰ式胃大部切除术
E.毕Ⅱ式胃大部切除术
C.胃空肠吻合术
4 2 2015 18. 胃癌发生血行转移的最常见器官是(2.5分) A.肺
B.脑
C.肝
D.肾
E.骨
C.肝
4 2 2015 19. B型慢性萎缩性胃炎好发于(2.5分) A.贲门部
B.胃底
C.胃体
D.胃窦部
E.幽门部
D.胃窦部
4 2 2015 20. 下列哪一项不符合胃癌的转移特点(2.5分) A.经淋巴道转移至锁骨上淋巴结
B.淋巴道扩散至肝门淋巴结
C.晚期经血道转移至肝脏
D.种植性转移至大网膜淋巴结
E.种植性转移至卵巢
D.种植性转移至大网膜淋巴结
4 2 2015 21. Krukenberg瘤是指(2.5分) A.胃黏液癌种植转移至膀胱上
B.肺癌种植转移至胸膜上
C.胃黏液癌种植转移至腹膜上
D.胃黏液癌种植转移至肠壁上
E.胃黏液癌种植转移至卵巢上
E.胃黏液癌种植转移至卵巢上
4 2 2015 22. 下列哪项不是溃疡型胃癌的大体特点?(2.5分) A.边缘隆起,不整齐
B.呈火山口状
C.溃疡直径一般>2cm
D.有出血坏死
E.周围粘膜向溃疡集中呈放射状
E.周围粘膜向溃疡集中呈放射状
4 2 2015 23. 以下哪项不是慢性胃溃疡的形态特点?(2.5分) A.常为单个
B.边缘整齐
C.底部平坦
D.往往较为表浅
E.多位于小弯侧
D.往往较为表浅
4 2 2015 24. 慢性萎缩性胃炎的光镜下特点包括(2.5分) A.粘膜全层内大量淋巴细胞、浆细胞浸润、并见淋巴滤泡形成
B.胃粘膜固有腺体萎缩或消失,并见囊状扩张,壁细胞、主细胞数量减少
C.常伴有肠上皮化生或假幽门腺化生
D.急性活动期可见大量中性粒细胞浸润
E.以上各项
E.以上各项
4 2 2015 25. 溃疡病的合并症不包括:(2.5分) A.出血
B.穿孔
C.幽门梗阻或狭窄
D.癌变
E.息肉形成
E.息肉形成
4 2 2015 26. 食管癌的类型不包括(2.5分) A.蕈伞型
B.髓质型
C.溃疡型
D.缩窄型
E.弥漫型
E.弥漫型
4 2 2015 27. 革囊胃是指:(2.5分) A.慢性胃炎
B.浸润型胃癌
C.溃疡型胃癌
D.结节性型胃癌
E.胃溃疡
B.浸润型胃癌
4 2 2015 28. 急性化脓性阑尾炎的特点不包括下列哪项?(2.5分) A.病变可达肌层及浆膜层
B.各层皆见大量中性粒细胞浸润,并有炎性水肿及纤维素渗出
C.浆膜面可见渗出的纤维素
D.合并腐败细菌感染
E.合并阑尾周围炎及局限性腹膜炎
D.合并腐败细菌感染
4 2 2015 29. 以下哪项不是慢性胃炎的病因(2.5分) A.幽门螺杆菌(H.pylori)
B.自身免疫
C.十二指肠液反流
D.化脓性
E.胃粘膜损伤因子
D.化脓性
4 2 2015 30. 哪些不是A型胃炎特点?(2.5分) A.好发于胃体部,
B.壁细胞抗体 PCA阳性
C.内因子抗体 IFA阳性,
D.血胃泌素水平升高
E.胃癌危险性降低
E.胃癌危险性降低
4 2 2015 31. 以下哪项不是萎缩性胃炎癌变监测可以使用的内镜技术?(2.5分) A.超声内镜
B.定标活检技术
C.NBI放大内镜
D.染色胃镜
E.荧光内镜
A.超声内镜
4 2 2015 32. 不同反流物对食管黏膜的损伤作用,哪句不正确(2.5分) A.肠液的反流可中和胃酸,减少食管黏膜的损伤
B.食管酸暴露的时间与损伤呈正相关
C.pH<2时损伤作用最强
D.胃蛋白酶的活性在pH<3时最强,pH>4时减弱
E.胃排空减缓增加胃食管反流
A.肠液的反流可中和胃酸,减少食管黏膜的损伤
4 2 2015 33. 常见的食管外症状,除外? (2.5分) A.咳嗽,
B.舌痛
C.哮喘,
D.声音嘶哑,
E.憋气
B.舌痛
4 2 2015 34. 出现食管黏膜破损并有2处融合,范围小于75%,属于洛杉矶分型(2.5分) A.A
B.B
C.C
D.D
C.C
4 2 2015 35. Which is not the main symptoms of GERD: (2.5分) A.Heartburn
B.nausea
C.acid regurgitation
D.dysphagia
B.nausea
4 2 2015 36. Which is not the advantages of empirical PPI trial for diagnosing GERD: (2.5分) A.the test is office based,
B.easily done,
C.relatively expensive,
D.available to all physicians,
E.avoids many needless procedures
C.relatively expensive,
4 2 2015 37. which is not the lifestyle Modifications for GERD(2.5分) A.Losing weight
B.Elevating the head of the bed,
C.Restricting alcohol and smoking
D.Refraining from lying down after meals,
E.Avoiding bedtime snacks
A.Losing weight
4 2 2015 38. 根除幽门螺杆菌的最佳时机?(2.5分) A.胃炎发生前
B.萎缩发生前
C.肠化发生前
D.异型增生发生前
E.胃癌发生前
B.萎缩发生前
4 2 2015 39. 萎缩性胃炎发生癌变的几率一般认为在(2.5分) A.0.01-0.1%
B.0.1-1%
C.1-2%
D.2-5%
E.>5%
B.0.1-1%
4 2 2015 40. 慢性胃炎的常见临床表现有一些,除外:(2.5分) A.上腹饱胀
B.嗳气
C.反酸
D.呕吐
E.上腹绞痛
E.上腹绞痛
4 3 2006 1. 我国门脉性肝硬化的常见原因是(2.0分) A.营养缺乏
B.慢性酒精中毒
C.毒物中毒
D.病毒性肝炎
E.药物中毒
D.病毒性肝炎
4 3 2006 2. Which of the following combinations of mutations could be associated with a tumor in the colon?(2.0分) A.Inherited APC mutation and loss of heterozygosity for APC in the tumor
B.Inherited APC mutation and hypermethylation of APC promoter in the tumor
C.A somatic deletion of APC and hypermethylation of the APC promoter in the tumor
D.A somatic nonsense mutation in APC and a somatic deletion of APC
E.Any of the above
E.Any of the above
4 3 2006 3. Which of the following is the largest contributor to cancer risk in the developed world?(1.0分) A.UV exposure
B.Diet
C.Infectious agents
D.Genetic variation
E.Smoking
E.Smoking
4 3 2006 4. ERCP以下哪种情况检查临床价值最大(2.0分) A.大量腹水
B.腹膜有广泛粘连
C.肠梗阻
D.消化性溃疡
E.胆原性胰腺炎
E.胆原性胰腺炎
4 3 2006 5. 关于萎缩性胃炎的病因,下列哪项错误:(2.0分) A.A型与自身免疫有关
B.A型血中有壁细胞抗体
C.B型与自身免疫无关
D.B型血中有内因子抗体
E.A型、B型胃粘膜病变基本一致
D.B型血中有内因子抗体
4 3 2006 6. 食管癌最好发的部位是:(2.0分) A.起始部
B.下段
C.中段
D.上段
E.末端
E.末端
4 3 2006 7. 以下哪项不常见于先天性巨结肠的婴儿(1.0分) A.胎便排出延迟
B.便秘
C.呕血
D.腹胀
E.消瘦
C.呕血
4 3 2006 8. 从腹腔取出的液体具有如下特征:高比重,静置时凝固,混浊且呈黄色,含纤维蛋白原,是下列哪一种原因引起(2.0分) A.腹膜炎
B.右心衰竭
C.门静脉高压
D.饥饿或蛋白丧失
E.以上都不是
A.腹膜炎
4 3 2006 9. 大肠癌的最好发部位是(2.0分) A.回盲部
B.升结肠
C.乙状结肠
D.降结肠
E.直肠
E.直肠
4 3 2006 10. 早期胃癌是指(2.0分) A.仅限于肌层以内
B.仅限于粘膜下层以内
C.仅限于浆膜以内
D.仅限于局部淋巴结转移
E.仅限于扩散至邻近器官
B.仅限于粘膜下层以内
4 3 2006 11. 关于中晚期食管癌的肉眼类型,下列哪项错误:(1.0分) A.髓质型
B.绒毛型
C.溃疡型
D.缩窄型
E.蕈伞型
B.绒毛型
4 3 2006 12. 既有肝细胞的大片坏死,又有肝细胞的结节状再生是下列哪型肝炎:(1.0分) A.亚急性重型
B.急性普通型
C.急性重型
D.轻度慢性肝炎
E.中度慢性肝炎
A.亚急性重型
4 3 2006 13. 目前认为与肝癌发生关系较为密切的原因有(2.0分) A.亚硝胺
B.乙型病毒性肝炎
C.肝硬化
D.黄曲霉毒素
E.以上都是
E.以上都是
4 3 2006 14. 关于胃癌的发生,下列哪项错误:(2.0分) A.与假幽门腺化生关系密切
B.来源于胃腺颈部的干细胞
C.与非典型增生有关
D.与肠上皮化生有关
E.来源于胃小凹底部的干细胞
A.与假幽门腺化生关系密切
4 3 2006 15. SAP常发生低血压或休克,休克的主要原因是 (1.0分) A.中毒
B.有效循环血容量不足
C.感染
D.过敏
E.心力衰竭
B.有效循环血容量不足
4 3 2006 16. 提高胃癌治愈率的关键在于: (1.0分) A.化疗
B.手术
C.放疗
D.手术 化疗 放疗等综合治疗
E.早期诊断
E.早期诊断
4 3 2006 17. 急性普通型肝炎肝细胞常见而普遍的病变是:(2.0分) A.再生
B.变性
C.坏死
D.萎缩
E.化生
B.变性
4 3 2006 18. 乙型肝炎病毒毒力一样,但免疫反应过强的人易发生:(1.0分) A.轻度慢性普通性肝炎
B.急性普通性肝炎
C.不发病,仅为病毒携带者
D.中度慢性普通性肝炎
E.重型肝炎
B.急性普通性肝炎
4 3 2006 19. Sylvia has recently learned she has cancer. She has a family history of this same type of cancer. Her doctor orders microsatellite instability testing on tumor tissue after Sylvia's surgery to remove her cancer. What inherited cancer syndrome does her doctor suspect in Sylvia's family?(2.0分) A.Li-Fraumeni syndrome
B.Hereditary breast and ovarian cancer
C.Hereditary nonpolyposis colon cancer
D.Familial adenomatous polyposis (FAP)
E.Neurofibromatosis
C.Hereditary nonpolyposis colon cancer
4 3 2006 20. 早期胃癌的诊断标准是:(1.0分) A.局部淋巴结无转移
B.癌肿浸润深达肌层
C.癌肿浸润未超过浆膜层
D.癌肿大小不超过2cm
E.瘤组织浸润仅限于粘膜层或粘膜下层
E.瘤组织浸润仅限于粘膜层或粘膜下层
4 3 2006 21. 慢性胃溃疡底部的小动脉常可出现何种变化(1.0分) A.细动脉玻璃样变性
B.增生性动脉内膜炎
C.小动脉硬化
D.动脉内血栓形成
E.C+D
E.C+D
4 3 2006 22. 早期胃癌最多见的类型是:(1.0分) A.表浅平坦型
B.表浅型
C.表浅凹陷型
D.隆起型
E.凹陷型
E.凹陷型
4 3 2006 23. 以下哪项不是先天性巨结肠的婴儿根治术的禁忌条件(1.0分) A.贫血
B.高热
C.营养不良
D.腹胀明显
E.体重<5kg
D.腹胀明显
4 3 2006 24. 胃肠间质瘤(GIST)在消化道最常发生的部位是:(2.0分) A.小肠
B.胃
C.胆道
D.结肠
E.食道
B.胃
4 3 2006 25. 重症急性胰腺炎胰腺坏死时,血液、胰酶及坏死组织液穿过筋膜与肌层渗入腹壁,可见脐周腹部皮肤青紫,预后差,称之为(1.0分) A.Grey-Turner征
B.Gilbert综合症
C.Cullen 征
D.Zollinger-Ellison 综合症
E.Dubin-Johnson综合症
C.Cullen 征
4 3 2006 26. 门脉高压症不包括:(1.0分) A.脾肿大
B.胃肠淤血
C.腹水
D.侧支循环形成
E.黄疸
E.黄疸
4 3 2006 27. 慢性萎缩性胃炎好发于:(2.0分) A.胃大弯
B.胃窦部
C.胃小弯
D.贲门
E.胃底部
B.胃窦部
4 3 2006 28. 关于十二指肠溃疡病变,下列哪项错误:(2.0分) A.多为圆形
B.通常一个
C.大而深
D.球部前壁多见
E.球部后壁亦常见
C.大而深
4 3 2006 29. 下列哪项不属于门脉高压症:(2.0分) A.腹水
B.胃肠道淤血
C.蜘蛛痣
D.脾肿大
E.侧支循环形成
C.蜘蛛痣
4 3 2006 30. 有关食管癌的组织类型,下列哪项最多见:(2.0分) A.未分化癌
B.类癌
C.鳞癌
D.腺癌
E.腺鳞癌
C.鳞癌
4 3 2006 31. 慢性胃炎是:(1.0分) A.细菌引起的胃粘膜的一种特异性炎症
B.病原微生物引起的胃粘膜的一种非特异性炎症
C.支原体引起的胃粘膜的一种特异性炎症
D.病毒引起的胃粘膜的一种特异性炎症
E.不同原因引起的胃粘膜的一种非特异性炎症
E.不同原因引起的胃粘膜的一种非特异性炎症
4 3 2006 32. 下列情况可行ERCP的是(1.0分) A.疑有胆胰疾病
B.疑有肠壁内病变
C.疑有十二指肠水平部憩室
D.下消化道出血原因不明
E.疑有食管源性胸痛
A.疑有胆胰疾病
4 3 2006 33. 上消化道大量出血最常见的原因是:(1.0分) A.慢性浅表性胃炎
B.慢性萎缩性胃炎
C.溃疡病
D.革囊胃
E.巨大肥厚性胃炎
C.溃疡病
4 3 2006 34. 胃溃疡病变最多见的部位是(1.0分) A.大弯近幽门部
B.小弯区近贲门部
C.小弯近幽门部
D.小弯角切迹部
E.贲门
C.小弯近幽门部
4 3 2006 35. 胃癌远处淋巴结转移的常见部位是: (2.0分) A.左颈部
B.右锁骨上
C.右颈部
D.左锁骨上
E.左腹股沟
D.左锁骨上
4 3 2006 36. 慢性胃溃疡肉眼形态通常是(2.0分) A.直径2cm以内圆形溃疡,贲门侧边缘呈阶梯状
B.直径2cm以上火山口或不规则溃疡
C.直径2cm以内圆形溃疡,幽门侧边缘呈阶梯状
D.直径2cm以内圆形溃疡,幽门侧边缘呈潜掘状
E.直径2cm以内圆形溃疡,周围粘膜绉襞排列紊乱
C.直径2cm以内圆形溃疡,幽门侧边缘呈阶梯状
4 3 2006 37. 先天性巨结肠的婴儿,以下哪项组织学检查错误(2.0分) A.痉挛肠段胆碱酯酶活性减少
B.痉挛肠段Ach含量增加
C.可用组化染色神经元特异性稀醇化酶
D.HE染色无神经节细胞
E.可用S-100蛋白组化染色检测
A.痉挛肠段胆碱酯酶活性减少
4 3 2006 38. 诊断急性胰腺炎血清最可靠的诊断方法是:(2.0分) A.淀粉酶增高超过正常上限4倍
B.淀粉酶增高超过正常上限2倍
C.淀粉酶增高超过正常上限3倍
D.淀粉酶增高超过正常上限1倍
E.淀粉酶增高超过正常上限5倍
C.淀粉酶增高超过正常上限3倍
4 3 2006 39. 急性胰腺炎患者饮食应该(2.0分) A.急性期禁食
B.低盐饮食
C.高维生素饮食
D.少量多餐
E.忌辛辣食物
A.急性期禁食
4 3 2006 40. 急性胰腺炎血清淀粉酶在起病多少时间达到高峰(2.0分) A.24小时
B.12小时
C.36小时
D.48小时
E.72小时
D.48小时
4 3 2006 41. 下列哪几项关于细菌性肠炎的发病机制不正确的(1.0分) A.渗出性腹泻
B.渗透性腹泻
C.耐热肠毒素激活鸟苷酸环化酶
D.不耐热肠毒素激活腺苷酸环化酶
E.抑制Na-和水吸收,促进Cl-分泌
B.渗透性腹泻
4 3 2006 42. 我国急性胰腺炎最常见的病因是(1.0分) A.暴饮暴食
B.酒精性
C.高脂血症
D.胆道疾病
E..病毒感染
D.胆道疾病
4 3 2006 43. 急性胰腺炎血清淀粉酶在起病多少时间开始升高(1.0分) A.18~24小时
B.6~12小时
C.12~18小时
D.0~6小时
E.24小时后
B.6~12小时
4 3 2006 44. 有关轮状病毒肠炎的发病机制,以下说法错误的是(2.0分) A.病毒侵入肠黏膜使绒毛受累、破坏
B.主要累及小肠
C.cAMP增加
D.继发双糖酶活性下降
E.NSP4作用
C.cAMP增加
4 3 2006 45. 急性胰腺炎临床诊断以血清淀粉酶为主,尿淀粉酶(2.0分) A.没有临床诊断价值
B.有重要临床诊断价值
C.有极大临床诊断价值
D.可用于确诊
E.仅作参考
E.仅作参考
4 3 2006 46. GIST发生的分子机制是下列哪一基因激活突变:(2.0分) A.c-erb2
B.c-met
C.c-kit
D.k-ras
E.c-myc
C.c-kit
4 3 2006 47. 诊断胃十二指肠肿瘤最可靠的诊断方法是: (1.0分) A.上腹部MRI
B.上腹部螺旋CT
C.胃肠造影
D.胃镜和超声内镜
E.血清肿瘤标记物检测
D.胃镜和超声内镜
4 3 2006 48. 胰腺假性囊肿多在急性胰腺炎起病2周后发生,多发生于(2.0分) A.胰头、胰体
B.胰头
C.胰体
D.胰体、胰尾
E.胰尾
D.胰体、胰尾
4 3 2006 49. 判断GIST临床预后最重要的指标是: (1.0分) A.肿瘤大小和细胞分裂数
B.肿瘤部位和年龄
C.肿瘤大小和部位
D.细胞核分裂数和肿瘤细胞多形性的程度
E.细胞核分裂数量和肿瘤细胞中梭形细胞的比例
A.肿瘤大小和细胞分裂数
4 3 2006 50. 先天性肥厚性幽门狭窄的特有体征为(1.0分) A.黄疸
B.消瘦
C.右上腹肿块
D.胃蠕动波
E.脱水
C.右上腹肿块
4 3 2006 51. 先天性肥厚性幽门狭窄的首选检查为(2.0分) A.B超
B.腹部平片
C.血气及电解质分析
D.上消化道钡餐
E.腹部CT
A.B超
4 3 2006 52. 下列病变中癌变可能性较大的是:(1.0分) A.慢性萎缩性胃炎
B.浅表性胃炎
C.肥厚性胃炎
D.十二指肠溃疡
E.疣状胃炎
A.慢性萎缩性胃炎
4 3 2006 53. 先天性肥厚性幽门狭窄的典型呕吐为(2.0分) A.呕吐含粪汁样物
B.呕血
C.出生即有呕吐
D.无胆汁的喷射性呕吐
E.溢乳样
D.无胆汁的喷射性呕吐
4 3 2006 54. 先天性巨结肠的婴儿,主要病变在哪个肠段(2.0分) A.移行段近端
B.痉挛段
C.移行段
D.扩张段
E.扩张段近端
B.痉挛段
4 3 2006 55. 下列不属于胃癌危险因素的是:(2.0分) A.胃腺瘤性息肉
B.恶性贫血
C.慢性浅表性胃炎
D.慢性萎缩性胃炎
E.胃部手术史
C.慢性浅表性胃炎
4 3 2006 56. 先天性巨结肠的并发症通常发生于生后(2.0分) A.6月内
B.2月内
C.12月
D.1周内
E.2周岁
B.2月内
4 3 2006 57. 胃癌最主要的转移途径是:(1.0分) A.种植转移
B.淋巴道转移
C.血液转移
D.直接蔓延
E.医源性转移
B.淋巴道转移
4 3 2006 58. 先天性巨结肠的婴儿基本的病理变化是(2.0分) A.脱髓鞘病变
B.肠管痉挛
C.黏膜炎症
D.神经节细胞缺乏
E.副交感神经纤维减少
D.神经节细胞缺乏
4 3 2006 59. Which of the following is NOT a characteristic of tumor suppressor genes?(2.0分) A.Act dominantly in tumor cells.
B.May be associated with loss of heterozygosity in tumors.
C.Transmitted as dominant traits in families.
D.Associated with "two-hit" model of carcinogenesis.
A.Act dominantly in tumor cells.
4 3 2006 60. 关于细菌性肠炎的描述,不正确的是(2.0分) A.粪便常规检查可无异常发现
B.粘液脓血便
C.水样便
D.可以有发热
E.禁食后腹泻即可明显缓解
E.禁食后腹泻即可明显缓解
4 3 2006 61. 下列哪种细菌性不会引起侵袭性肠炎(1.0分) A.空肠弯曲菌
B.耶尔森菌
C.金黄色葡萄球菌
D.志贺氏菌
E.致病性大肠杆菌
E.致病性大肠杆菌
4 3 2006 62. 下列哪项不是重型腹泻的临床表现(2.0分) A.嗜睡
B.电解质紊乱
C.明显脱水
D.粪便呈稀水样
E.休克
D.粪便呈稀水样
4 3 2006 1. 婴幼儿易患腹泻病,主要与下列易感因素有关(2.0分) A.肠道菌群易失调
B.机体防御功能差
C.婴幼儿消化系统发育未成熟
D.生长发育快,胃肠道负担重
E.人工喂养
A.肠道菌群易失调
B.机体防御功能差
C.婴幼儿消化系统发育未成熟
D.生长发育快,胃肠道负担重
E.人工喂养
4 3 2007 1. 坏死性胰腺炎诊断胰腺坏死的最佳方法是(2.0分) A.血清淀粉酶检测
B.B超检查
C.增强CT检查
D.MRI检查
E.MRCP检查
C.增强CT检查
4 3 2007 2. 如果尿淀粉酶升高而血淀粉酶正常,应该考虑(2.0分) A.Munchausen 综合症
B.Zollinger-Ellison 综合症
C.Cullen 征
D.Gilbert综合症
E.Dubin-Johnson综合症
A.Munchausen 综合症
4 3 2007 3. Which of the following is the largest contributor to cancer risk in the developed world?(2.0分) A.Infectious agents
B.Diet
C.Genetic variation
D.UV exposure
E.Smoking
E.Smoking
4 3 2007 4. 急性胰腺炎临床诊断以血清淀粉酶为主,尿淀粉酶(2.0分) A.可用于确诊
B.有重要临床诊断价值
C.没有临床诊断价值
D.有极大临床诊断价值
E.仅作参考
E.仅作参考
4 3 2007 5. 毛玻璃样肝细胞内嗜酸性颗粒的性质是:(2.0分) A.嗜酸性坏死
B.包涵体
C.细胞内玻变
D.乙型肝炎表面抗原
E.肿胀的内质网和线粒体
D.乙型肝炎表面抗原
4 3 2007 6. Which of the following combinations of mutations could be associated with a tumor in the colon?(2.0分) A.A somatic deletion of APC and hypermethylation of the APC promoter in the tumor
B.Inherited APC mutation and loss of heterozygosity for APC in the tumor
C.Inherited APC mutation and hypermethylation of APC promoter in the tumor
D.A somatic nonsense mutation in APC and a somatic deletion of APC
E.Any of the above
E.Any of the above
4 3 2007 7. Which of the following is NOT a characteristic of tumor suppressor genes?(2.0分) A.Act dominantly in tumor cells.
B.May be associated with loss of heterozygosity in tumors.
C.Transmitted as dominant traits in families.
D.Associated with "two-hit" model of carcinogenesis.
A.Act dominantly in tumor cells.
4 3 2007 8. 目前诊断胃癌可靠的手段是:(2.0分) A.X线钡餐检查和胃镜加活检
B.胃液分析
C.粪便隐血试验
D.幽门螺杆菌感染检查
E.胃癌血清标志物
A.X线钡餐检查和胃镜加活检
4 3 2007 9. A routine ophthalmologic examination in a 20-year-old woman demonstrates several well-defined oval, darkly pigmented lesions, one surrounded by a pale halo, on the retina. Identified as congenital hypertrophy of the retinal epithelium (CHEPE), this condit(2.0分) A.BRCA1
B.MLH1
C.BRCA2
D.APC
E.TP53
D.APC
4 3 2007 10. 先天性肥厚性幽门狭窄的首选检查为(2.0分) A.上消化道钡餐
B.血气及电解质分析
C.腹部平片
D.B超
E.腹部CT
D.B超
4 3 2007 11. 胰腺癌最常见的组织类型是(2.0分) A.纤毛细胞癌
B.多形性腺癌
C.腺泡细胞癌
D.导管细胞癌
E.黏液癌
D.导管细胞癌
4 3 2007 12. 先天性肥厚性幽门狭窄的主要治疗方法为(2.0分) A.对症治疗
B.加强营养
C.空肠喂养
D.手术
E.解痉药物
D.手术
4 3 2007 13. 胃溃疡病变部位最常见于:(2.0分) A.胃后壁
B.胃大弯及胃底
C.胃小弯近贲门处
D.胃前壁
E.胃小弯近幽门处
E.胃小弯近幽门处
4 3 2007 14. CRP 检测以下哪项提示胰腺广泛坏死(2.0分) A.CRP大于170mg/L
B.CRP大于130mg/L
C.CRP大于150mg/L
D.CRP大于100mg/L
E.CRP大于200mg/L
C.CRP大于150mg/L
4 3 2007 15. 哪项最不适合作为先天性肥厚性幽门狭窄的辅助检查(2.0分) A.B超
B.X线钡餐
C.24h食管pH值测定
D.腹部平片
E.上消化道内镜
D.腹部平片
4 3 2007 16. 以下哪项不是先天性巨结肠的婴儿根治术的禁忌条件(2.0分) A.营养不良
B.高热
C.腹胀明显
D.贫血
E.体重<5kg
C.腹胀明显
4 3 2007 17. 先天性巨结肠的婴儿,以下哪项不是其X线表现(2.0分) A.可显示痉挛段、移行段、扩张段
B.盆腔大量充气
C.低位不全肠梗阻
D.可见粘膜皱襞变粗
E.钡剂排出困难
B.盆腔大量充气
4 3 2007 18. 男性,42岁,上腹烧灼样痛半年,近一周排柏油便,为确诊,宜首选下列哪项检查方法:(2.0分) A.B型超声检查
B.X线钡灌肠透视检查
C.便潜血检查
D.X线胃肠钡餐透视
E.胃镜检查
E.胃镜检查
4 3 2007 19. 轮状病毒腹泻病人补液过程中突然出现抽搐,最不可能的原因为(2.0分) A.低镁血症
B.低钙血症
C.脑水肿
D.脑炎
E.颅内出血
D.脑炎
4 3 2007 20. 有关急性腹泻病人的饮食疗法中错误的是(2.0分) A.腹泻时应予以营养丰富饮食有利于腹泻恢复
B.严重呕吐可以暂时禁食
C.疑有继发性双糖酶缺乏,暂停乳类喂养
D.强调继续饮食
E.母乳喂养者继续母乳喂养
A.腹泻时应予以营养丰富饮食有利于腹泻恢复
4 3 2007 21. 有关急性腹泻病人的液体疗法中错误的是(2.0分) A.累积损失量通常在12-16小时内补足
B.判断脱水性质有困难时可先按等渗性脱水处理
C.等渗性脱水累积损失量的补充可予1/2张含钠液
D.累积损失量应根据脱水程度及性质补充
E.累积损失量补充约为8~10ml/kg/h
A.累积损失量通常在12-16小时内补足
4 3 2007 22. 早期肝癌是指:(2.0分) A.肿瘤直径或直径之和小于3cm,数目不超过2个
B.单个肿瘤,直径小于4cm
C.肿瘤直径或直径之和小于2cm,数目不超过3个
D.肿瘤直径或直径之和小于1cm,数目不超过4个
E.单个肿瘤,直径小于5cm,未发生转移
A.肿瘤直径或直径之和小于3cm,数目不超过2个
4 3 2007 23. 胃癌主要起源于:(2.0分) A.主细胞
B.壁细胞
C.幽门腺上皮细胞
D.腺颈部和胃小凹底部干细胞
E.间质细胞
D.腺颈部和胃小凹底部干细胞
4 3 2007 24. 早期胃癌的概念是:(2.0分) A.局限于胃窦内
B.无淋巴结转移
C.直径在2CM以内
D.局限于粘膜及粘膜下层
E.尚未侵及浆膜层
D.局限于粘膜及粘膜下层
4 3 2007 25. 提高胃癌治愈的关键在于:(2.0分) A.彻底手术
B.早期诊断
C.积极放疗
D.早期化疗
E.综合治疗
B.早期诊断
4 3 2007 26. 下列哪项不是胃溃疡的并发症(2.0分) A.出血
B.穿孔
C.反复发作
D.癌变
E.幽门狭窄
C.反复发作
4 3 2007 27. 胃溃疡病的合并症最常见的是:(2.0分) A.梗阻
B.穿孔
C.癌变
D.出血
E.粘连
D.出血
4 3 2007 28. 临床怀疑胰腺癌,首先的影像学检查方法是(2.0分) A.CT
B.超声
C.MRI
D.血管造影
E.ERCP
B.超声
4 3 2007 29. 关于急性普通型病毒性肝炎的病变下列哪项是错误的(2.0分) A.结缔组织增生
B.气球样变
C.嗜酸性变
D.肝细胞再生
E.溶解性坏死
A.结缔组织增生
4 3 2007 30. 男,40岁,上腹隐痛不适,近2个月来加剧,服胃痛片后有所缓解,食欲尚可,大便隐血试验(++),胃肠道钡餐检查见胃窦部小弯侧粘膜纹理紊乱,胃壁僵直不规则。首先应考虑: (2.0分) A.慢性胃窦炎
B.胃癌
C.胃溃疡
D.胃粘膜脱垂
E.萎缩性胃炎
B.胃癌
4 3 2007 31. 下列哪项不是先天性肥厚性幽门狭窄的典型B超特征(2.0分) A.幽门直径≥13mm
B.幽门肌厚度≥4mm
C.幽门胃窦"鸟嘴状"
D.幽门环肌低回声区
E.幽门管长≥17mm
C.幽门胃窦"鸟嘴状"
4 3 2007 32. 慢性萎缩性胃炎好发于(2.0分) A.胃小弯
B.胃大弯
C.胃窦部
D.贲门
E.胃底部
C.胃窦部
4 3 2007 33. 门脉高压症不包括:(2.0分) A.腹水
B.侧支循环形成
C.脾肿大
D.胃肠淤血
E.黄疸
E.黄疸
4 3 2007 34. 下列有关先天性肥厚性幽门狭窄的叙述错误的是(2.0分) A.男性多见
B.第一胎多见
C.占消化道畸形第三位
D.足月儿多见
E.多见于早产儿
E.多见于早产儿
4 3 2007 35. 下列哪项不属于门脉高压症:(2.0分) A.脾肿大
B.胃肠道淤血
C.腹水
D.蜘蛛痣
E.侧支循环形成
D.蜘蛛痣
4 3 2007 36. 关于胰腺癌黄疸不正确的是(2.0分) A.大部分病人出现黄疸时已属中晚期
B.黄疸时间长者可有出血倾向
C.黄疸出现早晚与癌肿部位有关
D.黄疸呈进行性加重,病人均有瘙痒
E.深度黄疸时大便可被染成浅黄色
D.黄疸呈进行性加重,病人均有瘙痒
4 3 2007 37. 十二指肠溃疡主要表现为:(2.0分) A.后壁之溃疡易穿孔
B.溃疡大小多为1cm以上
C.前壁之溃疡易出血
D.溃疡位置多在十二指肠降部
E.以上都不是
E.以上都不是
4 3 2007 38. 急性普通型病毒性肝炎的坏死多为(2.0分) A.大片坏死
B.凝固性坏死
C.桥接坏死
D.碎片状坏死
E.点状坏死
E.点状坏死
4 3 2007 39. 术中发现胰头部一轮廓不规则肿物,边界尚清,与周围组织无粘连,门静脉、肠系膜上静脉均完整,无破坏,宜行(2.0分) A.Whipple胰头十二脂肠切除术
B.胆囊十二指肠吻合术
C.全胰切除术
D.扩大切除术
E.胆总管十二指肠吻合术
A.Whipple胰头十二脂肠切除术
4 3 2007 40. 哪项不是先天性肥厚性幽门狭窄的血气电解质分析结果(2.0分) A.低氯血症
B.低钾血症
C.碱中毒
D.乳酸血症
E.高钾血症
E.高钾血症
4 3 2007 41. 恶性程度较高的胃癌可以超越常规所属淋巴结转移方式,而直接侵犯远处淋巴结,其中最常见为:(2.0分) A.腹腔淋巴结
B.肝门淋巴结
C.胰脾淋巴结
D.锁骨上淋巴结
E.直肠上动脉淋巴结
D.锁骨上淋巴结
4 3 2007 42. 关于胃癌的淋巴转移,下列哪项是正确的?(2.0分) A.胃癌浸润深度与淋巴结转移无相关关系
B.恶性程度较高或较晚期的胃癌可转移至锁骨上淋巴结
C.无跳跃式淋巴结转移
D.淋巴转移不是胃癌的主要转移途径
E.区域淋巴结转移部位与原发肿瘤的部位无关
B.恶性程度较高或较晚期的胃癌可转移至锁骨上淋巴结
4 3 2007 43. 腹泻病人导致酸中毒的原因不包括下列哪项(2.0分) A.腹泻致碱性物质丢失
B.肾脏排酸、保钠功能低下
C.组织缺氧,乳酸堆积
D.能量供应不足,脂肪分解增加
E.低钾血症
E.低钾血症
4 3 2007 44. Genes responsible for hereditary nonpolyposis colon cancer encode proteins with which of the following function?(2.0分) A.DNA mismatch repair
B.Transcription factor
C.Cell membrane receptor
D.Cell cycle control
A.DNA mismatch repair
4 3 2007 45. 从腹腔取出的液体具有如下特征:高比重,静置时凝固,混浊且呈黄色,含纤维蛋白原,是下列哪一种原因引起(2.0分) A.饥饿或蛋白丧失
B.右心衰竭
C.腹膜炎
D.门静脉高压
E.以上都不是
C.腹膜炎
4 3 2007 46. 行胃癌根治术时,手术切除最少应离癌肿边缘多远才算足够?(2.0分) A.6-8cm
B.4--6cm
C.1-2cm
D.2-4cm
E.8--10cm
A.6-8cm
4 3 2007 47. 关于胰岛外分泌物质,下列哪项是错误的(2.0分) A.胃泌素
B.淀粉酶
C.碳酸氢盐
D.脂肪酶
E.淀粉酶
A.胃泌素
4 3 2007 48. 我国门脉性肝硬化的常见原因是:(2.0分) A.营养缺乏
B.毒物中毒
C.病毒性肝炎
D.慢性酒精中毒
E.药物中毒
C.病毒性肝炎
4 3 2007 49. 胃腺癌最好发生在:(2.0分) A.胃窦部
B.胃体大弯
C.胃体小弯
D.贲门部
E.全胃
A.胃窦部
4 3 2007 50. 胃窦部肿块型胃癌约6×4×4CM大小,已累及浆膜层,CT检查左肝外叶亦有3CM大小转移灶,胰腺正常。该病人的治疗应选择: (2.0分) A.根治性胃大部切除术
B.已无法手术,行全身化学疗法
C.根治性胃大部分切除术+左肝动脉栓塞术
D.根治性胃大部切除+左肝外叶切除术
E.胃空肠吻合
D.根治性胃大部切除+左肝外叶切除术
4 3 2008 1. 以下哪些因素可以引起食管癌?(2.0分) A.饮酒
B.过热食物
C.腌制食品
D.食物中缺乏钼
E.以上都是
E.以上都是
4 3 2008 2. 食管癌的典型临床表现是:(2.0分) A.轻度下咽不适
B.胸骨后烧灼感
C.进行性吞咽困难
D.咽部异物感
E.轻度哽噎
C.进行性吞咽困难
4 3 2008 3. 根据2009年第七版AJCC/UICC食管鳞癌分期标准,T1-3 N2 M0 属哪一期?(2.0分) A.IA 期
B.IIB期
C.IV期
D. IIIC期
E.IIIA期
E.IIIA期
4 3 2008 4. 食道X线钡餐造影出现龛影提示哪一种病理类型肿瘤?(2.0分) A.髓质型
B.溃疡型
C.覃伞型
D.腔内型
E.缩窄型
B.溃疡型
4 3 2008 5. 以下哪项不是食管癌根治术(胃食道吻合)必须的操作内容?(2.0分) A.食管癌切除
B.淋巴结清扫
C.游离胃
D.胃食管吻合
E.胸导管结扎
E.胸导管结扎
4 3 2008 6. 恶性程度较高的胃癌可以超越常规所属淋巴结转移方式,而直接侵犯远处淋巴结,其中最常见为:(2.0分) A.胰脾淋巴结
B.肝门淋巴结
C.腹腔淋巴结
D.锁骨上淋巴结
E.直肠上动脉淋巴结
D.锁骨上淋巴结
4 3 2008 7. 下列哪项最能提示胃溃疡恶变?(2.0分) A.多发溃疡
B.复合溃疡
C.胃小弯侧溃疡
D.溃疡>2CM
E.溃疡位于胃腔廓之外,周围粘膜呈星状聚合
E.溃疡位于胃腔廓之外,周围粘膜呈星状聚合
4 3 2008 8. 提高胃癌治愈的关键在于(2.0分) A.早期诊断
B.彻底手术
C.积极放疗
D.早期化疗
E.综合治疗
A.早期诊断
4 3 2008 9. 关于胃癌的治疗,下列哪项是错误的?(2.0分) A.侵犯浆肌层的胃癌3.5.10年生存率较早期胃癌小50%以上
B.根治手术是最有效的治疗方法
C.常用的化疗药物为5-Fu及自力霉素
D.若胃癌已有肝左外叶浸润不应作根治术
E.如无远处转移,一般争取尽早手术
D.若胃癌已有肝左外叶浸润不应作根治术
4 3 2008 10. 关于胃癌,下列叙述哪项不正确?(2.0分) A.早期胃癌是指局限在粘膜和粘膜下层的胃癌
B.小胃癌是指病灶0.6-1.0cm的胃癌
C.早期胃癌均无淋巴结转移
D.微小胃癌是指直径≤0.5cm的胃癌
E.小胃癌和微小胃癌根治后5年存活率几乎达100
C.早期胃癌均无淋巴结转移
4 3 2008 11. 胃癌的癌前期状态不包括:(2.0分) A.胃息肉
B.胃平滑肌瘤
C.慢性萎缩性胃炎
D.胃酸缺乏症
E.恶性贫血者
B.胃平滑肌瘤
4 3 2008 12. 行胃癌根治术时,手术切除最少应离癌肿边缘多远才算足够?(2.0分) A.1-2cm
B.2-4cm
C.4--6cm
D.6-8cm
E.8--10cm
D.6-8cm
4 3 2008 13. 胃腺癌最好发生在:(2.0分) A.贲门部
B.胃体大弯
C.胃体小弯
D.胃窦部
E.全胃
D.胃窦部
4 3 2008 14. 胃癌的下列组织病理学分型,哪一型恶性程度最高:(2.0分) A.管状腺癌
B.粘液腺癌
C.印戒细胞癌
D.弥漫型癌
E.髓质癌
D.弥漫型癌
4 3 2008 15. 胃癌血行播散最常转移到:(2.0分) A.肝脏
B.腹膜
C.肺脏
D.肾脏
E.卵巢
A.肝脏
4 3 2008 16. Trousseau征是指胃癌患者伴有:(2.0分) A.黑棘皮病
B.反复发作性血栓性静脉炎
C.皮肌炎
D.微血管病性溶血性贫血
E.远处淋巴结转移
B.反复发作性血栓性静脉炎
4 3 2008 17. 便潜血试验呈持续阳性常提示:(2.0分) A.急性胃粘膜病变
B.胃溃疡
C.十二指肠球部溃疡
D.慢性萎缩性胃炎
E.胃癌
E.胃癌
4 3 2008 18. 男性,42岁,上腹烧灼样痛半年,近一周排柏油便,为确诊,宜首选下列哪项检查方法:(2.0分) A.X线胃肠钡餐透视
B.X线钡灌肠透视检查
C.便潜血检查
D.B型超声检查
E.胃镜检查
E.胃镜检查
4 3 2008 19. 胃窦部肿块型胃癌约6×4×4CM大小,已累及浆膜层,CT检查左肝外叶亦有3CM大小转移灶,胰腺正常。该病人的治疗应选择: (2.0分) A.已无法手术,行全身化学疗法
B.根治性胃大部切除术
C.根治性胃大部分切除术+左肝动脉栓塞术
D.根治性胃大部切除+左肝外叶切除术
E.胃空肠吻合术
D.根治性胃大部切除+左肝外叶切除术
4 3 2008 20. 男,48岁,因胃癌而行剖腹探查时,发现肿块已浸润至横结肠,但较局限,该病人宜: (2.0分) A.禁忌手术
B.行胃癌根治术+横结肠切除术
C.行胃大部切除术
D.行全胃切除术
E.行横结肠切除术
B.行胃癌根治术+横结肠切除术
4 3 2008 21. 胃癌主要起源于:(2.0分) A.主细胞
B.壁细胞
C.腺颈部和胃小凹底部干细胞幽门腺上皮细胞
D.间质细胞
D.间质细胞
4 3 2008 22. Krukenberg氏瘤是指(2.0分) A.卵巢的交界性粘液性囊腺瘤
B.卵巢腺癌
C.卵巢粘液性腺囊癌
D.腺癌伴广泛转移
E.卵巢的转移性粘液腺癌
E.卵巢的转移性粘液腺癌
4 3 2008 23. 与胃癌发生关系最为密切的化生类型是:(2.0分) A.不完全小肠型肠化
B.不完全大肠型肠化
C.完全小肠型肠化
D.完全大肠型肠化
E.假幽门腺化生
B.不完全大肠型肠化
4 3 2008 24. 胃溃疡病变部位最常见于:(2.0分) A.胃前壁
B.胃后壁
C.胃大弯及胃底
D.胃小弯近贲门处
E.胃小弯近幽门处
E.胃小弯近幽门处
4 3 2008 25. 毛玻璃样肝细胞内嗜酸性颗粒的性质是:(2.0分) A.包涵体
B.嗜酸性坏死
C.细胞内玻变
D.乙型肝炎表面抗原
E.肿胀的内质网和线粒体
D.乙型肝炎表面抗原
4 3 2008 26. 门脉高压症不包括:(2.0分) A.胃肠淤血
B.脾肿大
C.腹水
D.侧支循环形成
E.黄疸
E.黄疸
4 3 2008 27. 与慢性胃炎、胃溃疡和胃癌的发生有密切关系的病原体是:(2.0分) A.金黄色葡萄球菌
B.草绿色链球菌
C.大肠杆菌
D.黄曲霉菌
E.幽门螺杆菌
E.幽门螺杆菌
4 3 2008 28. 急性普通型肝炎肝细胞一般不出现:(2.0分) A..肝细胞疏松肿胀
B.气球样变
C.碎片状坏死
D.肝细胞点状坏死
E.汇管区炎细胞浸润
C.碎片状坏死
4 3 2008 29. 急性重型肝炎与亚急性重型肝炎肉眼观形态主要区别是:(2.0分) A.有无再生结节形成
B.肝体积缩小的程度
C.肝包膜皱缩的程度
D.肝实质黄染的程度
E.出血程度
A.有无再生结节形成
4 3 2008 30. 早期胃癌IIc型是指:(2.0分) A.表浅隆起型
B.表浅平坦型
C.表浅凹陷型
D.隆起型
E.凹陷型
C.表浅凹陷型
4 3 2008 31. Fatty change is most likely appeared in hepatitis of:(2.0分) A.HAV
B.HBV
C.HCV
D. HDV
E.HEV
C.HCV
4 3 2008 32. Carcinomas of the large intestine most frequently located in:(2.0分) A.rectum
B.sigmoid colon
C.cecum
D. ascending colon
E.descending colon
A.rectum
4 3 2008 33. Krukenberg瘤是指(2.0分) A.卵巢的交界性粘液性囊腺瘤
B.卵巢腺癌
C.卵巢粘液性腺囊癌
D.腺癌伴广泛转移
E.卵巢的转移性粘液腺癌
E.卵巢的转移性粘液腺癌
4 3 2008 34. 早期肝癌是指:(2.0分) A.肿瘤直径或直径之和小于1cm,数目不超过4个
B.肿瘤直径或直径之和小于2cm,数目不超过3个
C.肿瘤直径或直径之和小于3cm,数目不超过2个
D.单个肿瘤,直径小于4cm
E.单个肿瘤,直径小于5cm,未发生转移
C.肿瘤直径或直径之和小于3cm,数目不超过2个
4 3 2008 35. 慢性萎缩性胃炎好发于:(2.0分) A.胃大弯
B.胃窦部
C.胃小弯
D.贲门
E.胃底部
B.胃窦部
4 3 2008 36. 诊断上腹痛综合征,哪一项不是必须要包括的选项:(2.0分) A.至少中等程度的上腹部疼痛或烧灼感,每周至少1次
B.疼痛为间断性
C.排便或排气后可缓解
D.不符合胆囊或Oddi括约肌功能障碍的诊断标准
E.不放射或不在腹部其他区域/胸部出现
C.排便或排气后可缓解
4 3 2008 37. 哪一项不是消化不良的报警征象:(2.0分) A.进行性消瘦
B.年龄35岁以上的初发病者
C.频繁呕吐
D.呕血或黑便
E.有肿瘤家族史
B.年龄35岁以上的初发病者
4 3 2008 38. 功能性消化不良的病理生理学基础不包括:(2.0分) A.胃肠运动功能失调
B.幽门螺杆菌感染
C.社会-心理因素
D.内脏敏感性改变
E.遗传与免疫因素
E.遗传与免疫因素
4 3 2008 39. 胰腺假性囊肿实际上是胰腺周围的包裹性积液,囊壁由纤维组织和肉芽组织构成,囊液内含有组织碎片和大量胰酶,多发生于(2.0分) A.急性胰腺炎基础上
B.慢性胰腺炎基础上
C.急性胰腺炎好转基础上
D.慢性胰腺炎好转基础上
E.重症急性胰腺炎胰腺坏死基础上
E.重症急性胰腺炎胰腺坏死基础上
4 3 2008 40. 急性胰腺炎患者饮食应该(2.0分) A.少量多餐
B.低盐饮食
C.高维生素饮食
D.急性期禁食
E.忌辛辣食物
D.急性期禁食
4 3 2008 41. 我国急性胰腺炎最常见的病因是(2.0分) A.暴饮暴食
B.酒精性
C.胆道疾病
D.高脂血症
E.病毒感染
C.胆道疾病
4 3 2008 42. CRP 检测以下哪项提示胰腺广泛坏死(2.0分) A. CRP大于100mg/L
B.CRP大于130mg/L
C.CRP大于150mg/L
D. CRP大于170mg/L
E. CRP大于200mg/L
C.CRP大于150mg/L
4 3 2008 43. 胰腺假性囊肿多在急性胰腺炎起病2周后发生,多发生于(2.0分) A.胰头
B.胰头、胰体
C.胰体
D.胰体、胰尾
E.胰尾
D.胰体、胰尾
4 3 2008 44. 有关消化道功能的发育,不正确的是: (2.0分) A.12 wk gestation: A fetus can swallow amniotic fluid
B.34 wk gestation: Nutritive sucking in neonates first
C.the first few months of infants: coordinated oral and pharyngeal movements for swallowing solids
D.1 mo: show preferences for sweet and salty foods.
E.6 mo: Infants' interest in solids increases.
E.6 mo: Infants' interest in solids increases.
4 3 2008 45. 有关胃的功能,下列哪项不正确? (2.0分) A.Pepsinogen is a precursor of the proteolytic enzyme pepsin, and intrinsic factor is required for the absorption of vitamin B6.
B.Acid production and gastrin levels are inversely related to each other except in pathologic secretory states.
C.Acid secretion is low at birth but increases dramatically by 24 hr.
D.Acid and pepsin secretion peak during the first 10 days and decrease from 10-30 days after birth.
E.Intrinsic factor secretion rises slowly during the first 2 wk
A.Pepsinogen is a precursor of the proteolytic enzyme pepsin, and intrinsic factor is required for the absorption of vitamin B6.
4 3 2008 46. 双糖酶活性主要位于: (2.0分) A.绒毛上2/3上皮细胞
B.绒毛下1/3上皮细胞
C.隐窝部细胞
D.绒毛顶部上皮细胞
E.肠杯状细胞
A.绒毛上2/3上皮细胞
4 3 2008 47. 下列哪项不符合反流的特点? (2.0分) A.Effortless regurgitation may dribble out of an infant's mouth but also may be forceful
B.Volumes of emesis are commonly about 15-30ml, but may occasionally be larger
C.Remains unhappy, although possibly hungry, after an episode of regurgitation
D.Episodes may occur from less than one to several times per day.
E.Resolves in 80% of infants by 6 mo and in 90% by 12 mo
C.Remains unhappy, although possibly hungry, after an episode of regurgitation
4 3 2008 48. 肝、胆、胰、胃的疼痛,主要位于 (2.0分) A.脐周
B.上腹部
C.右下腹
D.左下腹
E.盆腔
B.上腹部
4 3 2008 49. 急性胃肠炎累及的器官主要为: (2.0分) A.食管
B.胃
C.小肠
D.大肠
E.盲肠
C.小肠
4 3 2008 50. 引起病毒性腹泻最常见的原因(2.0分) A.轮状病毒
B.腺病毒
C.杯状病毒
D.诺如病毒
E.星状病毒
A.轮状病毒
4 3 2009 1. 关于胰腺癌黄疸不正确的是(2.0分) A.黄疸出现早晚与癌肿部位有关
B.大部分病人出现黄疸时已属中晚期
C.黄疸呈进行性加重,病人均有瘙痒
D.黄疸时间长者可有出血倾向
E.深度黄疸时大便可被染成浅黄色
C.黄疸呈进行性加重,病人均有瘙痒
4 3 2009 2. 胰腺癌最常见的组织类型是(2.0分) A.腺泡细胞癌
B.多形性腺癌
C.纤毛细胞癌
D.导管细胞癌
E.黏液癌
NaN
4 3 2009 3. 胰头癌的主要临床特点是(2.0分) A.厌食、消瘦、乏力
B.上腹部隐痛
C.肝脏肿大
D.胆囊肿大
E.黄疸
NaN
4 3 2009 4. 关于胰岛外分泌物质,下列哪项是错误的(2.0分) A.碳酸氢盐
B.淀粉酶
C.胃泌素
D.脂肪酶
E.淀粉酶
NaN
4 3 2009 5. 有关胃泌素瘤的陈述下列哪项是不正确的(2.0分) A.超过半数病人的肿瘤是恶性的
B.1/4以上的病人同时合并其他内分泌肿瘤
C.部分病人的肿瘤发生于胰腺外的器官
D.主要表现为消化性溃疡的症状和腹泻
E.全胃切除可以治愈此病
NaN
4 3 2009 6. 食管癌病理中哪一种癌最常见?(2.0分) A.腺癌
B.鳞癌
C.小细胞癌
D.腺棘癌
E.黑色素瘤
NaN
4 3 2009 7. 以下哪项检查可以直观食管黏膜改变?(2.0分) A.拉网脱落细胞检查
B.胸部CT扫描
C.X线钡餐造影
D.食道超声检查
E.食管镜
NaN
4 3 2009 8. 胸上段食管以哪个平面分界?(2.0分) A.胸骨切迹
B. 气管分叉
C.奇静脉上缘
D.奇静脉下缘
E.下肺静脉
NaN
4 3 2009 9. 食管癌根治术中最常用的代食管器官是?(2.0分) A.空肠
B.回肠
C.横结肠
D.十二指肠
E.胃
NaN
4 3 2009 10. 食管癌出现以下哪项属手术禁忌?(2.0分) A.进行性吞咽困难
B.声音嘶哑
C.肿瘤长度超过10cm
D. 射血分数大于50%
E.T3 N1 M0患者
NaN
4 3 2009 11. 胃癌主要起源于(2.0分) A.主细胞
B.壁细胞
C.腺颈部和胃小凹底部干细胞
D.幽门腺上皮细胞
E.间质细胞
NaN
4 3 2009 12. Krukenberg氏瘤是指(2.0分) A.卵巢的交界性粘液性囊腺瘤
B.卵巢腺癌
C.卵巢粘液性腺囊癌
D.腺癌伴广泛转移
E.卵巢的转移性粘液腺癌
NaN
4 3 2009 13. 与胃癌发生关系最为密切的化生类型是:(2.0分) A.不完全小肠型肠化
B.不完全大肠型肠化
C.完全小肠型肠化
D.完全大肠型肠化
E.假幽门腺化生
NaN
4 3 2009 14. 胃溃疡病变部位最常见于:(2.0分) A.胃前壁
B.胃后壁
C.胃大弯及胃底
D.胃小弯近贲门处
E.胃小弯近幽门处
NaN
4 3 2009 15. 早期胃癌IIb型是指:(2.0分) A.表浅隆起型
B.表浅平坦型
C.表浅凹陷型
D.隆起型
E.凹陷型
NaN
4 3 2009 16. 慢性萎缩性胃炎一般不出现:(2.0分) A.胃粘膜腺体增生、分泌亢进
B.固有层淋巴细胞、浆细胞浸润
C.粘膜固有腺体减少
D.淋巴滤泡形成
E.肠上皮化生
NaN
4 3 2009 17. 早期胃癌的概念是:(2.0分) A.局限于胃窦内
B.局限于粘膜及粘膜下层
C.直径在2CM以内
D.无淋巴结转移
E.尚未侵及浆膜层
NaN
4 3 2009 18. 残胃癌多发生在第1次手术后约: (2.0分) A.5-10年
B.10-15年
C.20-25年
D.30-35年
E.35-40年
NaN
4 3 2009 19. 对有胃癌前期病变者,如胃酸减少或缺乏萎缩性胃炎.胃溃疡.胃息肉等,应作:(2.0分) A.纤维光束胃镜检查
B.X线钡餐检查
C.胃液细胞学检查
D.病理切片检查
E.定期系统随诊检查,早期积极治疗
NaN
4 3 2009 20. 关于胃癌,下列叙述哪项不正确? (2.0分) A.胃癌占我国消化道肿瘤的第一位
B.胃癌发病年龄经以40-60岁多见
C.胃癌发生部多在胃窦,其次是胃小弯
D.胃大弯和前壁一般不发生胃癌
E.贲门区的胃癌比胃大弯更常见
NaN
4 3 2009 21. 关于胃癌根治性切除,下列哪项是错误的?(2.0分) A.全胃切除可以明显提高5年生存率
B.切除域应距肿块边缘6cm以上
C.彻底清除胃和区段淋巴结
D.必要时作附近脏器联合切除
E.胃远端切除距幽门口前方5cm
NaN
4 3 2009 22. 提高胃癌早期发现的几项关键检查是:(2.0分) A.四环素荧光试验.OB试验.胃液细胞学
B.纤维光束胃镜.X线钡餐,胃液细胞学
C.游离胃液测定.胃液细胞学
D.X线钡餐.OB试验.纤维光束胃镜
E.纤维光束胃镜.胃液细胞学.四环素荧光试验
NaN
4 3 2009 23. 关于胃癌的淋巴转移,下列哪项是正确的?(2.0分) A.胃癌浸润深度与淋巴结转移无相关关系
B.淋巴转移不是胃癌的主要转移途径
C.无跳跃式淋巴结转移
D.恶性程度较高或较晚期的胃癌可转移至锁骨上淋巴结
E.区域淋巴结转移部位与原发肿瘤的部位无关
NaN
4 3 2009 24. 进展期胃癌Bormann分型中,最常见的是:(2.0分) A.息肉型
B.溃疡型
C.溃疡浸润型
D.弥漫浸润型
E.浅表扩散型
NaN
4 3 2009 25. 胃癌最常见的扩散方式是:(2.0分) A.直接蔓延
B.淋巴转移
C.血行播散
D.腹腔内种植
E.胃内转移
NaN
4 3 2009 26. 进展期胃癌最早出现的症状是:(2.0分) A.恶心呕吐
B.黑粪
C.上腹痛
D.贫血
E.咽下困难
NaN
4 3 2009 27. 微小胃癌是指:(2.0分) A.小于0.5cm
B.小于1.0cm
C.小于2.0cm
D.小于2.5cm
E.小于3.0cm
NaN
4 3 2009 28. 目前诊断胃癌可靠的手段是:(2.0分) A.幽门螺杆菌感染检查
B.胃液分析
C.粪便隐血试验
D.X线钡餐检查和胃镜加活检
E.胃癌血清标志物
NaN
4 3 2009 29. 男,40岁,上腹隐痛不适,近2个月来加剧,服胃痛片后有所缓解,食欲尚可,大便隐血试验(++),胃肠道钡餐检查见胃窦部小弯侧粘膜纹理紊乱,胃壁僵直不规则。首先应考虑: (2.0分) A.慢性胃窦炎
B.胃溃疡
C.胃癌
D.胃粘膜脱垂
E.萎缩性胃炎
NaN
4 3 2009 30. 男,60岁,上腹不适,隐痛,胀满,食欲减退30余天,服酵母片.苏打片等均无效,无胃病史。体查:略消瘦,腹部无阳性体征,余无特殊。实验检查:大便隐血试验(+),血红蛋白100g/l。首先应考虑: (2.0分) A.胃溃疡
B.十二指肠溃疡
C.胃息肉
D.慢性胃炎
E.胃癌
NaN
4 3 2009 31. 有关消化道功能的发育,不正确的是:(2.0分) A.12 wk gestation: A fetus can swallow amniotic fluid
B.34 wk gestation: Nutritive sucking in neonates first
C.the first few months of infants: coordinated oral and pharyngeal movements for swallowing solids
D.1 mo: show preferences for sweet and salty foods.
E.6 mo: Infants' interest in solids increases.
NaN
4 3 2009 32. 有关胃的功能,下列哪项不正确?(2.0分) A.Pepsinogen is a precursor of the proteolytic enzyme pepsin, and intrinsic factor is required for the absorption of vitamin B6.
B.Acid production and gastrin levels are inversely related to each other except in pathologic secretory states.
C.Acid secretion is low at birth but increases dramatically by 24 hr.
D.Acid and pepsin secretion peak during the first 10 days and decrease from 10-30 days after birth.
E.Intrinsic factor secretion rises slowly during the first 2 wk
NaN
4 3 2009 33. 双糖酶活性主要位于:(2.0分) A.绒毛上2/3上皮细胞
B.绒毛下1/3上皮细胞
C.隐窝部细胞
D.绒毛顶部上皮细胞
E.肠杯状细胞
NaN
4 3 2009 34. 下列哪项不符合反流的特点?(2.0分) A.Effortless regurgitation may dribble out of an infant's mouth but also may be forceful
B.Volumes of emesis are commonly about 15-30ml, but may occasionally be larger
C.Remains unhappy, although possibly hungry, after an episode of regurgitation
D.Episodes may occur from less than one to several times per day.
E.Resolves in 80% of infants by 6 mo and in 90% by 12 mo
NaN
4 3 2009 35. 与婴儿便秘相关的:(2.0分) A.婴儿排便困难
B.先天性巨结肠
C.先天性甲状腺功能低下症
D.假性肠梗阻
E.以上都是
NaN
4 3 2009 36. 有关病理性黄疸,不正确的是:(2.0分) A.It appears in the 1st 24-36hr of life
B.Serum bilirubin is rising at a rate faster than 5 mg/dl/24hr.
C.Serum bilirubin is ≥ 12 mg/dl in full-term or 10-14 mg/dl in preterm infants.
D.Direct-reacting bilirubin is ≥2 mg/dl at any time
E.Jaundice persists after one month of life.
NaN
4 3 2009 37. 有关肝内胆汁淤积,下列哪项是不正确的:(2.0分) A.Alagille syndrome
B. "Idiopathic" neonatal hepatitis
C.Bile duct stenosis
D.Progressive familial intrahepatic cholestasis
E.Caroli disease
NaN
4 3 2009 38. Alagille综合征的特征性面容,哪项不正确:(2.0分) A.前额宽阔
B.眼睛深陷
C.眼距宽
D.鼻子长而直
E.下巴小
NaN
4 3 2009 39. 有关进行性家族性肝内胆汁淤积症,不正确的是:(2.0分) A.临床上可表现为生长迟缓、脂肪泻、瘙痒、佝偻病
B.可发展为肝硬化
C.PFIC-1型:Byler病,ATP8B1基因缺陷,低γ谷氨酰转肽酶水平
D.PFIC-2型:ABCB11基因缺陷,高γ谷氨酰转肽酶水平
E.PFIC-3型:ABCB4/MDR3基因缺陷,高γ谷氨酰转肽酶水平
NaN
4 3 2009 40. 有关胆道闭锁的B超诊断,下列哪项可确诊:(2.0分) A.triangular cord sign
B.hepatomegaly
C.no biliary dilation
D.absent or small gallbladder
E.increased liver echogenicity
NaN
4 3 2009 41. 先天性肥厚性幽门狭窄典型的"三联征":(2.0分) A.nonbilious projectile vomiting, visible peristalsis, starvation
B.bilious projectile vomiting, visible peristalsis, palpable "olive"
C.nonbilious projectile vomiting, jaundice, palpable "olive"
D.nonbilious projectile vomiting, visible peristalsis, palpable "olive"
E.nonbilious projectile vomiting, dehydration, palpable "olive"
NaN
4 3 2009 42. 先天性肥厚性幽门狭窄的钡餐检查,正确的是:(2.0分) A.elongation of pyloric canal
B.pyloric canal outlined by string of contrast material ("string sign")
C.contrast material as linear tracts separated by mucosa ("double-track sign")
D.thickened pylorus causing external impression on gastric antrum ("shoulder sign")
E.Above all is correct.
NaN
4 3 2009 43. 有关先天性巨结肠的流行病学特征,不正确的是:(2.0分) A.The male to female ratio is 1:4
B.4% are premature infants, 96% are term infants
C.With an affected sibling, the incidence rises to 12-33%
D.occurs in 0.8% of patients with trisomy 21 Down syndrome
E.Incidence/Prevalence: 1/1,000-1,500 live births
NaN
4 3 2009 44. 先天性巨结肠的临床表现 (2.0分) A.胎便排出延迟
B.顽固性便秘
C.腹胀
D.直肠指检壶腹部空虚
E.以上均是
NaN
4 3 2009 45. 先天性巨结肠的常见并发症,(2.0分) A.小肠结肠炎
B.肠穿孔
C.败血症
D.肺炎
E.以上均是
NaN
4 3 2009 46. 关于慢性浅表性胃炎,下列叙述那项是正确的 (2.0分) A.常位于胃小弯近幽门侧
B.胃镜下可见充血、出血、糜烂
C.大多转为慢性萎缩性胃炎
D.炎症细胞浸润局限于粘膜浅1/3层
E.病变局部可有少量固有腺体破坏
NaN
4 3 2009 47. 慢性萎缩性胃炎好发于 (2.0分) A.胃窦部
B.胃大弯
C.胃小弯
D.贲门
E.胃底部
NaN
4 3 2009 48. 十二指肠溃疡的并发症不应有(2.0分) A.出血
B.癌变
C.穿孔
D.幽门梗阻
E.粘连
NaN
4 3 2009 49. 慢性胃溃疡底部的动脉常可出现何种变化 (2.0分) A.细动脉玻璃样变性
B.小动脉硬化
C.增殖性动脉内膜炎
D.动脉内血栓形成
E.C+D
NaN
4 3 2009 50. 符合早期食管癌的叙述有 (2.0分) A.癌组织浸润未及肌层
B.癌块直径必须小于3cm
C.肉眼形态多为平坦,也可轻微隆起或凹陷,或浅溃疡
D.可有淋巴结转移,但仅限于食管旁
E.可有淋巴结转移,但仅限于纵隔
NaN
4 3 2010 1. 以下哪些因素可以引起食管癌?(2.0分) A.饮酒
B.过热食物
C.腌制食品
D.食物中缺乏钼
E.以上都是
E.以上都是
4 3 2010 2. 食管癌的典型临床表现是:(2.0分) A.轻度下咽不适
B.胸骨后烧灼感
C.进行性吞咽困难
D.咽部异物感
E.轻度哽噎
C.进行性吞咽困难
4 3 2010 3. 根据2009年第七版AJCC/UICC食管鳞癌分期标准,T1-3 N2 M0 属哪一期?(2.0分) A.IA 期
B. IIB期
C. IV期
D. IIIC期
E. IIIA期
E. IIIA期
4 3 2010 4. 食道X线钡餐造影出现龛影提示哪一种病理类型肿瘤?(2.0分) A.髓质型
B.溃疡型
C.覃伞型
D.腔内型
E.缩窄型
B.溃疡型
4 3 2010 5. 以下哪项不是食管癌根治术(胃食道吻合)必须的操作内容?(2.0分) A.食管癌切除
B.淋巴结清扫
C.游离胃
D.胃食管吻合
E.胸导管结扎
E.胸导管结扎
4 3 2010 6. 恶性程度较高的胃癌可以超越常规所属淋巴结转移方式,而直接侵犯远处淋巴结,其中最常见为:(2.0分) A.胰脾淋巴结
B.肝门淋巴结
C.腹腔淋巴结
D.锁骨上淋巴结
E.直肠上动脉淋巴结
D.锁骨上淋巴结
4 3 2010 7. 下列哪项最能提示胃溃疡恶变?(2.0分) A.多发溃疡
B.复合溃疡
C.胃小弯侧溃疡
D.溃疡>2CM
E.溃疡位于胃腔廓之外,周围粘膜呈星状聚合
E.溃疡位于胃腔廓之外,周围粘膜呈星状聚合
4 3 2010 8. 提高胃癌治愈的关键在于:(2.0分) A.早期诊断
B.彻底手术
C.积极放疗
D.早期化疗
E.综合治疗
A.早期诊断
4 3 2010 9. 关于胃癌的治疗,下列哪项是错误的?(2.0分) A.侵犯浆肌层的胃癌3.5.10年生存率较早期胃癌小50%以上
B.根治手术是最有效的治疗方法
C.常用的化疗药物为5-Fu及自力霉素
D.若胃癌已有肝左外叶浸润不应作根治术
E.如无远处转移,一般争取尽早手术
D.若胃癌已有肝左外叶浸润不应作根治术
4 3 2010 10. 关于胃癌,下列叙述哪项不正确?(2.0分) A.早期胃癌是指局限在粘膜和粘膜下层的胃癌
B.小胃癌是指病灶0.6-1.0cm的胃癌
C.早期胃癌均无淋巴结转移
D.微小胃癌是指直径≤0.5cm的胃癌
E.小胃癌和微小胃癌根治后5年存活率几乎达100
C.早期胃癌均无淋巴结转移
4 3 2010 11. 胃癌的癌前期状态不包括:(2.0分) A.胃息肉
B.胃平滑肌瘤
C.慢性萎缩性胃炎
D.胃酸缺乏症
E.恶性贫血者
B.胃平滑肌瘤
4 3 2010 12. 行胃癌根治术时,手术切除最少应离癌肿边缘多远才算足够?(2.0分) A.1-2cm
B.2-4cm
C.4--6cm
D.6-8cm
E.8--10cm
D.6-8cm
4 3 2010 13. 胃腺癌最好发生在:(2.0分) A.贲门部
B.胃体大弯
C.胃体小弯
D.胃窦部
E.全胃
D.胃窦部
4 3 2010 14. 胃癌的下列组织病理学分型,哪一型恶性程度最高(2.0分) A.管状腺癌
B.粘液腺癌
C.印戒细胞癌
D.弥漫型癌
E.髓质癌
D.弥漫型癌
4 3 2010 15. 胃癌血行播散最常转移到:(2.0分) A.肝脏
B.腹膜
C.肺脏
D.肾脏
E.卵巢
A.肝脏
4 3 2010 16. Trousseau征是指胃癌患者伴有:(2.0分) A.黑棘皮病
B.反复发作性血栓性静脉炎
C.皮肌炎
D.微血管病性溶血性贫血
E.远处淋巴结转移
B.反复发作性血栓性静脉炎
4 3 2010 17. 便潜血试验呈持续阳性常提示:(2.0分) A.急性胃粘膜病变
B.胃溃疡
C.十二指肠球部溃疡
D.慢性萎缩性胃炎
E.胃癌
E.胃癌
4 3 2010 18. 男性,42岁,上腹烧灼样痛半年,近一周排柏油便,为确诊,宜首选下列哪项检查方法:(2.0分) A.X线胃肠钡餐透视
B.X线钡灌肠透视检查
C.便潜血检查
D.B型超声检查
E.胃镜检查
E.胃镜检查
4 3 2010 19. 胃窦部肿块型胃癌约6×4×4CM大小,已累及浆膜层,CT检查左肝外叶亦有3CM大小转移灶,胰腺正常。该病人的治疗应选择: (2.0分) A.已无法手术,行全身化学疗法
B.根治性胃大部切除术
C.根治性胃大部分切除术+左肝动脉栓塞术
D.根治性胃大部切除+左肝外叶切除术
E.胃空肠吻合术
D.根治性胃大部切除+左肝外叶切除术
4 3 2010 20. 男,48岁,因胃癌而行剖腹探查时,发现肿块已浸润至横结肠,但较局限,该病人宜: (2.0分) A.禁忌手术
B.行胃癌根治术+横结肠切除术
C.行胃大部切除术
D.行全胃切除术
E.行横结肠切除术
B.行胃癌根治术+横结肠切除术
4 3 2010 21. 有关病理性黄疸,不正确的是:(2.0分) A.It appears in the 1st 24-36hr of life
B.Jaundice persists after one month of life.
C.Serum bilirubin is rising at a rate faster than 5 mg/dl/24hr.
D.Serum bilirubin is ≥ 12 mg/dl in full-term or 10-14 mg/dl in preterm infants.
E.Direct-reacting bilirubin is ≥2 mg/dl at any time
B.Jaundice persists after one month of life.
4 3 2010 22. 有关肝内胆汁淤积,下列哪项是不正确的:(2.0分) A.Choledochal cyst
B. "Idiopathic" neonatal hepatitis
C.Alagille syndrome
D.Progressive familial intrahepatic cholestasis
E.Caroli disease
A.Choledochal cyst
4 3 2010 23. Alagille综合征的特征性面容,哪项不正确:(2.0分) A.broad forehead
B.wide between eyes
C.deep-set eyes
D.straight nose
E.pointed chin
B.wide between eyes
4 3 2010 24. 有关进行性家族性肝内胆汁淤积症,不正确的是(2.0分) A.临床上可表现为生长迟缓、脂肪泻、瘙痒、佝偻病
B.可发展为肝硬化
C.PFIC-1型:Byler病,ATP8B1基因缺陷,低γ谷氨酰转肽酶水平
D.PFIC-2型:ABCB11基因缺陷,高γ谷氨酰转肽酶水平
E.PFIC-3型:ABCB4/MDR3基因缺陷,高γ谷氨酰转肽酶水平
D.PFIC-2型:ABCB11基因缺陷,高γ谷氨酰转肽酶水平
4 3 2010 25. 有关胆道闭锁的B超诊断,下列哪项可确诊:(2.0分) A.absent or small gallbladder
B.hepatomegaly
C.no biliary dilation
D.triangular cord sign
E.increased liver echogenicity
D.triangular cord sign
4 3 2010 26. 有关消化道功能的发育,不正确的是(2.0分) A.12 wk gestation: A fetus can swallow amniotic fluid
B.34 wk gestation: Nutritive sucking in neonates first
C.the first few months of infants: coordinated oral and pharyngeal movements for swallowing solids
D.3 mo: show preferences for sweet and salty foods.
E.4 mo: Infants' interest in solids increases.
D.3 mo: show preferences for sweet and salty foods.
4 3 2010 27. 下列哪项不是构成胃肠道肠壁的4层结构之一?(2.0分) A.黏膜层
B.黏膜肌层
C.黏膜下层
D.肌层
E.浆膜层
B.黏膜肌层
4 3 2010 28. 双糖酶活性主要位于:(2.0分) A.绒毛上2/3上皮细胞
B.绒毛下1/3上皮细胞
C.隐窝部细胞
D.绒毛顶部上皮细胞
E.肠杯状细胞
A.绒毛上2/3上皮细胞
4 3 2010 29. 下列哪项不符合反流的特点?(2.0分) A.Effortless regurgitation may dribble out of an infant's mouth but also may be forceful
B.Volumes of emesis are commonly about 15-30ml, but may occasionally be larger
C.Remains unhappy, although possibly hungry, after an episode of regurgitation
D.Episodes may occur from less than one to several times per day.
E.Resolves in 80% of infants by 6 mo and in 90% by 12 mo
C.Remains unhappy, although possibly hungry, after an episode of regurgitation
4 3 2010 30. 有关呕吐,下列哪项是不正确的?(2.0分) A.Vomiting caused by obstruction of the GI is probably mediated by intestinal visceral afferent nerves stimulating the vomiting center.
B.Nonobstructive lesions of the digestive tract can also cause vomiting
C.With repeated vomiting in the absence of obstruction, the emesis may not become bile stained.
D.Most diseases of the upper bowel, pancreas, liver, or biliary tree are capable of provoking emesis.
E.CNS or metabolic derangements may lead to severe, persistent emesis.
C.With repeated vomiting in the absence of obstruction, the emesis may not become bile stained.
4 3 2010 31. 胃食管连接部的抗反流功能中,下列哪项是最重要的:(2.0分) A.diaphragm
B.intra-abdominal portion of the esophagus
C.angle of His
D.lower esophageal sphincter
E.esophageal peristalsis
D.lower esophageal sphincter
4 3 2010 32. 下列哪项不是胃食管反流的典型症状?(2.0分) A.heartburn
B.vomiting
C.feeding difficulties
D.chest pain
E.regurgitation
C.feeding difficulties
4 3 2010 33. 阻抗值大小,下列哪项正确?(2.0分) A.空气﹥反流物﹥唾液﹥食物
B.反流物﹥唾液﹥空气﹥食物
C.空气﹥唾液﹥食物﹥反流物
D.唾液﹥空气﹥反流物﹥食物
E.反流物﹥唾液﹥食物﹥空气
C.空气﹥唾液﹥食物﹥反流物
4 3 2010 34. 有关胃食管反流的诊断,下列哪项检查最有价值?(2.0分) A.食管pH动态监测
B.食管核素扫描
C.食管钡餐造影
D.食管内镜检查
E.食管pH-阻抗监测
E.食管pH-阻抗监测
4 3 2010 35. 有关胃食管反流的药物治疗中,目前最常用的治疗药物:(2.0分) A.H2受体拮抗剂
B.质子泵抑制剂
C.促动力剂
D.抗酸剂
E.黏膜保护剂
B.质子泵抑制剂
4 3 2010 36. 有关先天性肥厚性幽门狭窄,不正确的是:(2.0分) A.thickening of the pyloric portion of the stomach causing obstruction of gastric outflow
B.age distribution almost always between age 6-8 week
C.4-5 times more common in boys than girls
D.highest incidence in first-born infants
E.jaundice seen in 1%-2% of infants with hypertrophic pyloric stenosis
B.age distribution almost always between age 6-8 week
4 3 2010 37. 先天性肥厚性幽门狭窄的B超检查,下列哪项为阳性结果:(2.0分) A.pylorus muscle wall thickness > 4 mm
B.pylorus muscle wall thickness < 3 mm
C.pylorus canal length < 14 mm
D.pylorus diameter < 12 mm
E.absent or minimal vascular flow
A.pylorus muscle wall thickness > 4 mm
4 3 2010 38. 下列哪项不符合先天性肥厚性幽门狭窄的临床特征:(2.0分) A.forceful (projectile) nonbilious vomiting
B.bad appetite
C.visible peristalsis
D.hypochloremic alkalosis
E.poor weight gain
B.bad appetite
4 3 2010 39. 有关先天性巨结肠的病理生理,不正确的是(2.0分) A.Hypertrophic cholinergic nerve trunks
B.Limited number of adrenergic fiber
C.dilated colon found proximal to aganglionic segment
D.absence of submucosal and myenteric sympathetic ganglion cells
E.absence of submucosal and myenteric parasympathetic ganglion cells
D.absence of submucosal and myenteric sympathetic ganglion cells
4 3 2010 40. 先天性巨结肠的辅助检查,可排除诊断的是(2.0分) A.腹部立位片,盆腔无气体或少量气体
B.直肠肛门测压,直肠肛门抑制反射阴性
C.直肠黏膜活检,痉挛段肠管乙酰胆碱含量减少
D.直肠黏膜活检,痉挛段肠管胆碱脂酶活性增加
E.直肠肌层活检,病变段无髓鞘的神经纤维数量增加,形态增粗增大
C.直肠黏膜活检,痉挛段肠管乙酰胆碱含量减少
4 3 2010 41. 起生理性调节和激素样作用的胃肠肽不包括 (2.0分) A.促胃液素
B.缩胆囊素
C.促胰液素
D.抑胃肽
E.生长抑素
E.生长抑素
4 3 2010 42. 多种胃肠肽也分布于中枢神经系统中,这种双重分布的肽,被称为 (2.0分) A.脑肠肽
B.糖皮质激素
C.内因子
D.胃泌素
E.胆囊收缩素
A.脑肠肽
4 3 2010 43. 下面不属于代谢综合征的成分是 (2.0分) A.超重/肥胖
B.高血压
C.血脂异常
D.高血糖
E.冠心病
E.冠心病
4 3 2010 44. 炎症性肠病的特征性皮肤表现有:(2.0分) A.结节性红斑
B.红斑狼疮
C.天胞疮
D.剥脱性皮炎
E.银屑病
A.结节性红斑
4 3 2010 45. PCOS和脂肪肝的最重要共同发病机制:(2.0分) A.脂质代谢紊乱
B.糖代谢紊乱
C.胰岛素抵抗
D.基因多态性
E.蛋白质代谢紊乱
C.胰岛素抵抗
4 3 2010 46. 食管癌最典型的临床表现是(2.0分) A.进行性吞咽困难
B.咽下疼痛
C.咽部异物感
D.食物反流
E.左锁骨上淋巴结肿大
A.进行性吞咽困难
4 3 2010 47. 卓-艾综合征的发生与下列胃肠激素分泌过多有关(2.0分) A.胰岛素
B.胰高血糖素
C.胆囊收缩素
D.促胰酶素
E.胃泌素
E.胃泌素
4 3 2010 48. 诊断肝性脑病最有意义的实验室检查是(2.0分) A.血糖
B.血氨
C.血胆红素
D.血尿素氮
E.血转氨酶
B.血氨
4 3 2010 49. 血清淀粉酶测定正确的是(2.0分) A.发病后立即升高
B.起病后6-12小时开始升高
C.淀粉酶的高低与疾病严重程度呈正比
D.超过正常值2倍即可确诊
E.E持续一周以上
B.起病后6-12小时开始升高
4 3 2010 50. 上消化道出血是指出血的部位是(2.0分) A.食管至幽门
B.十二指肠屈氏韧带以上的消化器官
C.十二指肠以上的消化器官
D.胃以上的消化器官
E.食管至空肠
B.十二指肠屈氏韧带以上的消化器官
4 3 2011 1. 根据AJCC第7版食管癌TNM分期,关于食管分段叙述不正确的有(2.5分) A.颈段食管:始于下咽,向下至胸骨切迹平面的胸廓入口
B.胸上段食管:自胸廓入口向下至气管分叉水平
C.胸中段食管:自奇静脉弓下缘向下至肺静脉水平
D.胸下段食管:自下肺静脉水平向下至胃
E.腹段食管包括在胸下段食管中
B.胸上段食管:自胸廓入口向下至气管分叉水平
4 3 2011 2. 早期食管癌钡餐造影分型,除外(2.5分) A.斑块型
B.乳头型
C.糜烂型
D.平坦型
E.髓质型
E.髓质型
4 3 2011 3. 患者,男,32岁,因吞咽不顺3个月就诊,查体无特殊发现,上消化道造影:胸中段食管处卵圆形肿物,食管管腔偏心性狭窄,边缘光滑锐利,食管黏膜无紊乱和中断。最有可能的诊断(2.5分) A.食管平滑肌瘤
B.食管良性狭窄
C.食管癌
D.消化性食管炎
E.贲门失迟缓症
A.食管平滑肌瘤
4 3 2011 4. 题干同第3题,不宜采用的诊断手段是(2.5分) A.X线钡餐造影
B.胸部CT
C.超声内镜
D.胃镜活检
E.胸部MRI
D.胃镜活检
4 3 2011 5. 下列哪项不是食管癌切除、食管胃重建的常用手术方法(2.5分) A.结肠代食管术
B.空肠代食管术
C.食管胃转流吻合术
D.主动脉弓下食管胃吻合术
E.主动脉弓上食管胃吻合术
C.食管胃转流吻合术
4 3 2011 6. 有关食管癌的临床特点,下列不正确的是(2.5分) A.男性多于女性,以40岁以上居多
B.早期症状可不明显,中晚期典型的症状是进行性吞咽困难
C.食管吞钡造影是诊断食管癌最常用的方法
D.血行转移是最主要的转移途径
E.强调早期发现、早期诊断及早期治疗
D.血行转移是最主要的转移途径
4 3 2011 7. 与食管癌发病因素无相关性的(2.5分) A.亚硝胺类化合物及真菌
B.环境气候与季节
C.饮食习惯、营养不良和微量元素缺乏
D.遗传因素
E.Barret食管
B.环境气候与季节
4 3 2011 8. 男性,60岁。进食梗阻感2个月。下列X线钡餐检查的影像学表现特点中,不支持早期食管癌的是(2.5分) A.局限性小的充盈缺损
B.局限性管壁僵硬
C.局限性黏膜皱襞增粗和断裂
D.小龛影
E.食管下端呈"鸟嘴状"狭窄
E.食管下端呈"鸟嘴状"狭窄
4 3 2011 9. 急性坏疽性阑尾炎会发生 (2.5分) A.阑尾肿胀充血
B.粘膜上皮缺损
C.阑尾腐败穿孔
D.阑尾全层中性粒细胞浸润
E.阑尾腔闭塞,壁变硬
C.阑尾腐败穿孔
4 3 2011 10. 十二指肠溃疡主要表现为 (2.5分) A.溃疡多在十二指肠降部
B.溃疡大小多在 1cm 以上
C.前壁之溃疡易穿孔
D.后壁之溃疡不易出血
E.以上都不是
C.前壁之溃疡易穿孔
4 3 2011 11. 胃粘液癌中可出现 (2.5分) A.印戒细胞
B.朗罕氏巨细胞
C.镜影细胞
D.肠上皮化生细胞
E.瘤巨细胞
A.印戒细胞
4 3 2011 12. Typically,the diameter of the main pancreatic duct is(2.5分) A.3-4mm
B.2-3mm
C.<1mm
D.4-5mm
E.5-6mm
B.2-3mm
4 3 2011 13. 胰腺癌与胆总管结石的主要鉴别点是(2.5分) A.腹痛的性质与程度
B.肝功能变化
C.血尿淀粉酶改变
D.胆囊增大
E.进行性加重黄疸
E.进行性加重黄疸
4 3 2011 14. 胰头癌的最常见的首发症状是(2.5分) A.食欲不振
B.上腹部隐痛和饱胀不适
C.消瘦
D.胆囊肿大
E.发热
B.上腹部隐痛和饱胀不适
4 3 2011 15. 胰腺十二指肠切除术最常见的并发症是(2.5分) A.术后出血
B.糖尿病
C.肝衰竭
D.腹腔脓肿
E.胰瘘和胆瘘
D.腹腔脓肿
4 3 2011 16. 早期胃癌(Early Gastric Cancer)的定义是: 正确答案(2.5分) A.局限于胃窦内
B.局限于粘膜及粘膜下层
C.直径在2CM以内
D. 无淋巴结转移
B.局限于粘膜及粘膜下层
4 3 2011 17. 提高胃癌治愈的关键在于:正确答案(2.5分) A. 早期诊断
B. 彻底手术
C. 积极放疗
D.综合治疗
A. 早期诊断
4 3 2011 18. 胃癌的癌前期状态不包括: 正确答案(2.5分) A.胃息肉
B.胃平滑肌瘤
C.慢性萎缩性胃炎
D.胃酸缺乏症
B.胃平滑肌瘤
4 3 2011 19. 胃窦部肿块型胃癌,大小约6×5CM,已累及浆膜层,CT检查提示左肝外叶有一枚3CM大小转移灶,余脏器未见转移灶。该病人的治疗应选择: 正确答案(2.5分) A. 已无法手术,行全身化学疗法
B. 根治性胃大部切除术
C.根治性胃大部切除+左肝外叶切除术
D.胃空肠吻合术
C.根治性胃大部切除+左肝外叶切除术
4 3 2011 20. 胃癌可引起:正确答案(2.5分) A.出血
B.梗阻
C.两者均有
D.两者均无
C.两者均有
4 3 2011 21. 胃癌的癌前病变有:正确答案(2.5分) A.胃大部切除术后残胃
B.慢性萎缩性胃炎
C.胃溃疡
D.以上均是
D.以上均是
4 3 2011 22. 男性,60岁,上腹部饱胀不适伴隐痛30余天,服酵母片、苏打片等均无效,既往无胃病史。查体:消瘦貌,腹部无阳性体征。实验检查:大便隐血试验(+),血红蛋白88g/l。首先应考虑: 正确答案(2.5分) A.胃溃疡
B.十二指肠溃疡
C.胃癌
D.慢性胃炎
C.胃癌
4 3 2011 23. Endoscopic ultrasound is used in the staging of gastric cancer. Which of the following statements is true: (2.5分) A.when good quality CT scan is available, EUS adds little to staging accuracy
B.CT scan is better than EUS in assessing N stage
C.the availability of EUS removes the need for staging laparoscopy
D.EUS is better than CT in assessing T stage
D.EUS is better than CT in assessing T stage
4 3 2011 24. 胃癌的好发部位依次是(2.5分) A.胃小弯、胃窦、胃大弯
B.胃窦、胃小弯、贲门
C.贲门、胃窦、胃大弯
D.胃小弯、贲门、胃窦
B.胃窦、胃小弯、贲门
4 3 2011 25. 男性,75岁,胃癌根治术后7天,剧烈咳嗽时,突然出现切口疼痛,并流出少量淡红色液体,病人最可能出现了(2.5分) A.切口内癌细胞种植
B.切口感染
C.切口裂开
D.切口脂肪液化
C.切口裂开
4 3 2011 26. 溃疡型胃癌X线钡餐造影的特点是(2.5分) A.龛影边缘不整齐
B.龛影位于胃腔之内
C.龛影>2.5cm
D.以上都是
D.以上都是
4 3 2011 27. 有关先天性肥厚性幽门狭窄,不正确的是(2.5分) A.thickening of the pyloric portion of the stomach causing obstruction of gastric outflow
B.age distribution almost always between age 3-8 week
C.1-2 times more common in boys than girls
D.highest incidence in first-born infants
E.jaundice seen in 1%-2% of infants with hypertrophic pyloric stenosis
C.1-2 times more common in boys than girls
4 3 2011 28. 先天性肥厚性幽门狭窄的钡餐检查,正确的是(2.5分) A.elongation of pyloric canal
B.pyloric canal outlined by string of contrast material ("string sign")
C.contrast material as linear tracts separated by mucosa ("double-track sign")
D.thickened pylorus causing external impression on gastric antrum ("shoulder sign")
E.Above all is correct.
E.Above all is correct.
4 3 2011 29. 先天性巨结肠的临床表现(2.5分) A.胎便排出延迟
B.顽固性便秘
C.腹胀
D.直肠指检壶腹部空虚
E.以上均是
E.以上均是
4 3 2011 30. 先天性巨结肠的辅助检查,可排除诊断的是(2.5分) A.腹部立位片,盆腔无气体或少量气体
B.直肠肛门测压,直肠肛门抑制反射阴性
C.直肠黏膜活检,痉挛段肠管乙酰胆碱含量减少
D.直肠黏膜活检,痉挛段肠管胆碱脂酶活性增加
E.直肠肌层活检,病变段无髓鞘的神经纤维数量增加,形态增粗增大
C.直肠黏膜活检,痉挛段肠管乙酰胆碱含量减少
4 3 2011 31. 胃食管连接部的抗反流功能中,下列哪项是最重要的(2.5分) A.diaphragm
B.intra-abdominal portion of the esophagus
C.angle of His
D.lower esophageal sphincter
E.esophageal peristalsis
D.lower esophageal sphincter
4 3 2011 32. 有关钡餐造影,不正确的是(2.5分) A.常用于诊断胃食管反流病
B.可观察食管黏膜的完整性
C.常用于诊断食管解剖结构的异常
D.常用于诊断食管裂孔疝
E.可评估食管动力
A.常用于诊断胃食管反流病
4 3 2011 33. 有关胃食管反流的诊断,下列哪项检查最有价值?(2.5分) A.食管pH动态监测
B.食管核素扫描
C.食管钡餐造影
D.食管内镜检查
E.食管pH-阻抗监测
E.食管pH-阻抗监测
4 3 2011 34. 有关消化道功能的发育,不正确的是(2.5分) A.12 wk gestation: A fetus can swallow amniotic fluid
B.34 wk gestation: Nutritive sucking in neonates first
C.the first few months of infants: coordinated oral and pharyngeal movements for swallowing solids
D.3 mo: show preferences for sweet and salty foods.
E.4 mo: Infants' interest in solids increases.
D.3 mo: show preferences for sweet and salty foods.
4 3 2011 35. 单纯母乳喂养儿肠道中占绝对优势的菌群是(2.5分) A.大肠杆菌
B.嗜酸乳杆菌
C.肠球菌
D.双歧杆菌
E.芽胞杆菌
D.双歧杆菌
4 3 2011 36. 有关病理性黄疸,不正确的是(2.5分) A.It appears in the 1st 24-36hr of life
B.Jaundice persists after one month of life.
C.Serum bilirubin is rising at a rate faster than 5 mg/dl/24hr.
D.Serum bilirubin is ≥ 12 mg/dl in full-term or 10-14 mg/dl in preterm infants.
E.Direct-reacting bilirubin is ≥2 mg/dl at any time
B.Jaundice persists after one month of life.
4 3 2011 37. 消化道出血,下列哪项不相关?(2.5分) A.门脉高压出血
B.胰腺假性囊肿
C.麦克尔憩室
D.肠重复畸形
E.胆道出血
B.胰腺假性囊肿
4 3 2011 38. 有关胆汁淤积,正确的是(2.5分) A.当总胆红素<85umol/L,直接胆红素>17umol/L
B.当总胆红素>85umol/L,直接胆红素>17umol/L
C.当总胆红素>85umol/L,直接胆红素占总胆红素的比例<20%
D.当总胆红素>85umol/L,直接胆红素<17umol/L
E.当总胆红素<85umol/L,直接胆红素<17umol/L
A.当总胆红素<85umol/L,直接胆红素>17umol/L
4 3 2011 39. 有关肝内胆汁淤积,下列哪项是不正确的(2.5分) A.Choledochal cyst
B. "Idiopathic" neonatal hepatitis
C.Alagille syndrome
D.Progressive familial intrahepatic cholestasis
E.Caroli disease
A.Choledochal cyst
4 3 2011 40. Alagille综合征的饮食治疗(2.5分) A.补充脂肪
B.无乳糖饮食
C.补充中链甘油三酯
D.补充短肽
E.低敏配方奶粉
C.补充中链甘油三酯
4 3 2012 1. 以下对胰腺癌的论述哪项不正确(2.5分) A.可发生于胰腺头、体和尾部,40岁以上男性好发,生存率低。
B.早期诊断困难,手术切除率低,预后差。
C.组织类型主要是腺泡细胞癌
D.浸润方式以淋巴转移为主
E.早期可广泛浸润周围组织、器官、较早经淋巴转移
E.早期可广泛浸润周围组织、器官、较早经淋巴转移
4 3 2012 2. 以下关于粘液性囊性瘤的叙述,不正确的是(2.5分) A.约占全部外分泌肿瘤的2%
B.腺瘤几乎全都发生于女性,平均发病年龄40-50岁之间多见
C.在胰腺体尾部最多发
D.免疫组化染色显示CEA(+),CA19-9多数为(-)
E.肿瘤内容物一般为粘液性和粘液血性
D.免疫组化染色显示CEA(+),CA19-9多数为(-)
4 3 2012 3. 目前已知与胰腺癌最相关的基因是(2.5分) A.P53基因
B.DPC4基因
C.SPINK1基因
D.hTR基因
E.K-ras基因
E.K-ras基因
4 3 2012 4. 溃疡病最好发于: (2.5分) A.十二指肠下段
B.十二指肠球部
C.胃小弯近幽门处
D.胃体部
E.胃及十二指肠球部
E.胃及十二指肠球部
4 3 2012 5. 胃癌最主要的转移途径是 (2.5分) A.直接蔓延
B.淋巴道转移
C.血行转移
D.腹腔内种植
E.消化道转移
B.淋巴道转移
4 3 2012 6. 哪种肠疾病可能发生癌变(2.5分) A.慢性肠结核性溃疡
B.慢性溃疡性结肠炎
C.出血坏死性肠炎
D.先天性巨结肠
E.阿米巴痢疾
B.慢性溃疡性结肠炎
4 3 2012 7. 根据AJCC第7版食管癌TNM分期,关于食管分段叙述不正确的有(2.5分) A.颈段食管:始于下咽,向下至胸骨切迹平面的胸廓入口
B.胸上段食管:自胸廓入口向下至主动脉弓上缘水平
C.胸中段食管:自奇静脉弓下缘向下至肺静脉水平
D.胸下段食管:自下肺静脉水平向下至胃
E.腹段食管包括在胸下段食管中
B.胸上段食管:自胸廓入口向下至主动脉弓上缘水平
4 3 2012 8. 早期食管癌钡餐造影分型,除外(2.5分) A.斑块型
B.乳头型
C.糜烂型
D.平坦型
E.髓质型
E.髓质型
4 3 2012 9. 下列哪项不是食管癌的手术禁忌(2.5分) A.严重吞咽困难
B.声音嘶哑
C.气管食管瘘
D.左锁骨上淋巴结转移
E.严重恶病质者
A.严重吞咽困难
4 3 2012 10. 食管炎与早期食管癌的鉴别,主要方法是(2.5分) A.免疫诊断方法
B.食管钡餐造影检查示黏膜紊乱
C.纤维内镜检查及病理活检
D.胸骨后灼痛
E.试验治疗
C.纤维内镜检查及病理活检
4 3 2012 11. 与食管癌发病因素无相关性的(2.5分) A.亚硝胺类化合物及真菌
B.环境气候与季节
C.饮食习惯、营养不良和微量元素缺乏
D.遗传因素
E.Barret食管
B.环境气候与季节
4 3 2012 12. 食管癌患者出现声音嘶哑,说明肿瘤已侵及(2.5分) A.迷走神经
B.声带
C.气管隆嵴
D.喉返神经
E.喉上神经
D.喉返神经
4 3 2012 13. 关于食管鳞癌的淋巴转移特点,下列错误的是(2.5分) A.仅次于血行转移的转移途径
B.上段食管癌常转移至锁骨上及颈淋巴结
C.中下段多转移至气管旁、贲门及胃左动脉旁淋巴结
D.各段均可向上或向下方的淋巴结转移
E.约1/4病例的淋巴结转移是跳跃式
A.仅次于血行转移的转移途径
4 3 2012 14. 根据AJCC第7版食管鳞癌TNM分期中,如肿瘤侵及食管外膜,则T分级是以下哪项(2.5分) A.Tis
B.T1
C.T2
D.T3
E.T4
D.T3
4 3 2012 15. 恶性程度较高的胃癌可以超越常规所属淋巴结转移方式,而直接侵犯远处淋巴结,其中最常见是 (2.5分) A.胰脾淋巴结
B.肝门淋巴结
C.腹腔淋巴结
D.锁骨上淋巴结
E.以上全是
D.锁骨上淋巴结
4 3 2012 16. 下列哪项最能提示胃溃疡恶变(2.5分) A.多发溃疡
B.胃小弯侧溃疡
C.溃疡>2CM
D.溃疡位于胃腔廓之外,周围粘膜呈星状聚合
E.以上均有误
D.溃疡位于胃腔廓之外,周围粘膜呈星状聚合
4 3 2012 17. 关于胃癌,下列叙述哪项不正确的是(2.5分) A.早期胃癌是指局限在粘膜和粘膜下层的胃癌
B.早期胃癌均无淋巴结转移
C.微小胃癌是指直径≤0.5cm的胃癌
D.小胃癌和微小胃癌根治后预后非常好
E.A和C都不正确
B.早期胃癌均无淋巴结转移
4 3 2012 18. 男性,42岁,上腹烧灼样痛半年,近一周排柏油便,为确诊,宜首选下列哪项检查方法:(2.5分) A.X线胃肠钡餐
B.便潜血检查
C.B型超声检查
D.胃镜检查
E.A和B
D.胃镜检查
4 3 2012 19. 胃癌的常见转移方式(2.5分) A.直接蔓延
B.淋巴结转移
C.血行转移
D.以上均有
E.A或B
D.以上均有
4 3 2012 20. 男性,50岁,反复上腹痛伴消瘦,X线钡餐检查发现胃窦呈持续性向心性狭窄伴充盈缺损,首选的进一步处理: (2.5分) A.治疗后复查
B.胃镜及活检
C.胃液分析
D.手术探查
E.A和D
B.胃镜及活检
4 3 2012 21. 关于胃癌化疗的说法正确的是(2.5分) A.是早期胃癌首选疗法
B.是根治手术后的辅助治疗
C.是不能手术者的姑息疗法
D.比单一药物疗效高
E.以上都对
C.是不能手术者的姑息疗法
4 3 2012 22. 下列不属于特殊型胃癌的是(2.5分) A.未分化癌
B.腺鳞癌
C.类癌
D.印戒细胞癌
E.A和B
D.印戒细胞癌
4 3 2012 23. 下列哪项不是胃癌的组织学分型(2.5分) A.I型(隆起型)
B.III型(凹陷型)
C.II型(平坦型)
D.II型(局限溃疡型)
E.A和C
D.II型(局限溃疡型)
4 3 2012 24. 下列哪项与胃癌的发生有关(2.5分) A.过多食用腌制烟熏食品
B.幽门螺杆菌感染
C.遗传因素
D.以上都是
E.以上均不是
D.以上都是
4 3 2012 25. 发生于贲门的进展期胃癌,可出现(2.5分) A.进行性吞咽困难
B.上腹部饱胀、呕吐宿食
C.呕血、黑便
D.持续性疼痛
E.C和D
A.进行性吞咽困难
4 3 2012 26. 有关先天性肥厚性幽门狭窄,不正确的是(2.5分) A.thickening of the pyloric portion of the stomach causing obstruction of gastric outflow
B.age distribution almost always between age 1-2 week
C.4-5 times more common in boys than girls
D.highest incidence in first-born infants
E.jaundice seen in 1%-2% of infants with hypertrophic pyloric stenosis
B.age distribution almost always between age 1-2 week
4 3 2012 27. 先天性肥厚性幽门狭窄的钡餐检查,正确的是(2.5分) A.elongation of pyloric canal
B.pyloric canal outlined by string of contrast material ("string sign")
C.contrast material as linear tracts separated by mucosa ("double-track sign")
D.thickened pylorus causing external impression on gastric antrum ("shoulder sign")
E.Above all is correct.
E.Above all is correct.
4 3 2012 28. 有关先天性巨结肠的病理生理,不正确的是(2.5分) A.Hypertrophic cholinergic nerve trunks
B.Limited number of adrenergic fiber
C.dilated colon found proximal to aganglionic segment
D.absence of submucosal and myenteric sympathetic ganglion cells
E.absence of submucosal and myenteric parasympathetic ganglion cells
D.absence of submucosal and myenteric sympathetic ganglion cells
4 3 2012 29. 先天性巨结肠的常见并发症(2.5分) A.小肠结肠炎
B.肠穿孔
C.败血症
D.肺炎
E.以上均是
E.以上均是
4 3 2012 30. The duration of transient lower esophageal sphincter relaxations (TLESR) (2.5分) A.2-4 s
B.4-6 s
C.6-8 s
D.8-10 s
E.10-12 s
D.8-10 s
4 3 2012 31. 酸性反流指数是指(2.5分) A.Number of reflux episodes
B.Percent total time with a pH<4.0
C.Percent supine time with a pH<4.0
D.Longest reflux episode
E.Number of reflux episodes lasting≥5 min
B.Percent total time with a pH<4.0
4 3 2012 32. 阻抗值大小,下列哪项正确?(2.5分) A.空气﹥反流物﹥唾液﹥食物
B.反流物﹥唾液﹥空气﹥食物
C.空气﹥唾液﹥食物﹥反流物
D.唾液﹥空气﹥反流物﹥食物
E.反流物﹥唾液﹥食物﹥空气
C.空气﹥唾液﹥食物﹥反流物
4 3 2012 33. 有关胃食管反流的药物治疗中,目前最常用的治疗药物(2.5分) A.雷尼替丁
B.西米替丁
C.奥美拉唑
D.多潘立酮
E.红霉素
C.奥美拉唑
4 3 2012 34. 有关消化道功能的发育,不正确的是(2.5分) A.12 wk gestation: A fetus can swallow amniotic fluid
B.34 wk gestation: Nutritive sucking in neonates first
C.the first few months of infants: coordinated oral and pharyngeal movements for swallowing solids
D.3 mo: show preferences for sweet and salty foods.
E.4 mo: Infants' interest in solids increases.
D.3 mo: show preferences for sweet and salty foods.
4 3 2012 35. 小肠上皮细胞更新时间(2.5分) A.1-2天
B.2-3天
C.4-5天
D.6-7天
E.8-9天
C.4-5天
4 3 2012 36. 口服补液盐配方的主要原理(2.5分) A.Na+- K+ATPase
B.Na+-H+ exchanger
C.Cl--HCO3- exchanger
D.glucose-sodium co-transporter
E.NaCl- coupled pathway
D.glucose-sodium co-transporter
4 3 2012 37. 有关病理性黄疸,不正确的是(2.5分) A.It appears in the 1st 24-36hr of life
B.Jaundice persists after one month of life.
C.Serum bilirubin is rising at a rate faster than 5 mg/dl/24hr.
D.Serum bilirubin is ≥ 12 mg/dl in full-term or 10-14 mg/dl in preterm infants.
E.Direct-reacting bilirubin is ≥2 mg/dl at any time
B.Jaundice persists after one month of life.
4 3 2012 38. 有关胆汁淤积,正确的是(2.5分) A.当总胆红素<85umol/L,直接胆红素>17umol/L
B.当总胆红素>85umol/L,直接胆红素>17umol/L
C.当总胆红素>85umol/L,直接胆红素占总胆红素的比例<20%
D.当总胆红素>85umol/L,直接胆红素<17umol/L
E.当总胆红素<85umol/L,直接胆红素<17umol/L
A.当总胆红素<85umol/L,直接胆红素>17umol/L
4 3 2012 39. Alagille综合征的特征性面容,哪项不正确(2.5分) A.前额宽阔
B.眼睛深陷
C.眼距宽
D.鼻子长而直
E.下巴小
C.眼距宽
4 3 2012 40. 胆道闭锁的临床表现,哪项不是(2.5分) A.生后2周起黄疸持续加深
B.生后4周起黄疸退而复现
C.营养不良
D.肝脾肿大
E.灰白色粪便、深色尿
B.生后4周起黄疸退而复现
4 3 2013 1. 根据AJCC第7版食管癌TNM分期,关于食管分段叙述不正确的有(2.5分) A.颈段食管:始于下咽,向下至胸骨切迹平面的胸廓入口
B.胸上段食管:自胸廓入口向下至主动脉弓上缘水平
C.胸中段食管:自奇静脉弓下缘向下至肺静脉水平
D.胸下段食管:自下肺静脉水平向下至胃
E.腹段食管包括在胸下段食管中
B.胸上段食管:自胸廓入口向下至主动脉弓上缘水平
4 3 2013 2. 患者,男,57岁,因进行性吞咽困难2月入院,查体:双侧锁骨上区淋巴结未触及肿大,心肺腹无特殊发现,上消化道造影提示自气管分叉水平至下肺静脉水平食管充盈缺损,黏膜紊乱中断,局部有软组织影。最有可能的诊断(2.5分) A.食管平滑肌瘤
B.食管息肉
C.食管良性狭窄
D.食管囊肿
E.食管癌
E.食管癌
4 3 2013 3. 下列哪项不是食管癌的手术禁忌(2.5分) A.严重吞咽困难
B.声音嘶哑
C.气管食管瘘
D.左锁骨上淋巴结转移
E.严重恶病质者
A.严重吞咽困难
4 3 2013 4. 有关食管的解剖特点,下列不正确的是(2.5分) A.全长约25cm,上端平第6颈椎下缘,下端平11胸椎平面
B.分为颈部、胸部和腹部3段,全程有3处较狭窄,也是肿瘤好发部位
C.颈段食管的血供主要来源于左、右甲状腺下动脉的食管支
D.胸段食管的血供接受纵隔血管的食管支
E.腹段食管的供血主要来自胃左动脉分支,其次为膈下动脉分支
D.胸段食管的血供接受纵隔血管的食管支
4 3 2013 5. 食管癌患者出现声音嘶哑,说明肿瘤已侵及(2.5分) A.迷走神经
B.声带
C.气管隆嵴
D.喉返神经
E.喉上神经
D.喉返神经
4 3 2013 6. 胃溃疡病的病变部位最常见于(2.5分) A.胃大弯
B.胃小弯近贲门处
C.胃前壁
D.胃小弯近幽门处
E.胃后壁
D.胃小弯近幽门处
4 3 2013 7. 胃溃疡病最常见的并发症是(2.5分) A.出血
B.幽门梗阻
C.穿孔
D.粘连
E.癌变
A.出血
4 3 2013 8. 十二指肠溃疡病最好发于(2.5分) A.十二指肠各段
B.十二指肠降部
C.十二指肠球部
D.十二指肠降部和球部
E.十二指肠下段后壁
C.十二指肠球部
4 3 2013 9. 胃癌的最主要转移途径是(2.5分) A.血道转移
B.直接蔓延
C.淋巴道转移
D.消化道内转
E.腹腔内种植
C.淋巴道转移
4 3 2013 10. 胃、肠道好发下列哪种肿瘤?(2.5分) A.肉瘤
B.腺癌
C.转移癌
D.平滑肌瘤
E.横纹肌瘤
B.腺癌
4 3 2013 11. 胃粘膜活体组织检查有肠上皮化生,很可能是来源(2.5分) A.胃溃疡病
B.慢性萎缩性胃炎
C.先天性肠粘膜异位
D.慢性轻度浅表性胃炎
E.肠型胃癌边缘胃组织
B.慢性萎缩性胃炎
4 3 2013 12. 下列哪种类型阑尾炎最易引起阑尾穿孔?(2.5分) A.急性单纯性阑尾炎
B.急性坏疽性阑尾炎
C.慢性阑尾炎
D.阑尾急性蜂窝织炎
E.慢性阑尾炎急性发作
B.急性坏疽性阑尾炎
4 3 2013 13. 结肠癌最常转移的器官是(2.5分) A.肺
B.心
C.肝
D.胰腺
E.脾
C.肝
4 3 2013 14. 阑尾炎最严重的并发症是(2.5分) A.败血症
B.急性弥漫性腹膜炎
C.肝脓肿
D.肾脓肿
E.毒血症
B.急性弥漫性腹膜炎
4 3 2013 15. 直肠癌最常见的症状是(2.5分) A.肠梗阻
B.贫血
C.体重减轻
D.大便习惯改变
E.粘液血便
E.粘液血便
4 3 2013 16. 男性,50岁,反复上腹痛伴消瘦,X线钡餐检查发现胃窦呈持续性向心性狭窄伴充盈缺损,首选的进一步处理(2.5分) A.治疗后复查
B.胃镜及活检
C.B超
D.手术探查
B.胃镜及活检
4 3 2013 17. Following a gastric resection for a stage III gastric cancer the patient asks whether any further therapy will improve their prognosis. Which of the following statements is true? (2.5分) A.chemotherapy is of no use in this setting
B.chemoradiotherapy may improve outcome
C.radiotherapy may improve outcome
D.adjuvant chemotherapy is standard treatment
D.adjuvant chemotherapy is standard treatment
4 3 2013 18. 以下哪项是胃癌癌前疾病(2.5分) A.慢性萎缩性胃炎
B.胃溃疡
C.胃粘膜巨大皱襞症
D.以上全是
D.以上全是
4 3 2013 19. 胃癌最常见、最早的转移方式是(2.5分) A.直接蔓延
B.淋巴转移
C.血行转移
D.种植转移
B.淋巴转移
4 3 2013 20. 早期胃癌的定义是(2.5分) A.局限于胃窦内
B.侵犯粘膜及粘膜下层
C.无淋巴结转移
D.直径在2cm以内
B.侵犯粘膜及粘膜下层
4 3 2013 21. 远端胃切除+毕II式胃空肠吻合术后,远端空肠段梗阻,呕吐物性质为(2.5分) A.呕吐物为食物,无胆汁
B.呕吐物为胆汁,无食物
C.呕吐物既有胆汁,又有食物
D.呕吐物呈酸臭味,有宿食
C.呕吐物既有胆汁,又有食物
4 3 2013 22. 胃癌可以超越常规所属淋巴结转移方式,最常受累的部位为(2.5分) A.胰脾淋巴结
B.肝门淋巴结
C.腹腔淋巴结
D.锁骨上淋巴结
D.锁骨上淋巴结
4 3 2013 23. 患者,男性,48岁,因胃癌行剖腹探查时,发现肿块已浸润至横结肠,但较局限,该病人应该(2.5分) A.行胃癌根治术+横结肠切除术
B.无法手术
C.行全胃切除术
D.行横结肠切除术
A.行胃癌根治术+横结肠切除术
4 3 2013 24. 男性,75岁,胃癌根治术后7天,剧烈咳嗽时,突然出现切口疼痛,并流出少量淡红色液体,病人最可能出现了(2.5分) A.切口内癌细胞种植
B.切口感染
C.切口裂开
D.切口脂肪液化
C.切口裂开
4 3 2013 25. 有关先天性肥厚性幽门狭窄,不正确的是(2.5分) A.thickening of the pyloric portion of the stomach causing obstruction of gastric outflow
B.age distribution almost always between age 6-8 week
C.4-5 times more common in boys than girls
D.highest incidence in first-born infants
E.jaundice seen in 1%-2% of infants with hypertrophic pyloric stenosis
B.age distribution almost always between age 6-8 week
4 3 2013 26. 先天性肥厚性幽门狭窄的钡餐检查,正确的是.(2.5分) A.elongation of pyloric canal
B.pyloric canal outlined by string of contrast material ("string sign")
C.contrast material as linear tracts separated by mucosa ("double-track sign")
D.thickened pylorus causing external impression on gastric antrum ("shoulder sign")
E.Above all is correct.
E.Above all is correct.
4 3 2013 27. 有关先天性巨结肠的病理生理,不正确的是(2.5分) A.Hypertrophic cholinergic nerve trunks
B.Limited number of adrenergic fiber
C.dilated colon found proximal to aganglionic segment
D.absence of submucosal and myenteric sympathetic ganglion cells
E.absence of submucosal and myenteric parasympathetic ganglion cells
D.absence of submucosal and myenteric sympathetic ganglion cells
4 3 2013 28. 先天性巨结肠的常见并发症(2.5分) A.小肠结肠炎
B.肠穿孔
C.败血症
D.肺炎
E.以上均是
E.以上均是
4 3 2013 29. 胃食管连接部的抗反流功能中,下列哪项是最重要的(2.5分) A.diaphragm
B.intra-abdominal portion of the esophagus
C.angle of His
D.lower esophageal sphincter
E.esophageal peristalsis
D.lower esophageal sphincter
4 3 2013 30. 酸性反流指数是指(2.5分) A.Number of reflux episodes
B.Percent total time with a pH<4.0
C.Percent supine time with a pH<4.0
D.Longest reflux episode
E.Number of reflux episodes lasting≥5 min
B.Percent total time with a pH<4.0
4 3 2013 31. 有关胃食管反流的诊断,下列哪项检查最有价值?(2.5分) A.食管pH动态监测
B.食管核素扫描
C.食管钡餐造影
D.食管内镜检查
E.食管pH-阻抗监测
E.食管pH-阻抗监测
4 3 2013 32. 有关胃食管反流,下列哪项是不正确的?(2.5分) A.胃食管反流的症状随年龄不同而异
B.6个月后,大部分婴儿的反流症状好转
C.有关胃食管反流的诊断,主要依赖实验室检查
D.胃食管反流的治疗要强调个体化
E.抑酸治疗是目前最主要的抗反流治疗方法
C.有关胃食管反流的诊断,主要依赖实验室检查
4 3 2013 33. 有关消化道功能的发育,不正确的是(2.5分) A.12 wk gestation: A fetus can swallow amniotic fluid
B.34 wk gestation: Nutritive sucking in neonates first
C.the first few months of infants: coordinated oral and pharyngeal movements for swallowing solids
D.3 mo: show preferences for sweet and salty foods.
E.4 mo: Infants' interest in solids increases.
D.3 mo: show preferences for sweet and salty foods.
4 3 2013 34. 小肠上皮细胞更新时间(2.5分) A.1-2天
B.2-3天
C.4-5天
D.6-7天
E.8-9天
C.4-5天
4 3 2013 35. 口服补液盐配方的主要原理(2.5分) A.Na+- K+ATPase
B.Na+-H+ exchanger
C.Cl--HCO3- exchanger
D.glucose-sodium co-transporter
E.NaCl- coupled pathway
D.glucose-sodium co-transporter
4 3 2013 36. 下列哪种婴儿粪便的颜色与黄疸发病机制相关?(2.5分) A.金黄色
B.墨绿色
C.白陶土色
D.暗褐色
E.淡黄色
C.白陶土色
4 3 2013 37. 消化道出血,下列哪项不相关?(2.5分) A.门脉高压出血
B.胰腺假性囊肿
C.麦克尔憩室
D.肠重复畸形
E.胆道出血
B.胰腺假性囊肿
4 3 2013 38. 胆汁淤积,是由于下列物质排泄不畅引起的(2.5分) A.胆固醇
B.直接胆红素
C.胆汁酸
D.微量元素
E.以上都是
E.以上都是
4 3 2013 39. Alagille综合征的特征性面容,哪项不正确(2.5分) A.broad forehead
B.wide between eyes
C.deep-set eyes
D.straight nose
E.pointed chin
B.wide between eyes
4 3 2013 40. 胆道闭锁的临床表现,哪项不是(2.5分) A.生后2周起黄疸持续加深
B.生后4周起黄疸退而复现
C.营养不良
D.肝脾肿大
E.灰白色粪便、深色尿
B.生后4周起黄疸退而复现
4 3 2014 1. 早期食管癌的症状除外(2.5分) A.吞咽哽噎感
B.食管内异物感
C.胸骨后烧灼样痛
D.声音嘶哑
E.可无任何症状
D.声音嘶哑
4 3 2014 2. 中晚期食管癌病理分型包括(2.5分) A.斑块型,蕈伞型,溃疡型,乳头型
B.乳头型,蕈伞型,溃疡型,缩窄型
C.髓质型,蕈伞型,溃疡型,乳头型
D.髓质型,蕈伞型,溃疡型,斑块型
E.髓质型,蕈伞型,溃疡型,缩窄型
E.髓质型,蕈伞型,溃疡型,缩窄型
4 3 2014 3. 下列哪项不是食管癌的手术禁忌(2.5分) A.严重吞咽困难
B.声音嘶哑
C.气管食管瘘
D.左锁骨上淋巴结转移
E.严重恶病质者
A.严重吞咽困难
4 3 2014 4. 下列哪项是食管癌切除,消化道重建的最常用手术方法(2.5分) A.结肠代食管术
B.空肠代食管术
C.食管胃转流吻合术
D.胃代食管吻合术
E.回肠代食管胃吻合术
D.胃代食管吻合术
4 3 2014 5. 食管癌患者出现声音嘶哑,说明肿瘤已侵及(2.5分) A.迷走神经
B.声带
C.气管隆突
D.喉返神经
E.膈神经
D.喉返神经
4 3 2014 6. 食管癌患者出现进食呛咳,最可能的原因是(2.5分) A.上段食管癌梗阻
B.食管气管瘘
C.肿瘤侵犯迷走神经
D.肿瘤侵及气管
E.患者伴有气管炎
B.食管气管瘘
4 3 2014 7. 关于食管鳞癌的淋巴转移特点,下列错误的是(2.5分) A.仅次于血行转移的转移途径
B.上段食管癌常转移至锁骨上及颈淋巴结
C.中下段多转移至气管旁、贲门及胃左动脉旁淋巴结
D.各段均可向上或向下方的淋巴结转移
E.约1/4病例的淋巴结转移是跳跃式
A.仅次于血行转移的转移途径
4 3 2014 8. 根据AJCC第8版食管鳞癌TNM分期中,如肿瘤侵及食管外膜,则T分级是以下哪项(2.5分) A.ATis
B.T1C
C.2D
D.T3E
E.T4
D.T3E
4 3 2014 9. 先天性肥厚性幽门狭窄典型的"三联征"(2.5分) A.nonbilious projectile vomiting, visible peristalsis, starvation
B.bilious projectile vomiting, visible peristalsis, palpable "olive"
C.nonbilious projectile vomiting, jaundice, palpable "olive"
D.nonbilious projectile vomiting, visible peristalsis, palpable "olive"
E.nonbilious projectile vomiting, dehydration, palpable "olive"
D.nonbilious projectile vomiting, visible peristalsis, palpable "olive"
4 3 2014 10. 下列哪项不符合先天性肥厚性幽门狭窄的临床特征(2.5分) A.forceful (projectile) nonbilious vomiting
B.bad appetite
C.visible peristalsis
D.hypochloremic alkalosis
E.poor weight gain
B.bad appetite
4 3 2014 11. 先天性巨结肠的临床表现(2.5分) A.胎便排出延迟
B.顽固性便秘
C.腹胀
D.直肠指检壶腹部空虚
E.以上均是
E.以上均是
4 3 2014 12. 先天性巨结肠的常见并发症(2.5分) A.小肠结肠炎
B.肠穿孔
C.败血症
D.肺炎
E.以上均是
E.以上均是
4 3 2014 13. 胃食管连接部的抗反流功能中,下列哪项是最重要的(2.5分) A.Diaphragm
B.intra-abdominal portion of the esophagus
C.angle of His
D.lower esophageal sphincter
E.esophageal peristalsis
D.lower esophageal sphincter
4 3 2014 14. 下列哪项不是胃食管反流的典型症状?(2.5分) A.Heartburn
B.Vomiting
C.feeding difficulties
D.chest pain
E.regurgitation
C.feeding difficulties
4 3 2014 15. 阻抗值大小,下列哪项正确?(2.5分) A.空气﹥反流物﹥唾液﹥食物
B.反流物﹥唾液﹥空气﹥食物
C.空气﹥唾液﹥食物﹥反流物
D.唾液﹥空气﹥反流物﹥食物
E.反流物﹥唾液﹥食物﹥空气
C.空气﹥唾液﹥食物﹥反流物
4 3 2014 16. 有关钡餐造影,不正确的是(2.5分) A.常用于诊断胃食管反流病
B.可观察食管黏膜的完整性
C.常用于诊断食管解剖结构的异常
D.常用于诊断食管裂孔疝
E.可评估食管动力
A.常用于诊断胃食管反流病
4 3 2014 17. 单纯母乳喂养儿肠道中占绝对优势的菌群是(2.5分) A.大肠杆菌
B.嗜酸乳杆菌
C.肠球菌
D.双歧杆菌
E.芽胞杆菌
D.双歧杆菌
4 3 2014 18. 双糖酶活性主要位于(2.5分) A.绒毛上2/3上皮细胞
B.绒毛下1/3上皮细胞
C.隐窝部细胞
D.绒毛顶部上皮细胞
E.肠杯状细胞
A.绒毛上2/3上皮细胞
4 3 2014 19. 下列哪种婴儿粪便的颜色与黄疸发病机制相关?(2.5分) A.金黄色
B.墨绿色
C.白陶土色
D.暗褐色
E.淡黄色
C.白陶土色
4 3 2014 20. 消化道出血,下列哪项不相关?(2.5分) A.门脉高压出血
B.胰腺假性囊肿
C.麦克尔憩室
D.肠重复畸形
E.胆道出血
B.胰腺假性囊肿
4 3 2014 21. 有关病理性黄疸,不正确的是(2.5分) A.It appears in the 1st 24-36hr of life
B.Jaundice persists after one month of life.
C.Serum bilirubin is rising at a rate faster than 5 mg/dl/24hr.
D.Serum bilirubin is ≥ 12 mg/dl in full-term or 10-14 mg/dl in preterm infants.
E.Direct-reacting bilirubin is ≥2 mg/dl at any time
B.Jaundice persists after one month of life.
4 3 2014 22. Alagille综合征的饮食治疗(2.5分) A.补充脂肪
B.无乳糖饮食
C.补充中链甘油三酯
D.补充短肽
E.低敏配方奶粉
C.补充中链甘油三酯
4 3 2014 23. 非胆汁淤积性黄疸,哪项不正确(2.5分) A.生理性黄疸
B.胆道闭锁
C.溶血性黄疸
D.母乳性黄疸
E.先天性非溶血性胆红素增高症
B.胆道闭锁
4 3 2014 24. 胆道闭锁的肝脏组织学检查,可发现(2.5分) A.Cholestasis
B.bile duct proliferation
C.extramedullary hematopoiesis
D.giant cell transformation of hepatocytes
E.above all is correct
E.above all is correct
4 3 2014 25. 以下哪项是胃癌癌前疾病(2.5分) A.慢性萎缩性胃炎
B.胃溃疡
C.胃粘膜巨大皱襞症
D.以上全是
D.以上全是
4 3 2014 26. 胃癌最常见、最早的转移方式是(2.5分) A.直接蔓延
B.淋巴转移
C.血行转移
D.种植转移
B.淋巴转移
4 3 2014 27. 早期胃癌的定义是(2.5分) A.局限于胃窦内
B.侵犯粘膜及粘膜下层
C.无淋巴结转移
D.直径在2cm以内
B.侵犯粘膜及粘膜下层
4 3 2014 28. 远端胃切除+毕II式胃空肠吻合术后,远端空肠段梗阻,呕吐物性质为(2.5分) A.呕吐物为食物,无胆汁
B.呕吐物为胆汁,无食物
C.呕吐物既有胆汁,又有食物
D.呕吐物呈酸臭味,有宿食
C.呕吐物既有胆汁,又有食物
4 3 2014 29. 哪项不属于胃溃疡的并发症( )(2.5分) A.梗阻
B.穿孔
C.出血
D.癌变
E.粘连
E.粘连
4 3 2014 30. 食管癌最常见的组织学类型是(2.5分) A.腺癌
B.粘液癌
C.未分化癌
D.腺棘皮癌
E.鳞状细胞癌
E.鳞状细胞癌
4 3 2014 31. 胃溃疡的显微镜下结构不包括(2.5分) A.渗出层
B.坏死层
C.出血层
D.幽门梗阻
E.癌变
C.出血层
4 3 2014 32. 哪个不是血道转移癌的特点(2.5分) A.多发性
B.结节性
C.界限不清
D.边缘性
E.癌脐
C.界限不清
4 3 2014 33. 胃溃疡病的病变部位最常见于(2.5分) A.胃大弯
B.胃小弯近贲门处
C.胃前壁
D.胃小弯近幽门处
E.胃后壁
D.胃小弯近幽门处
4 3 2014 34. 十二指肠溃疡病最好发于(2.5分) A.十二指肠各段
B.十二指肠降部
C.十二指肠球部
D.十二指肠降部和球部
E.十二指肠下段后壁
C.十二指肠球部
4 3 2014 35. 胃粘膜活体组织检查有肠上皮化生,很可能是来源(2.5分) A.胃溃疡病
B.慢性萎缩性胃炎
C.先天性肠粘膜异位
D.慢性轻度浅表性胃炎
E.肠型胃癌边缘胃组织
B.慢性萎缩性胃炎
4 3 2014 36. 结肠癌最常转移的器官是(2.5分) A.肺
B.心
C.肝
D.胰腺
E.脾
C.肝
4 3 2014 37. 胰腺癌最多见的组织学类型是(2.5分) A.腺泡细胞癌
B.导管腺癌
C.多形性腺癌
D.纤毛细胞癌
E.鳞状上皮癌
B.导管腺癌
4 3 2014 38. 胰腺癌与总胆管结石的主要鉴别点是(2.5分) A.进行性黄疸
B.肝功能改变
C.淀粉酶改变
D.胆囊肿大
E.皮肤瘙痒
A.进行性黄疸
4 3 2014 39. 胰腺癌最好发的部位是(2.5分) A.胰腺头部
B.胰腺体部
C.胰腺尾部
D.全胰腺
E.异位胰腺
A.胰腺头部
4 3 2014 40. 以下哪种不属于胰腺囊性肿瘤(2.5分) A.浆液性囊性肿瘤
B.黏液性囊性肿瘤
C.胰腺假性囊肿
D.实性假乳头状瘤
E.导管内乳头状黏液瘤
C.胰腺假性囊肿
4 3 2015 1. 常见于胰腺癌的临床表现不包括哪些(2.5分) A.食欲不振
B.便秘
C.黑便
D.黄疸
E.消瘦
B.便秘
4 3 2015 2. 诊断胰岛素瘤的最准确方法是(2.5分) A.Wipple三联征
B.饥饿试验
C.葡萄糖耐量试验
D.血胰岛素测定
E.血糖空腹时2.2mmol.L以下
D.血胰岛素测定
4 3 2015 3. 胃溃疡病最常见的并发症是(2.5分) A.出血
B.幽门梗阻
C.穿孔
D.粘连
E.癌变
A.出血
4 3 2015 4. 阑尾炎最严重的并发症是(2.5分) A.败血症
B.急性弥漫性腹膜炎
C.肝脓肿
D.肾脓肿
E.毒血症
B.急性弥漫性腹膜炎
4 3 2015 5. 根据AJCC第8版食管癌TNM分期,关于食管分段叙述正确是(2.5分) A.颈段食管:环状软骨至胸骨角平面
B.胸上段食管:胸骨角向下至主动脉弓上缘水平
C.胸中段食管:奇静脉弓上缘向下至下肺静脉下缘水平
D.胸下段食管:下肺静脉下缘水平向下至食管裂孔上缘
E.腹段食管:食管裂孔下缘至胃食管交界处
D.胸下段食管:下肺静脉下缘水平向下至食管裂孔上缘
4 3 2015 6. 中晚期食管癌病理分型包括(2.5分) A.斑块型,蕈伞型,溃疡型,乳头型
B.乳头型,蕈伞型,溃疡型,缩窄型
C.髓质型,蕈伞型,溃疡型,乳头型
D.髓质型,蕈伞型,溃疡型,斑块型
E.髓质型,蕈伞型,溃疡型,缩窄型
E.髓质型,蕈伞型,溃疡型,缩窄型
4 3 2015 7. 下列哪项不是食管癌的手术禁忌(2.5分) A.严重吞咽困难
B.声音嘶哑
C.气管食管瘘
D.左锁骨上淋巴结转移
E.严重恶病质者
A.严重吞咽困难
4 3 2015 8. 有关食管的解剖特点,下列不正确的是(2.5分) A.全长约25cm,上端平第6颈椎下缘,下端平11胸椎平面
B.分为颈部、胸部和腹部3段,全程有3处较狭窄,也是肿瘤好发部位
C.颈段食管的血供主要来源于左、右甲状腺下动脉的食管支
D.胸段食管的血供接受纵隔血管的食管支
E.腹段食管的供血主要来自胃左动脉分支,其次为膈下动脉分支
D.胸段食管的血供接受纵隔血管的食管支
4 3 2015 9. 食管癌的发病具有明显的地理分布特征,食管癌的高发区除外(2.5分) A.太行山麓
B.川北
C.闽粤交界区
D.云南
E.福建
D.云南
4 3 2015 10. 食管癌患者出现声音嘶哑,说明肿瘤已侵及(2.5分) A.迷走神经
B.声带
C.气管隆突
D.喉返神经
E.膈神经
D.喉返神经
4 3 2015 11. 关于食管鳞癌的淋巴转移特点,下列错误的是(2.5分) A.仅次于血行转移的转移途径
B.上段食管癌常转移至锁骨上及颈淋巴结
C.中下段多转移至气管旁、贲门及胃左动脉旁淋巴结
D.各段均可向上或向下方的淋巴结转移
E.约1/4病例的淋巴结转移是跳跃式
A.仅次于血行转移的转移途径
4 3 2015 12. 根据AJCC第8版食管鳞癌TNM分期中,如肿瘤侵及食管外膜,则T分级是(2.5分) A.Tis
B.T1
C.T2
D.T3
E.T4
D.T3
4 3 2015 13. 有关先天性肥厚性幽门狭窄,不正确的是(2.5分) A.thickening of the pyloric portion of the stomach causing obstruction of gastric outflow
B.age distribution almost always between age 3-8 week
C.1-2 times more common in boys than girls
D.highest incidence in first-born infants
E.jaundice seen in 1%-2% of infants with hypertrophic pyloric stenosis
C.1-2 times more common in boys than girls
4 3 2015 14. 先天性肥厚性幽门狭窄的B超检查,下列哪项为阳性结果(2.5分) A.pylorus muscle wall thickness > 4 mm
B.pylorus muscle wall thickness < 3 mm
C.pylorus canal length < 14 mm
D.pylorus diameter < 12 mm
E.absent or minimal vascular flow
A.pylorus muscle wall thickness > 4 mm
4 3 2015 15. 下列哪项不符合先天性肥厚性幽门狭窄的临床特征(2.5分) A.forceful (projectile) nonbilious vomiting
B.bad appetite
C.visible peristalsis
D.hypochloremic alkalosis
E.poor weight gain
B.bad appetite
4 3 2015 16. 先天性巨结肠的临床表现(2.5分) A.胎便排出延迟
B.顽固性便秘
C.腹胀
D.直肠指检壶腹部空虚
E.以上均是
E.以上均是
4 3 2015 17. 有关病理性黄疸,不正确的是(2.5分) A.It appears in the 1st 24-36hr of life
B.Jaundice persists after one month of life.
C.Serum bilirubin is rising at a rate faster than 5 mg/dl/24hr.
D.Serum bilirubin is ≥ 12 mg/dl in full-term or 10-14 mg/dl in preterm infants.
E.Direct-reacting bilirubin is ≥2 mg/dl at any time
B.Jaundice persists after one month of life.
4 3 2015 18. 有关婴儿胆汁淤积症,下列哪些不符合(2.5分) A.1岁以下
B.肝细胞性黄疸
C.肝脏肿大
D.脾脏肿大
E.肝功能异常
D.脾脏肿大
4 3 2015 19. 肝内疾病引起的胆汁淤积症,不正确的是(2.5分) A.特发性新生儿肝炎
B.Alagille综合征
C.Citrin缺乏症
D.硬化性胆管炎
E.先天性肝纤维化
D.硬化性胆管炎
4 3 2015 20. 胆道闭锁的临床表现,哪项不是(2.5分) A.生后2周起黄疸持续加深
B.生后4周起黄疸退而复现
C.营养不良
D.肝脏肿大
E.灰白色粪便、深色尿
B.生后4周起黄疸退而复现
4 3 2015 21. 单纯母乳喂养儿肠道中占绝对优势的菌群是(2.5分) A.大肠杆菌
B.嗜酸乳杆菌
C.肠球菌
D.双歧杆菌
E.芽胞杆菌
D.双歧杆菌
4 3 2015 22. 双糖酶活性主要位于(2.5分) A.绒毛上2/3上皮细胞
B.绒毛下1/3上皮细胞
C.隐窝部细胞
D.绒毛顶部上皮细胞
E.肠杯状细胞
A.绒毛上2/3上皮细胞
4 3 2015 23. 下列哪种婴儿粪便的颜色与黄疸发病机制相关?(2.5分) A.金黄色
B.墨绿色
C.白陶土色
D.暗褐色
E.淡黄色
C.白陶土色
4 3 2015 24. 小肠上皮细胞更新时间(2.5分) A.1-2天
B.2-3天
C.4-5天
D.6-7天
E.8-9天
C.4-5天
4 3 2015 25. 消化道出血,下列哪项不相关?(2.5分) A.门脉高压出血
B.胰腺假性囊肿
C.麦克尔憩室
D.肠重复畸形
E.胆道出血
B.胰腺假性囊肿
4 3 2015 26. 胃食管反流的主要原因是(2.5分) A.LES压力降低
B.食管廓清能力下降
C.食管粘膜的屏障功能降低
D.胃、十二指肠功能失常
E.His角较大
A.LES压力降低
4 3 2015 27. 下列哪项不是胃食管反流的典型症状?(2.5分) A.heartburn
B.vomiting
C.feeding difficulties
D.chest pain
E.regurgitation
C.feeding difficulties
4 3 2015 28. 质子泵抑制是由于抑制了下列哪种酶而抑制了胃酸的分泌?(2.5分) A.H-K-ATPase
B.Na-K-ATPase
C.Cl-K-ATPase
D.Na-K-GTPase
E.H-K-GTPase
B.Na-K-ATPase
4 3 2015 29. 有关胃食管反流的体位治疗,下列哪项最有疗效?(2.5分) A.俯卧位
B.仰卧位
C.右侧卧位
D.左侧卧位
E.上述均不是
D.左侧卧位
4 3 2015 30. 男性,50岁,反复上腹痛伴消瘦,X线钡餐检查发现胃窦呈持续性向心性狭窄伴充盈缺损,首选的进一步处理(2.5分) A.治疗后复查
B.胃镜及活检
C.B超
D.手术探查
E.胃液分析
B.胃镜及活检
4 3 2015 31. 以下哪项是胃癌癌前病变(2.5分) A.慢性萎缩性胃炎
B.胃溃疡
C.胃粘膜巨大皱襞症
D.胃大部切除术后残胃
E.以上全是
E.以上全是
4 3 2015 32. 胃癌最常见、最早的转移方式是(2.5分) A.直接蔓延
B.淋巴转移
C.血行转移
D.种植转移
E.以上都不是
B.淋巴转移
4 3 2015 33. 早期胃癌的定义是(2.5分) A.局限于胃窦内
B.侵犯粘膜及粘膜下层
C.侵犯固有肌层
D.无淋巴结转移
E.直径在2cm以内
B.侵犯粘膜及粘膜下层
4 3 2015 34. 远端胃切除+毕II式胃空肠吻合术后,远端空肠段梗阻,呕吐物性质为(2.5分) A.呕吐物为食物,无胆汁
B.呕吐物为胆汁,无食物
C.呕吐物既有胆汁,又有食物
D.呕吐物呈酸臭味,有宿食
E.以上都不是
C.呕吐物既有胆汁,又有食物
4 3 2015 35. 胃癌可以超越常规所属淋巴结转移方式,最常受累的部位为(2.5分) A.胰脾淋巴结
B.肝门淋巴结
C.腹腔淋巴结
D.锁骨上淋巴结
E.胸腔淋巴结
D.锁骨上淋巴结
4 3 2015 36. 下列哪项与胃癌的发生有关(2.5分) A.过多食用腌制烟熏食品
B.幽门螺杆菌感染
C.遗传因素
D.胃大部切除术后残胃
E.以上都是
E.以上都是
4 3 2015 37. 患者,男性,63岁,3月余前无明显诱因下出现上腹饱胀感,无腹痛腹泻、反酸嗳气、恶心呕吐等不适。查胃镜及病检提示:胃角溃疡,病检提示:胃角高级别上皮内瘤变,局灶癌变。全腹CT平扫+增强提示:胃角小弯侧胃壁稍增厚。该患者术中应切除胃的范围至少是(2.5分) A.胃远端的1/3至1/2
B.胃远端的1/2至2/3
C.胃远端的2/3至3/4
D.全胃切除
E.保留全胃的局部切除
C.胃远端的2/3至3/4
4 3 2015 38. 患者,男性,66岁,主诉"餐后饱胀、反酸、嗳气1年余",来我院门诊,查肿瘤标志物提示:CEA 4.3 ng/ml,AFP 130.3 ng/ml,CA-199 12.0U/ml。胃镜提示:胃窦占位,胃潴留。胃镜病理:胃(窦后壁)腺癌,既往无甲肝、乙肝、戊肝等肝炎病史,为进一步评估患者,下列检查哪项最合适(2.5分) A.B超
B.全腹部增强CT
C.超声胃镜
D.全身PET-CT
E.骨骼ECT
B.全腹部增强CT
4 3 2015 39. 患者,男,62岁,因"体检发现胃壁增厚1月",患者31月前体检查胃镜见胃壁广泛增厚,未见溃疡,取活检提示:高级别上皮内瘤变;一月后再次行胃镜复查活检提示:高级别上皮内瘤变,局灶癌变。增强CT提示胃窦胃角胃壁增厚,最厚处约1.8cm.该患者胃癌大体分型是什么(2.5分) A.早癌IIc型
B.Borrman I型
C.Borrman II型
D.Borrman III型
E.Borrman IV型
E.Borrman IV型
4 3 2015 40. 患者,女性,65岁。诊断胃癌,术前检查未提示远处转移。行胃癌根治术后,病理报告为:浸润至黏膜肌层,送检淋巴结27枚未见癌转移。切缘阴性。该患者的5年生存率约为(2.5分) A.90%
B.75%
C.50%
D.25%
E.10%
A.90%
4 4 2006 1. 糖皮质激素类药物与水盐代谢相关的不良反应是(2.0分) A.多毛
B.痤疮
C.胃、十二指肠溃疡
D.高血压
E.向心性肥胖
D.高血压
4 4 2006 2. Insulin的不良反应有(2.0分) A.骨髓抑制
B.低血糖昏迷
C.高血糖高渗性昏迷
D.酮症酸中毒
E.乳酸血症
B.低血糖昏迷
4 4 2006 3. 关于糖尿病病因和发病机制的论述,下列说法正确的是:(2.0分) A.环境因素导致胰岛素分泌不足
B.遗传与环境因素共同参与其发病过程
C.主要与遗传及免疫因素有关
D.遗传易感性是其主要发病因素
B.遗传与环境因素共同参与其发病过程
4 4 2006 4. 甲状腺激素主要促进哪些组织生长发育(2.0分) A.神经系统和肌肉
B.骨骼和肌肉
C.内脏和骨骼
D.神经系统和骨骼
E.肌肉和内脏
D.神经系统和骨骼
4 4 2006 5. 下列哪一项不属于下丘脑调节肽(2.0分) A.促甲状腺激素释放激素
B.促性腺激素释放激素
C.抗利尿激素
D.生长抑素
E.促肾上腺皮质激素释放激素
C.抗利尿激素
4 4 2006 6. 能促进胰岛β细胞释放insulin的降血糖药是 (2.0分) A.小剂量insulin
B.Phenformin(苯乙福明)
C.Tolbutamide
D.Carbimazole
E.精蛋白锌insulin
C.Tolbutamide
4 4 2006 7. 糖皮质激素不宜用于胃溃疡患者是因为糖皮质激素能:(2.0分) A.抑制糖的利用,使组织能源减少
B.分解蛋白质,影响伤口愈合
C.促进盐酸和胃蛋白酶原的分泌,加剧溃疡病变
D.使胃肠道血管收缩,血供减少
E.抑制纤维母细胞的增殖和功能,延长溃疡愈合
C.促进盐酸和胃蛋白酶原的分泌,加剧溃疡病变
4 4 2006 8. 下列关于催产素的叙述,哪一项是错误的?(2.0分) A.由下丘脑合成
B.由神经垂体释放
C.促进妊娠期乳腺生长发育
D.促进妊娠子宫收缩
E.促进哺乳期乳腺排乳
C.促进妊娠期乳腺生长发育
4 4 2006 9. Wolff-Chaikoff效应是指:(2.0分) A.过量的酪氨酸所产生的抗甲状腺效应
B.过量的碘所产生的抗甲状腺效应
C.缺乏酪氨酸产生的抗甲状腺效应
D.缺乏碘而产生的抗甲状腺效应
E.碘和酪氨酸缺乏而产生的促甲状腺效应
B.过量的碘所产生的抗甲状腺效应
4 4 2006 10. Which of the following hormones exerts the least effect on growth?(2.0分) A.T4
B.Growth hormone
C.Insulin
D.Testosterone
E.Vasopressin
E.Vasopressin
4 4 2006 11. 食物中长期缺碘可引起:(2.0分) A.腺垂体功能减退
B.甲状腺功能亢进
C.甲状腺组织萎缩
D.单纯性甲状腺肿
E.神经垂体功能减退
D.单纯性甲状腺肿
4 4 2006 12. In pars nervosa of hypophysis, Herring bodies are composed of (2.0分) A.neuroglial cell
B.groups of neurosecretory granules
C.dendrite of neuron
D.dendrite of neuron
E.axon of neuron
B.groups of neurosecretory granules
4 4 2006 13. 与甲状腺功能减退(hypothyroidism)发生无关的因素是(2.0分) A.慢性淋巴细胞性甲状腺炎
B.手术切除或放射线治疗损伤过多正常甲状腺组织
C.自身免疫性损伤
D.TGI抗体
D.TGI抗体
4 4 2006 14. 非肾上腺皮质增生所引起的临床表现是(2.0分) A.低血糖
B.高血压
C.满月脸
D.低血钾
A.低血糖
4 4 2006 15. 临床上长期大剂量使用糖皮质激素时可以引起:(2.0分) A.淋巴细胞数目增加
B.肾上腺皮质渐趋萎缩
C.血中ACTH浓度增高
D.肢端肥大症
E.CHR分泌增多
B.肾上腺皮质渐趋萎缩
4 4 2006 16. 内分泌系统的反馈调节是指: (2.0分) A.免疫系统对内分泌系统的调节
B.内分泌系统对神经系统的调节
C.神经系统对内分泌系统的调节
D.下丘脑-垂体-靶腺之间的相互调节
D.下丘脑-垂体-靶腺之间的相互调节
4 4 2006 17. Insulin的适应证有(2.0分) A.糖尿病合并酮症酸中毒
B.重症糖尿病(1型)
C.糖尿病合并严重感染
D.口服降血糖药无效的非胰岛素依赖性糖尿病
E.以上都是
E.以上都是
4 4 2006 18. 2型糖尿病的发病机制是:(2.0分) A.胰岛B细胞自身免疫反应性损伤
B.胰岛B细胞遗传性缺陷
C.胰岛B细胞破坏,胰岛素绝对不足
D.胰岛素抵抗和胰岛素分泌缺陷
D.胰岛素抵抗和胰岛素分泌缺陷
4 4 2006 19. Thyroid hormone作用的主要机制是(2.0分) A.与核受体结合,促进cGMP生成
B.与核受体结合,刺激mRNA生成
C.与膜受体结合,促进cAMP生成
D.与膜受体结合,抑制cAMP生成
B.与核受体结合,刺激mRNA生成
4 4 2006 20. 以胰高血糖素样肽-1(GLP-1)为靶点的降血糖药物是 (2.0分) A.醋酸普兰林肽
B.依克那肽
C.氯磺丙脲
D.罗格列酮
E.阿卡波糖
B.依克那肽
4 4 2006 21. 胰岛素缺乏时可导致(2.0分) A.细胞内葡萄糖增加
B.无尿糖
C.血糖浓度下降
D.肝糖原合成减少
E.促进蛋白质合成
D.肝糖原合成减少
4 4 2006 22. On electron microscopy, the most characteristic feature of component cells of the zona fasciculat is (2.0分) A.well-developed SER and numerous mitochondria with tubular cristae and lipid droplets
B.well-developed RER and numerous mitochondria
C.well-developed SER and numerous mitochondria
D.numerous SER and RER
E.well-developed RER and numerous mitochondria with tubular cristae and lipid droplets
A.well-developed SER and numerous mitochondria with tubular cristae and lipid droplets
4 4 2006 23. 促进胰高血糖素分泌的主要因素是:(2.0分) A.高血糖
B.血中氨基酸增多
C.低血糖
D.血中脂肪酸减少
E.迷走神经兴奋
C.低血糖
4 4 2006 24. 影响神经系统发育的最重要的激素是:(2.0分) A.胰岛素
B.生长素
C.甲状腺激素
D.糖皮质激素
E.肾上腺素
C.甲状腺激素
4 4 2006 25. 关于激素的正确概念是(2.0分) A.直接作用于靶细胞膜受体而起作用
B.只能加强靶细胞的功能活动
C.由内分泌细胞产生的具有生物活性的物质
D.每种激素对其他激素的作用均无影响
C.由内分泌细胞产生的具有生物活性的物质
4 4 2006 26. 将动物去除双侧肾上腺后,容易引起死亡的原因是(2.0分) A.失去"应急"反应能力
B. "应激"反应能力过强
C.儿茶酚胺缺乏引起的血管功能紊乱
D.水盐代谢紊乱和物质代谢紊乱
E.以上都对
D.水盐代谢紊乱和物质代谢紊乱
4 4 2006 27. 长期大量注射氢化可的松时主要可引起血中:(2.0分) A.红细胞减少
B.血糖下降
C.ACTH↓
D.血钙升高
E.CRH↑
C.ACTH↓
4 4 2006 28. 糖皮质激素用于急性炎症的主要目的在于(2.0分) A.降低血管通透性,减轻肿胀、疼痛等症状
B.增强机体防御功能,促进炎症消退
C.加快消除致病因素,促进炎症消退
D.改善葡萄糖、蛋白质、水盐代谢,提高机体抗病能力
E.抑制肉芽组织生长,防止粘连和疤痕形成,减轻后遗症
A.降低血管通透性,减轻肿胀、疼痛等症状
4 4 2006 29. 肾上腺髓质的描述,哪项正确 (2.0分) A.嗜铬细胞只分泌肾上腺素
B.髓质细胞又称嗜铬细胞
C.腺内只有嗜铬细胞
D.位于肾上腺的被膜下
E.嗜铬细胞只分泌去甲肾上腺素
B.髓质细胞又称嗜铬细胞
4 4 2006 30. 关于突眼和甲亢的关系(2.0分) A.有甲亢一定有浸润性突眼
B.有浸润性突眼一定同时有甲亢
C.甲亢越严重突眼越明显
D.突眼的程度与甲亢轻重无平行关系
D.突眼的程度与甲亢轻重无平行关系
4 4 2006 31. 甲状腺素的主要适应证是 (2.0分) A.甲亢的手术前准备
B.轻、中度甲状腺功能亢进
C.甲状腺危象
D.交感神经活性增强引起的病变
E.黏液性水肿
E.黏液性水肿
4 4 2006 32. 胰岛素分泌不足将产生(2.0分) A.侏儒症
B.呆小症
C.阿狄森病
D.糖尿病
E.柯兴综合征
D.糖尿病
4 4 2006 33. 糖皮质激素抗炎作用的基本机制是(2.0分) A.抑制毛细血管和纤维母细胞的增生
B.稳定溶酶体膜
C.减轻渗出、水肿、毛细血管扩张等炎症反应
D.影响了参与炎症的一些基因转录
E.增加肥大细胞颗粒的稳定性
D.影响了参与炎症的一些基因转录
4 4 2006 34. 以下哪一项对糖皮质激素作用的描述是错误的?(2.0分) A.能对抗各种原因(如化学、物理、生物、免疫等)引起的炎症反应
B.能抑制巨噬细胞对抗原的吞噬和处理
C.具有抗休克作用
D.具有强大的抗炎作用
E.可对抗细菌内毒素而缓解中毒性休克的症状
E.可对抗细菌内毒素而缓解中毒性休克的症状
4 4 2006 35. 糖皮质激素用于急性炎症的主要目的在于 (2.0分) A.降低血管通透性,减轻肿胀、疼痛等症状
B.增强机体防御功能,促进炎症消退
C.加快消除致病因素,促进炎症消退
D.改善葡萄糖、蛋白质、水盐代谢,提高机体抗病能力
E.抑制肉芽组织生长,防止粘连和疤痕形成,减轻后遗症
A.降低血管通透性,减轻肿胀、疼痛等症状
4 4 2006 36. 糖皮质激素用于严重感染的目的在于 (2.0分) A.有抗菌和抗毒素作用
B.具有中和抗毒作用,提高机体对毒素的耐受力
C.利用其强大的抗炎作用,缓解症状,使病人度过危险期
D.由于加强心肌收缩力,帮助病人度过危险期
E.消除危害机体的炎症和过敏反应
C.利用其强大的抗炎作用,缓解症状,使病人度过危险期
4 4 2006 37. 糖皮质激素抗炎作用的基本机制是 (2.0分) A.稳定溶酶体膜
B.影响了参与炎症的一些基因转录
C.减轻渗出、水肿、毛细血管扩张等炎症反应
D.抑制毛细血管和纤维母细胞的增生
E.增加肥大细胞颗粒的稳定性
B.影响了参与炎症的一些基因转录
4 4 2006 38. 下列哪种激素对腺垂体具有负反馈作用?(2.0分) A.肾上腺素
B.肾素
C.促肾上腺皮质激素
D.氢化可的松
E.胰岛素
D.氢化可的松
4 4 2006 39. Propylthiouracil的抗甲状腺作用主要在于 (2.0分) A.抑制甲状腺中碘的有机化过程
B.作用于甲状腺细胞膜受体
C.作用于甲状腺细胞核内受体
D.抑制已合成的甲状腺素的释放
E.以上都不是
A.抑制甲状腺中碘的有机化过程
4 4 2006 40. 成人生长素分泌过多将导致: (2.0分) A.粘液水肿
B.腺瘤
C.侏儒症
D.甲状腺肿
E.肢端肥大症
E.肢端肥大症
4 4 2006 41. 关于胰岛素的正确论述是:(2.0分) A.由胰岛的A细胞合成分泌
B.血糖浓度对胰岛素分泌有负反馈作用
C.具有降血糖作用
D.促进脂肪的分解
E.交感神经兴奋,胰岛素分泌增加
C.具有降血糖作用
4 4 2006 42. 糖皮质激素用于严重感染的目的在于(2.0分) A.有抗菌和抗毒素作用
B.由于加强心肌收缩力,帮助病人度过危险期
C.具有中和抗毒作用,提高机体对毒素的耐受力
D.利用其强大的抗炎作用,缓解症状,使病人度过危险期
E.消除危害机体的炎症和过敏反应
C.具有中和抗毒作用,提高机体对毒素的耐受力
4 4 2006 43. 胰岛中的β细胞可以 (2.0分) A.分泌糖原
B.合成、释放胰岛素
C.分泌可的松
D.合成甘油三酯
E.产生肾上腺素
B.合成、释放胰岛素
4 4 2006 44. 不能用于治疗甲状腺危象的药物是(2.0分) A.Propranolol
B.Tolbutamide
C.Propylthiouracil
D.大剂量碘剂
E.Carbimazole
B.Tolbutamide
4 4 2006 45. A scientist finds that infusion of growth hormone into the median eminence of the hypothalamus in experimental animals inhibits the secretion of growth hormone, and concludes that this proves that growth hormone feeds back to inhibit GRH secretion. Do you(2.0分) A.No, because substances placed in the median eminence could be transported to the anterior pituitary
B.No, because the infused growth hormone could be stimulating dopamine secretion
C.No, because growth hormone does not cross the blood-brain barrier
D.Yes, because systemically administered growth hormone inhibits growth hormone secretion
E.Yes, because growth hormone binds GRH, inactivating it
A.No, because substances placed in the median eminence could be transported to the anterior pituitary
4 4 2006 46. 以下哪一项对糖皮质激素作用的描述是错误的?(2.0分) A.能对抗各种原因(如化学、物理、生物、免疫等)引起的炎症反应
B.具有强大的抗炎作用
C.能抑制巨噬细胞对抗原的吞噬和处理
D.具有抗休克作用
E.可对抗细菌内毒素而缓解中毒性休克的症状
C.能抑制巨噬细胞对抗原的吞噬和处理
4 4 2006 47. 机体处于"应急"反应时,血中主要升高的激素是(2.0分) A.去甲肾上腺素
B.肾上腺素
C.ACTH和糖皮质激素
D.醛固酮
E.以上都是
B.肾上腺素
4 4 2006 48. 合成甲状腺激素的主要原料有:(2.0分) A.碘和亚铁
B.碘和酪氨酸
C.铁和酪氨酸
D.球蛋白和类固醇
E.甲状腺球蛋白
B.碘和酪氨酸
4 4 2006 49. 垂体神经部分泌 (2.0分) A.生长激素
B.促甲状腺素
C.抗利尿激素和催产素
D.促肾上腺皮质激素
E.卵泡刺激素
C.抗利尿激素和催产素
4 4 2006 50. 下列哪一种降血糖药禁用于有严重肝病的糖尿病病人?(2.0分) A.甲苯磺丁脲
B.阿卡波糖
C.正规胰岛素
D.氯磺丙脲
E.结晶锌胰岛素
D.氯磺丙脲
4 4 2007 1. -葡萄糖苷酶抑制药是 (2.0分) A.甲福明
B.阿卡波糖
C.胰岛素
D.甲苯磺丁脲
E.罗格列酮
B.阿卡波糖
4 4 2007 2. 关于甲状旁腺描述错误的是(2.0分) A.嗜酸性细胞体积大,胞质嗜酸性
B.腺细胞分为主细胞和嗜酸性细胞
C.主细胞分泌的激素属肽类激素
D.分泌的激素参与血钙浓度调节
E.嗜酸性细胞随年龄增长而减少
E.嗜酸性细胞随年龄增长而减少
4 4 2007 3. Which of the following hormones exerts the least effect on growth?(2.0分) A.Growth hormone
B.Insulin
C.T4
D.Testosterone
E.Vasopressin
E.Vasopressin
4 4 2007 4. 下列哪一降血糖药可用于治疗尿崩症(2.0分) A.格列吡嗪
B.格列齐特
C.格列本脲
D.吸人型胰岛素
E.氯磺丙脲
E.氯磺丙脲
4 4 2007 5. 产生肾上腺素的细胞是(2.0分) A.嗜铬细胞
B.嗜酸性细胞
C.主细胞
D.交感神经节细胞
E.嫌色细胞
A.嗜铬细胞
4 4 2007 6. 能促进脂肪组织摄取葡萄糖,降低肠道葡萄糖吸收的药物是(2.0分) A.甲福明
B.阿卡波糖
C.胰岛素
D.甲苯磺丁脲
E.罗格列酮
A.甲福明
4 4 2007 7. 目前腺垂体功能减退症患者最佳的治疗方法是(2.0分) A.靶腺激素替代
B.应用促激素释放激素
C.垂体促激素刺激靶腺
D.补充靶腺激素调节物质
E.腺垂体皮下移植治疗
A.靶腺激素替代
4 4 2007 8. 可产生雄性激素的细胞是(2.0分) A.胰岛细胞
B.肾上腺皮质球状带细胞
C.肾间质细胞
D.肾上腺皮质网状带细胞
E.垂体嗜碱性细胞
D.肾上腺皮质网状带细胞
4 4 2007 9. 下列哪一项不属于下丘脑调节肽:(2.0分) A.生长抑素
B.抗利尿激素
C.促性腺激素释放激素
D.促甲状腺激素释放激素
E.促肾上腺皮质激素释放激素
B.抗利尿激素
4 4 2007 10. 神经激素是指:(2.0分) A.具有激素功能的神经递质
B.作用于神经细胞的激素
C.神经细胞分泌的激素
D.神经系统内存在的激素
E.来自神经细胞,并只作用于神经系统的激素
C.神经细胞分泌的激素
4 4 2007 11. 肾上腺皮质球状带细胞分泌的激素的功能是(2.0分) A.排钾保钠
B.排钠保钾
C.保钠
D.排钾
E.以上均不是
A.排钾保钠
4 4 2007 12. 生长素对糖、脂肪代谢的作用是:(2.0分) A.抑制脂肪分解,使游离脂肪酸增加,促进葡萄糖氧化
B.促进脂肪分解,使游离脂肪酸增加,抑制葡萄糖氧化
C.促进脂肪分解,使游离脂肪酸减少,促进葡萄糖氧化
D.促进脂肪分解,使游离脂肪酸减少,抑制葡萄糖氧化
E.抑制脂肪分解,使游离脂肪酸增加,抑制葡萄糖氧化
B.促进脂肪分解,使游离脂肪酸增加,抑制葡萄糖氧化
4 4 2007 13. 生长素分泌过多的患者可出现:(2.0分) A.血糖过高并可产生糖尿
B.尿氮增加
C.血中脂肪酸含量减少
D.组织脂肪增加
E.血中生长激素介质含量减少
A.血糖过高并可产生糖尿
4 4 2007 14. 成人生长素分泌过多将导致: (2.0分) A.腺瘤
B.粘液水肿
C.甲状腺肿
D.侏儒症
E.肢端肥大症
E.肢端肥大症
4 4 2007 15. 系统性红斑狼疮患者服用强的松数月,目前剂量30mg/d,因外出旅游,未随身带药而停药半月余。近一周出现疲倦乏力,头晕,恶心呕吐,体重减轻,血压下降,低血钠,该病人应考虑:(2.0分) A.急性胃肠炎
B.医源性腺垂体功能减退症
C.肾功能不全
D.激素副作用
E.继发性糖尿病
B.医源性腺垂体功能减退症
4 4 2007 16. Wolff-Chaikoff效应是指:(2.0分) A.过量的碘所产生的抗甲状腺效应
B.缺乏碘而产生的抗甲状腺效应
C.缺乏酪氨酸产生的抗甲状腺效应
D.过量的酪氨酸所产生的抗甲状腺效应
E.碘和酪氨酸缺乏而产生的促甲状腺效应
A.过量的碘所产生的抗甲状腺效应
4 4 2007 17. Propylthiouracil的抗甲状腺作用主要在于 (2.0分) A.抑制已合成的甲状腺素的释放
B.作用于甲状腺细胞膜受体
C.抑制甲状腺中碘的有机化过程
D.作用于甲状腺细胞核内受体
E.以上都不是
C.抑制甲状腺中碘的有机化过程
4 4 2007 18. 生理剂量时能促进蛋白质分解的激素是:(2.0分) A.甲状腺激素
B.生长素
C.胰岛素
D.糖皮质激素
E.雄激素
D.糖皮质激素
4 4 2007 19. 关于Graves病的发病机制,下列选项中与目前观点不符的是:(2.0分) A.与TSAb的关系十分密切
B.与一定的HLA类型有关
C.有一定的家族倾向
D.是一种器官特异性自身免疫病
A.与TSAb的关系十分密切
4 4 2007 20. 腺垂体功能减退症的治疗,首先应补充 (2.0分) A.甲状腺素
B.性激素
C.皮质醇
D.TSH
E.ACTH
C.皮质醇
4 4 2007 21. 大剂量碘剂用于甲状腺切除手术前的主要目的是(2.0分) A.使甲状腺对碘摄取增加,抑制甲状腺激素合成的作用
B.为甲状腺激素合成提供原料
C.抑制甲状腺中碘的有机化过程
D.使腺体血管增生,腺体增大
E.使腺体血管萎缩,腺体变小、变硬
E.使腺体血管萎缩,腺体变小、变硬
4 4 2007 22. 胰高血糖素对糖和脂肪代谢效应的靶器官是(2.0分) A.心脏
B.肝脏
C.肌肉
D.肾脏
E.神经系统
B.肝脏
4 4 2007 23. 盐皮质激素主要作用于肾脏的(2.0分) A.细段
B.近端小管直部
C.近端小管曲部
D.远端小管直部
E.远端小管曲部
E.远端小管曲部
4 4 2007 24. 腺垂体嗜酸性细胞能分泌(2.0分) A.促性腺激素
B.促甲状腺激素
C.促肾上腺皮质激素
D.生长激素
E.黄体生长素
D.生长激素
4 4 2007 25. 胰岛素增敏剂是 (2.0分) A.甲苯磺丁脲
B.阿卡波糖
C.甲福明
D.胰岛素
E.罗格列酮
E.罗格列酮
4 4 2007 26. 长期大量注射氢化可的松时主要可引起血中:(2.0分) A.红细胞减少
B.血糖下降
C.ACTH↓
D.血钙升高
E.CRH↑
C.ACTH↓
4 4 2007 27. 糖皮质激素对血液有形成分作用描述错误的是(2.0分) A.红细胞增多
B.血小板增多
C.淋巴细胞增多
D.中性粒细胞增多
E.血红蛋白增多
C.淋巴细胞增多
4 4 2007 28. 糖皮质激素对血液和造血系统的作用是 (2.0分) A.使血小板减少
B.使红细胞与血红蛋白减少
C.使中性粒细胞减少
D.刺激骨髓造血功能
E.使淋巴细胞增加
D.刺激骨髓造血功能
4 4 2007 29. 糖皮质激素用于严重感染的目的在于(2.0分) A.有抗菌和抗毒素作用
B.由于加强心肌收缩力,帮助病人度过危险期
C.具有中和抗毒作用,提高机体对毒素的耐受力
D.利用其强大的抗炎作用,缓解症状,使病人度过危险期
E.消除危害机体的炎症和过敏反应
D.利用其强大的抗炎作用,缓解症状,使病人度过危险期
4 4 2007 30. 甲状旁腺的腺细胞可分为(2.0分) A.主细胞和嗜酸性细胞
B.嗜酸性细胞和嗜碱性细胞
C.滤泡上皮细胞与C细胞
D.A细胞和B细胞
E.主细胞和嫌色细胞
A.主细胞和嗜酸性细胞
4 4 2007 31. 促进胰高血糖素分泌的主要因素是:(2.0分) A.低血糖
B.高血糖
C.血中氨基酸增多
D.血中脂肪酸减少
E.迷走神经兴奋
A.低血糖
4 4 2007 32. 肾上腺皮质细胞超微结构特点是富含(2.0分) A.滑面内质网和溶酶体
B.粗面内质网和滑面内质网
C.粗面内质网和高尔基复合体
D.高尔基复合体和溶酶体
E.滑面内质网和脂滴
E.滑面内质网和脂滴
4 4 2007 33. 急性严重中毒性感染时,糖皮质激素治疗采用(2.0分) A.大剂量突击静脉滴注
B.大剂量肌内注射
C.小剂量多次给药
D.一次负荷量,然后给予维持量
E.较长时间大剂量给药
A.大剂量突击静脉滴注
4 4 2007 34. 下列哪项试验不是检测下丘脑-垂体-肾上腺皮质轴功能的方法(2.0分) A.ACTH兴奋试验
B.CRH兴奋试验
C.胰岛素低血糖试验
D.地塞米松抑制试验
E.安体舒通试验
E.安体舒通试验
4 4 2007 35. 糖皮质激素类药物与水盐代谢相关的不良反应是 (2.0分) A.多毛
B.高血压
C.痤疮
D.胃、十二指肠溃疡
E.向心性肥胖
B.高血压
4 4 2007 36. 甲状腺素的主要适应证是(2.0分) A.轻、中度甲状腺功能亢进
B.甲状腺危象
C.甲亢的手术前准备
D.交感神经活性增强引起的病变
E.黏液性水肿
E.黏液性水肿
4 4 2007 37. 下列疾病中禁用糖皮质激素类药物的是 (2.0分) A.感染性休克
B.活动性消化性溃疡
C.中毒性痢疾
D.重症伤寒
E.肾病综合征
B.活动性消化性溃疡
4 4 2007 38. 下列那种激素对蛋白质合成与储存是不可缺少的?(2.0分) A.生长抑素
B.胰岛素
C.胰多肽
D.胰高血糖素
E.降钙素
B.胰岛素
4 4 2007 39. 胰岛中的β细胞可以 (2.0分) A.合成、释放胰岛素
B.分泌糖原
C.合成甘油三酯
D.分泌可的松
E.产生肾上腺素
A.合成、释放胰岛素
4 4 2007 40. Tolbutamide的适应证是 (2.0分) A.胰岛功能全部丧失的糖尿病患者
B.糖尿病合并高热
C.糖尿病昏迷
D.中、轻度糖尿病
E.糖尿病合并酮症酸中毒
D.中、轻度糖尿病
4 4 2007 41. 关于甲状腺激素合成、分泌描述错误的是(2.0分) A.滤泡上皮细胞自血中摄取氨基酸
B.分泌颗粒以胞吐方式排入滤泡腔贮存
C.甲状腺球蛋白与摄入的碘在滤泡上皮细胞内结合
D.在粗面内质网和高尔基复合体合成甲状腺球蛋白
E.释放入血的是经溶酶体水解的甲状腺激素
C.甲状腺球蛋白与摄入的碘在滤泡上皮细胞内结合
4 4 2007 42. 格列本脲降血糖的主要作用机制是(2.0分) A.妨碍葡萄糖的肠道吸收
B.增强肌肉组织糖的无氧酵解
C.拮抗胰高血糖素的作用
D.刺激胰岛B细胞释放胰岛素
E.升高血清糖原水平
D.刺激胰岛B细胞释放胰岛素
4 4 2007 43. 下列哪种情况不会引起PRL增高 (2.0分) A.垂体瘤
B.原发性甲状腺功能减退症
C.原发性甲状腺功能亢进症
D.下丘脑病变
E.药物
C.原发性甲状腺功能亢进症
4 4 2007 44. 胰高血糖素的主要生理作用是:(2.0分) A.促进胃液和胆汁分泌
B.促进葡萄糖利用
C.促进脂肪合成
D.促进肝糖原分解和糖异生
E.加快心率
D.促进肝糖原分解和糖异生
4 4 2007 45. Insulin的适应证有(2.0分) A.糖尿病合并酮症酸中毒
B.口服降血糖药无效的非胰岛素依赖性糖尿病
C.重症糖尿病(1型)
D.糖尿病合并严重感染
E.以上都是
E.以上都是
4 4 2007 46. 糖皮质激素用于急性炎症的主要目的在于(2.0分) A.加快消除致病因素,促进炎症消退
B.改善葡萄糖、蛋白质、水盐代谢,提高机体抗病能力
C.降低血管通透性,减轻肿胀、疼痛等症状
D.增强机体防御功能,促进炎症消退
E.抑制肉芽组织生长,防止粘连和疤痕形成,减轻后遗症
C.降低血管通透性,减轻肿胀、疼痛等症状
4 4 2007 47. 下列哪些属于先垂体功能减退症(2.0分) A.ACTH下降,皮质醇增高
B.TSH增高,T3、T4增高
C.TSH下降,T3、T4下降
D.FSH、LH增高,雌激素下降
A.ACTH下降,皮质醇增高
4 4 2007 48. Graves病浸润性突眼主要与:(2.0分) A.TPO有关
B.TSH有关
C.TSAb有关
D.细胞免疫有关
D.细胞免疫有关
4 4 2007 49. 甲状腺激素主要促进哪些组织生长发育:(2.0分) A.骨骼和肌肉
B.内脏和骨骼
C.神经系统和肌肉
D.神经系统和骨骼
E.肌肉和内脏
D.神经系统和骨骼
4 4 2007 50. 关于糖尿病病因和发病机制的论述,下列说法正确的是:(2.0分) A.环境因素导致胰岛素分泌不足
B.遗传与环境因素共同参与其发病过程
C.主要与遗传及免疫因素有关
D.遗传易感性是其主要发病因素
B.遗传与环境因素共同参与其发病过程
4 4 2008 1. 腺垂体嗜酸性细胞能分泌(2.0分) A.促肾上腺皮质激素
B.促甲状腺激素
C.促性腺激素
D.生长激素
E.黄体生长素
D.生长激素
4 4 2008 2. 盐皮质激素主要作用于肾脏的(2.0分) A.近端小管曲部
B.近端小管直部
C.细段
D.远端小管直部
E.远端小管曲部
E.远端小管曲部
4 4 2008 3. 与肢端肥大症相关的是(2.0分) A.垂体细胞
B.嗜酸性细胞
C.嗜碱性细胞
D.嫌色细胞
E.以上都不是
B.嗜酸性细胞
4 4 2008 4. 神经垂体的功能是(2.0分) A.合成加压素和催产素
B.调节腺垂体的功能活动
C.贮存和释放下丘脑视上核和室旁核分泌的激素
D.受下丘脑弓状核分泌物的影响
E.分泌黑素细胞刺激素
C.贮存和释放下丘脑视上核和室旁核分泌的激素
4 4 2008 5. 甲状腺滤泡腔中贮存的物质是(2.0分) A.四碘甲状腺原氨酸
B.三碘甲状腺原氨酸
C.甲状腺球蛋白
D.碘化甲状腺球蛋白
E.酪氨酸
D.碘化甲状腺球蛋白
4 4 2008 6. 产生肾上腺素的细胞是(2.0分) A.嗜酸性细胞
B.嗜铬细胞
C.主细胞
D.交感神经节细胞
E.嫌色细胞
B.嗜铬细胞
4 4 2008 7. 关于甲状腺激素合成、分泌描述错误的是(2.0分) A.滤泡上皮细胞自血中摄取氨基酸
B.在粗面内质网和高尔基复合体合成甲状腺球蛋白
C.甲状腺球蛋白与摄入的碘在滤泡上皮细胞内结合
D.分泌颗粒以胞吐方式排入滤泡腔贮存
E.释放入血的是经溶酶体水解的甲状腺激素
C.甲状腺球蛋白与摄入的碘在滤泡上皮细胞内结合
4 4 2008 8. 神经激素是指:(2.0分) A.作用于神经细胞的激素
B.具有激素功能的神经递质
C.神经细胞分泌的激素
D.神经系统内存在的激素
E.来自神经细胞,并只作用于神经系统的激素
C.神经细胞分泌的激素
4 4 2008 9. 下列为腺垂体所分泌的激素是(2.0分) A.促肾上腺皮质激素释放激素
B.促肾上腺皮质激素
C.肾上腺皮质激素
D.催产素
E.抗利尿激素
B.促肾上腺皮质激素
4 4 2008 10. 生长素对糖、脂肪代谢的作用是:(2.0分) A.促进脂肪分解,使游离脂肪酸增加,抑制葡萄糖氧化
B.抑制脂肪分解,使游离脂肪酸增加,促进葡萄糖氧化
C.促进脂肪分解,使游离脂肪酸减少,促进葡萄糖氧化
D.促进脂肪分解,使游离脂肪酸减少,抑制葡萄糖氧化
E.抑制脂肪分解,使游离脂肪酸增加,抑制葡萄糖氧化
A.促进脂肪分解,使游离脂肪酸增加,抑制葡萄糖氧化
4 4 2008 11. 幼儿时生长素分泌不足可导致:(2.0分) A.呆小症
B.巨人症
C.侏儒症
D.向心性肥胖
E.肢端肥大症
C.侏儒症
4 4 2008 12. Which of the following hormones exerts the least effect on growth?(2.0分) A.Growth hormone
B.Testosterone
C.T4
D.Insulin
E.Vasopressin
E.Vasopressin
4 4 2008 13. 甲状腺激素主要促进哪些组织生长发育:(2.0分) A.骨骼和肌肉
B.神经系统和肌肉
C.内脏和骨骼
D.神经系统和骨骼
E.肌肉和内脏
D.神经系统和骨骼
4 4 2008 14. 甲状腺激素反馈调节主要影响:(2.0分) A.丘脑下部
B.神经垂体
C.腺垂体
D.甲状腺
E.大脑皮层
C.腺垂体
4 4 2008 15. 糖皮质激素不宜用于胃溃疡患者是因为糖皮质激素能:(2.0分) A.抑制糖的利用,使组织能源减少
B.使胃肠道血管收缩,血供减少
C.促进盐酸和胃蛋白酶原的分泌,加剧溃疡病变
D.分解蛋白质,影响伤口愈合
E.抑制纤维母细胞的增殖和功能,延长溃疡愈合
C.促进盐酸和胃蛋白酶原的分泌,加剧溃疡病变
4 4 2008 16. 糖皮质激素的存在对去甲肾上腺素的作用是:(2.0分) A.不存在任何影响
B.存在相互拮抗作用
C.存在允许作用
D.存在相互加强作用
E.有拮抗作用也有加强作用
C.存在允许作用
4 4 2008 17. 长期大量注射氢化可的松时主要可引起血中:(2.0分) A.红细胞减少
B.ACTH↓
C.血糖下降
D.血钙升高
E.CRH↑
B.ACTH↓
4 4 2008 18. 糖皮质激素对物质代谢的影响是促进(2.0分) A.肝内蛋白质分解
B.肝外蛋白质合成
C.糖原合成
D.核酸合成
E.使脂肪重新分配,四肢脂肪分解
E.使脂肪重新分配,四肢脂肪分解
4 4 2008 19. 下列那种激素对蛋白质合成与储存是不可缺少的?(2.0分) A.胰高血糖素
B.胰岛素
C.胰多肽
D.生长抑素
E.降钙素
B.胰岛素
4 4 2008 20. 关于胰岛素的正确论述是:(2.0分) A.由胰岛的A细胞合成分泌
B.具有降血糖作用
C.血糖浓度对胰岛素分泌有负反馈作用
D.促进脂肪的分解
E.交感神经兴奋,胰岛素分泌增加
B.具有降血糖作用
4 4 2008 21. 胰岛素分泌不足将产生:(2.0分) A.侏儒症
B.呆小症
C.糖尿病
D.阿狄森病
E.柯兴综合征
C.糖尿病
4 4 2008 22. 胰高血糖素的主要生理作用是:(2.0分) A.促进肝糖原分解和糖异生
B.促进葡萄糖利用
C.促进脂肪合成
D.促进胃液和胆汁分泌
E.加快心率
A.促进肝糖原分解和糖异生
4 4 2008 23. 幼儿时生长素分泌不足可导致(2.0分) A.呆小症
B.巨人症
C.侏儒症
D.肢端肥大症
C.侏儒症
4 4 2008 24. Thyroid hormone作用的主要机制是(2.0分) A.与核受体结合,刺激mRNA生成
B.与核受体结合,促进cGMP生成
C.与膜受体结合,促进cAMP生成
D.与膜受体结合,抑制cAMP生成
A.与核受体结合,刺激mRNA生成
4 4 2008 25. 在对克汀病(cretinism)的描述中,错误的说法是(2.0分) A.地方性克汀病可以用碘化食盐预防
B.散发性克汀病的发生可能与遗传因素有关
C.患儿全身组织和骨骼发育障碍
D.脑发育基本正常
D.脑发育基本正常
4 4 2008 26. 影响神经系统发育的最重要的hormone是(2.0分) A.生长素
B.甲状腺激素
C.糖皮质激素
D.肾上腺素
B.甲状腺激素
4 4 2008 27. 下列哪些属于先垂体功能减退症(2.0分) A.TSH下降,T3、T4下降
B.TSH增高,T3、T4增高
C.ACTH下降,皮质醇增高
D.FSH、LH增高,雌激素下降
C.ACTH下降,皮质醇增高
4 4 2008 28. 内分泌系统的反馈调节是指: (2.0分) A.神经系统对内分泌系统的调节
B.内分泌系统对神经系统的调节
C.免疫系统对内分泌系统的调节
D.下丘脑-垂体-靶腺之间的相互调节
D.下丘脑-垂体-靶腺之间的相互调节
4 4 2008 29. 关于Graves病的发病机制,下列选项中与目前观点不符的是:(2.0分) A.有一定的家族倾向
B.与一定的HLA类型有关
C.与TSAb的关系十分密切
D.是一种器官特异性自身免疫病
C.与TSAb的关系十分密切
4 4 2008 30. 2型糖尿病的发病机制是:(2.0分) A.胰岛B细胞遗传性缺陷
B.胰岛B细胞自身免疫反应性损伤
C.胰岛B细胞破坏,胰岛素绝对不足
D.胰岛素抵抗和胰岛素分泌缺陷
D.胰岛素抵抗和胰岛素分泌缺陷
4 4 2008 31. 非毒性甲状腺肿胶质贮积期的主要病变是(2.0分) A.甲状腺不均匀性肿大
B.滤泡高度扩张,内含大量胶质,上皮细胞扁平
C.滤泡上皮呈高柱状,间质充血
D.甲状腺形成结节
E.滤泡上皮明显增生,呈立方形,伴小滤泡形成
B.滤泡高度扩张,内含大量胶质,上皮细胞扁平
4 4 2008 32. 地方性甲状腺肿的最主要原因是(2.0分) A.致甲状腺肿物质
B.缺碘
C.甲状腺刺激免疫球蛋白
D.遗传
E.碘摄入过高
B.缺碘
4 4 2008 33. 甲状腺恶性肿瘤中生长慢,恶性程度低,发病率最高的是(2.0分) A.滤泡性癌
B.乳头状腺癌
C.髓样癌
D.未分化癌
E.肉瘤
B.乳头状腺癌
4 4 2008 34. 颈中部淋巴结转移性肿瘤灶中可见淀粉样物,首先考虑的原发灶是(2.0分) A.鼻咽泡状核细胞癌
B.甲状腺髓样癌
C.肺小细胞癌
D.甲状腺乳头状癌
E.甲状腺滤泡癌
B.甲状腺髓样癌
4 4 2008 35. 出现毛玻璃状核的甲状腺癌是(2.0分) A.滤泡性腺癌
B.乳头状癌
C.髓样癌
D.小细胞癌
E.梭形细胞癌
B.乳头状癌
4 4 2008 36. 嗜铬细胞瘤常引起下列哪项病症?(2.0分) A.Cushing's综合征
B.Sheehan's综合征
C.甲亢
D.阵发性高血压
E.高血糖症
D.阵发性高血压
4 4 2008 37. 关于胰岛素依赖型糖尿病,以下哪项是错的?(2.0分) A. 多见于青少年
B.胰岛B细胞破坏
C. 血中可查见抗胰岛细胞的自身抗体
D.血中胰岛素升高
E.易合并酮症酸中毒
D.血中胰岛素升高
4 4 2008 38. 下列哪种肿瘤不属于APUD瘤?(2.0分) A.甲状腺髓样癌
B.胃泌素瘤
C.肾上腺皮质腺瘤
D.肺燕麦细胞癌
E.嗜铬细胞瘤
C.肾上腺皮质腺瘤
4 4 2008 39. 最常见的胰腺肿瘤是:(2.0分) A.胰岛细胞瘤
B.胰高血糖素瘤
C.生长抑素瘤
D.纤维瘤
E.胃泌素瘤
A.胰岛细胞瘤
4 4 2008 40. 胰岛细胞瘤的特点除外:(2.0分) A.单发,较小
B.圆形结节状
C.有完整或不完整包膜
D.组织学特点多样
E.常伴出血
E.常伴出血
4 4 2008 41. 有关垂体激素,以下哪一组合是正确的 (2.0分) A.生长激素、泌乳素、雌二醇、促肾上腺皮质激素、促甲状腺素
B.促黑素、促甲状腺素、促肾上腺皮质激素、黄体生成素、泌乳素
C.抗利尿激素、心钠素、促甲状腺素、生长激素、泌乳素
D.黄体生成素、泌乳素、炔诺酮、游离皮质醇、促生长激素释放激素
E.内皮素、抗利尿激素、肾上腺素、促黑素、甲状旁腺素
B.促黑素、促甲状腺素、促肾上腺皮质激素、黄体生成素、泌乳素
4 4 2008 42. 下列哪种激素不是腺垂体分泌的激素 (2.0分) A.GH
B.PRL
C.ADH
D.TSH
E.FSH、LH
C.ADH
4 4 2008 43. 与控制生长发育无关的激素为 (2.0分) A.甲状腺素
B.胰岛素
C.雄激素
D.生长激素
E.醛固酮
E.醛固酮
4 4 2008 44. 内分泌系统的反馈调节不包括 (2.0分) A.代谢物质对激素分泌的调节
B.免疫-内分泌系统之间的互相调节
C.下丘脑-垂体-甲状腺轴的调节
D.下丘脑-垂体-性腺轴的调节
E.下丘脑-垂体-肾上腺轴的调节
B.免疫-内分泌系统之间的互相调节
4 4 2008 45. 下列哪一降血糖药可用于治疗尿崩症(2.0分) A.格列吡嗪
B.格列齐特
C.吸人型胰岛素
D.格列本脲
E.氯磺丙脲
E.氯磺丙脲
4 4 2008 46. 通过激动PPARγ发挥胰岛素增敏作用的是 (2.0分) A.胰岛素
B.阿卡波糖
C.二甲双胍
D.甲苯磺丁脲
E.罗格列酮
E.罗格列酮
4 4 2008 47. 糖皮质激素用于慢性炎症的目的在于(2.0分) A.具有强大抗炎作用,促进炎症消散
B.抑制花生四烯酸释放,使炎症介质PG合成减少
C.促进炎症区的血管收缩,降低其通透性
D.稳定溶酶体膜,减少蛋白水解酶的释放
E.抑制肉芽组织生长,防止粘连和疤痕
E.抑制肉芽组织生长,防止粘连和疤痕
4 4 2008 48. 糖皮质激素对血液有形成分作用描述错误的是 (2.0分) A.淋巴细胞增多
B.血小板增多
C.红细胞增多
D.中性粒细胞增多
E.血红蛋白增多
A.淋巴细胞增多
4 4 2008 49. 糖皮质激素类药物与蛋白质代谢相关的不良反应是(2.0分) A.骨质疏松
B.高血压
C.精神失常
D.多毛
E.向心性肥胖
A.骨质疏松
4 4 2008 50. 糖皮质激素类药物与水盐代谢相关的不良反应是(2.0分) A.胃、十二指肠溃疡
B.高血压
C.痤疮
D.多毛
E.向心性肥胖
B.高血压
4 4 2009 1. Wolff-Chaikoff效应是指:(2.0分) A.缺乏碘而产生的抗甲状腺效应
B.过量的碘所产生的抗甲状腺效应
C.缺乏酪氨酸产生的抗甲状腺效应
D.过量的酪氨酸所产生的抗甲状腺效应
E.碘和酪氨酸缺乏而产生的促甲状腺效应
B.过量的碘所产生的抗甲状腺效应
4 4 2009 2. 甲状腺激素主要促进哪些组织生长发育:(2.0分) A.骨骼和肌肉
B.神经系统和肌肉
C.内脏和骨骼
D.神经系统和骨骼
E.肌肉和内脏
D.神经系统和骨骼
4 4 2009 3. 甲状腺激素反馈调节主要影响(2.0分) A.丘脑下部
B.神经垂体
C.腺垂体
D.甲状腺
E.大脑皮层
C.腺垂体
4 4 2009 4. 以下关于甲状腺球蛋白的说法哪个是错误的:(2.0分) A.在甲状腺滤泡细胞和胶质中工作
B.是碘化酪氨酸的载体
C.属于同源二聚体糖蛋白
D.是甲状腺素的载体
E.是甲状腺激素的储存形式
D.是甲状腺素的载体
4 4 2009 5. 糖皮质激素的作用是 (2.0分) A.抑制蛋白质合成
B.使血糖浓度降低
C.使肾脏排水能力降低
D.使红细胞和血小板数量增加
E.使肾脏排钾能力降低
D.使红细胞和血小板数量增加
4 4 2009 6. 生理剂量时能促进蛋白质分解的激素是:(2.0分) A.糖皮质激素
B.生长素
C.胰岛素
D.甲状腺激素
E.雄激素
A.糖皮质激素
4 4 2009 7. 临床上长期大剂量使用糖皮质激素时可以引起(2.0分) A.血中ACTH浓度增高
B.淋巴细胞数目增加
C.肢端肥大症
D.肾上腺皮质渐趋萎缩
E.CHR分泌增多
D.肾上腺皮质渐趋萎缩
4 4 2009 8. 肾上腺髓质激素的分泌主要受下列哪项的调节?(2.0分) A.交感神经节前纤维
B.副交感神经节后纤维
C.血糖浓度
D.ACTH
E.肾上腺髓质激素的反馈调节
A.交感神经节前纤维
4 4 2009 9. 促进胰高血糖素分泌的主要因素是:(2.0分) A.高血糖
B.低血糖
C.血中氨基酸增多
D.血中脂肪酸减少
E.迷走神经兴奋
B.低血糖
4 4 2009 10. 胰岛素缺乏时可导致 (2.0分) A.细胞内葡萄糖增加
B.肝糖原合成减少
C.血糖浓度下降
D.无尿糖
E.促进蛋白质合成
B.肝糖原合成减少
4 4 2009 11. 调节胰岛素分泌的主要因素是:(2.0分) A.血中脂肪酸浓度
B.血中氨基酸浓度
C.血糖浓度
D.胃肠激素
E.植物神经活动
C.血糖浓度
4 4 2009 12. 胰高血糖素对糖和脂肪代谢效应的靶器官是:(2.0分) A.心脏
B.肝脏
C.肾脏
D.肌肉
E.神经系统
B.肝脏
4 4 2009 13. 下列哪一项不属于下丘脑调节肽:(2.0分) A.促甲状腺激素释放激素
B.抗利尿激素
C.促性腺激素释放激素
D.生长抑素
E.促肾上腺皮质激素释放激素
B.抗利尿激素
4 4 2009 14. 下列为下丘脑所分泌的激素是(2.0分) A.促甲状腺激素
B.黄体生成素
C.卵泡刺激素
D.生长素
E.抗利尿激素
E.抗利尿激素
4 4 2009 15. 生长素分泌过多的患者可出现:(2.0分) A.尿氮增加
B.血糖过高并可产生糖尿
C.血中脂肪酸含量减少
D.组织脂肪增加
E.血中生长激素介质含量减少
B.血糖过高并可产生糖尿
4 4 2009 16. 成人生长素分泌过多将导致: (2.0分) A.腺瘤
B.粘液水肿
C.侏儒症
D.甲状腺肿
E.肢端肥大症
E.肢端肥大症
4 4 2009 17. The growth hormone receptor(2.0分) A.Activates Gs
B.Requires dimerization to exert its effects
C.Must be internalized to exert its effects
D.Resembles the IGF-I receptor
E.Resembles the ACTH receptor
B.Requires dimerization to exert its effects
4 4 2009 18. 非毒性甲状腺肿胶质贮积期的主要病变是(2.0分) A.甲状腺不均匀性肿大
B.滤泡高度扩张,内含大量胶质,上皮细胞扁平
C.滤泡上皮呈高柱状,间质充血
D.甲状腺形成结节
E.滤泡上皮明显增生,呈立方形,伴小滤泡形成
B.滤泡高度扩张,内含大量胶质,上皮细胞扁平
4 4 2009 19. 细胞分泌降钙素的甲状腺癌是 (2.0分) A.滤泡性癌
B.乳头状腺癌
C.髓样癌
D.未分化癌
E.嗜酸细胞癌
C.髓样癌
4 4 2009 20. 甲状腺恶性肿瘤中发病率最高,恶性程度低,生长慢的是 (2.0分) A.滤泡性癌
B.乳头状腺癌
C.髓样癌
D.未分化癌
E.肉瘤
B.乳头状腺癌
4 4 2009 21. 颈中部淋巴结转移性肿瘤灶中可见淀粉样物,首先考虑的原发灶是(2.0分) A.鼻咽泡状核细胞癌
B.甲状腺髓样癌
C.肺小细胞癌
D.甲状腺乳头状癌
E.甲状腺滤泡癌
B.甲状腺髓样癌
4 4 2009 22. 出现毛玻璃状核的甲状腺癌是(2.0分) A.滤泡性腺癌
B.乳头状癌
C.髓样癌
D.小细胞癌
E.梭形细胞癌
B.乳头状癌
4 4 2009 23. 腺垂体嗜酸性细胞能分泌:(2.0分) A.促肾上腺皮质激素
B.促甲状腺激素
C.促性腺激素
D.生长激素
E.黄体生长素
D.生长激素
4 4 2009 24. 盐皮质激素主要作用于肾脏的(2.0分) A.近端小管曲部
B.近端小管直部
C.细段
D.远端小管直部
E.远端小管曲部
E.远端小管曲部
4 4 2009 25. 肾上腺皮质球状带细胞分泌的激素的功能是(2.0分) A.排钾
B.排钠保钾
C.保钠
D.排钾保钠
E.以上均不是
D.排钾保钠
4 4 2009 26. 神经垂体的功能是(2.0分) A.合成加压素和催产素
B.调节腺垂体的功能活动
C.贮存和释放下丘脑视上核和室旁核分泌的激素
D.受下丘脑弓状核分泌物的影响
E.分泌黑素细胞刺激素
C.贮存和释放下丘脑视上核和室旁核分泌的激素
4 4 2009 27. 甲状腺滤泡腔中贮存的物质是(2.0分) A.四碘甲状腺原氨酸
B.三碘甲状腺原氨酸
C.甲状腺球蛋白
D.碘化甲状腺球蛋白
E.酪氨酸
D.碘化甲状腺球蛋白
4 4 2009 28. 产生肾上腺素的细胞是(2.0分) A.嗜酸性细胞
B.嗜铬细胞
C.主细胞
D.交感神经节细胞
E.嫌色细胞
B.嗜铬细胞
4 4 2009 29. 下列哪项不是糖尿病人的典型症状:(2.0分) A.多饮
B.多食
C.多尿
D.水肿
E.消瘦
D.水肿
4 4 2009 30. 下列描述中,哪项不是Ⅰ型糖尿病的特点?(2.0分) A.疾病早期就有胰岛B细胞破坏和减少,血中胰岛素降低
B.多为青少年发病
C.有自身免疫反应的表现
D.与病毒感染无关
E.与遗传因素有关
D.与病毒感染无关
4 4 2009 31. 在糖尿病肾脏的病变中,下列哪项是错误的?(2.0分) A.结节性肾小球硬化
B.肾细动脉硬化
C.肾盂肾炎伴乳头坏死
D.肾小管上皮纤维素样变性
E.肾小管上皮细胞空泡样变性
D.肾小管上皮纤维素样变性
4 4 2009 32. 胰岛细胞瘤的特点除外:(2.0分) A.单发,较小
B.圆形结节状
C.有完整或不完整包膜
D.组织学特点多样
E.常伴出血
E.常伴出血
4 4 2009 33. Propylthiouracil(丙硫氧嘧啶)的抗甲状腺作用主要在于 (2.0分) A.作用于甲状腺细胞核内受体
B.作用于甲状腺细胞膜受体
C.抑制甲状腺中碘的有机化过程
D.抑制已合成的甲状腺素的释放
E.以上都不是
C.抑制甲状腺中碘的有机化过程
4 4 2009 34. Methylthiouracil(甲硫氧嘧啶)最严重的不良反应是 (2.0分) A.药疹、药热
B.关节痛、淋巴结肿大
C.腹痛、腹泻、恶心、呕吐
D.血管神经性水肿,喉头水肿
E.粒经胞缺乏症
E.粒经胞缺乏症
4 4 2009 35. 大剂量碘剂用于甲状腺切除手术前的主要目的是 (2.0分) A.为甲状腺激素合成提供原料
B.使甲状腺对碘摄取增加,抑制甲状腺激素合成的作用
C.抑制甲状腺中碘的有机化过程
D.使腺体血管增生,腺体增大
E.使腺体血管萎缩,腺体变小、变硬
E.使腺体血管萎缩,腺体变小、变硬
4 4 2009 36. Insulin的不良反应有 (2.0分) A.骨髓抑制
B.酮症酸中毒
C.高血糖高渗性昏迷
D.低血糖昏迷
E.乳酸血症
D.低血糖昏迷
4 4 2009 37. Tolbutamide(甲苯磺丁脲)的适应证是 (2.0分) A.胰岛功能全部丧失的糖尿病患者
B.糖尿病昏迷
C.糖尿病合并高热
D.中、轻度糖尿病
E.糖尿病合并酮症酸中毒
D.中、轻度糖尿病
4 4 2009 38. 下列哪一降血糖药可用于治疗尿崩症 (2.0分) A.Glipizide(格列吡嗪)
B.Gliclazide(格列齐特)
C.吸人型胰岛素
D.Glibenclamide(格列本脲)
E.Chlorpropamide(氯磺丙脲)
E.Chlorpropamide(氯磺丙脲)
4 4 2009 39. 以胰高血糖素样肽-1(GLP-1)为靶点的降血糖药物是 (2.0分) A.依克那肽
B.醋酸普兰林肽
C.氯磺丙脲
D.罗格列酮
E.阿卡波糖
A.依克那肽
4 4 2009 40. 胰岛素增敏剂是 (2.0分) A.Insulin(胰岛素)
B.Acarbose(阿卡波糖)
C.Metformin(甲福明)
D.Tolbutamide(甲苯磺丁脲)
E.Rosiglitazone(罗格列酮)
E.Rosiglitazone(罗格列酮)
4 4 2009 41. Repaglinide(瑞格列奈)主要作用是 (2.0分) A.抑制肠道 -葡萄糖苷酶
B.促进脂肪组织摄取葡萄糖
C.促进胰岛素分泌
D.抑制胰岛素降解
E.激活胰岛素受体
C.促进胰岛素分泌
4 4 2009 42. 糖皮质激素用于严重感染的目的在于 (2.0分) A.具有中和抗毒作用,提高机体对毒素的耐受力
B.有抗菌和抗内毒素作用
C.利用其强大的抗炎作用,缓解症状,使病人度过危险期
D.由于加强心肌收缩力,帮助病人度过危险期
E.消除危害机体的炎症和过敏反应
C.利用其强大的抗炎作用,缓解症状,使病人度过危险期
4 4 2009 43. 糖皮质激素用于慢性炎症的目的在于 (2.0分) A.具有强大抗炎作用,促进炎症消散
B.抑制花生四烯酸释放,使炎症介质PG合成减少
C.促进炎症区的血管收缩,降低其通透性
D.稳定溶酶体膜,减少蛋白水解酶的释放
E.抑制肉芽组织生长,防止粘连和疤痕
D.稳定溶酶体膜,减少蛋白水解酶的释放
4 4 2009 44. 以下哪一种疾病不宜应用糖皮质激素? (2.0分) A.严重支气管哮喘发作
B.急性淋巴细胞白血病
C.癫痫
D.肾移植排斥反应
E.类风湿性关节炎
C.癫痫
4 4 2009 45. 糖皮质激素的不良反应不包括 (2.0分) A.类肾上腺皮质功能亢进综合征
B.诱发或加重高血压
C.诱发或加重支气管哮喘
D.诱发或加重溃疡病
E.骨质疏松、肌肉萎缩
C.诱发或加重支气管哮喘
4 4 2009 46. 长期应用糖皮质激素可引起 (2.0分) A.向心性肥胖
B.高血钙
C.低血糖
D.高血钾
E.磷的排泄减少
A.向心性肥胖
4 4 2009 47. 早晨一次给予糖皮质激素的治疗方法是根据 (2.0分) A.口服吸收更加完全
B.体内代谢灭活缓慢,有效血药浓度稳定
C.与靶细胞受体结合牢固,作用持久
D.与体内糖皮质激素分泌的昼夜节律一致
E.有效血浓度更加持久
D.与体内糖皮质激素分泌的昼夜节律一致
4 4 2009 48. 下列哪组激素由神经垂体分泌(2.0分) A.生长素和催产素
B.胰岛素和催乳素
C.催产素和抗利尿激素
D.加压素和催乳素
C.催产素和抗利尿激素
4 4 2009 49. 在对克汀病(cretinism)的描述中,错误的说法是(2.0分) A.地方性克汀病可以用碘化食盐预防
B.散发性克汀病的发生可能与遗传因素有关
C.患儿全身组织和骨骼发育障碍
D.脑发育基本正常
D.脑发育基本正常
4 4 2009 50. 影响神经系统发育的最重要的hormone是(2.0分) A.生长素
B.甲状腺激素
C.糖皮质激素
D.肾上腺素
B.甲状腺激素
4 4 2010 1. 非毒性甲状腺肿胶质贮积期的主要病变是 (2.0分) A.甲状腺不均匀性肿大
B.滤泡高度扩张,内含大量胶质,上皮细胞扁平
C.滤泡上皮呈高柱状,间质充血
D.甲状腺形成结节
E.滤泡上皮明显增生,呈立方形,伴小滤泡形成
B.滤泡高度扩张,内含大量胶质,上皮细胞扁平
4 4 2010 2. 地方性甲状腺肿的最主要原因是 (2.0分) A.致甲状腺肿物质
B.缺碘
C.甲状腺刺激免疫球蛋白
D.遗传
E.碘摄入过高
B.缺碘
4 4 2010 3. 甲状腺恶性肿瘤中生长慢,恶性程度低,发病率最高的是 (2.0分) A.滤泡性癌
B.乳头状腺癌
C.髓样癌
D.未分化癌
E.肉瘤
B.乳头状腺癌
4 4 2010 4. 颈中部淋巴结转移性肿瘤灶中可见淀粉样物,首先考虑的原发灶是 (2.0分) A.鼻咽泡状核细胞癌
B.甲状腺髓样癌
C.肺小细胞癌
D.甲状腺乳头状癌
E.甲状腺滤泡癌
B.甲状腺髓样癌
4 4 2010 5. 出现毛玻璃状核的甲状腺癌是 (2.0分) A.滤泡性腺癌
B.乳头状癌
C.髓样癌
D.小细胞癌
E.梭形细胞癌
B.乳头状癌
4 4 2010 6. 下列哪项不是非胰岛素依赖型糖尿病的特点 (2.0分) A.患者年龄在40岁以上
B.早期就有胰岛B细胞破坏和减少,血中胰岛素降低
C.无自身免疫反应的表现
D.肥胖是重要因素
E.常见胰岛淀粉样变性
B.早期就有胰岛B细胞破坏和减少,血中胰岛素降低
4 4 2010 7. 下列哪项与血中胰岛素明显降低有关 (2.0分) A.Ⅰ型糖尿病
B.Ⅱ型糖尿病
C.Ⅰ型和Ⅱ型糖尿病
D.低血糖
E.以上均不是
A.Ⅰ型糖尿病
4 4 2010 8. 导致Ⅱ型糖尿病发生的最重要因素是 (2.0分) A.肥胖
B.肾功能不全
C.肝脂肪变性
D.垂体疾病
E.肿瘤
A.肥胖
4 4 2010 9. 下面叙述中哪项不符合Ⅱ型糖尿病 (2.0分) A.成年发病,起病较轻,发展缓慢
B.无抗胰岛细胞抗体
C.胰岛数目正常或轻度减少
D.血胰岛素水平明显降低
E.肥胖者多见
D.血胰岛素水平明显降低
4 4 2010 10. 关于胰岛细胞瘤的叙述,下列哪项是错误的 (2.0分) A.部分肿瘤可分泌胰岛素
B.瘤细胞可呈巢索状或菊形团排列
C.间质常出现淀粉样物质或钙盐沉积
D.瘤细胞与正常B细胞相似
E.大多数为恶性肿瘤
E.大多数为恶性肿瘤
4 4 2010 11. 神经激素是指:(2.0分) A.作用于神经细胞的激素
B.具有激素功能的神经递质
C.神经细胞分泌的激素
D.神经系统内存在的激素
E.来自神经细胞,并只作用于神经系统的激素
C.神经细胞分泌的激素
4 4 2010 12. 下列为腺垂体所分泌的激素是:(2.0分) A.促肾上腺皮质激素释放激素
B.促肾上腺皮质激素
C.肾上腺皮质激素
D.催产素
E.抗利尿激素
B.促肾上腺皮质激素
4 4 2010 13. 生长素对糖、脂肪代谢的作用是:(2.0分) A.促进脂肪分解,使游离脂肪酸增加,抑制葡萄糖氧化
B.抑制脂肪分解,使游离脂肪酸增加,促进葡萄糖氧化
C.促进脂肪分解,使游离脂肪酸减少,促进葡萄糖氧化
D.促进脂肪分解,使游离脂肪酸减少,抑制葡萄糖氧化
E.抑制脂肪分解,使游离脂肪酸增加,抑制葡萄糖氧化
A.促进脂肪分解,使游离脂肪酸增加,抑制葡萄糖氧化
4 4 2010 14. 成人生长素分泌过多将导致: (2.0分) A.腺瘤
B.粘液水肿
C.侏儒症
D.甲状腺肿
E.肢端肥大症
E.肢端肥大症
4 4 2010 15. The growth hormone receptor(2.0分) A.Activates Gs
B.Requires dimerization to exert its effects
C.Must be internalized to exert its effects
D.Resembles the IGF-I receptor
E.Resembles the ACTH receptor
B.Requires dimerization to exert its effects
4 4 2010 16. 甲状腺素的主要用于 (2.0分) A.甲状腺危象
B.轻、中度甲状腺功能亢进
C.甲亢的手术前准备
D.黏液性水肿
E.交感神经活性增强引起的病变
D.黏液性水肿
4 4 2010 17. Propylthiouracil(丙硫氧嘧啶)的抗甲状腺作用主要在于 (2.0分) A.抑制甲状腺中碘的有机化过程
B.抑制已合成的甲状腺素的释放
C.作用于甲状腺细胞核内受体
D.作用于甲状腺细胞膜受体
E.以上都不是
A.抑制甲状腺中碘的有机化过程
4 4 2010 18. Methylthiouracil(甲硫氧嘧啶)最严重的不良反应是 (2.0分) A.药疹、药热
B.关节痛、淋巴结肿大
C.腹痛、腹泻、恶心、呕吐
D.粒经胞缺乏症
E.血管神经性水肿,喉头水肿
D.粒经胞缺乏症
4 4 2010 19. 大剂量碘剂不能单独用于甲状腺功能亢进长期内科治疗的主要原因是 (2.0分) A.为甲状腺素合成提供原料
B.使T4转化为T3
C.使甲状腺对碘摄取减少,失去抑制甲状腺素合成的作用
D.使腺体血管增生,腺体增大
E.引起慢性碘中毒
C.使甲状腺对碘摄取减少,失去抑制甲状腺素合成的作用
4 4 2010 20. 大剂量碘剂的不良反应一般不包括 (2.0分) A.咽喉不适、口内金属味、呼吸道刺激等
B.血管神经性水肿,严重者有喉头水肿
C.可能诱发甲状腺功能亢进
D.可能诱发甲状腺功能减退
E.可能诱发血糖降低
E.可能诱发血糖降低
4 4 2010 21. Insulin的不良反应有 (2.0分) A.骨髓抑制
B.低血糖昏迷
C.高血糖高渗性昏迷
D.酮症酸中毒
E.乳酸血症
B.低血糖昏迷
4 4 2010 22. Glibenclamide(格列本脲)降血糖的主要作用机制是 (2.0分) A.拮抗胰高血糖素的作用
B.增强肌肉组织糖的无氧酵解
C.刺激胰岛B细胞释放胰岛素
D.妨碍葡萄糖的肠道吸收
E.升高血清糖原水平
C.刺激胰岛B细胞释放胰岛素
4 4 2010 23. 以胰高血糖素样肽-1(GLP-1)为靶点的降血糖药物是 (2.0分) A.阿卡波糖
B.醋酸普兰林肽
C.氯磺丙脲
D.罗格列酮
E.依克那肽
E.依克那肽
4 4 2010 24. 哪一药物可促进胰岛素分泌,是胰岛素增敏剂? (2.0分) A.Insulin(胰岛素)
B.Acarbose(阿卡波糖)
C.Metformin(甲福明)
D.Tolbutamide(甲苯磺丁脲)
E.Rosiglitazone(罗格列酮)
E.Rosiglitazone(罗格列酮)
4 4 2010 25. 哪一基本作用是糖皮质激素抗炎作用的基础? (2.0分) A.影响了参与炎症的一些基因转录
B.稳定溶酶体膜
C.抑制毛细血管和纤维母细胞的增生
D.减轻渗出、水肿、毛细血管扩张等炎症反应
E.增加肥大细胞颗粒的稳定性
A.影响了参与炎症的一些基因转录
4 4 2010 26. 对于急性炎症,糖皮质激素的主要作用是 (2.0分) A.加快消除致病因素,促进炎症消退
B.增强机体防御功能,促进炎症消退
C.改善葡萄糖、蛋白质、水盐代谢,提高机体抗病能力
D.降低血管通透性,减轻肿胀、疼痛等症状
E.抑制肉芽组织生长,防止粘连和疤痕形成,减轻后遗症
D.降低血管通透性,减轻肿胀、疼痛等症状
4 4 2010 27. 糖皮质激素和抗生素合用治疗严重感染的目的是 (2.0分) A.增强机体应激性
B.用激素缓解症状,度过危险期;而用抗生素控制感染
C.对抗抗生素的某些不良反应
D.增强机体防御能力
E.增强抗生素的抗菌作用
B.用激素缓解症状,度过危险期;而用抗生素控制感染
4 4 2010 28. 糖皮质激素的禁忌症还包括 (2.0分) A.中毒性痢疾
B.活动性消化性溃疡
C.感染性休克
D.重症伤寒
E.肾病综合征
B.活动性消化性溃疡
4 4 2010 29. 糖皮质激素诱发和加重感染,主要由于 (2.0分) A.用量不足,无法控制症状而造成
B.病人对激素不敏感而未反映出相应的疗效
C.促使许多病原微生物繁殖所致
D.抑制炎症反应和免疫反应,降低机体的防御能力
E.抑制促肾上腺皮质激素的释放
D.抑制炎症反应和免疫反应,降低机体的防御能力
4 4 2010 30. 治疗急性严重中毒性感染时,糖皮质激素给药方式是 (2.0分) A.小剂量多次给药
B.大剂量肌内注射
C.大剂量突击静脉滴注
D.一次负荷量,然后给予维持量
E.较长时间大剂量给药
C.大剂量突击静脉滴注
4 4 2010 31. 下述激素分泌方式中哪项是错误的 (2.0分) A.内分泌
B.邻分泌
C.自分泌
D.胞内分泌
E.外分泌
E.外分泌
4 4 2010 32. 有关垂体激素,以下哪一组合是正确的 (2.0分) A.生长激素、泌乳素、雌二醇、促肾上腺皮质激素、促甲状腺素
B.促黑素、促甲状腺素、促肾上腺皮质激素、黄体生成素、泌乳素
C.抗利尿激素、心钠素、促甲状腺素、生长激素、泌乳素
D.黄体生成素、泌乳素、炔诺酮、游离皮质醇、促生长激素释放激素
E.内皮素、抗利尿激素、肾上腺素、促黑素、甲状旁腺素
B.促黑素、促甲状腺素、促肾上腺皮质激素、黄体生成素、泌乳素
4 4 2010 33. 下列哪种激素不是腺垂体分泌的激素(2.0分) A.GH
B.PRL
C.ADH
D.TSH
E.FSH、LH
C.ADH
4 4 2010 34. 与控制生长发育无关的激素为(2.0分) A.甲状腺素
B.胰岛素
C.雄激素
D.生长激素
E.醛固酮
E.醛固酮
4 4 2010 35. 内分泌系统的反馈调节不包括(2.0分) A.代谢物质对激素分泌的调节
B.免疫-内分泌系统之间的互相调节
C.下丘脑-垂体-甲状腺轴的调节
D.下丘脑-垂体-性腺轴的调节
E.下丘脑-垂体-肾上腺轴的调节
B.免疫-内分泌系统之间的互相调节
4 4 2010 36. Wolff-Chaikoff效应是指:(2.0分) A.缺乏碘而产生的抗甲状腺效应
B.过量的碘所产生的抗甲状腺效应
C.缺乏酪氨酸产生的抗甲状腺效应
D.过量的酪氨酸所产生的抗甲状腺效应
E.碘和酪氨酸缺乏而产生的促甲状腺效应
B.过量的碘所产生的抗甲状腺效应
4 4 2010 37. 食物中长期缺碘可引起(2.0分) A.甲状腺功能亢进
B.甲状腺组织萎缩
C.单纯性甲状腺肿
D.腺垂体功能减退
E.神经垂体功能减退
C.单纯性甲状腺肿
4 4 2010 38. 影响神经系统发育的最重要的激素是:(2.0分) A.胰岛素
B.甲状腺激素
C.生长素
D.糖皮质激素
E.肾上腺素
B.甲状腺激素
4 4 2010 39. 有关甲状腺激素的工作机制说法错误的是:(2.0分) A.T3 and T4 act by binding to nuclear receptors
B.T3 has 10 times the affinity for thyroid receptor as T4
C.increase transcription of large numbers of genes
D.increase synthesis of great numbers of proteins
E.T4 has 10 times the affinity for thyroid receptor as T3
E.T4 has 10 times the affinity for thyroid receptor as T3
4 4 2010 40. 糖皮质激素的作用是 (2.0分) A.抑制蛋白质合成
B.使血糖浓度降低
C.使肾脏排水能力降低
D.使红细胞和血小板数量增加
E.使肾脏排钾能力降低
D.使红细胞和血小板数量增加
4 4 2010 41. 生理剂量时能促进蛋白质分解的激素是:(2.0分) A.糖皮质激素
B.生长素
C.胰岛素
D.甲状腺激素
E.雄激素
A.糖皮质激素
4 4 2010 42. 临床上长期大剂量使用糖皮质激素时可以引起:(2.0分) A.血中ACTH浓度增高
B.淋巴细胞数目增加
C.肢端肥大症
D.肾上腺皮质渐趋萎缩
E.CHR分泌增多
D.肾上腺皮质渐趋萎缩
4 4 2010 43. 肾上腺髓质激素的分泌主要受下列哪项的调节?(2.0分) A.交感神经节前纤维
B.副交感神经节后纤维
C.血糖浓度
D.ACTH
E.肾上腺髓质激素的反馈调节
A.交感神经节前纤维
4 4 2010 44. 促进胰高血糖素分泌的主要因素是:(2.0分) A.高血糖
B.低血糖
C.血中氨基酸增多
D.血中脂肪酸减少
E.迷走神经兴奋
B.低血糖
4 4 2010 45. 胰岛素缺乏时可导致(2.0分) A.细胞内葡萄糖增加
B.肝糖原合成减少
C.血糖浓度下降
D.无尿糖
E.促进蛋白质合成
B.肝糖原合成减少
4 4 2010 46. 调节胰岛素分泌的主要因素是:(2.0分) A.血中脂肪酸浓度
B.血中氨基酸浓度
C.血糖浓度
D.胃肠激素
E.植物神经活动
C.血糖浓度
4 4 2010 47. 胰高血糖素对糖和脂肪代谢效应的靶器官是:(2.0分) A.心脏
B.肝脏
C.肾脏
D.肌肉
E.神经系统
B.肝脏
4 4 2010 48. 腺垂体嗜酸性细胞能分泌:(2.0分) A.催乳素
B.促甲状腺激素
C.促性腺激素
D.黑素细胞刺激素
E.黄体生长素
A.催乳素
4 4 2010 49. 盐皮质激素主要作用于肾脏的 (2.0分) A.近端小管曲部
B.近端小管直部
C.远端小管曲部
D.远端小管直部
E.细段
C.远端小管曲部
4 4 2010 50. 肾上腺皮质球状带细胞分泌的激素的功能是 (2.0分) A.排钾
B.保钠排钾
C.保钠
D.排钠保钾
E.以上均不是
B.保钠排钾
4 4 2011 1. 甲状腺素的主要用于 (2.5分) A.甲状腺危象
B.轻、中度甲状腺功能亢进
C.甲亢的手术前准备
D.黏液性水肿
E.交感神经活性增强引起的病变
D.黏液性水肿
4 4 2011 2. 丙硫氧嘧啶的主要适应证是 (2.5分) A.单纯性甲状腺肿
B.轻、中度甲状腺功能亢进
C.甲状腺功能低下
D.黏液性水肿
E.糖尿病酮症酸中毒
B.轻、中度甲状腺功能亢进
4 4 2011 3. 大剂量碘剂不能单独用于甲状腺功能亢进长期内科治疗的主要原因是 (2.5分) A.为甲状腺素合成提供原料
B.使T4转化为T3
C.使甲状腺对碘摄取减少,失去抑制甲状腺素合成的作用
D.使腺体血管增生,腺体增大
E.引起慢性碘中毒
C.使甲状腺对碘摄取减少,失去抑制甲状腺素合成的作用
4 4 2011 4. Insulin(胰岛素)用于 (2.5分) A.重症糖尿病(1型)
B.口服降血糖药无效的非胰岛素依赖性糖尿病
C.糖尿病合并酮症酸中毒
D.糖尿病合并严重感染
E.以上都是
E.以上都是
4 4 2011 5. Glibenclamide(格列本脲)降血糖的主要作用机制是 (2.5分) A.拮抗胰高血糖素的作用
B.增强肌肉组织糖的无氧酵解
C.刺激胰岛B细胞释放胰岛素
D.妨碍葡萄糖的肠道吸收
E.升高血清糖原水平
C.刺激胰岛B细胞释放胰岛素
4 4 2011 6. 哪一药物能促进脂肪组织摄取葡萄糖,降低肠道葡萄糖吸收? (2.5分) A.Insulin(胰岛素)
B.Acarbose(阿卡波糖)
C.Tolbutamide(甲苯磺丁脲)
D.Metformin(甲福明)
E.Rosiglitazone(罗格列酮)
D.Metformin(甲福明)
4 4 2011 7. 哪一基本作用是糖皮质激素抗炎作用的基础? (2.5分) A.影响了参与炎症的一些基因转录(基因组效应)
B.稳定溶酶体膜
C.抑制毛细血管和纤维母细胞的增生
D.减轻渗出、水肿、毛细血管扩张等炎症反应
E.增加肥大细胞颗粒的稳定性
A.影响了参与炎症的一些基因转录(基因组效应)
4 4 2011 8. 糖皮质激素和抗生素合用治疗严重感染的目的是 (2.5分) A.增强机体应激性
B.用激素缓解症状,度过危险期;而用抗生素控制感染
C.对抗抗生素的某些不良反应
D.增强机体防御能力
E.增强抗生素的抗菌作用
B.用激素缓解症状,度过危险期;而用抗生素控制感染
4 4 2011 9. 哪一病变不是糖皮质激素的不良反应? (2.5分) A.类肾上腺皮质功能亢进综合征
B.诱发或加重高血压
C.诱发或加重支气管哮喘
D.诱发或加重溃疡病
E.骨质疏松、肌肉萎缩
C.诱发或加重支气管哮喘
4 4 2011 10. 治疗急性严重中毒性感染时,糖皮质激素给药方式是 (2.5分) A.小剂量多次给药
B.大剂量肌内注射
C.大剂量突击静脉滴注
D.一次负荷量,然后给予维持量
E.较长时间大剂量给药
C.大剂量突击静脉滴注
4 4 2011 11. 腺垂体嗜酸性细胞能分泌:(2.5分) A.催乳素
B.促甲状腺激素
C.促性腺激素
D.黑素细胞刺激素
E.黄体生长素
A.催乳素
4 4 2011 12. 腺垂体分为 (2.5分) A.前叶和后叶
B.前叶和垂体柄
C.远侧部、中间部和漏斗部
D.远侧部、结节部和中间部
E.前叶、中间部和正中隆起
D.远侧部、结节部和中间部
4 4 2011 13. 神经垂体的功能是(2.5分) A.合成加压素和催产素
B.调节腺垂体的功能活动
C.贮存和释放下丘脑视上核和室旁核分泌的激素
D.受下丘脑弓状核分泌物的影响
E.分泌黑素细胞刺激素
C.贮存和释放下丘脑视上核和室旁核分泌的激素
4 4 2011 14. 甲状旁腺的腺细胞可分为(2.5分) A.主细胞和嗜酸性细胞
B.A细胞和B细胞
C.滤泡上皮细胞与C细胞
D.嗜酸性细胞和嗜碱性细胞
E.主细胞和嫌色细胞
A.主细胞和嗜酸性细胞
4 4 2011 15. Wolff-Chaikoff效应是指:(2.5分) A. 缺乏碘而产生的抗甲状腺效应
B.过量的碘所产生的抗甲状腺效应
C. 缺乏酪氨酸产生的抗甲状腺效应
D.过量的酪氨酸所产生的抗甲状腺效应
E.碘和酪氨酸缺乏而产生的促甲状腺效应
B.过量的碘所产生的抗甲状腺效应
4 4 2011 16. 甲状腺激素反馈调节主要影响:(2.5分) A.丘脑下部
B.神经垂体
C.腺垂体
D.甲状腺
E.大脑皮层
C.腺垂体
4 4 2011 17. 有关甲状腺激素的工作机制说法错误的是(2.5分) A.T3 and T4 act by binding to nuclear receptors
B.T3 has 10 times the affinity for thyroid receptor as T4
C.increase transcription of large numbers of genes
D.increase synthesis of great numbers of proteins
E.T4 has 10 times the affinity for thyroid receptor as T3
E.T4 has 10 times the affinity for thyroid receptor as T3
4 4 2011 18. 糖皮质激素的存在对去甲肾上腺素的作用是(2.5分) A.不存在任何影响
B.存在相互拮抗作用
C.存在允许作用
D.存在相互加强作用
E.有拮抗作用也有加强作用
C.存在允许作用
4 4 2011 19. 临床上长期大剂量使用糖皮质激素时可以引起(2.5分) A.血中ACTH浓度增高
B.淋巴细胞数目增加
C.肢端肥大症
D.肾上腺皮质渐趋萎缩
E.CHR分泌增多
D.肾上腺皮质渐趋萎缩
4 4 2011 20. 下列那种激素对蛋白质合成与储存是不可缺少的?(2.5分) A.胰高血糖素
B.胰岛素
C.胰多肽
D.生长抑素
E.降钙素
B.胰岛素
4 4 2011 21. 胰岛素缺乏时可导致(2.5分) A.细胞内葡萄糖增加
B.肝糖原合成减少
C.血糖浓度下降
D.无尿糖
E.促进蛋白质合成
B.肝糖原合成减少
4 4 2011 22. 胰高血糖素的主要生理作用是(2.5分) A.促进肝糖原分解和糖异生
B.促进葡萄糖利用
C.促进脂肪合成
D.促进胃液和胆汁分泌
E.加快心率
A.促进肝糖原分解和糖异生
4 4 2011 23. 下述激素分泌方式中哪项是错误的(2.5分) A.内分泌
B.邻分泌
C.自分泌
D.胞内分泌
E.外分泌
E.外分泌
4 4 2011 24. 关于ACTH的分泌下列哪项是错误的(2.5分) A.应激情况下分泌增加
B.受下丘脑分泌的CRH调节
C.受皮质醇的负反馈调节
D.存在脉冲式分泌(阵发性)
E.存在着清晨渐低午夜增高的昼夜节律
E.存在着清晨渐低午夜增高的昼夜节律
4 4 2011 25. 与控制生长发育无关的激素为(2.5分) A.甲状腺素
B.胰岛素
C.雄激素
D.生长激素
E.醛固酮
E.醛固酮
4 4 2011 26. 关于激素的正确概念是 (2.5分) A.由内分泌细胞产生的具有生物活性的物质
B.只能加强靶细胞的功能活动
C.直接作用于靶细胞膜受体而起作用
D.每种激素对其他激素的作用均无影响
A.由内分泌细胞产生的具有生物活性的物质
4 4 2011 27. Thyroid hormone作用的主要机制是(2.5分) A.与核受体结合,刺激mRNA生成
B.与核受体结合,促进cGMP生成
C.与膜受体结合,促进cAMP生成
D.与膜受体结合,抑制cAMP生成
A.与核受体结合,刺激mRNA生成
4 4 2011 28. 非肾上腺皮质增生所引起的临床表现是(2.5分) A.满月脸
B.高血压
C.低血糖
D.低血钾
C.低血糖
4 4 2011 29. 下列哪些属于先垂体功能减退症(2.5分) A.TSH下降,T3、T4下降
B.TSH增高,T3、T4增高
C.ACTH下降,皮质醇增高
D.FSH、LH增高,雌激素下降
C.ACTH下降,皮质醇增高
4 4 2011 30. Graves病浸润性突眼主要与(2.5分) A.TSAb有关
B.TSH有关
C.TPO有关
D.细胞免疫有关
D.细胞免疫有关
4 4 2011 31. 神经激素是指(2.5分) A.作用于神经细胞的激素
B.具有激素功能的神经递质
C.神经细胞分泌的激素
D.神经系统内存在的激素
E.来自神经细胞,并只作用于神经系统的激素
C.神经细胞分泌的激素
4 4 2011 32. Glucocorticoids comes mainly from(2.5分) A.hypothalamus
B.adrenal medulla
C.pituitary (gland)
D.adrenal cortex
E.ovary
D.adrenal cortex
4 4 2011 33. 成人生长素分泌过多将导致 (2.5分) A.腺瘤
B.粘液水肿
C.侏儒症
D.甲状腺肿
E.肢端肥大症
E.肢端肥大症
4 4 2011 34. 关于单纯性甲状腺肿,下列哪项说法不正确(2.5分) A.女性多于男性
B.发病与碘缺乏有关
C.可表现为多发结节
D.常伴有甲状腺功能低下
E.甲状腺体积增大
A.女性多于男性
4 4 2011 35. 关于结节性甲状腺肿的叙述,下列哪项是正确的 (2.5分) A.结节常为单个
B.结节边界清楚,有完整包膜
C.甲状腺滤泡小而一致,排列拥挤
D.甲状腺滤泡过度扩大,滤泡上皮增生与复旧不一致
E.结节内甲状腺与周围不一致,周围组织有受压现象
D.甲状腺滤泡过度扩大,滤泡上皮增生与复旧不一致
4 4 2011 36. 未经碘治疗的毒性甲状腺肿的形态学特征应除外(2.5分) A.滤泡增生,滤泡大小不等
B.滤泡上皮细胞呈立方形或高柱状
C.滤泡内胶质稀薄,可见吸收空泡
D.间质血管减少
E.多量淋巴细胞浸润并有滤泡形成
D.间质血管减少
4 4 2011 37. 甲状腺腺瘤与结节性甲状腺肿的主要区别是(2.5分) A.发病年龄和性别不同
B.有无完整的包膜,瘤内组织结构是否一致
C.结节的数目不同
D.滤泡中有无胶质
E.结节大小不同
B.有无完整的包膜,瘤内组织结构是否一致
4 4 2011 38. 下列哪项与甲状腺滤泡性癌有关(2.5分) A.有无滤泡形成
B.包膜或血管有无肿瘤细胞浸润
C.有无完整的包膜
D.多发于青少年
E.以上均不是
B.包膜或血管有无肿瘤细胞浸润
4 4 2011 39. 颈中部淋巴结转移性肿瘤灶中可见淀粉样物,首先考虑的原发灶是(2.5分) A.鼻咽泡状核细胞癌
B.甲状腺髓样癌
C.肺小细胞癌
D.甲状腺乳头状癌
E.甲状腺滤泡癌
B.甲状腺髓样癌
4 4 2011 40. 糖尿病胰岛中常有(2.5分) A.脂肪变性
B.颗粒变性
C.淀粉样变性
D.肾细动脉硬化
E.肾小管上皮细胞空泡样变性
C.淀粉样变性
4 4 2011 1. 甲状腺素的主要用于 (2.5分) A.甲状腺危象
B.轻、中度甲状腺功能亢进
C.甲亢的手术前准备
D.黏液性水肿
E.交感神经活性增强引起的病变
NaN
4 4 2011 2. 丙硫氧嘧啶的主要适应证是 (2.5分) A.单纯性甲状腺肿
B.轻、中度甲状腺功能亢进
C.甲状腺功能低下
D.黏液性水肿
E.糖尿病酮症酸中毒
NaN
4 4 2011 3. 大剂量碘剂不能单独用于甲状腺功能亢进长期内科治疗的主要原因是 (2.5分) A.为甲状腺素合成提供原料
B.使T4转化为T3
C.使甲状腺对碘摄取减少,失去抑制甲状腺素合成的作用
D.使腺体血管增生,腺体增大
E.引起慢性碘中毒
NaN
4 4 2011 4. Insulin(胰岛素)用于 (2.5分) A.重症糖尿病(1型)
B.口服降血糖药无效的非胰岛素依赖性糖尿病
C.糖尿病合并酮症酸中毒
D.糖尿病合并严重感染
E.以上都是
NaN
4 4 2011 5. Glibenclamide(格列本脲)降血糖的主要作用机制是 (2.5分) A.拮抗胰高血糖素的作用
B.增强肌肉组织糖的无氧酵解
C.刺激胰岛B细胞释放胰岛素
D.妨碍葡萄糖的肠道吸收
E.升高血清糖原水平
NaN
4 4 2011 6. 哪一药物能促进脂肪组织摄取葡萄糖,降低肠道葡萄糖吸收? (2.5分) A.Insulin(胰岛素)
B.Acarbose(阿卡波糖)
C.Tolbutamide(甲苯磺丁脲)
D.Metformin(甲福明)
E.Rosiglitazone(罗格列酮)
NaN
4 4 2011 7. 哪一基本作用是糖皮质激素抗炎作用的基础? (2.5分) A.影响了参与炎症的一些基因转录(基因组效应)
B.稳定溶酶体膜
C.抑制毛细血管和纤维母细胞的增生
D.减轻渗出、水肿、毛细血管扩张等炎症反应
E.增加肥大细胞颗粒的稳定性
NaN
4 4 2011 8. 糖皮质激素和抗生素合用治疗严重感染的目的是 (2.5分) A.增强机体应激性
B.用激素缓解症状,度过危险期;而用抗生素控制感染
C.对抗抗生素的某些不良反应
D.增强机体防御能力
E.增强抗生素的抗菌作用
NaN
4 4 2011 9. 哪一病变不是糖皮质激素的不良反应? (2.5分) A.类肾上腺皮质功能亢进综合征
B.诱发或加重高血压
C.诱发或加重支气管哮喘
D.诱发或加重溃疡病
E.骨质疏松、肌肉萎缩
NaN
4 4 2011 10. 治疗急性严重中毒性感染时,糖皮质激素给药方式是 (2.5分) A.小剂量多次给药
B.大剂量肌内注射
C.大剂量突击静脉滴注
D.一次负荷量,然后给予维持量
E.较长时间大剂量给药
NaN
4 4 2011 11. 腺垂体嗜酸性细胞能分泌:(2.5分) A.催乳素
B.促甲状腺激素
C.促性腺激素
D.黑素细胞刺激素
E.黄体生长素
NaN
4 4 2011 12. 腺垂体分为 (2.5分) A.前叶和后叶
B.前叶和垂体柄
C.远侧部、中间部和漏斗部
D.远侧部、结节部和中间部
E.前叶、中间部和正中隆起
NaN
4 4 2011 13. 神经垂体的功能是(2.5分) A.合成加压素和催产素
B.调节腺垂体的功能活动
C.贮存和释放下丘脑视上核和室旁核分泌的激素
D.受下丘脑弓状核分泌物的影响
E.分泌黑素细胞刺激素
NaN
4 4 2011 14. 甲状旁腺的腺细胞可分为(2.5分) A.主细胞和嗜酸性细胞
B.A细胞和B细胞
C.滤泡上皮细胞与C细胞
D.嗜酸性细胞和嗜碱性细胞
E.主细胞和嫌色细胞
NaN
4 4 2011 15. Wolff-Chaikoff效应是指:(2.5分) A. 缺乏碘而产生的抗甲状腺效应
B.过量的碘所产生的抗甲状腺效应
C. 缺乏酪氨酸产生的抗甲状腺效应
D.过量的酪氨酸所产生的抗甲状腺效应
E.碘和酪氨酸缺乏而产生的促甲状腺效应
NaN
4 4 2011 16. 甲状腺激素反馈调节主要影响:(2.5分) A.丘脑下部
B.神经垂体
C.腺垂体
D.甲状腺
E.大脑皮层
NaN
4 4 2011 17. 有关甲状腺激素的工作机制说法错误的是(2.5分) A.T3 and T4 act by binding to nuclear receptors
B.T3 has 10 times the affinity for thyroid receptor as T4
C.increase transcription of large numbers of genes
D.increase synthesis of great numbers of proteins
E.T4 has 10 times the affinity for thyroid receptor as T3
NaN
4 4 2011 18. 糖皮质激素的存在对去甲肾上腺素的作用是(2.5分) A.不存在任何影响
B.存在相互拮抗作用
C.存在允许作用
D.存在相互加强作用
E.有拮抗作用也有加强作用
NaN
4 4 2011 19. 临床上长期大剂量使用糖皮质激素时可以引起(2.5分) A.血中ACTH浓度增高
B.淋巴细胞数目增加
C.肢端肥大症
D.肾上腺皮质渐趋萎缩
E.CHR分泌增多
NaN
4 4 2011 20. 下列那种激素对蛋白质合成与储存是不可缺少的?(2.5分) A.胰高血糖素
B.胰岛素
C.胰多肽
D.生长抑素
E.降钙素
NaN
4 4 2011 21. 胰岛素缺乏时可导致(2.5分) A.细胞内葡萄糖增加
B.肝糖原合成减少
C.血糖浓度下降
D.无尿糖
E.促进蛋白质合成
NaN
4 4 2011 22. 胰高血糖素的主要生理作用是(2.5分) A.促进肝糖原分解和糖异生
B.促进葡萄糖利用
C.促进脂肪合成
D.促进胃液和胆汁分泌
E.加快心率
NaN
4 4 2011 23. 下述激素分泌方式中哪项是错误的(2.5分) A.内分泌
B.邻分泌
C.自分泌
D.胞内分泌
E.外分泌
NaN
4 4 2011 24. 关于ACTH的分泌下列哪项是错误的(2.5分) A.应激情况下分泌增加
B.受下丘脑分泌的CRH调节
C.受皮质醇的负反馈调节
D.存在脉冲式分泌(阵发性)
E.存在着清晨渐低午夜增高的昼夜节律
NaN
4 4 2011 25. 与控制生长发育无关的激素为(2.5分) A.甲状腺素
B.胰岛素
C.雄激素
D.生长激素
E.醛固酮
NaN
4 4 2011 26. 关于激素的正确概念是 (2.5分) A.由内分泌细胞产生的具有生物活性的物质
B.只能加强靶细胞的功能活动
C.直接作用于靶细胞膜受体而起作用
D.每种激素对其他激素的作用均无影响
NaN
4 4 2011 27. Thyroid hormone作用的主要机制是(2.5分) A.与核受体结合,刺激mRNA生成
B.与核受体结合,促进cGMP生成
C.与膜受体结合,促进cAMP生成
D.与膜受体结合,抑制cAMP生成
NaN
4 4 2011 28. 非肾上腺皮质增生所引起的临床表现是(2.5分) A.满月脸
B.高血压
C.低血糖
D.低血钾
NaN
4 4 2011 29. 下列哪些属于先垂体功能减退症(2.5分) A.TSH下降,T3、T4下降
B.TSH增高,T3、T4增高
C.ACTH下降,皮质醇增高
D.FSH、LH增高,雌激素下降
NaN
4 4 2011 30. Graves病浸润性突眼主要与(2.5分) A.TSAb有关
B.TSH有关
C.TPO有关
D.细胞免疫有关
NaN
4 4 2011 31. 神经激素是指(2.5分) A.作用于神经细胞的激素
B.具有激素功能的神经递质
C.神经细胞分泌的激素
D.神经系统内存在的激素
E.来自神经细胞,并只作用于神经系统的激素
NaN
4 4 2011 32. Glucocorticoids comes mainly from(2.5分) A.hypothalamus
B.adrenal medulla
C.pituitary (gland)
D.adrenal cortex
E.ovary
NaN
4 4 2011 33. 成人生长素分泌过多将导致 (2.5分) A.腺瘤
B.粘液水肿
C.侏儒症
D.甲状腺肿
E.肢端肥大症
NaN
4 4 2011 34. 关于单纯性甲状腺肿,下列哪项说法不正确(2.5分) A.女性多于男性
B.发病与碘缺乏有关
C.可表现为多发结节
D.常伴有甲状腺功能低下
E.甲状腺体积增大
NaN
4 4 2011 35. 关于结节性甲状腺肿的叙述,下列哪项是正确的 (2.5分) A.结节常为单个
B.结节边界清楚,有完整包膜
C.甲状腺滤泡小而一致,排列拥挤
D.甲状腺滤泡过度扩大,滤泡上皮增生与复旧不一致
E.结节内甲状腺与周围不一致,周围组织有受压现象
NaN
4 4 2011 36. 未经碘治疗的毒性甲状腺肿的形态学特征应除外(2.5分) A.滤泡增生,滤泡大小不等
B.滤泡上皮细胞呈立方形或高柱状
C.滤泡内胶质稀薄,可见吸收空泡
D.间质血管减少
E.多量淋巴细胞浸润并有滤泡形成
NaN
4 4 2011 37. 甲状腺腺瘤与结节性甲状腺肿的主要区别是(2.5分) A.发病年龄和性别不同
B.有无完整的包膜,瘤内组织结构是否一致
C.结节的数目不同
D.滤泡中有无胶质
E.结节大小不同
NaN
4 4 2011 38. 下列哪项与甲状腺滤泡性癌有关(2.5分) A.有无滤泡形成
B.包膜或血管有无肿瘤细胞浸润
C.有无完整的包膜
D.多发于青少年
E.以上均不是
NaN
4 4 2011 39. 颈中部淋巴结转移性肿瘤灶中可见淀粉样物,首先考虑的原发灶是(2.5分) A.鼻咽泡状核细胞癌
B.甲状腺髓样癌
C.肺小细胞癌
D.甲状腺乳头状癌
E.甲状腺滤泡癌
NaN
4 4 2011 40. 糖尿病胰岛中常有(2.5分) A.脂肪变性
B.颗粒变性
C.淀粉样变性
D.肾细动脉硬化
E.肾小管上皮细胞空泡样变性
NaN
4 4 2012 1. 腺垂体嗜酸性细胞能分泌(2.5分) A.促肾上腺皮质激素
B.促甲状腺激素
C.促性腺激素
D.生长激素
E.黄体生长素
E.黄体生长素
4 4 2012 2. 与肢端肥大症相关的是(2.5分) A.垂体细胞
B.嗜酸性细胞
C.嗜碱性细胞
D.嫌色细胞
E.以上都不是
B.嗜酸性细胞
4 4 2012 3. 甲状腺滤泡上皮细胞内与甲状腺激素合成和释放有关的细胞器是(2.5分) A.粗面内质网、高尔基复合体、溶酶体、微体
B.中心体、粗面内质网、高尔基复合体、溶酶体
C.高尔基复合体、微体、溶酶体、线粒体
D.高尔基复合体、溶酶体、线粒体、中心体
E.粗面内质网、高尔基复合体、溶酶体、线粒体
A.粗面内质网、高尔基复合体、溶酶体、微体
4 4 2012 4. 神经激素是指(2.5分) A.作用于神经细胞的激素
B.具有激素功能的神经递质
C.神经细胞分泌的激素
D.神经系统内存在的激素
E.来自神经细胞,并只作用于神经系统的激素
C.神经细胞分泌的激素
4 4 2012 5. Glucocorticoids comes mainly from(2.5分) A.hypothalamus
B.adrenal medulla
C.pituitary (gland)
D.Zona glomerulosa of adrenal cortex
E.Zona fasciculate of adrenal cortex
E.Zona fasciculate of adrenal cortex
4 4 2012 6. 成人生长素分泌过多将导致(2.5分) A.腺瘤
B.粘液水肿
C.侏儒症
D.甲状腺肿
E.肢端肥大症
E.肢端肥大症
4 4 2012 7. 下列哪项不是腺垂体功能减退症患者的实验室检查结果(2.5分) A.雌二醇水平降低
B.FSH、LH、TSH、ACTH、GH、PRL均减少
C.24小时尿17-羟皮质类固醇及游离皮质醇降低
D.血浆皮质醇浓度降低,ADH降低
E.血清总T4、游离T4均降低,总T3、游离T3正常
D.血浆皮质醇浓度降低,ADH降低
4 4 2012 8. 关于ACTH的分泌下列哪项是错误的(2.5分) A.应激情况下分泌增加
B.受下丘脑分泌的CRH调节
C.受皮质醇的负反馈调节
D.存在脉冲式分泌(阵发性)
E.存在着清晨渐低午夜增高的昼夜节律
E.存在着清晨渐低午夜增高的昼夜节律
4 4 2012 9. 下丘脑激素通过哪一途径调节腺垂体各激素的分泌(2.5分) A.下丘脑垂体神经束
B.神经递质
C.激素第二信使
D.下丘脑激素代谢产物
E.垂体门脉系统
E.垂体门脉系统
4 4 2012 10. 关于激素的正确概念是(2.5分) A.由内分泌细胞产生的具有生物活性的物质
B.只能加强靶细胞的功能活动
C.直接作用于靶细胞膜受体而起作用
D.每种激素对其他激素的作用均无影响
A.由内分泌细胞产生的具有生物活性的物质
4 4 2012 11. Thyroid hormone作用的主要机制是(2.5分) A.与核受体结合,刺激mRNA生成
B.与核受体结合,促进cGMP生成
C.与膜受体结合,促进cAMP生成
D.与膜受体结合,抑制cAMP生成
A.与核受体结合,刺激mRNA生成
4 4 2012 12. 下列哪些属于先垂体功能减退症(2.5分) A.TSH下降,T3、T4下降
B.TSH增高,T3、T4增高
C.ACTH下降,皮质醇增高
D.FSH、LH增高,雌激素下降
E.A和D
C.ACTH下降,皮质醇增高
4 4 2012 13. 内分泌系统的反馈调节是指(2.5分) A.神经系统对内分泌系统的调节
B.内分泌系统对神经系统的调节
C.免疫系统对内分泌系统的调节
D.下丘脑-垂体-靶腺之间的相互调节
E.A和B
D.下丘脑-垂体-靶腺之间的相互调节
4 4 2012 14. 非肾上腺皮质增生所引起的临床表现是(2.5分) A.满月脸
B.高血压
C.低血糖
D.低血钾
E.以上均有误
C.低血糖
4 4 2012 15. 2型糖尿病的发病机制是(2.5分) A.胰岛B细胞遗传性缺陷
B.胰岛B细胞自身免疫反应性损伤
C.胰岛B细胞破坏,胰岛素绝对不足
D.胰岛素抵抗和胰岛素分泌缺陷
E.以上均有误
D.胰岛素抵抗和胰岛素分泌缺陷
4 4 2012 16. Wolff-Chaikoff效应是指(2.5分) A.缺乏碘而产生的抗甲状腺效应
B.过量的碘所产生的抗甲状腺效应
C.缺乏酪氨酸产生的抗甲状腺效应
D.过量的酪氨酸所产生的抗甲状腺效应
E.碘和酪氨酸缺乏而产生的促甲状腺效应
B.过量的碘所产生的抗甲状腺效应
4 4 2012 17. 甲状腺激素反馈调节主要影响(2.5分) A.丘脑下部
B.神经垂体
C.腺垂体
D.甲状腺
E.大脑皮层
C.腺垂体
4 4 2012 18. 糖皮质激素的存在对去甲肾上腺素的作用是(2.5分) A.不存在任何影响
B.存在相互拮抗作用
C.存在允许作用
D.存在相互加强作用
E.有拮抗作用也有加强作用
C.存在允许作用
4 4 2012 19. 临床上长期大剂量使用糖皮质激素时可以引起(2.5分) A.血中ACTH浓度增高
B.淋巴细胞数目增加
C.肢端肥大症
D.肾上腺皮质渐趋萎缩
E.CHR分泌增多
D.肾上腺皮质渐趋萎缩
4 4 2012 20. 下列那种激素对蛋白质合成与储存是不可缺少的?(2.5分) A.胰高血糖素
B.胰岛素
C.胰多肽
D.生长抑素
E.降钙素
B.胰岛素
4 4 2012 21. 胰岛素缺乏时可导致(2.5分) A.细胞内葡萄糖增加
B.肝糖原合成减少
C.血糖浓度下降
D.无尿糖
E.促进蛋白质合成
B.肝糖原合成减少
4 4 2012 22. 胰高血糖素对糖和脂肪代谢效应的靶器官是(2.5分) A.心脏
B.肝脏
C.肾脏
D.肌肉
E.神经系统
B.肝脏
4 4 2012 23. 甲状腺癌中,哪种类型最常见(2.5分) A.髓样癌
B.乳头状癌
C.滤泡癌
D.未分化癌
E.嗜酸细胞腺癌
B.乳头状癌
4 4 2012 24. 常出现砂粒体的甲状腺疾病是(2.5分) A.胎儿型腺瘤
B.慢性淋巴细胞性甲状腺炎
C.弥漫性非毒性甲状腺肿结节期
D.髓样癌
E.乳头状癌
E.乳头状癌
4 4 2012 25. 由滤泡旁细胞发生的甲状腺肿瘤是(2.5分) A.乳头状癌
B.髓样癌
C.滤泡性癌
D.未分化癌
E.胚胎型腺瘤
B.髓样癌
4 4 2012 26. 属于弥漫神经内分泌细胞源性的甲状腺癌是(2.5分) A.滤泡性癌
B.髓样癌
C.未分化癌
D.乳头状癌
E.间变性癌
B.髓样癌
4 4 2012 27. 下列哪项不是未经治疗的毒性甲状腺肿的形态学特征(2.5分) A.滤泡上皮细胞呈立方形或高柱状
B.滤泡上皮细胞增生形成乳头状结构和小滤泡
C.滤泡内胶质少而稀薄,可见吸收空泡
D.间质内纤维组织增生明显、包绕多少不等的滤泡形成结节状病灶
E.间质内可见多量淋巴细胞浸润并有淋巴滤泡形成
D.间质内纤维组织增生明显、包绕多少不等的滤泡形成结节状病灶
4 4 2012 28. 间质常有淀粉样物质沉着的甲状腺肿瘤是(2.5分) A.乳头状癌
B.滤泡癌o
C.髓样癌
D.未分化癌
E.滤泡性腺瘤
C.髓样癌
4 4 2012 29. 胰岛β细胞瘤与下列哪项有关(2.5分) A.阵发性高血压
B.低血糖症
C.高血糖症
D.高钙血症
E.低钙血症
B.低血糖症
4 4 2012 30. 甲状腺素的主要用于 (2.5分) A.甲状腺危象
B.轻、中度甲状腺功能亢进
C.甲亢的手术前准备
D.黏液性水肿
E.交感神经活性增强引起的病变
D.黏液性水肿
4 4 2012 31. 丙硫氧嘧啶的主要适应证是 (2.5分) A.单纯性甲状腺肿
B.轻、中度甲状腺功能亢进
C.甲状腺功能低下
D.黏液性水肿
E.糖尿病酮症酸中毒
B.轻、中度甲状腺功能亢进
4 4 2012 32. 大剂量碘剂不能单独用于甲状腺功能亢进长期内科治疗的主要原因是 (2.5分) A.为甲状腺素合成提供原料
B.使T4转化为T3
C.使甲状腺对碘摄取减少,失去抑制甲状腺素合成的作用
D.使腺体血管增生,腺体增大
E.引起慢性碘中毒
C.使甲状腺对碘摄取减少,失去抑制甲状腺素合成的作用
4 4 2012 33. Insulin(胰岛素)用于 (2.5分) A.重症糖尿病(1型)
B.口服降血糖药无效的非胰岛素依赖性糖尿病
C.糖尿病合并酮症酸中毒
D.糖尿病合并严重感染
E.以上都是
E.以上都是
4 4 2012 34. Glibenclamide(格列本脲)降血糖的主要作用机制是 (2.5分) A.拮抗胰高血糖素的作用
B.增强肌肉组织糖的无氧酵解
C.刺激胰岛B细胞释放胰岛素
D.妨碍葡萄糖的肠道吸收
E.升高血清糖原水平
C.刺激胰岛B细胞释放胰岛素
4 4 2012 35. 下列哪一项是二甲双胍的特征? (2.5分) A.不是2型糖尿病的首选治疗
B.抑制肝脏糖异生
C.促进脂肪生成
D.不能与磺酰脲类或胰岛素合用
E.会引起体重增加
B.抑制肝脏糖异生
4 4 2012 36. 哪一基本作用是糖皮质激素抗炎作用的基础? (2.5分) A.影响了参与炎症的一些基因转录(基因组效应)
B.稳定溶酶体膜
C.抑制毛细血管和纤维母细胞的增生
D.减轻渗出、水肿、毛细血管扩张等炎症反应
E.增加肥大细胞颗粒的稳定性
A.影响了参与炎症的一些基因转录(基因组效应)
4 4 2012 37. 对于慢性炎症,糖皮质激素的主要作用是 (2.5分) A.抑制炎症性细胞因子合成,减轻炎症
B.抑制花生四烯酸释放,使炎症介质PG合成减少
C.促进炎症区的血管收缩,降低其通透性
D.稳定溶酶体膜,减少蛋白水解酶的释放
E.抑制肉芽组织生长,防止粘连和疤痕
E.抑制肉芽组织生长,防止粘连和疤痕
4 4 2012 38. 糖皮质激素的禁忌症还包括 (2.5分) A.中毒性痢疾
B.活动性消化性溃疡
C.感染性休克
D.重症伤寒
E.肾病综合征
B.活动性消化性溃疡
4 4 2012 39. 长期应用糖皮质激素的不良反应之一是 (2.5分) A.向心性肥胖
B.高血钙
C.低血糖
D.高血钾
E.磷的排泄减少
A.向心性肥胖
4 4 2012 40. 早晨一次给予糖皮质激素的治疗方法,其依据是 (2.5分) A.口服吸收更加完全
B.体内代谢灭活缓慢,有效血药浓度稳定
C.与靶细胞受体结合牢固,作用持久
D.与体内糖皮质激素分泌的昼夜节律一致
E.有效血浓度更加持久
D.与体内糖皮质激素分泌的昼夜节律一致
4 4 2013 1. 神经激素是指(2.5分) A.作用于神经细胞的激素
B.具有激素功能的神经递质
C.神经细胞分泌的激素
D.神经系统内存在的激素
E.来自神经细胞,并只作用于神经系统的激素
C.神经细胞分泌的激素
4 4 2013 2. 下列为腺垂体所分泌的激素是(2.5分) A.促肾上腺皮质激素释放激素
B.促肾上腺皮质激素
C.肾上腺皮质激素
D.催产素
E.抗利尿激素
B.促肾上腺皮质激素
4 4 2013 3. 成人生长素分泌过多将导致 (2.5分) A.腺瘤
B.粘液水肿
C.侏儒症
D.甲状腺肿
E.肢端肥大症
E.肢端肥大症
4 4 2013 4. Pro-opiomelanocortin (POMC) is a precursor for the peptides below except for(2.5分) A.ACTH
B.β-lipotropin (β-LPH)
C.γ-lipotropin (γ-LPH),
D.β-endorphin,
E.CRH
E.CRH
4 4 2013 5. 内分泌腺或组织所分泌的激素主要通过下列哪种方式传递(2.5分) A.通过血液传递
B.通过细胞外液局部传递
C.通过细胞外液邻近传递
D.直接作用于自身细胞
E.细胞内直接作用
A.通过血液传递
4 4 2013 6. 有关功能减退的内分泌疾病的治疗下列哪项是错误的(2.5分) A.激素替代治疗
B.相关药物治疗
C.放射性同位素治疗
D.器官组织移植
E.细胞移植
C.放射性同位素治疗
4 4 2013 7. 关于ACTH的分泌下列哪项是错误的(2.5分) A.应激情况下分泌增加
B.受下丘脑分泌的CRH调节
C.受皮质醇的负反馈调节
D.存在脉冲式分泌(阵发性)
E.存在着清晨渐低午夜增高的昼夜节律
E.存在着清晨渐低午夜增高的昼夜节律
4 4 2013 8. 下丘脑激素通过哪一途径调节腺垂体各激素的分泌(2.5分) A.下丘脑垂体神经束
B.神经递质
C.激素第二信使
D.下丘脑激素代谢产物
E.垂体门脉系统
E.垂体门脉系统
4 4 2013 9. 腺垂体嗜酸性细胞能分泌(2.5分) A.催乳素
B.促甲状腺激素
C.促性腺激素
D.黑素细胞刺激素
E.黄体生长素
A.催乳素
4 4 2013 10. 腺垂体分为(2.5分) A.前叶和后叶
B.前叶和垂体柄
C.远侧部、中间部和漏斗部
D.远侧部、结节部和中间部
E.前叶、中间部和正中隆起
D.远侧部、结节部和中间部
4 4 2013 11. 神经垂体的功能是(2.5分) A.合成加压素和催产素
B.调节腺垂体的功能活动
C.贮存和释放下丘脑视上核和室旁核分泌的激素
D.受下丘脑弓状核分泌物的影响
E.分泌黑素细胞刺激素
C.贮存和释放下丘脑视上核和室旁核分泌的激素
4 4 2013 12. 甲状腺滤泡腔中贮存的物质是(2.5分) A.四碘甲状腺原氨酸
B.三碘甲状腺原氨酸
C.甲状腺球蛋白
D.碘化甲状腺球蛋白
E.酪氨酸
D.碘化甲状腺球蛋白
4 4 2013 13. 产生肾上腺素的细胞是(2.5分) A.嗜酸性细胞
B.嗜铬细胞
C.主细胞
D.交感神经节细胞
E.嫌色细胞
B.嗜铬细胞
4 4 2013 14. For which of the following thyroid disorders is levothyroxine not the recommended treatment? (2.5分) A.Hashmoto's thyroiditis
B.Cretinism
C.Myxedema
D.Endemic goiter
E.All of the above
D.Endemic goiter
4 4 2013 15. chlorpropamide (Diabinese)(2.5分) A.useful adjunct in treatment of type 1 diabets(IDDM)
B.absorbed slowly from GI tract
C.action is potentiated by phenytoin and propranolol
D.main side effect is hypoglycemia
E.all of the above
E.all of the above
4 4 2013 16. The initial and crucial event that enables glyburide to promote insulin release is:(2.5分) A.Increasing K+ efflux
B.Binding to ATP-sensitive K+ channels
C.Closing of voltage dependent Ca2+ channels
D.Decreasing phosphorylation
E.Increasing ADH secretion
B.Binding to ATP-sensitive K+ channels
4 4 2013 17. Thiocarbamide drugs(2.5分) A.Result in increased TSH formation
B.Inhibit iodide uptake by the thyroid gland
C.May cause agranulocytosis
D.Cross the placenta but do not affect the fetus
E.A and C are correct
E.A and C are correct
4 4 2013 18. Which of the following oral hypoglycemic agents can be used in diabetes insipidus?(2.5分) A.Glyburide
B.Metformin
C.Acarbose
D.Pioglitazone
E.Repaglinide
A.Glyburide
4 4 2013 19. 大剂量碘能抑制 (2.5分) A.过氧化酶
B.甲状腺球蛋白水解酶
C.磷酸二酯酶
D.碱性磷酸酶
E.磷脂酶
A.过氧化酶
4 4 2013 20. 既有降血糖作用,又有抗利尿作用的药是(2.5分) A.二甲双胍
B.氯磺丙脲
C.阿卡波糖
D.罗格列酮
E.甲苯磺丁脲
B.氯磺丙脲
4 4 2013 21. 下列哪种病变很少有可能成为内分泌激素增多症的原因(2.5分) A.内分泌腺的增生
B.内分泌腺的肿瘤
C.内分泌腺的炎症
D.靶细胞异常
E.激素灭活机制的障碍
C.内分泌腺的炎症
4 4 2013 22. 关于亚急性甲状腺炎,下列哪项是正确的(2.5分) A.与细菌感染有关,常伴有发热
B.甲状腺增大,质软,结节状
C.一般认为属于自身免疫疾病
D.形成类似结核结节的肉芽肿
E.本病男性多于女性
D.形成类似结核结节的肉芽肿
4 4 2013 23. 关于结节性甲状腺肿,下列描述哪一项是错误的(2.5分) A.结节具有完整的包膜
B.结节明显压迫周围甲状腺组织
C.部分滤泡上皮可呈柱状或乳头状增生
D.结节内常有出血、坏死、纤维增生等改变
E.从弥漫性甲状腺肿可以移行为结节性甲状腺肿
A.结节具有完整的包膜
4 4 2013 24. 下列肿瘤中,哪一项不属于甲状腺滤泡性肿瘤(2.5分) A.胚胎型腺瘤
B.胎儿型腺瘤
C.单纯型腺瘤
D.乳头型腺瘤
E.胶样型腺瘤
D.乳头型腺瘤
4 4 2013 25. 有血中降钙素增多的甲状腺肿瘤是(2.5分) A.嗜酸性细胞腺瘤
B.乳头状癌
C.胎儿型腺癌
D.髓样癌
E.未分化癌
D.髓样癌
4 4 2013 26. 关于糖尿病,下列哪一项是错误的(2.5分) A.发病后病人多肥胖
B.青年人发病者家族内常可证明其他人亦有糖尿病患者
C.糖尿病主要是由胰岛素相对的缺乏所引起
D.糖尿病病人比非糖尿病病人较早而且较重地出现动脉粥样硬化
E.糖代谢发生障碍后脂肪和蛋白质代谢也可出现异常
A.发病后病人多肥胖
4 4 2013 27. 下列哪项对区别结节性甲状腺肿和甲状腺瘤没有帮助(2.5分) A.完整的包膜
B.肿块压迫周围甲状腺组织
C.肿块内组织结构比较一致
D.肿块内组织形态与周围甲状腺组织不同
E.肿块为单个,大小不超过3cm
E.肿块为单个,大小不超过3cm
4 4 2013 28. APUD细胞系不包括下列哪种细胞(2.5分) A.肺的嗜银细胞
B.甲状腺滤泡旁细胞
C.胰岛细胞
D.肾上腺皮质细胞
E.消化管的嗜银细胞
D.肾上腺皮质细胞
4 4 2013 29. 关于胰岛细胞瘤的叙述,下列哪项是错误的(2.5分) A.部分肿瘤可分泌胰岛素
B.瘤细胞可呈菊形团排列
C.间质常出现淀粉样物质沉积
D.核有不同程度的异型性
E.绝大多数为恶性肿瘤
E.绝大多数为恶性肿瘤
4 4 2013 30. Wolff-Chaikoff效应是指(2.5分) A.缺乏碘而产生的抗甲状腺效应
B.过量的碘所产生的抗甲状腺效应
C.缺乏酪氨酸产生的抗甲状腺效应
D.过量的酪氨酸所产生的抗甲状腺效应
E.碘和酪氨酸缺乏而产生的促甲状腺效应
B.过量的碘所产生的抗甲状腺效应
4 4 2013 31. 甲状腺激素反馈调节主要影响(2.5分) A.丘脑下部
B.神经垂体
C.腺垂体
D.甲状腺
E.大脑皮层
C.腺垂体
4 4 2013 32. 下列哪种激素对儿茶酚胺心血管效应起允许作用(2.5分) A.抗利尿激素
B.甲状腺激素
C.生长素
D.糖皮质激素
E.醛固酮
B.甲状腺激素
4 4 2013 33. 糖皮质激素的作用是 (2.5分) A.抑制蛋白质合成
B.使血糖浓度降低
C.使肾脏排水能力降低
D.使红细胞和血小板数量增加
E.使肾脏排钾能力降低
D.使红细胞和血小板数量增加
4 4 2013 34. 长期大量注射氢化可的松时主要可引起血中(2.5分) A.红细胞减少
B.ACTH↓
C.血糖下降
D.血钙升高
E.CRH↑
B.ACTH↓
4 4 2013 35. 糖皮质激素对物质代谢的影响是促进(2.5分) A.肝内蛋白质分解
B.肝外蛋白质合成
C.糖原合成
D.核酸合成
E.使脂肪重新分配,四肢脂肪分解
E.使脂肪重新分配,四肢脂肪分解
4 4 2013 36. 关于胰岛素的正确论述是(2.5分) A.由胰岛的A细胞合成分泌
B.具有降血糖作用
C.血糖浓度对胰岛素分泌有负反馈作用
D.促进脂肪的分解
E.交感神经兴奋,胰岛素分泌增加
B.具有降血糖作用
4 4 2013 37. 调节胰岛素分泌的主要因素是(2.5分) A.血中脂肪酸浓度
B.血中氨基酸浓度
C.血糖浓度
D.胃肠激素
E.植物神经活动
C.血糖浓度
4 4 2013 38. 下列那种激素对蛋白质合成与储存是不可缺少的?(2.5分) A.胰高血糖素
B.胰岛素
C.胰多肽
D.生长抑素
E.降钙素
B.胰岛素
4 4 2013 39. 胰高血糖素对糖和脂肪代谢效应的靶器官是(2.5分) A.心脏
B.肝脏
C.肾脏
D.肌肉
E.神经系统
B.肝脏
4 4 2013 40. 以下关于胰高血糖素作用的说法哪个是错误的(2.5分) A.主要工作地点在肾脏
B.促进肝糖原分解和糖异生
C.促进蛋白质分解和产尿增加
D.生酮作用增加
E.在胰岛素存在的条件下,对骨骼肌的效应很小
A.主要工作地点在肾脏
4 4 2014 1. 内分泌腺或组织所分泌的激素主要通过下列哪种方式传递(2.5分) A.通过血液传递
B.通过细胞外液局部传递
C.通过细胞外液邻近传递
D.直接作用于自身细胞
E.细胞内直接作用
A.通过血液传递
4 4 2014 2. 下列哪项不是腺垂体功能减退症患者的实验室检查结果(2.5分) A.雌二醇水平降低
B.FSH、LH、TSH、ACTH、GH、PRL均减少
C.24小时尿17-羟皮质类固醇及游离皮质醇降低
D.血浆皮质醇浓度降低,ADH降低
E.血清总T4、游离T4均降低,总T3、游离T3正常
D.血浆皮质醇浓度降低,ADH降低
4 4 2014 3. 有关功能减退的内分泌疾病的治疗下列哪项是错误的(2.5分) A.激素替代治疗
B.相关药物治疗
C.放射性同位素治疗
D.器官组织移植
E.细胞移植
C.放射性同位素治疗
4 4 2014 4. 关于ACTH的分泌下列哪项是错误的(2.5分) A.应激情况下分泌增加
B.受下丘脑分泌的CRH调节
C.受皮质醇的负反馈调节
D.可以在非垂体部位分泌
E.清晨渐低午夜增高的昼夜节律
E.清晨渐低午夜增高的昼夜节律
4 4 2014 5. 腺垂体嗜酸性细胞能分泌: (2.5分) A.催乳素
B.促甲状腺激素
C.促性腺激素
D.黑素细胞刺激素
E.黄体生长素
A.催乳素
4 4 2014 6. 盐皮质激素主要作用于肾脏的(2.5分) A.近端小管曲部
B.近端小管直部
C.远端小管曲部
D.远端小管直部
E.细段
C.远端小管曲部
4 4 2014 7. 肾上腺皮质细胞超微结构特点是富含(2.5分) A.粗面内质网和滑面内质网
B.滑面内质网和溶酶体
C.粗面内质网和高尔基复合体
D.高尔基复合体和溶酶体
E.滑面内质网和脂滴
E.滑面内质网和脂滴
4 4 2014 8. 甲状腺滤泡腔中贮存的物质是(2.5分) A.四碘甲状腺原氨酸
B.三碘甲状腺原氨酸
C.甲状腺球蛋白
D.碘化甲状腺球蛋白
E.酪氨酸
D.碘化甲状腺球蛋白
4 4 2014 9. 成年人甲状腺激素缺乏会引起甲状腺肿大,主要原因是(2.5分) A.TSH的大量分泌
B.Thyroglobulin的大量分泌
C.甲状腺对I-的大量摄取
D.自身抗体抗甲状腺的炎症效应
E.碘和酪氨酸缺乏而产生的促甲状腺效应
A.TSH的大量分泌
4 4 2014 10. 食物中长期缺碘可引起(2.5分) A.甲状腺功能亢进
B.甲状腺组织萎缩
C.单纯性甲状腺肿
D.腺垂体功能减退
E.神经垂体功能减退
C.单纯性甲状腺肿
4 4 2014 11. 影响神经系统发育的最重要的激素是(2.5分) A.胰岛素
B.甲状腺激素
C.生长素
D.糖皮质激素
E.肾上腺素
B.甲状腺激素
4 4 2014 12. 下列哪种激素对儿茶酚胺心血管效应起允许作用(permissive effects)(2.5分) A.抗利尿激素
B.甲状腺激素
C.生长素
D.糖皮质激素
E.醛固酮
B.甲状腺激素
4 4 2014 13. 最常发生嗜铬细胞瘤的器官是(2.5分) A.甲状腺
B.胰岛
C.脑垂体
D.肾上腺髓质
E.肾上腺皮质
D.肾上腺髓质
4 4 2014 14. 嗜铬细胞瘤常引起下列哪项病症? (2.5分) A.Cushing's综合征
B.Sheehan's综合征
C.甲亢
D.阵发性高血压
E.高血糖症
D.阵发性高血压
4 4 2014 15. 下列哪种肿瘤不属于APUD瘤? (2.5分) A.甲状腺髓样癌
B.胃泌素瘤
C.肾上腺皮质腺瘤
D.肺燕麦细胞癌
E.嗜铬细胞瘤
C.肾上腺皮质腺瘤
4 4 2014 16. 最常见的胰腺肿瘤是(2.5分) A.胰岛细胞瘤
B.胰高血糖素瘤
C.生长抑素瘤
D.纤维瘤
E.胃泌素瘤
A.胰岛细胞瘤
4 4 2014 17. 胰岛细胞瘤的特点除外(2.5分) A.单发,较小
B.圆形结节状
C.有完整或不完整包膜
D.组织学特点多样
E.常伴出血
E.常伴出血
4 4 2014 18. 下列除哪项外与甲亢有关? (2.5分) A.体温升高
B.食欲增加,体重减轻
C.心律失常
D.低血压
E.有刺激甲状腺增生的免疫球蛋白
D.低血压
4 4 2014 19. 下列除哪项外都是单纯性甲状腺肿的表现? (2.5分) A.甲状腺肿大
B.肿大的甲状腺压迫食管、气管
C.甲状腺功能障碍
D.土壤及水中缺碘
E.肿大的甲状腺压迫喉返神经
E.肿大的甲状腺压迫喉返神经
4 4 2014 20. 可发生癌变的甲状腺疾病是(2.5分) A.甲状腺功能亢进
B.甲状腺功能减退
C.结节性甲状腺肿
D.慢性甲状腺炎
E.甲状腺腺瘤
C.结节性甲状腺肿
4 4 2014 21. 诊断亚急性甲状腺炎的特征性病理学改变是甲状腺(2.5分) A.体积增大、不对称
B.切面斑片状坏死
C.滤泡坏死
D.肉芽肿性炎
E.纤维化
D.肉芽肿性炎
4 4 2014 22. 慢性甲状腺炎的病理学特点不包括下列哪项? (2.5分) A.甲状腺质地较软
B.结节状
C.炎细胞浸润
D.甲状腺滤泡萎缩、破坏
E.甲状腺与周围组织黏连紧密
A.甲状腺质地较软
4 4 2014 23. 大剂量碘能抑制(2.5分) A.过氧化酶
B.甲状腺球蛋白水解酶
C.磷酸二酯酶
D.碱性磷酸酶
E.磷脂酶
A.过氧化酶
4 4 2014 24. 甲亢术前准备正确给药应是(2.5分) A.只给碘化物
B.先给硫脲类,术前两周再给碘化物
C.只给硫脲类
D.先给碘化物,术前两周再给硫脲类
E.同时给予硫脲类和碘化物
B.先给硫脲类,术前两周再给碘化物
4 4 2014 25. 硫脲类药物治疗甲亢的机理是(2.5分) A.减少甲状腺激素的释放
B.对抗甲状腺素的作用
C.抑制甲状腺组织摄碘
D.主要是抑制甲状腺过氧化酶
E.抑制甲状腺球蛋白水解酶
D.主要是抑制甲状腺过氧化酶
4 4 2014 26. 能抑制T4转化为T3的抗甲状腺药物是(2.5分) A.碘化钾
B.丙硫氧嘧啶
C.普奈洛尔
D.复方碘溶液
E.他巴唑
B.丙硫氧嘧啶
4 4 2014 27. 关于糖皮质激素的抗炎作用机制,下列哪种说法是错误的(2.5分) A.抑制磷脂酶A2
B.稳定溶酶体酶
C.直接杀灭致病菌
D.抑制多种炎性细胞因子
E.抑制诱导型NO合成酶
C.直接杀灭致病菌
4 4 2014 28. 糖皮质激素的抗休克作用,以下哪点是错误的(2.5分) A.扩张痉挛的血管
B.减少心肌抑制因子形成
C.降解细菌毒素
D.提高机体对毒素的耐受力
E.稳定溶酶体酶
C.降解细菌毒素
4 4 2014 29. 小剂量糖皮质激素替代疗法用于治疗(2.5分) A.败血症
B.肾病综合症
C.肾上腺嗜铬细胞瘤
D.严重中毒性感染
E.垂体前叶功能减退
E.垂体前叶功能减退
4 4 2014 30. 糖皮质激素的抗毒作用是指 (2.5分) A.破坏内毒素
B.中和内毒素
C.直接杀灭内毒素
D.提高机体对内毒素的耐受力
E.杀灭病毒
D.提高机体对内毒素的耐受力
4 4 2014 31. 经体内转化后才有活性的糖皮质激素是(2.5分) A.氢化可的松
B.地塞米松
C.泼尼松
D.氟氢松
E.泼尼松龙
C.泼尼松
4 4 2014 32. Corticosteroid inhibits: (2.5分) A.gluconeogenesis
B.leukotriene C4 and D4 synthesis
C.platelet thromboxane A2 synthesis
D.neutrophil production
E.lipocortin production
B.leukotriene C4 and D4 synthesis
4 4 2014 33. 神经激素是指 (2.5分) A.作用于神经细胞的激素
B.具有激素功能的神经递质
C.神经细胞分泌的激素
D.神经系统内存在的激素
E.来自神经细胞,并只作用于神经系统的激素
C.神经细胞分泌的激素
4 4 2014 34. 下列哪一项不属于下丘脑调节肽(2.5分) A.促甲状腺激素释放激素
B.生长抑素
C.促性腺激素释放激素
D.催产素
E.促肾上腺皮质激素释放激素
D.催产素
4 4 2014 35. 下列为腺垂体所分泌的激素是(2.5分) A.促肾上腺皮质激素释放激素
B.促肾上腺皮质激素
C.肾上腺皮质激素
D.催产素
E.抗利尿激素
B.促肾上腺皮质激素
4 4 2014 36. Glucocorticoids comes mainly from(2.5分) A.hypothalamus
B.adrenal medulla
C.pituitary (gland)
D.adrenal cortex
E.ovary
D.adrenal cortex
4 4 2014 37. 幼儿时生长素分泌不足可导致(2.5分) A.呆小症
B.巨人症
C.侏儒症
D.肢端肥大症
C.侏儒症
4 4 2014 38. 下列哪组激素由神经垂体分泌(2.5分) A.生长素和催产素
B.胰岛素和催乳素
C.催产素和抗利尿激素
D.加压素和催乳素
C.催产素和抗利尿激素
4 4 2014 39. 影响神经系统发育的最重要的hormone是(2.5分) A.生长素
B.甲状腺激素
C.糖皮质激素
D.肾上腺素
B.甲状腺激素
4 4 2014 40. Graves病浸润性突眼主要与(2.5分) A.TSAb有关
B.TSH有关
C.TPO有关
D.细胞免疫有关
D.细胞免疫有关
4 4 2015 1. 与甲状腺功能减退(hypothyroidism)发生无关的因素是(2.5分) A.手术切除或放射线治疗损伤过多正常甲状腺组织
B.TGI抗体
C.自身免疫性损伤
D.慢性淋巴细胞性甲状腺炎
B.TGI抗体
4 4 2015 2. 下列哪些属于先垂体功能减退症(2.5分) A.TSH下降,T3、T4下降
B.TSH增高,T3、T4增高
C.ACTH下降,皮质醇增高
D.FSH、LH增高,雌激素下降
C.ACTH下降,皮质醇增高
4 4 2015 3. 2型糖尿病的发病机制是(2.5分) A.胰岛素抵抗和胰岛素分泌缺陷
B.胰岛B细胞自身免疫反应性损伤
C.胰岛B细胞破坏,胰岛素绝对不足
D.胰岛B细胞遗传性缺陷
A.胰岛素抵抗和胰岛素分泌缺陷
4 4 2015 4. 腺垂体嗜酸性细胞能分泌(2.5分) A.催乳素
B.促甲状腺激素
C.促性腺激素
D.黑素细胞刺激素
E.黄体生长素
A.催乳素
4 4 2015 5. 盐皮质激素主要作用于肾脏的(2.5分) A.近端小管曲部
B.近端小管直部
C.远端小管曲部
D.远端小管直部
E.细段
C.远端小管曲部
4 4 2015 6. 甲状腺滤泡腔中贮存的物质是(2.5分) A.四碘甲状腺原氨酸
B.三碘甲状腺原氨酸
C.甲状腺球蛋白
D.碘化甲状腺球蛋白
E.酪氨酸
D.碘化甲状腺球蛋白
4 4 2015 7. 成年人甲状腺激素缺乏会引起甲状腺肿大,主要原因是(2.5分) A.TSH的大量分泌
B.Thyroglobulin的大量分泌
C.甲状腺对I- 的大量摄取
D.自身抗体抗甲状腺的炎症效应
E.碘和酪氨酸缺乏而产生的促甲状腺效应
A.TSH的大量分泌
4 4 2015 8. 食物中长期缺碘可引起(2.5分) A.甲状腺功能亢进
B.甲状腺组织萎缩
C.单纯性甲状腺肿
D.腺垂体功能减退
E.神经垂体功能减退
C.单纯性甲状腺肿
4 4 2015 9. 影响神经系统发育的最重要的激素是(2.5分) A.胰岛素
B.甲状腺激素
C.生长素
D.糖皮质激素
E.肾上腺素
B.甲状腺激素
4 4 2015 10. 下列哪种激素对儿茶酚胺心血管效应起允许作用(permissive effects):(2.5分) A.抗利尿激素
B.甲状腺激素
C.生长素
D.糖皮质激素
E.醛固酮
B.甲状腺激素
4 4 2015 11. 糖皮质激素的作用是(2.5分) A.抑制蛋白质合成
B.使血糖浓度降低
C.使肾脏排水能力降低
D.使红细胞和血小板数量增加
E.使肾脏排钾能力降低
D.使红细胞和血小板数量增加
4 4 2015 12. 生理剂量时能促进蛋白质分解的激素是(2.5分) A.糖皮质激素
B.生长素
C.胰岛素
D.甲状腺激素
E.雄激素
A.糖皮质激素
4 4 2015 13. 临床上长期大剂量使用糖皮质激素时可以引起(2.5分) A.血中ACTH浓度增高
B.淋巴细胞数目增加
C.肢端肥大症
D.肾上腺皮质渐趋萎缩
E.CHR分泌增多
D.肾上腺皮质渐趋萎缩
4 4 2015 14. 肾上腺髓质激素的分泌主要受下列哪项的调节?(2.5分) A.交感神经节前纤维
B.副交感神经节后纤维
C.血糖浓度
D.ACTH
E.肾上腺髓质激素的反馈调节
A.交感神经节前纤维
4 4 2015 15. 下列那种激素对蛋白质合成与储存是不可缺少的?(2.5分) A.胰高血糖素
B.胰岛素
C.胰多肽
D.生长抑素
E.降钙素
B.胰岛素
4 4 2015 16. 胰岛素缺乏时可导致(2.5分) A.细胞内葡萄糖增加
B.肝糖原合成减少
C.血糖浓度下降
D.无尿糖
E.促进蛋白质合成
B.肝糖原合成减少
4 4 2015 17. 胰高血糖素的主要生理作用是(2.5分) A.促进肝糖原分解和糖异生
B.促进葡萄糖利用
C.促进脂肪合成
D.促进胃液和胆汁分泌
E.加快心率
A.促进肝糖原分解和糖异生
4 4 2015 18. 以下关于胰高血糖素作用的说法哪个是错误的(2.5分) A.主要工作地点在肾脏
B.促进肝糖原分解和糖异生
C.促进蛋白质分解和产尿增加
D.生酮作用增加
E.在胰岛素存在的条件下,对骨骼肌的效应很小
A.主要工作地点在肾脏
4 4 2015 19. 最常发生嗜铬细胞瘤的器官是(2.5分) A.甲状腺
B.胰岛
C.脑垂体
D.肾上腺髓质
E.肾上腺皮质
D.肾上腺髓质
4 4 2015 20. 关于胰岛素依赖型糖尿病,以下哪项是错的?(2.5分) A.多见于青少年
B.胰岛B细胞破坏
C.血中可查见抗胰岛细胞的自身抗体
D.血中胰岛素升高
E.易合并酮症酸中毒
D.血中胰岛素升高
4 4 2015 21. 下列疾病的病因组合中,哪项不正确?(2.5分) A.急性甲状腺炎一化脓菌
B.慢性淋巴细胞性甲状腺炎-自身免疫反应
C.非毒性甲状腺肿一缺碘
D.甲状腺腺癌一自身免疫反应
E.糖尿病一神经内分泌失调
D.甲状腺腺癌一自身免疫反应
4 4 2015 22. 毒性甲状腺肿的发病机理目前认为最大可能是(2.5分) A.细菌引起的化脓性炎
B.食物中含致肿素
C.自身免疫反应
D.甲状腺异常增生
E.土壤及水中缺碘
C.自身免疫反应
4 4 2015 23. 甲状腺功能亢进症的发病机理:(2.5分) A.缺碘引起的滤泡上皮弥漫性增生
B.缺碘引起的滤泡上皮结节状增生
C.原因不明的滤泡上皮结节状增生
D.自身免疫异常引起的滤泡上皮弥漫性增生
E.促甲状腺素分泌过多
D.自身免疫异常引起的滤泡上皮弥漫性增生
4 4 2015 24. 关于地方性甲状腺肿,哪一项是错误的?(2.5分) A.病区多为山区
B.女性显著多于男性
C.可压迫气管
D.当地居民均患有甲状腺肿
E.含碘食盐可预防本病发生
D.当地居民均患有甲状腺肿
4 4 2015 25. 诊断桥本病最特征的病理改变是甲状腺:(2.5分) A.中等度对称性肿大
B.切面似肉样
C.光镜下见大量淋巴细胞浸润伴淋巴滤泡形成
D.滤泡被扭曲、变形
E.伴有嗜酸细胞形成
C.光镜下见大量淋巴细胞浸润伴淋巴滤泡形成
4 4 2015 26. 颈部淋巴结转移性肿瘤灶中可见淀粉洋物,首先考虑的原发灶是:(2.5分) A.鼻咽癌
B.肺小细胞癌
C.胃癌
D.甲状腺癌
E.腮腺瘤
D.甲状腺癌
4 4 2015 27. 关于内分泌系统的最佳描述是(2.5分) A.区别于外分泌腺的系统
B.无导管,分泌物直接进入血液
C.分泌物通过体液传递的系统
D.由内分泌腺及全身内分泌细胞组成的信息传递系统
E.内分泌腺分泌激素通过细胞间传递的系统
D.由内分泌腺及全身内分泌细胞组成的信息传递系统
4 4 2015 28. 有关功能减退的内分泌疾病的治疗,下列哪项是错误的(2.5分) A.激素替代治疗
B.相关药物治疗
C.放射性同位素治疗
D.器官组织移植
E.细胞移植
C.放射性同位素治疗
4 4 2015 29. 下丘脑激素通过哪一途径调节腺垂体各激素的分泌(2.5分) A.下丘脑垂体神经束
B.神经递质
C.激素第二信使
D.下丘脑激素代谢产物
E.垂体门脉系统
E.垂体门脉系统
4 4 2015 30. 关于糖皮质激素的抗炎作用机制,下列哪种说法是错误的:(2.5分) A.抑制磷脂酶A2
B.稳定溶酶体酶
C.直接杀灭致病菌
D.抑制多种炎性细胞因子
E.抑制诱导型NO合成酶
C.直接杀灭致病菌
4 4 2015 31. 糖皮质激素的抗休克作用,以下哪点是错误的:(2.5分) A.扩张痉挛的血管
B.减少心肌抑制因子形成
C.降解细菌毒素
D.提高机体对毒素的耐受力
E.稳定溶酶体酶
C.降解细菌毒素
4 4 2015 32. 小剂量糖皮质激素替代疗法用于治疗(2.5分) A.败血症
B.肾病综合症
C.肾上腺嗜铬细胞瘤
D.严重中毒性感染
E.垂体前叶功能减退
E.垂体前叶功能减退
4 4 2015 33. 糖皮质激素的抗毒作用是指(2.5分) A.破坏内毒素
B.中和内毒素
C.直接杀灭内毒素
D.提高机体对内毒素的耐受力
E.杀灭病毒
D.提高机体对内毒素的耐受力
4 4 2015 34. 大剂量碘能抑制(2.5分) A.过氧化酶
B.甲状腺球蛋白水解酶
C.磷酸二酯酶
D.碱性磷酸酶
E.磷脂酶
A.过氧化酶
4 4 2015 35. 硫脲类药物治疗甲亢的机理是(2.5分) A.减少甲状腺激素的释放
B.对抗甲状腺素的作用
C.抑制甲状腺组织摄碘
D.主要是抑制甲状腺过氧化酶
E.抑制甲状腺球蛋白水解酶
D.主要是抑制甲状腺过氧化酶
4 4 2015 36. 能抑制T4转化为T3的抗甲状腺药物是(2.5分) A.碘化钾
B.丙硫氧嘧啶
C.普奈洛尔
D.复方碘溶液
E.他巴唑
B.丙硫氧嘧啶
4 4 2015 37. 神经激素是指(2.5分) A.作用于神经细胞的激素
B.具有激素功能的神经递质
C.神经细胞分泌的激素
D.神经系统内存在的激素
E.来自神经细胞,并只作用于神经系统的激素
C.神经细胞分泌的激素
4 4 2015 38. 下列哪一项不属于下丘脑调节肽:(2.5分) A.促甲状腺激素释放激素
B.生长抑素
C.促性腺激素释放激素
D.催产素
E.促肾上腺皮质激素释放激素
D.催产素
4 4 2015 39. 下列为腺垂体所分泌的激素是(2.5分) A.促肾上腺皮质激素释放激素
B.促肾上腺皮质激素
C.肾上腺皮质激素
D.催产素
E.抗利尿激素
B.促肾上腺皮质激素
4 4 2015 40. 关于ACTH的分泌下列哪项是错误的(2.5分) A.应激情况下分泌增加
B.受下丘脑分泌的CRH调节
C.受皮质醇的负反馈调节
D.可以在非垂体部位分泌
E.清晨渐低午夜增高的昼夜节律
E.清晨渐低午夜增高的昼夜节律
4 5 2006 1. 原发性甲状旁腺亢进症最有价值的诊断依据是:(2.0分) A.血钙、血磷
B.甲状旁腺素+血磷
C.甲状旁腺素+血钙
D.肾小管磷再吸收试验
E.磷清除试验
C.甲状旁腺素+血钙
4 5 2006 2. 体内调节血钙水平最重要的激素是:(2.0分) A.醛固酮
B.降钙素
C.甲状旁腺激素
D.维生素D
E.以上都不是
C.甲状旁腺激素
4 5 2006 3. 女性, 40岁, 多饮多尿, Bp160/100mmHg, 血pH7.5,血钾2.8mmol/L(2.8mEq/L),下列疾病诊断哪种可能性大(2.0分) A.尿崩症
B.原发性醛固酮增多症
C.糖尿病
D.嗜咯细胞瘤
E.甲亢
B.原发性醛固酮增多症
4 5 2006 4. 下列哪项不是急性高钙血症的治疗方法(2.0分) A.维生素D制剂
B.糖皮质激素
C.血液透析
D.容量扩张
E.降钙素
A.维生素D制剂
4 5 2006 5. 总钙浓度小于多少,无论有无症状均应进行治疗(2.0分) A.2.175mmol/L
B.1.875mmol/L
C.2.575mmol/L
D.1.975mmol/L
E.2.075mmol/L
B.1.875mmol/L
4 5 2006 6. 肢端肥大症系腺垂体分泌哪种内分泌激素细胞腺瘤或增生引起(2.0分) A.GH
B.LH
C.FSH
D.ACTH
E.TSH
A.GH
4 5 2006 7. 甲状旁腺素对血液钙磷浓度的调节作用表现为(2.0分) A.降低血钙浓度、升高血磷浓度
B.降低血钙浓度、不影响血磷浓度
C.升高血钙浓度、不影响血磷浓度
D.升高血钙浓度、降低血磷浓度
E.升高血钙、血磷浓度
D.升高血钙浓度、降低血磷浓度
4 5 2006 8. 肢端肥大症中最多见于哪一段年龄 (2.0分) A.51~60岁
B.21~30岁
C.41~50岁
D.31~40岁
E.61~70岁
D.31~40岁
4 5 2006 9. 甲状旁腺激素对肾脏的主要作用是(2.0分) A.促进钙离子排除
B.促钠、钾排出
C.抑制磷的回吸收
D.促进磷的回吸收
E.尿酸化
C.抑制磷的回吸收
4 5 2006 10. ARR比值对进一步原醛筛查有意义?(2.0分) A.20倍
B.30倍
C.10倍
D.5倍
E.40倍
A.20倍
4 5 2006 11. 血钙低于多少为低钙血症(2.0分) A.2.2mmol/L
B.2.3mmol/L
C.2.5mmol/L
D.2.6mmol/L
E.2.0mmol/L
A.2.2mmol/L
4 5 2006 12. 血钙高于多少为高钙血症(2.0分) A.2.5mmol/L
B.2.8mmol/L
C.2.3mmol/L
D.2.6mmol/L
E.3.0mmol/L
D.2.6mmol/L
4 5 2006 13. AVP主要受哪些因素的调控?(2.0分) A.渗透压感受器、容量感受器、压力感受器
B.容量感受器、压力感受器与神经递质
C.渗透压感受器、神经递质、药物
D.渗透压感受器、肾小管的反应、喝水的量
E.血容量、渗透压感受器、和体液平衡
A.渗透压感受器、容量感受器、压力感受器
4 5 2006 14. 嗜铬细瘤恶性的可能占百分比?(2.0分) A.5%
B.20%
C.15%
D.10%
E.12%
D.10%
4 5 2006 15. 下列哪项不是高钙血症的心血管系统的临床表现(2.0分) A.易出现洋地黄中毒
B.可合并高血压
C.易出现心律失常
D.心肌兴奋性增加
E.QT期间和ST段明显延长
E.QT期间和ST段明显延长
4 5 2006 16. 正常情况下,蛋白质结合钙占血钙的(2.0分) A.50%
B.40%
C.30%
D.60%
B.40%
4 5 2006 17. 有关三发性甲旁亢的叙述正确的是(2.0分) A.临床上最常见的甲旁亢是三发性甲旁亢
B.有三个甲状旁腺腺体原发疾病引起的甲旁亢,叫三发性甲旁亢
C.恶性肿瘤转移至三个甲状旁腺引起的甲旁亢叫三发性甲旁亢
D.除了甲状旁腺外,还有另外两个内分泌腺出现病变,临床上以甲旁亢症状为主,叫三发性甲旁亢
E.在继发甲旁亢的基础上,由于腺体受到持久的刺激,部分增生组织转变成腺瘤,自主性分泌过多的PTH,叫三发性甲旁亢
E.在继发甲旁亢的基础上,由于腺体受到持久的刺激,部分增生组织转变成腺瘤,自主性分泌过多的PTH,叫三发性甲旁亢
4 5 2006 18. 肢端肥大症内分泌检查,生长激素一般大于(2.0分) A.20ug/L
B.10ug/L
C.15ug/L
D.5ug/L
E.30ug/L
A.20ug/L
4 5 2006 19. 下列哪项疾病需与肢端肥大症鉴别(2.0分) A.甲状腺功能亢进
B.糖尿病
C.手足皮肤骨膜肥厚症
D.皮质醇增多症
E.泌乳素瘤
C.手足皮肤骨膜肥厚症
4 5 2006 20. 下列哪种方法是血液总钙测定的参考方法(2.0分) A.原子吸收分光光度法
B.同位素稀释质谱法
C.火焰分光光度法
D.邻甲酚酞络合酮法
E.分光光度法
A.原子吸收分光光度法
4 5 2006 21. 诊断尿崩症的初筛标准?(2.0分) A.尿量大于4000ml/day,尿比重小于1.005
B.尿量大于2500ml/day ,尿比重小于1.005
C.尿量大于4000ml/day ,尿比重小于1.010
D.尿量大于5000ml/day,尿比重小于1.008
E.尿量大于3000ml/day,尿比重大于1.005
A.尿量大于4000ml/day,尿比重小于1.005
4 5 2006 22. 下列哪项属于甲状旁腺激素的作用?(2.0分) A.使血磷升高
B.抑制远曲小管和集合管对钙的重吸收
C.促进1,25-二羟维生素D3形成
D.抑制骨钙入血
E.促进磷酸盐的重吸收
C.促进1,25-二羟维生素D3形成
4 5 2006 23. 下列哪个不是肾上腺腺瘤所致皮质醇增多症的临床表现: (2.0分) A.皮肤黏膜色素沉着
B.向心性肥胖
C.紫纹
D.高血压
E.骨质疏松
A.皮肤黏膜色素沉着
4 5 2006 24. 引起手足搐搦的原因是以下何种改变(2.0分) A.结合钙浓度升高
B.结合钙浓度降低
C.离子钙浓度升高
D.离子钙浓度降低
E.离子钙浓度升高,结合钙浓度降低
D.离子钙浓度降低
4 5 2006 25. Which of the following is not involved in regulating plasma Ca2+ levels? (2.0分) A.Kidneys
B.Skin
C.Lungs
D.Liver
E.Intestine
C.Lungs
4 5 2006 26. 正常人血浆钙、磷乘积为(2.0分) A.31-40
B.35-40
C.41-50
D.20-30
E.51-60
B.35-40
4 5 2006 27. 女性,29岁,发作性心悸,头痛1年,洗碗时或手受凉时常有症状,测血压26.6/17.3kPa,本次就诊时血压正常,空腹血糖6.8mmol/L,电解质正常,尿蛋白(+)下述哪个检查改变通常不会出现(2.0分) A.胰高血糖素试验血儿茶酚胺升高3倍以上
B.24小时尿VMA定量增高近1倍
C.可乐定试验后3小时,血儿茶酚胺降低2倍
D.8小时尿VMA测定值在正常高限2倍以上
E.冷加压试验,血压升高达20.15mmHg以上
C.可乐定试验后3小时,血儿茶酚胺降低2倍
4 5 2006 28. 食物中Ca﹕P为多少时有利于这两种元素的吸收(2.0分) A.2﹕1
B.1﹕2
C.1﹕1
D.1﹕3
A.2﹕1
4 5 2006 29. 影响PTH分泌最重要的因素是:(2.0分) A.血钙浓度
B.血磷浓度
C.血镁浓度
D.血中儿茶酚胺浓度
E.血中前列腺素浓度
A.血钙浓度
4 5 2006 30. 下列关于血钙的叙述哪一项是正确的?(2.0分) A.可扩散钙即为游离钙
B.非扩散钙包括柠檬酸钙
C.非扩散钙即为结合钙
D.结合钙均不能透过毛细血管
E.以上都是错误的
E.以上都是错误的
4 5 2006 31. 尿崩症的诊断要点?(2.0分) A.烦渴、夜尿量增加、禁水试验尿量正常
B.口干、多饮、多尿、禁水试验 无法耐受
C.尿频、口渴、多饮、莫氏试验夜尿量大于日尿量
D.禁水试验尿渗压、尿比重不能增加
E.尿频、尿量增加、尿比重1.010、血钠偏低
D.禁水试验尿渗压、尿比重不能增加
4 5 2006 32. 什么往往是骨质疏松症的首发症状或就医原因。 (2.0分) A.身高变矮,驼背畸形
B.疼痛
C.骨密度低下
D.骨折
D.骨折
4 5 2006 33. 关于原醛(2.0分) A.原醛均表现为RAS系统ARR比值增高
B.原醛均表现为高血压、低钾
C.难治性高血压均是原醛
D.原醛的病因是由于醛固酮腺瘤所致
E.原醛的尿钾排泄不增高
A.原醛均表现为RAS系统ARR比值增高
4 5 2006 34. 关于原醛的肾性失钾的证据以下哪个是正确的?(2.0分) A.夏季出汗增加,尿钾排泄增加
B.钾的摄 入增加,尿钾增加
C.血钾正常,尿钾排泄增加
D.血钾高,尿钾排泄增加
E.血钾低,尿钾排泄增加
E.血钾低,尿钾排泄增加
4 5 2006 35. 下列哪项指标是反映破骨的生化指标(2.0分) A.骨钙素
B.ALP
C.CICP
D.CTX
D.CTX
4 5 2006 36. 发生脆性骨折的常见部位为(2.0分) A.桡尺骨远端
B.肱骨近端
C.髋部
D.胸腰椎
E.以上都是
E.以上都是
4 5 2006 37. 甲状旁腺激素对骨的主要作用是(2.0分) A.抑制骨吸收
B.促进钙离子释出
C.抑制磷离子释出
D.抑制钙离子释出
E.释出镁离子
B.促进钙离子释出
4 5 2006 38. 嗜铬细胞瘤患者为什么会发生低血压?(2.0分) A.用药过度
B.儿茶酚胺受体不敏感
C.肿瘤突然停止释放血管活性物质
D.分泌舒血管肠肽、肾上腺髓质素致血压下降
E.以上均不是
D.分泌舒血管肠肽、肾上腺髓质素致血压下降
4 5 2006 39. 女性,29岁,发作性心悸,头痛1年,洗碗时或手受凉时常有症状,测血压26.6/17.3kPa,本次就诊时血压正常,空腹血糖6.8mmol/L,电解质正常,尿蛋白(+)本例最应考虑的诊断是(2.0分) A.慢性肾炎高血压型
B.原发性醛固酮增多症
C.周期性库欣综合征
D.嗜铬细胞瘤
E.不稳定型原发性高血压。
D.嗜铬细胞瘤
4 5 2006 40. PTH and PTHrP activate the same receptor, but their effects are different. How is this possible? (2.0分) A.Plasma PTH is high when plasma PTHrP is low and vice versa
B.For unknown reasons, PTHrP acts mainly on fetal tissues, whereas PTH acts mainly on adult tissues
C.PTH acts on the brain, whereas PTHrP does not
D.When PTH binds to the common receptor, it activates adenylyl cyclase, whereas PTHrP activates phospholipase C
E.There is an additional receptor in the body that responds to PTH but not to PTHrP
E.There is an additional receptor in the body that responds to PTH but not to PTHrP
4 5 2006 41. 糖皮质激素是由下列哪个部位分泌的: (2.0分) A.肾上腺皮质球状带
B.肾上腺皮质束状带
C.肾上腺皮质网状带
D.肾脏
E.肾上腺髓质
B.肾上腺皮质束状带
4 5 2006 42. I 型(绝经后)骨质疏松是什么型骨质疏松 (2.0分) A.平衡型
B.低转换型
C.高转换型
D.快速型
C.高转换型
4 5 2006 43. 内分泌性高血压为继发性高血压的常见病因之一,下列疾病能引起高血压的是: (2.0分) A.嗜铬细胞瘤
B.原发性醛固酮增多症
C.甲状腺功能亢进症
D.库欣氏综合症
E.以上均可
E.以上均可
4 5 2006 44. 关于垂体性肾上腺皮质增生,下列哪项描述是正确的: (2.0分) A.小剂量和大剂量DXM抑制试验均不能被抑制
B.小剂量DXM抑制试验能被抑制,大剂量DXM抑制试验不能被抑制
C.小剂量和大剂量DXM抑制试验均能被抑制
D.小剂量DXM抑制试验不能被抑制,大剂量DXM抑制试验能被抑制
E.以上均不对
D.小剂量DXM抑制试验不能被抑制,大剂量DXM抑制试验能被抑制
4 5 2006 45. 皮质醇增多症系由多种原因引起肾上腺皮质分泌过多糖皮质激素所致。病因以下列哪种为最常见:(2.0分) A.肾上腺腺瘤
B.垂体瘤致肾上腺双侧增生
C.肾上腺腺癌
D.异位ACTH综合症
E.先天性肾上腺增生
B.垂体瘤致肾上腺双侧增生
4 5 2006 46. 肾上腺皮质癌伴转移者首选(2.0分) A.氨基导眠能
B.安体舒通
C.双氯苯二氯乙烷
D.甲吡酮
E.酮康
C.双氯苯二氯乙烷
4 5 2006 47. 肾上腺X线平片发现肾上腺区钙化影提示(2.0分) A.肾上腺结核
B.柯兴综合症
C.嗜铬细胞瘤
D.原发性醛固增多症
E.垂体ACTH腺瘤
A.肾上腺结核
4 5 2006 1. 骨质疏松症与骨组织内哪几种细胞有关(2.0分) A.骨髓前体细胞
B.软骨细胞
C.破骨细胞
D.骨村细胞
E.成骨细胞
C.破骨细胞
E.成骨细胞
4 5 2006 2. 骨质疏松症的治疗药物大致可以分为 (2.0分) A.抑制骨重吸
B.刺激骨形成
C.基础补充
D.增加矿物质
A.抑制骨重吸
B.刺激骨形成
C.基础补充
4 5 2006 3. 临床上骨密度常规检测部位是(2.0分) A.桡骨远端
B.跟骨
C.股骨颈
D.腰椎
C.股骨颈
D.腰椎
4 5 2007 1. 垂体功能减退症最早出现的靶腺功能减退是(2.0分) A.性腺功能减退
B.肾上腺皮质功能减退
C.甲状腺功能减退
D.肾上腺与甲状腺功能减退
E.甲状腺与性腺功能减退
A.性腺功能减退
4 5 2007 2. 下列哪项对诊断腺垂体功能减退症无意义(2.0分) A.性腺激素测定
B.甲状腺素测定
C.甲状旁腺素测定
D.皮质醇测定
E.泌乳素测定
C.甲状旁腺素测定
4 5 2007 3. 垂体危象时,下列何种情况最为多见(2.0分) A.谵妄
B.低钾性麻痹
C.低血糖性昏迷
D.脑梗死
E.高钠高渗性昏迷
C.低血糖性昏迷
4 5 2007 4. 下列哪项为最常见的垂体瘤?(2.0分) A.泌乳素瘤
B.生长激素瘤
C.促肾上腺皮质激素瘤
D.促性腺激素瘤
E.促甲状腺激素瘤
A.泌乳素瘤
4 5 2007 5. 男性,40岁,头痛,视力下降3个月,查血压140/80mmHg, 左眼颞侧偏盲,血GH 35 g/L (正常<10 g/L)。进一步应行下列哪项检查?(2.0分) A.垂体MRI
B.头颅血管造影
C.头颅X线检查
D.肌电图
E.脑电图
A.垂体MRI
4 5 2007 6. 关于催乳素瘤下列哪项不正确?(2.0分) A.多为微腺瘤
B.多见于女性
C.为最常见的垂体瘤
D.溴隐亭治疗效果好
E.停药易复发,故无论何种情况均不宜停药
E.停药易复发,故无论何种情况均不宜停药
4 5 2007 7. 关于PRL下列哪项不正确?(2.0分) A.多巴胺可促进其分泌
B.雌激素的作用是妊娠期PRL增高的主要原因
C.〉200ug/L提示催乳素瘤
D.甲状腺功能减退症患者可出现PRL 升高
E.对FSH/LH 的分泌和作用均有抑制作用
A.多巴胺可促进其分泌
4 5 2007 8. 钙离子在细胞中的分布特征 (2.0分) A.细胞外液游离钙浓度低
B.细胞内液的钙离子主要储存于细胞内钙库中
C.细胞内液的钙离子含量很高
D.胞液Ca2+可以顺浓度梯度返回细胞外或胞内钙库
B.细胞内液的钙离子主要储存于细胞内钙库中
4 5 2007 9. 影响磷吸收因素正确的是(2.0分) A.肠内酸度增加时磷酸盐的吸收减少
B.碱性磷酸盐、草酸盐和植酸盐有利于磷的吸收
C.体内钙、镁、铁等离子过多时易与磷酸结合成不溶性盐,不易吸收。
D.不能在体内保存,易缺乏
C.体内钙、镁、铁等离子过多时易与磷酸结合成不溶性盐,不易吸收。
4 5 2007 10. 有关甲状旁腺素叙述错误的是(2.0分) A.利于骨基质的分解和吸收
B.糖酵解加强
C.血钙浓度与甲状旁腺素分泌呈负相关。
D.甲状旁腺激素的总体作用是升高血钙和血磷,
B.糖酵解加强
4 5 2007 11. 以下叙述错误的是 (2.0分) A.1,25-(OH)2-D3能促进肾小管对钙、磷的重吸收
B.1,25-(OH)2-D3促进小肠对钙磷的吸收.
C.维生素D 的活性形式是1,25-(OH)2-D3
D.肝肾佝偻病能用普通的维生素D治疗
D.肝肾佝偻病能用普通的维生素D治疗
4 5 2007 12. 有关钙的功能叙述错误的是 (2.0分) A.血钙的正常水平可维持骨骼内骨盐的含量
B.参与血液凝固过程
C.当线粒体钙离子浓度升高时,抑制三羧酸循环和氧化磷酸化
D.钙调蛋白可看作是细胞内Ca2+的受体
C.当线粒体钙离子浓度升高时,抑制三羧酸循环和氧化磷酸化
4 5 2007 13. 影响PTH分泌最重要的因素是:(2.0分) A.血钙浓度
B.血磷浓度
C.血镁浓度
D.血中儿茶酚胺浓度
E.血中前列腺素浓度
A.血钙浓度
4 5 2007 14. 降钙素的主要靶器官是:(2.0分) A.胃肠道
B.甲状旁腺
C.甲状腺
D.肾脏
E.骨组织
E.骨组织
4 5 2007 15. 血浆中降钙素的主要来源是:(2.0分) A.甲状腺C细胞
B.甲状旁腺细胞
C.胰岛D细胞
D.肾上腺皮质网状带
E.肾上腺皮质球状带
A.甲状腺C细胞
4 5 2007 16. 下列哪项属于甲状旁腺激素的作用?(2.0分) A.抑制骨钙入血
B.促进1,25-二羟维生素D3形成
C.抑制远曲小管和集合管对钙的重吸收
D.使血磷升高
E.促进磷酸盐的重吸收
B.促进1,25-二羟维生素D3形成
4 5 2007 17. Which of the following is not involved in regulating plasma Ca2+ levels? (2.0分) A.Liver
B.Lungs
C.Skin
D.Kidneys
E.Intestine
B.Lungs
4 5 2007 18. PTH and PTHrP activate the same receptor, but their effects are different. How is this possible? (2.0分) A.For unknown reasons, PTHrP acts mainly on fetal tissues, whereas PTH acts mainly on adult tissues
B.When PTH binds to the common receptor, it activates adenylyl cyclase, whereas PTHrP activates phospholipase C
C.PTH acts on the brain, whereas PTHrP does not
D.Plasma PTH is high when plasma PTHrP is low and vice versa
E.There is an additional receptor in the body that responds to PTH but not to PTHrP
E.There is an additional receptor in the body that responds to PTH but not to PTHrP
4 5 2007 19. 有关三发性甲旁亢的叙述正确的是:(2.0分) A.除了甲状旁腺外,还有另外两个内分泌腺出现病变,临床上以甲旁亢症状为主,叫三发性甲旁亢
B.有三个甲状旁腺腺体原发疾病引起的甲旁亢,叫三发性甲旁亢
C.临床上最常见的甲旁亢是三发性甲旁亢
D.恶性肿瘤转移至三个甲状旁腺引起的甲旁亢叫三发性甲旁亢
E.在继发甲旁亢的基础上,由于腺体受到持久的刺激,部分增生组织转变成腺瘤,自主性分泌过多的PTH,叫三发性甲旁亢
E.在继发甲旁亢的基础上,由于腺体受到持久的刺激,部分增生组织转变成腺瘤,自主性分泌过多的PTH,叫三发性甲旁亢
4 5 2007 20. 抢救高钙危象时需应用:(2.0分) A.尼群地平
B.糖皮质激素
C.大量生理盐水
D.酚妥拉明
E.大量10%葡萄糖
C.大量生理盐水
4 5 2007 21. 甲状旁腺功能减退症患者,静脉滴注200UPTH后,尿磷和尿cAMP不增加的是(2.0分) A.特发性甲状旁腺功能减退症
B.假性甲状旁腺功能减退症II型
C.假性甲状旁腺功能减退症I型
D.继发性甲状旁腺功能减退症
E.低血镁性甲状旁腺功能减退症
C.假性甲状旁腺功能减退症I型
4 5 2007 22. 甲状旁腺素对血液钙磷浓度的调节作用表现为(2.0分) A.升高血钙浓度、不影响血磷浓度
B.升高血钙浓度、降低血磷浓度
C.降低血钙浓度、升高血磷浓度
D.降低血钙浓度、不影响血磷浓度
E.升高血钙、血磷浓度
B.升高血钙浓度、降低血磷浓度
4 5 2007 23. 下列哪项不引起高钙血症:(2.0分) A.肾功能衰竭
B.长期的制动
C.脂质代谢异常
D.原发性甲状旁腺功能亢进
E.肢端肥大症
C.脂质代谢异常
4 5 2007 24. 纠正低钙搐搦首选:(2.0分) A.口服钙剂和维生素D
B.应用利尿剂
C.静脉注射葡萄糖酸钙
D.迅速输注生理盐水
E.应用镇静剂
C.静脉注射葡萄糖酸钙
4 5 2007 25. 低磷血症是指:(2.0分) A.指血磷低于0.6mmol/L
B.指血磷低于0.4mmol/L
C.指血磷低于0.7mmol/L
D.指血磷低于1.0mmol/L
E.指血磷低于0.8mmol/L
E.指血磷低于0.8mmol/L
4 5 2007 26. 下列哪项不是引起高磷血症的病因:(2.0分) A.肉芽肿病
B.甲状旁腺功能减退
C.维生素D缺乏
D.肾功能衰竭
E.含磷酸盐泻剂或灌肠液
C.维生素D缺乏
4 5 2007 27. 肢端肥大症系腺垂体分泌哪种内分泌激素细胞腺瘤或增生引起(2.0分) A.LH
B.ACTH
C.FSH
D.GH
E.TSH
D.GH
4 5 2007 28. 肢端肥大症内分泌检查,葡萄糖抑制试验血浆GH不被抑制到多少以下(2.0分) A.5ug/L
B.15ug/L
C.12ug/L
D.10ug/L
E.20ug/L
A.5ug/L
4 5 2007 29. 经治疗后,GH一般控制指随机血GH小于(2.0分) A.6ug/L
B.5ug/L
C.8ug/L
D.10ug/L
E.12ug/L
B.5ug/L
4 5 2007 30. 血钙低于多少为低钙血症(2.0分) A.2.3mmol/L
B.2.6mmol/L
C.2.5mmol/L
D.2.4mmol/L
E.2.0mmol/L
E.2.0mmol/L
4 5 2007 31. 总钙浓度小于多少,无论有无症状均应进行治疗(2.0分) A.1.975mmol/L
B.2.175mmol/L
C.2.575mmol/L
D.1.875mmol/L
E.2.075mmol/L
D.1.875mmol/L
4 5 2007 32. 低钙血症时表现神经肌肉兴奋性(2.0分) A.不变
B.降低
C.升高
D.先降低后升高
E.先升高后降低
C.升高
4 5 2007 33. 下列哪项不是高钙血症的心血管系统的临床表现(2.0分) A.易出现心律失常
B.心肌兴奋性增加
C.易出现洋地黄中毒
D.可合并高血压
E.QT期间和ST段明显延长
E.QT期间和ST段明显延长
4 5 2007 34. 慢性肾功能衰竭低血钙发生的主要机制(2.0分) A.骨钙动员受阻
B.低蛋白血症
C.维生素D羟化障碍
D.高血磷
E.肠钙吸收减少
B.低蛋白血症
4 5 2007 35. 血管加压素(ArginineVasopressin,AVP)是由什么.部位?以下哪种细胞产生的?(2.0分) A.第三脑室旁细胞产生沿神经垂体束下行至垂体后叶
B.垂体前叶内分泌细胞产生作用于下丘脑室旁核
C.下丘脑视上核心与室旁核神经元产生,沿神经垂体束下行至垂体后叶
D.下丘脑弓状核的APUD细胞与AgRP细胞产生
E.侧脑室旁神经细胞产生
C.下丘脑视上核心与室旁核神经元产生,沿神经垂体束下行至垂体后叶
4 5 2007 36. 尿崩症的诊断要点?(2.0分) A.烦渴、夜尿量增加、禁水试验尿量正常
B.口干、多饮、多尿、禁水试验 无法耐受
C.尿频、口渴、多饮、莫氏试验夜尿量大于日尿量
D.禁水试验尿渗压、尿比重不能增加
E.尿频、尿量增加、尿比重1.010、血钠偏低
D.禁水试验尿渗压、尿比重不能增加
4 5 2007 37. 关于精神性烦渴(2.0分) A.口干、多饮、多尿、高比重尿、血钠、血糖升高、禁水后出现高渗
B.口干、饮凉水、低比重尿、禁水试验尿量不减少、尿渗透压不上升
C.口干、多饮、低比重尿、禁水试验尿量不减少、加压素无效
D.极度口渴、自诉尿次增加、禁水试验尿量减少、尿比重增加
E.口干、眼干、多饮、反复关节痛、球蛋白增高、
D.极度口渴、自诉尿次增加、禁水试验尿量减少、尿比重增加
4 5 2007 38. 关于原醛(2.0分) A.难治性高血压均是原醛
B.原醛的病因是由于醛固酮腺瘤所致
C.原醛均表现为RAS系统ARR比值增高
D.原醛均表现为高血压、低钾
E.原醛的尿钾排泄不增高
C.原醛均表现为RAS系统ARR比值增高
4 5 2007 39. ARR比值对进一步原醛筛查有意义?(2.0分) A.30倍
B.10倍
C.20倍
D.5倍
E.40倍
C.20倍
4 5 2007 40. 原发性醛固酮增多症的最常见原因是(2.0分) A.特醛症
B.遗传性酶缺陷
C.醛固酮癌
D.醛固酮腺瘤
E.骨质疏松症
D.醛固酮腺瘤
4 5 2007 41. 女性, 40岁, 多饮多尿, Bp160/100mmHg, 血pH7.5,血钾2.8mmol/L(2.8mEq/L),下列疾病诊断哪种可能性大(2.0分) A.糖尿病
B.嗜咯细胞瘤
C.尿崩症
D.原发性醛固酮增多症
E.甲亢
D.原发性醛固酮增多症
4 5 2007 42. 柯兴病是(2.0分) A.肾上腺皮质瘤产生大量糖皮质激素
B.垂体瘤分泌大量的ACTH, 引起肾上腺皮质增生
C.异位ACTH所致肾上腺皮质增生
D.长期服用大量的糖皮质激素, 引起向心性肥胖
E.不依赖ACTH的肾上腺瘤结节增生
B.垂体瘤分泌大量的ACTH, 引起肾上腺皮质增生
4 5 2007 43. 皮质醇增多症的病因, 以下哪种最常见(2.0分) A.双侧肾上腺皮质增生
B.肾上腺皮质腺癌
C.肾上腺皮质腺瘤
D.异位ACTH综合征
E.不依赖ACTH的肾上腺瘤结节性增生
A.双侧肾上腺皮质增生
4 5 2007 44. 女性, 20岁, 肥胖4年, 头晕半年, 多血质(+), 多毛, 紫纹(+), 血压20/13.3kPa(160/100mmHg), 为诊断皮质醇增多症除外单纯性肥胖症, 最重要的试验是(2.0分) A.血ACTH激素测定
B.24小时, 尿游离皮质醇
C.血皮质醇测定
D.小剂量地塞米松抑制试验
E.大剂量地塞米松抑制试验
D.小剂量地塞米松抑制试验
4 5 2007 45. 女性,29岁,发作性心悸,头痛1年,洗碗时或手受凉时常有症状,测血压26.6/17.3kPa,本次就诊时血压正常,空腹血糖6.8mmol/L,电解质正常,尿蛋白(+)下述哪个检查改变通常不会出现(2.0分) A.8小时尿VMA测定值在正常高限2倍以上
B.可乐定试验后3小时,血儿茶酚胺降低2倍
C.24小时尿VMA定量增高近1倍
D.胰高血糖素试验血儿茶酚胺升高3倍以上
E.冷加压试验,血压升高达20.15mmHg以上
B.可乐定试验后3小时,血儿茶酚胺降低2倍
4 5 2007 46. 醛固酮的主要作用是促进肾脏(2.0分) A.近曲小管保钠排钾
B.远曲小管保钾排钠
C.远曲小管保钠排钾
D.近曲小管保钾排钠
E.髓质保钠排钾
C.远曲小管保钠排钾
4 5 2007 47. 肾上腺X线平片发现肾上腺区钙化影提示(2.0分) A.柯兴综合症
B.嗜铬细胞瘤
C.肾上腺结核
D.原发性醛固增多症
E.垂体ACTH腺瘤
C.肾上腺结核
4 5 2007 48. 一位39岁女性患者,乏力、恶心、消瘦二年,伴面部、手掌指纹等处皮肤变黑一年。入院前一天腹泻三次而神志模糊入院。查体发现乳房萎缩、阴毛稀少。血压45/30mmHg.心率120次/分。血糖60mg/dl,血钠132mmol/dl.诊断上最大可能是(2.0分) A.严重结核病晚期
B.恶性肿瘤晚期
C.垂体功能减退症危象
D.肾上腺危象
E.中毒性肠炎、感染性休克
D.肾上腺危象
4 5 2007 49. 上题,治疗上首要措施是(2.0分) A.静滴大剂量雷米封,口服利福平
B.姑息支持疗法、通知家属准备后事
C.补充糖盐水、静注琥珀酰氢可的松
D.补充糖盐水、加强抗炎
E.补液、升压、抗炎
C.补充糖盐水、静注琥珀酰氢可的松
4 5 2007 50. 上题,若抢救成功、病情稳定后应给予(2.0分) A.正规抗痨治疗、至少两年
B.强的松、甲状腺片、雌激素替代治疗
C.继续口服抗生素一周后停药
D.手术切除肿瘤或化疗或放疗
E.皮质醇终身替代治疗
E.皮质醇终身替代治疗
4 5 2008 1. 腺垂体功能减退症最早出现的靶腺功能减退是(2.0分) A.肾上腺皮质功能减退
B.性腺功能减退
C.甲状腺功能减退
D.肾上腺与甲状腺功能减退
E.甲状腺与性腺功能减退
B.性腺功能减退
4 5 2008 2. 下列哪项对诊断腺垂体功能减退症无意义(2.0分) A.甲状旁腺素测定
B.甲状腺素测定
C.性腺激素测定
D.皮质醇测定
E.泌乳素测定
A.甲状旁腺素测定
4 5 2008 3. 下列哪项替代治疗是错误的?(2.0分) A.甲状腺功能减退使用左甲状腺素50 150 g/d
B.肾上腺皮质功能减退使用氢化可的松20 30mg/d,9α-氟氢可的松0.05~0.1 mg/d
C.女性闭经:炔雌醇5 20 g/d(月经周期第1 25天),甲羟孕酮(安宫黄体酮)5 10mg/d(月经周期第12 25天)
D. 男子性腺功能减退症丙酸睾酮50mg/周肌注, 或十一酸睾酮40mg,每日3次口服
E. 女性不育:促性素(HMG) 75 ~ 150 IU/d,持续二周,并肌注绒促性素(HCG) 2000IU
B.肾上腺皮质功能减退使用氢化可的松20 30mg/d,9α-氟氢可的松0.05~0.1 mg/d
4 5 2008 4. 男性,45岁,鼻咽癌放疗后出现性欲减退、阳痿、怕冷、思睡、思维迟钝、精神淡漠。该病人需考虑下列哪种情况:(2.0分) A.放疗引起甲状腺破坏,至甲状腺功能减退症
B. 鼻咽癌复发
C. 放疗破坏垂体组织,引起腺垂体功能减退症
D. 植物神经功能紊乱
E. 前列腺炎
C. 放疗破坏垂体组织,引起腺垂体功能减退症
4 5 2008 5. 女性,30岁,结婚4年未育,双乳溢乳,垂体发现微腺瘤,首先应行哪项检查?(2.0分) A.ACTH
B.TSH
C.LH
D.PRL
E.GH
D.PRL
4 5 2008 6. 男性,40岁,头痛,视力下降3个月,查血压140/80mmHg, 左眼颞侧偏盲,血GH 35 g/L (正常<10 g/L)。进一步应行下列哪项检查?(2.0分) A.垂体MRI
B.肌电图
C.头颅X线检查
D.头颅血管造影
E.脑电图
A.垂体MRI
4 5 2008 7. 关于垂体瘤分泌的激素下列哪项不正确?(2.0分) A.生长激素瘤分泌GH
B.催乳素瘤分泌PRL
C.促甲状腺激素瘤分泌TSH
D.促性腺激素瘤分泌FSH/LH
E.促肾上腺皮质激素瘤分泌ACTH、黑素细胞刺激素
A.生长激素瘤分泌GH
4 5 2008 8. 关于催乳素瘤下列哪项不正确?(2.0分) A.为最常见的垂体瘤
B.多见于女性
C.多为微腺瘤
D.溴隐亭治疗效果好
E.停药易复发,故无论何种情况均不宜停药
E.停药易复发,故无论何种情况均不宜停药
4 5 2008 9. 体内调节血钙水平最重要的激素是:(2.0分) A.甲状旁腺激素
B.降钙素
C.醛固酮
D.维生素D
E.以上都不是
A.甲状旁腺激素
4 5 2008 10. 血浆中降钙素的主要来源是:(2.0分) A.甲状旁腺细胞
B.胰岛D细胞
C.肾上腺皮质网状带
D.甲状腺C细胞
E.肾上腺皮质球状带
D.甲状腺C细胞
4 5 2008 11. 下列哪种维生素能显著影响钙的吸收和代谢?(2.0分) A.维生素A
B.维生素B
C.维生素C
D.维生素D
E.维生素E
D.维生素D
4 5 2008 12. Which of the following is not involved in regulating plasma Ca2+ levels?(2.0分) A.Kidneys
B.Skin
C.Liver
D.Lungs
E.Intestine
D.Lungs
4 5 2008 13. PTH and PTHrP activate the same receptor, but their effects are different. How is this possible? (2.0分) A.For unknown reasons, PTHrP acts mainly on fetal tissues, whereas PTH acts mainly on adult tissues
B.Plasma PTH is high when plasma PTHrP is low and vice versa
C.PTH acts on the brain, whereas PTHrP does not
D.When PTH binds to the common receptor, it activates adenylyl cyclase, whereas PTHrP activates phospholipase C
E.There is an additional receptor in the body that responds to PTH but not to PTHrP
E.There is an additional receptor in the body that responds to PTH but not to PTHrP
4 5 2008 14. 影响钙磷代谢的因素下列哪项不是的:(2.0分) A.甲状旁腺激素
B.维生素D
C.肾脏功能
D.心脏功能
E.肠道功能
D.心脏功能
4 5 2008 15. 低钙血症最具特征的临床表现是:(2.0分) A.手足搐搦
B.肌痉挛
C.骨痛、骨折
D.心律不齐、心脏传导阻滞
E.认知能力减退
A.手足搐搦
4 5 2008 16. 低磷血症是指:(2.0分) A.指血磷低于0.4mmol/L
B.指血磷低于0.6mmol/L
C.指血磷低于0.7mmol/L
D.指血磷低于1.0mmol/L
E.指血磷低于0.8mmol/L
E.指血磷低于0.8mmol/L
4 5 2008 17. 下列哪项不是引起高磷血症的病因:(2.0分) A.肾功能衰竭
B.甲状旁腺功能减退
C.维生素D缺乏
D.肉芽肿病
E.含磷酸盐泻剂或灌肠液
C.维生素D缺乏
4 5 2008 18. 有关三发性甲旁亢的叙述正确的是:(2.0分) A.有三个甲状旁腺腺体原发疾病引起的甲旁亢,叫三发性甲旁亢
B.临床上最常见的甲旁亢是三发性甲旁亢
C.恶性肿瘤转移至三个甲状旁腺引起的甲旁亢叫三发性甲旁亢
D.除了甲状旁腺外,还有另外两个内分泌腺出现病变,临床上以甲旁亢症状为主,叫三发性甲旁亢
E.在继发甲旁亢的基础上,由于腺体受到持久的刺激,部分增生组织转变成腺瘤,自主性分泌过多的PTH,叫三发性甲旁亢
E.在继发甲旁亢的基础上,由于腺体受到持久的刺激,部分增生组织转变成腺瘤,自主性分泌过多的PTH,叫三发性甲旁亢
4 5 2008 19. 抢救高钙危象时需应用(2.0分) A.糖皮质激素
B.酚妥拉明
C.尼群地平
D.大量生理盐水
E.大量10%葡萄糖
D.大量生理盐水
4 5 2008 20. 甲状旁腺功能减退症患者,静脉滴注200UPTH后,尿磷和尿cAMP都增加的是(2.0分) A.假性甲状旁腺功能减退症I型
B.假性甲状旁腺功能减退症II型
C.特发性甲状旁腺功能减退症
D.继发性甲状旁腺功能减退症
E.低血镁性甲状旁腺功能减退症
C.特发性甲状旁腺功能减退症
4 5 2008 21. 甲状旁腺素对血液钙磷浓度的调节作用表现为(2.0分) A.降低血钙浓度、升高血磷浓度
B.升高血钙浓度、降低血磷浓度
C.升高血钙浓度、不影响血磷浓度
D.降低血钙浓度、不影响血磷浓度
E.升高血钙、血磷浓度
B.升高血钙浓度、降低血磷浓度
4 5 2008 22. I 型(绝经后)骨质疏松是什么型骨质疏松 (2.0分) A.高转换型
B.低转换型
C.平衡型
D.快速型
A.高转换型
4 5 2008 23. 发生脆性骨折的常见部位为 (2.0分) A.胸腰椎
B.髋部
C.桡尺骨远端
D.肱骨近端
E.以上都是
E.以上都是
4 5 2008 24. 一个60岁妇女,测得骨密度T值为-3.4,全身检查无骨折,该病人诊断为(2.0分) A.正常
B.骨质疏松
C.骨量减少
D.严重骨质疏松
B.骨质疏松
4 5 2008 25. 肢端肥大症中最多见于哪一段年龄(2.0分) A.31~40岁
B.21~30岁
C.41~50岁
D.51~60岁
E.61~70岁
A.31~40岁
4 5 2008 26. 肢端肥大症内分泌检查,葡萄糖抑制试验血浆GH不被抑制到多少以下(2.0分) A.5ug/L
B.10ug/L
C.12ug/L
D.15ug/L
E.20ug/L
A.5ug/L
4 5 2008 27. 下列哪种药物不能用于治疗肢端肥大症(2.0分) A.兰瑞肽
B.奥曲肽
C.卡麦角林
D.降钙素
E.溴隐亭
D.降钙素
4 5 2008 28. 经治疗后,GH严格控制指随机血GH小于(2.0分) A.5ug/L
B.4ug/L
C.3ug/L
D.2ug/L
E.1ug/L
D.2ug/L
4 5 2008 29. 下列哪种疾病中患者血中的ACTH增多(2.0分) A.席汉综合征
B.肾上腺皮质腺瘤或癌引起的柯兴氏综合征
C.原发性醛固酮增多症
D.Cushing病
E.肢端肥大症
D.Cushing病
4 5 2008 30. Cushing病是(2.0分) A.肾上腺皮质瘤产生大量糖皮质激素
B.垂体瘤分泌大量的ACTH, 引起肾上腺皮质增生
C.长期服用大量的糖皮质激素, 引起向心性肥胖
D.异位ACTH所致肾上腺皮质增生
E.不依赖ACTH的肾上腺瘤结节增生
B.垂体瘤分泌大量的ACTH, 引起肾上腺皮质增生
4 5 2008 31. 关于皮质醇增多症下列哪一项是错误的(2.0分) A.主要临床表现向心性肥胖, 高血压, 骨质疏松
B.肾上腺皮质增生少见
C.成人多于儿童
D.儿童患者腺癌多见
E.女性男性化明显者提示腺癌
B.肾上腺皮质增生少见
4 5 2008 32. 女性, 35岁, 肥胖, 紫纹(-), 血浆皮质醇552nmol/L(20μg/dl)(正常值6-16μg/dl), 为了明确诊断应首选下列哪项检查(2.0分) A.测定血浆ACTH
B.肾上腺CT扫描
C.垂体磁共振检查
D.葡萄糖耐量试验
E.小剂量地塞米松抑制试验
E.小剂量地塞米松抑制试验
4 5 2008 33. 下列哪种病不属于肾上腺皮质疾病(2.0分) A.柯兴综合征
B.嗜铬细胞瘤
C.原发性醛固酮增多症
D.先天性11B-羟类固醇脱氢酶缺乏
E.阿狄森病
B.嗜铬细胞瘤
4 5 2008 34. 肾上腺髓质血液供应的特点是(2.0分) A.仅由肾上腺动脉分支直接供应
B.仅由经皮质的窦状毛细血管进入髓质供应
C.由肾上腺动脉形成第二级毛细血管供应
D.由肾上腺动脉分枝的髓质小动脉经皮质直接进入髓质与经皮质的窦状毛细血管网相通的血液供给
E.肾动脉的分支直接进入髓质供给
D.由肾上腺动脉分枝的髓质小动脉经皮质直接进入髓质与经皮质的窦状毛细血管网相通的血液供给
4 5 2008 35. 下列哪种病不属于肾上腺皮质疾病(2.0分) A.柯兴综合征
B.嗜铬细胞瘤
C.原发性醛固酮增多症
D.先天性11B-羟类固醇脱氢酶缺乏
E.阿狄森病
B.嗜铬细胞瘤
4 5 2008 36. 肾上腺皮质肿瘤引起的Cushing综合征与 Cushing病的鉴别,最有意义的试验检查是(2.0分) A.血皮质醇昼夜节律消失
B.葡萄糖耐量试验
C.测试24小时尿17-羟类固醇
D.小剂量地塞米松抑制试验
E.大剂量地塞米松抑制试验
E.大剂量地塞米松抑制试验
4 5 2008 37. 糖皮质激素的作用是(2.0分) A.抑制肝脏糖异生
B.抑制外周组织利用葡萄糖
C.抑制蛋白质分解
D.抑制脂肪合成
B.抑制外周组织利用葡萄糖
4 5 2008 38. 肾上腺X线平片发现肾上腺区钙化影提示(2.0分) A.嗜铬细胞瘤
B.柯兴综合症
C.肾上腺结核
D.原发性醛固增多症
E.垂体ACTH腺瘤
C.肾上腺结核
4 5 2008 39. 患者,女,38岁。主诉:头昏、乏力、恶心、呕吐、色素增加6月。有体重减轻、四肢无力、食欲不振、腹胀、闭经等。体检:BP:85/45mmHg,T:35.6℃;神疲乏力,消瘦,颜面、口腔粘膜、四肢皮肤等见色素沉着,HR:64bpm, 律齐,心音低,未及杂音。化验:血Na+:125mmol/L,K+:5.1mmol/L,Glu:66mg/dl。该患者应诊断什么疾病?(2.0分) A.垂体瘤
B.原发性慢性肾上腺皮质功能减退症
C.垂体前叶功能减退
D.异位ACTH 综合征
B.原发性慢性肾上腺皮质功能减退症
4 5 2008 40. 睾酮由睾丸的下列哪种细胞分泌? (2.0分) A.间质细胞
B.支持细胞
C.精原细胞
D.精母细胞
E.精子
A.间质细胞
4 5 2008 41. 雄激素结合蛋白是由下列哪种细胞产生的? (2.0分) A.肝细胞
B.睾丸间质细胞
C.睾丸支持细胞
D.睾丸生精细胞
E.睾丸毛细血管内皮细胞
C.睾丸支持细胞
4 5 2008 42. 抑制素是由睾丸的哪种细胞分泌的? (2.0分) A.间质细胞
B.支持细胞
C.精原细胞
D.精母细胞
E.精子
B.支持细胞
4 5 2008 43. 能促进支持细胞分泌抑制素的激素是(2.0分) A.LH
B.ACTH
C.FSHD
D.ADH
E.FSH
E.FSH
4 5 2008 44. 下列关于雌激素生理作用的描述,错误的是(2.0分) A.使输卵管平滑肌活动增强
B.促进阴道上皮细胞增生、角化
C.抑制子宫内膜增生、腺体分泌
D.刺激乳腺导管和结缔组织增生
E.促进肾小管对钠和水的重吸收
C.抑制子宫内膜增生、腺体分泌
4 5 2008 45. 下列有关孕激素作用的叙述,正确的是(2.0分) A.促进子宫内膜发生增生期变化
B.促进子宫内膜发生分泌期变化
C.子宫发育
D.促进并维持女性特征
E.促进子宫收缩
B.促进子宫内膜发生分泌期变化
4 5 2008 46. 雌激素和孕激素作用的相同点是(2.0分) A.促进阴道上皮细胞角化
B.促进乳腺导管增生和延长
C.使子宫输卵管平滑肌活动减弱
D.使子宫内膜增生变厚
E.减少宫颈粘液的分泌
D.使子宫内膜增生变厚
4 5 2008 47. 下列关于月经的叙述,哪一项是错误的? (2.0分) A.出血期中,GnRH分泌处于较低水平
B.黄体退化,雌激素和孕激素分泌骤减
C.前列腺素F2 释放,子宫内膜血管痉挛
D.溶酶体释出蛋白水解酶,使组织溶解,内膜剥脱、出血
E.出血期中由于雌激素和孕激素的正反馈,FSH和LH分泌增加
E.出血期中由于雌激素和孕激素的正反馈,FSH和LH分泌增加
4 5 2008 48. 正常情况下精子与卵子在何处相遇而发生受精? (2.0分) A.输卵管壶腹部
B.输卵管伞部
C.输卵管漏斗部
D.子宫腔
E.子宫颈
A.输卵管壶腹部
4 5 2008 1. 骨质疏松症的治疗药物大致可以分为(2.0分) A.基础补充
B.促进骨形成
C.抑制骨重吸
D.增加矿物质
A.基础补充
B.促进骨形成
C.抑制骨重吸
4 5 2008 2. 临床上骨密度常规检测部位是(2.0分) A.股骨颈
B.跟骨
C.桡骨远端
D.腰椎
A.股骨颈
D.腰椎
4 5 2009 1. 体内调节血钙水平最重要的激素是:(2.5分) A.甲状旁腺激素
B.降钙素
C.醛固酮
D.维生素D
E.以上都不是
A.甲状旁腺激素
4 5 2009 2. 血浆中降钙素的主要来源是:(2.5分) A.甲状旁腺细胞
B.胰岛D细胞
C.肾上腺皮质网状带
D.甲状腺C细胞
E.肾上腺皮质球状带
D.甲状腺C细胞
4 5 2009 3. 下列哪种维生素能显著影响钙的吸收和代谢?(2.5分) A.维生素A
B.维生素B
C.维生素C
D.维生素D
E.维生素E
D.维生素D
4 5 2009 4. Which of the following is not involved in regulating plasma Ca2+ levels? (2.5分) A.Kidneys
B.Skin
C.Liver
D.Lungs
E.Intestine
D.Lungs
4 5 2009 5. PTH and PTHrP activate the same receptor, but their effects are different. How is this possible? (2.5分) A.For unknown reasons, PTHrP acts mainly on fetal tissues, whereas PTH acts mainly on adult tissues
B.Plasma PTH is high when plasma PTHrP is low and vice versa
C.PTH acts on the brain, whereas PTHrP does not
D.When PTH binds to the common receptor, it activates adenylyl cyclase, whereas PTHrP activates phospholipase C
E.There is an additional receptor in the body that responds to PTH but not to PTHrP
E.There is an additional receptor in the body that responds to PTH but not to PTHrP
4 5 2009 6. 影响钙磷代谢的因素下列哪项不是的(2.5分) A.甲状旁腺激素
B.维生素D
C.肾脏功能
D.心脏功能
E.肠道功能
D.心脏功能
4 5 2009 7. 低钙血症最具特征的临床表现是:(2.5分) A.手足搐搦
B.肌痉挛
C.骨痛、骨折
D.心律不齐、心脏传导阻滞
E.认知能力减退
A.手足搐搦
4 5 2009 8. 低磷血症是指:(2.5分) A.指血磷低于0.4mmol/L
B.指血磷低于0.6mmol/L
C.指血磷低于0.7mmol/L
D.指血磷低于1.0mmol/L
E.指血磷低于0.8mmol/L
E.指血磷低于0.8mmol/L
4 5 2009 9. 下列哪项不是引起高磷血症的病因:(2.5分) A.肾功能衰竭
B.甲状旁腺功能减退
C.维生素D缺乏
D.肉芽肿病
E.含磷酸盐泻剂或灌肠液
C.维生素D缺乏
4 5 2009 10. 维生素D中毒可引起高钙、高磷血症,主要的针对病因的治疗措施是:(2.5分) A.大量补液
B.使用速尿等快速利尿
C.糖皮质激素的使用
D.降钙素
E.低钙饮食
C.糖皮质激素的使用
4 5 2009 11. 下列哪项为最常见的垂体瘤?(2.5分) A.生长激素瘤
B.泌乳素瘤
C.促肾上腺皮质激素瘤
D.促性腺激素瘤
E.促甲状腺激素瘤
B.泌乳素瘤
4 5 2009 12. 下列哪现不是垂体分泌的激素?(2.5分) A.ACTH
B.TSH
C.LH
D.E
E.GH
D.E
4 5 2009 13. 于垂体瘤下列哪项正确?(2.5分) A.一定有某一激素升高或减少
B.一种腺瘤分泌一种激素
C.TSH瘤常见
D.药物治疗效果好
E.催乳素瘤首选药物治疗
E.催乳素瘤首选药物治疗
4 5 2009 14. 有关垂体微腺瘤下列哪项不正确?(2.5分) A.直径<10mm
B.PRL瘤最常见
C.一般采取经蝶显微外科手术切除腺瘤
D.需开颅经额途径切除腺瘤
E.最后诊断依赖于病理免疫细胞化学分析
D.需开颅经额途径切除腺瘤
4 5 2009 15. 关于PRL下列哪项不正确?(2.5分) A.甲状腺功能减退症患者可出现PRL 升高
B.〉200ug/L提示催乳素瘤
C.多巴胺可促进其分泌
D.雌激素的作用是妊娠期PRL增高的主要原因
E.对FSH/LH 的分泌和作用均有抑制作用
C.多巴胺可促进其分泌
4 5 2009 16. 关于腺垂体功能减退症患者的治疗,下列哪项不对(2.5分) A.给予左旋甲状腺素50~150μg/d
B.给予泼尼松5.0~7.5mg/d
C.性激素替代,可用人工月经周期
D..感染时糖皮质激素用量可适当加大
E.可放心使用镇静安眠药
E.可放心使用镇静安眠药
4 5 2009 17. 以下哪项不符合腺垂体功能减退症垂体危象的临床类型(2.5分) A.低血糖型
B.高血糖型
C.低温型
D.低血压型
E.混合型
B.高血糖型
4 5 2009 18. 垂体危象时,下列何种情况最为多见(2.5分) A.低血糖性昏迷
B.低钾性麻痹
C.谵妄
D.脑梗死
E.高钠高渗性昏迷
A.低血糖性昏迷
4 5 2009 19. 女性,32岁,因前置胎盘,分娩时大出血,剖腹产一健康男孩,产后无乳,乳房萎缩,无性欲,无月经再来,疲倦乏力,怕冷,嗜睡。 该病人宜考虑:(2.5分) A.产后抑郁症
B. 希恩综合征
C. 肾上腺皮质功能减退症
D. 产后甲状腺炎
E. 催乳素瘤
B. 希恩综合征
4 5 2009 20. 抢救高钙危象时需应用:(2.5分) A.糖皮质激素
B.酚妥拉明
C.尼群地平
D.大量生理盐水
E.大量10%葡萄糖
D.大量生理盐水
4 5 2009 21. 女性患者,因甲状腺肿大出现呼吸困难而行手术切除术,术后患者自觉指端和嘴边有麻木感、刺痛,一遇劳累便出现"抽搐"发作,该患者最有可能的诊断:(2.5分) A.癫痫
B.甲状旁腺功能减退症
C.甲状腺功能减退症
D.植物神经功能紊乱
E.神经系统疾病
B.甲状旁腺功能减退症
4 5 2009 22. 甲状旁腺素对血钙调节主要通过(2.5分) A.肠和胃
B.肝和胆
C.胰和胆
D.骨、肾和肠
E.脑垂体
D.骨、肾和肠
4 5 2009 23. 有关原发性甲旁亢,下列错误的是(2.5分) A.该病主要的生化表现为高血钙、高尿钙,低血磷、低尿磷
B.PTH均增高明显
C.因维生素D充足而多合并佝偻病/骨软化症
D.腺癌患者多于术后诊断明确
E.B超和核素检查是其主要的定位诊断手段
C.因维生素D充足而多合并佝偻病/骨软化症
4 5 2009 24. 关于醛固酮增多症,筛查指标是:(2.5分) A.血钾水平;
B.24小时尿醛固酮水平
C.随机血醛固酮水平
D.ARR比值
E.卡托普利试验
D.ARR比值
4 5 2009 25. 原发性醛固酮增多症的生化改变不包括: (2.5分) A.高醛固酮水平
B.低肾素活性
C.低钾血症
D.代谢性碱中毒
E.血皮质醇水平升高
E.血皮质醇水平升高
4 5 2009 26. 原醛症的临床表现为高血压、低钾,高血压合并低钾的比例为; (2.5分) A.10%,
B.20%
C. 30-50%
D.50-70%
E.均合并有低钾
C. 30-50%
4 5 2009 27. 分型诊断金标准(2.5分) A.肾上腺CT
B.奥曲肽显像
C.AVS
D.肾上腺CTA
E.肾上腺MRI
C.AVS
4 5 2009 28. AVP主要受哪些因素的调控?(2.5分) A.渗透压感受器、容量感受器、压力感受器
B.渗透压感受器、神经递质、药物
C.容量感受器、压力感受器与神经递质
D.渗透压感受器、肾小管的反应、喝水的量
E.血容量、渗透压感受器、和体液平衡
A.渗透压感受器、容量感受器、压力感受器
4 5 2009 29. 诊断尿崩症的初筛标准?(2.5分) A.尿量大于2500ml/day ,尿比重小于1.005
B.尿量大于4000ml/day,尿比重小于1.005
C.尿量大于4000ml/day ,尿比重小于1.010
D.尿量大于5000ml/day,尿比重小于1.008
E.尿量大于3000ml/day,尿比重大于1.005
B.尿量大于4000ml/day,尿比重小于1.005
4 5 2009 30. 加压素(抗利尿激素)的下述作用,哪项是正确的(2.5分) A.使肾远曲小管及集合管对水的通透性增加,促进水的重吸收
B.临床上常用加压素来提高血压、治疗休克
C.加压素在生理状态上参与机体血压调节
D.调节体内水总量,使血浆渗透压增高
E.肾性尿崩症可用加压素治疗
A.使肾远曲小管及集合管对水的通透性增加,促进水的重吸收
4 5 2009 31. 最方便有效的中枢性尿崩症治疗药物: (2.5分) A.垂体加压素水剂
B.血管升压素
C.去氨加压素(弥凝片)
D.氢氯噻嗪片
E.氯磺丙脲片
C.去氨加压素(弥凝片)
4 5 2009 32. 影响PTH分泌最重要的因素是:(2.5分) A.血磷浓度
B.血钙浓度
C.血镁浓度
D.血中儿茶酚胺浓度
E.血中前列腺素浓度
B.血钙浓度
4 5 2009 33. 降钙素的主要靶器官是:(2.5分) A.甲状腺
B.甲状旁腺
C.胃肠道
D.肾脏
E.骨组织
E.骨组织
4 5 2009 34. 下列哪种激素可促进1,25-(OH)2-VD3形成:(2.5分) A.PTH
B.T3
C.T4
D.CT
E.TSH
A.PTH
4 5 2009 35. Which of the following would you expect to find in a patient whose diet has been low in calcium for 2 months?(2.5分) A.Increased formation of 24, 25-dihydroxycholecalciferol
B.Decreased amounts of calcium-binding protein in intestinal epithelial cells
C.Increased parathyroid hormone secretion
D.A high plasma calcitonin concentration
E.Increased plasma phosphates
C.Increased parathyroid hormone secretion
4 5 2009 36. 男性,40岁,头痛,视力下降3个月,查血压140/80mmHg, 左眼颞侧偏盲,血GH 35 g/L (正常<10 g/L)。进一步应行下列哪项检查?(2.5分) A.垂体MRI
B.肌电图
C.头颅X线检查
D.头颅血管造影
E.脑电图
A.垂体MRI
4 5 2009 37. 关于催乳素瘤下列哪项不正确?(2.5分) A.为最常见的垂体瘤
B.多见于女性
C.多为微腺瘤
D.溴隐亭治疗效果好
E.停药易复发,故无论何种情况均不宜停药
E.停药易复发,故无论何种情况均不宜停药
4 5 2009 38. 下列哪项不引起高钙血症:(2.5分) A.原发性甲状旁腺功能亢进
B.脂质代谢异常
C.肾功能衰竭
D.长期的制动
E.肢端肥大症
B.脂质代谢异常
4 5 2009 39. 纠正低钙搐搦首选:(2.5分) A.迅速输注生理盐水
B.静脉注射葡萄糖酸钙
C.口服钙剂和维生素D
D.应用利尿剂
E.应用镇静剂
B.静脉注射葡萄糖酸钙
4 5 2009 40. 纠正高钙血症的治疗措施下列哪项是错误的:(2.5分) A.二膦酸盐
B.输入生理盐水
C.速尿
D.双氢克尿噻
E.透析
D.双氢克尿噻
4 5 2010 1. 临床怀疑胰腺癌,首先的影像学检查方法是 (2.0分) A.超声
B.CT
C.MRI
D.血管造影
E.ERCP
A.超声
4 5 2010 2. 胰腺癌的临床特征哪项不正桷 (2.0分) A.好发年龄在40~70岁之间,男性多于女性
B.胰腺癌中2/3为胰头癌
C.90%以上为导管细胞癌
D.易早期发现,易手术切除、预后好
E.广泛浸润周围组织、器官、较早经淋巴转移
D.易早期发现,易手术切除、预后好
4 5 2010 3. 术中发现胰头部一轮廓不规则肿物,边界尚清,与周围组织无粘连,门静脉、肠系膜上静脉均完整,无破坏,宜行 (3.0分) A.Whipple胰头十二脂肠切除术
B.扩大切除术
C.全胰切除术
D.胆囊十二指肠吻合术
E.胆总管十二指肠吻合术
A.Whipple胰头十二脂肠切除术
4 5 2010 4. 有关胰岛素瘤的陈述下列哪项是不正确的(B)(2.0分) A.是胰腺内分泌肿瘤中最常见的
B.绝大多数是恶性肿瘤
C.单发肿瘤占90%以上
D.典型症状是清晨自发性低血糖
E.给予葡萄糖或进餐后症状缓解
B.绝大多数是恶性肿瘤
4 5 2010 5. 醛固酮增多症病因分类占比最大的是: (2.0分) A.醛固酮腺瘤
B.肾上腺皮质球状带增生
C.醛固酮癌
D.家族性醛固酮增多症
E.特醛症
E.特醛症
4 5 2010 6. 原发性醛固酮增多症的典型临床特征为(2.0分) A.高血压和向心性肥胖
B.高血压和发作性心悸
C.高血压和低钾血症
D.发作性软瘫和低钾血症
E.多饮、多尿和低钾血症
C.高血压和低钾血症
4 5 2010 7. 确诊诊断应进行的检查是 (2.0分) A.血醛固酮水平
B.生理盐水抑制试验
C.24h尿醛固酮
D.肾动脉造影
E.肾上腺CT
B.生理盐水抑制试验
4 5 2010 8. 关于难治性高血压:(2.0分) A.三种降压药联用血压仍大于140/90mmHg
B.二种降压药联用血压仍大于140/90mmHg
C.三种降压药联用血压仍大于140/85mmHg
D.二种降压药联用血压仍大于130/85mmHg
E.三种降压药联用血压仍大于150/90mmHg
E.三种降压药联用血压仍大于150/90mmHg
4 5 2010 9. 糖皮质激素是由下列哪个部位分泌的: (2.0分) A.肾脏
B.肾上腺皮质束状带
C.肾上腺皮质网状带
D.肾上腺皮质球状带
E.肾上腺髓质
B.肾上腺皮质束状带
4 5 2010 10. 醛固酮的主要作用是促进肾脏(2.0分) A.近曲小管保钠排钾
B.远曲小管保钠排钾
C.远曲小管保钾排钠
D.近曲小管保钾排钠
E.髓质保钠排钾
B.远曲小管保钠排钾
4 5 2010 11. 肾上腺素是由下列哪个部位分泌的: (2.0分) A.肾脏
B.肾上腺皮质束状带
C.肾上腺皮质网状带
D.肾上腺皮质球状带
E.肾上腺髓质
E.肾上腺髓质
4 5 2010 12. 一位39岁女性患者,乏力、恶心、消瘦二年,伴面部、手掌指纹等处皮肤变黑一年。入院前一天腹泻三次而神志模糊入院。查体发现乳房萎缩、阴毛稀少。血压45/30mmHg.心率120次/分。血糖60mg/dl,血钠132mmol/dl.诊断上最大可能是(2.0分) A.恶性肿瘤晚期
B.严重结核病晚期
C.垂体功能减退症危象
D.肾上腺危象
E.中毒性肠炎、感染性休克
D.肾上腺危象
4 5 2010 13. 有关三发性甲旁亢的叙述正确的是:(2.0分) A.有三个甲状旁腺腺体原发疾病引起的甲旁亢,叫三发性甲旁亢
B.临床上最常见的甲旁亢是三发性甲旁亢
C.恶性肿瘤转移至三个甲状旁腺引起的甲旁亢叫三发性甲旁亢
D.除了甲状旁腺外,还有另外两个内分泌腺出现病变,临床上以甲旁亢症状为主,叫三发性甲旁亢
E.在继发甲旁亢的基础上,由于腺体受到持久的刺激,自主性分泌过多的PTH,叫三发性甲旁亢
E.在继发甲旁亢的基础上,由于腺体受到持久的刺激,自主性分泌过多的PTH,叫三发性甲旁亢
4 5 2010 14. 抢救高钙危象时需应用:(2.0分) A.糖皮质激素
B.酚妥拉明
C.尼群地平
D.大量生理盐水
E.大量10%葡萄糖
D.大量生理盐水
4 5 2010 15. 女性患者,因甲状腺肿大出现呼吸困难而行手术切除术,术后患者自觉指端和嘴边有麻木感、刺痛,一遇劳累便出现"抽搐"发作,该患者最有可能的诊断:(2.0分) A.癫痫
B.甲状旁腺功能减退症
C.甲状腺功能减退症
D.植物神经功能紊乱
E.神经系统疾病
B.甲状旁腺功能减退症
4 5 2010 16. 甲状旁腺素对血钙调节主要通过(2.0分) A.肠和胃
B.肝和胆
C.胰和胆
D.骨、肾和肠
E.脑垂体
D.骨、肾和肠
4 5 2010 17. 有关原发性甲旁亢,下列错误的是(2.0分) A.该病主要的生化表现为高血钙、高尿钙,低血磷、低尿磷
B.PTH均增高明显
C.因维生素D充足而多合并佝偻病/骨软化症
D.腺癌患者多于术后诊断明确
E.B超和核素检查是其主要的定位诊断手段
C.因维生素D充足而多合并佝偻病/骨软化症
4 5 2010 18. 影响钙磷代谢的因素下列哪项不是的:(2.0分) A.甲状旁腺激素
B.维生素D
C.肾脏功能
D.心脏功能
E.肠道功能
D.心脏功能
4 5 2010 19. 低钙血症最具特征的临床表现是:(2.0分) A.手足搐搦
B.肌痉挛
C.骨痛、骨折
D.心律不齐、心脏传导阻滞
E.认知能力减退
A.手足搐搦
4 5 2010 20. 低磷血症是指:(2.0分) A.指血磷低于0.4mmol/L
B.指血磷低于0.6mmol/L
C.指血磷低于0.7mmol/L
D.指血磷低于1.0mmol/L
E.指血磷低于0.8mmol/L
E.指血磷低于0.8mmol/L
4 5 2010 21. 下列哪项不是引起高磷血症的病因:(2.0分) A.肾功能衰竭
B.甲状旁腺功能减退
C.维生素D缺乏
D.肉芽肿病
E.含磷酸盐泻剂或灌肠液
C.维生素D缺乏
4 5 2010 22. 维生素D中毒可引起高钙、高磷血症,主要的针对病因的治疗措施是:(3.0分) A.大量补液
B.使用速尿等快速利尿
C.糖皮质激素的使用
D.降钙素
E.低钙饮食
C.糖皮质激素的使用
4 5 2010 23. 腺垂体功能减退症最早出现的靶腺功能减退是(2.0分) A.肾上腺皮质功能减退
B.性腺功能减退
C.甲状腺功能减退
D.肾上腺与甲状腺功能减退
E.甲状腺与性腺功能减退
B.性腺功能减退
4 5 2010 24. 下列哪项对诊断腺垂体功能减退症无意义(2.0分) A.甲状旁腺素测定
B.甲状腺素测定
C.性腺激素测定
D.皮质醇测定
E.泌乳素测定
A.甲状旁腺素测定
4 5 2010 25. 下列哪项替代治疗是错误的?(2.0分) A.甲状腺功能减退使用左甲状腺素50 150 g/d
B.肾上腺皮质功能减退使用氢化可的松20 30mg/d,9α-氟氢可的松0.05~0.1 mg/d
C.女性闭经:炔雌醇5 20 g/d(月经周期第1 25天),甲羟孕酮(安宫黄体酮)5 10mg/d(月经周期第12 25天)男子性腺功能减退症丙酸睾酮50mg/周肌注, 或十一酸睾酮40mg,每日3次口服
D. 女性不育:促性素(HMG) 75 ~ 150 IU/d,持续二周,并肌注绒促性素(HCG) 2000IU
B.肾上腺皮质功能减退使用氢化可的松20 30mg/d,9α-氟氢可的松0.05~0.1 mg/d
4 5 2010 26. 男性,45岁,鼻咽癌放疗后出现性欲减退、阳痿、怕冷、思睡、思维迟钝、精神淡漠。该病人需考虑下列哪种情况:(2.0分) A.放疗引起甲状腺破坏,至甲状腺功能减退症
B. 鼻咽癌复发
C. 放疗破坏垂体组织,引起腺垂体功能减退症
D. 植物神经功能紊乱
E. 前列腺炎
C. 放疗破坏垂体组织,引起腺垂体功能减退症
4 5 2010 27. 下列哪项为最常见的垂体瘤?(2.0分) A.生长激素瘤
B.泌乳素瘤
C.促肾上腺皮质激素瘤
D.促性腺激素瘤
E.促甲状腺激素瘤
B.泌乳素瘤
4 5 2010 28. 下列哪现不是垂体分泌的激素?(2.0分) A.ACTH
B.TSH
C.LH
D.E
E.GH
D.E
4 5 2010 29. 于垂体瘤下列哪项正确?(2.0分) A.一定有某一激素升高或减少
B.一种腺瘤分泌一种激素
C.TSH瘤常见
D.药物治疗效果好
E.催乳素瘤首选药物治疗
E.催乳素瘤首选药物治疗
4 5 2010 30. 有关垂体微腺瘤下列哪项不正确?(2.0分) A.直径<10mm
B.PRL瘤最常见
C.一般采取经蝶显微外科手术切除腺瘤
D.需开颅经额途径切除腺瘤
E.最后诊断依赖于病理免疫细胞化学分析
D.需开颅经额途径切除腺瘤
4 5 2010 31. 关于PRL下列哪项不正确?(2.0分) A.甲状腺功能减退症患者可出现PRL 升高
B.〉200ug/L提示催乳素瘤
C.多巴胺可促进其分泌
D.雌激素的作用是妊娠期PRL增高的主要原因
E.对FSH/LH 的分泌和作用均有抑制作用
C.多巴胺可促进其分泌
4 5 2010 32. 肢端肥大症系腺垂体分泌哪种内分泌激素细胞腺瘤或增生引起(2.0分) A.ACTH
B.LH
C.FSH
D.GH
E.TSH
D.GH
4 5 2010 33. 肢端肥大症内分泌检查,生长激素一般大于(2.0分) A.5ug/L
B.20ug/L
C.10ug/L
D.15ug/L
E.30ug/L
B.20ug/L
4 5 2010 34. 下列哪项疾病需与肢端肥大症鉴别(2.0分) A.糖尿病
B.甲状腺功能亢进
C.手足皮肤骨膜肥厚症
D.皮质醇增多症
E.泌乳素瘤
C.手足皮肤骨膜肥厚症
4 5 2010 35. 经治疗后,GH一般控制指随机血GH小于(2.0分) A.5 ug/L
B.6 ug/L
C.8 ug/L
D.10 ug/L
E.12 ug/L
A.5 ug/L
4 5 2010 36. 经治疗后,GH严格控制指随机血GH小于(2.0分) A.5 ug/L
B.4 ug/L
C.3 ug/L
D.2 ug/L
E.1 ug/L
D.2 ug/L
4 5 2010 37. I 型(绝经后)骨质疏松是什么型骨质疏松(2.0分) A.高转换型
B.低转换型
C.平衡型
D.快速型
A.高转换型
4 5 2010 38. 发生脆性骨折的常见部位为 (2.0分) A.胸腰椎
B.髋部
C.桡尺骨远端
D.肱骨近端
E.以上都是
E.以上都是
4 5 2010 39. 一个60岁妇女,测得骨密度T值为-3.4,全身检查无骨折,该病人诊断为(2.0分) A.正常
B.骨质疏松
C.骨量减少
D.严重骨质疏松
B.骨质疏松
4 5 2010 40. 影响PTH分泌最重要的因素是:(2.0分) A.血磷浓度
B.血钙浓度
C.血镁浓度
D.血中儿茶酚胺浓度
E.血中前列腺素浓度
B.血钙浓度
4 5 2010 41. 降钙素的主要靶器官是:(2.0分) A.甲状腺
B.甲状旁腺
C.胃肠道
D.肾脏
E.骨组织
E.骨组织
4 5 2010 42. 下列哪种激素可促进1,25-(OH)2-VD3形成:(2.0分) A.PTH
B.T3
C.T4
D.CT
E.TSH
A.PTH
4 5 2010 43. 下列哪项属于甲状旁腺激素的作用?(2.0分) A.抑制骨钙入血
B.使血磷升高
C.抑制远曲小管和集合管对钙的重吸收
D.促进1,25-二羟维生素D3形成
E.促进磷酸盐的重吸收
D.促进1,25-二羟维生素D3形成
4 5 2010 44. Which of the following would you expect to find in a patient whose diet has been low in calcium for 2 months?(2.0分) A.Increased formation of 24, 25-dihydroxycholecalciferol
B.Decreased amounts of calcium-binding protein in intestinal epithelial cells
C.Increased parathyroid hormone secretion
D.A high plasma calcitonin concentration
E.Increased plasma phosphates
C.Increased parathyroid hormone secretion
4 5 2010 45. Which of the following is not involved in regulating plasma Ca2+ levels? (2.0分) A.Kidneys
B.Skin
C.Liver
D.Lungs
E.Intestine
D.Lungs
4 5 2010 46. 血浆中降钙素的主要来源是:(2.0分) A.甲状旁腺细胞
B.胰岛D细胞
C.肾上腺皮质网状带
D.甲状腺C细胞
E.肾上腺皮质球状带
D.甲状腺C细胞
4 5 2010 47. 体内调节血钙水平最重要的激素是:(2.0分) A.甲状旁腺激素
B.降钙素
C.醛固酮
D.维生素D
E.以上都不是
A.甲状旁腺激素
4 5 2010 48. 下列哪种药物不能用于治疗肢端肥大症(2.0分) A.兰曲肽
B.奥曲肽
C.卡麦角林
D.降钙素
E.溴隐亭
D.降钙素
4 5 2010 49. 经治疗后,GH一般控制指口服葡萄糖抑制试验GH谷值小于(2.0分) A.4 ug/L
B.3 ug/L
C.2 ug/L
D.1 ug/L
D.1 ug/L
4 5 2011 1. I 型(绝经后)骨质疏松是什么型骨质疏松(2.5分) A.高转换型
B.低转换型
C.平衡型
D.快速型
A.高转换型
4 5 2011 2. 骨强度是指 (2.5分) A.骨量
B.骨矿物质含量
C.骨密度
D.骨质量+骨密度
D.骨质量+骨密度
4 5 2011 3. 发生脆性骨折的常见部位为 (2.5分) A.胸腰椎
B.髋部
C.桡尺骨远端
D.肱骨近端
E.以上都是
E.以上都是
4 5 2011 4. 目前国际学术界公认的骨质疏松症诊断的金标准(2.5分) A.超声
B.Q-CT
C.双能X线骨密度仪(DXA)
D.MRI
C.双能X线骨密度仪(DXA)
4 5 2011 5. 血管加压素(ArginineVasopressin,AVP)是由什么.部位?以下哪种细胞产生的?(2.5分) A.下丘脑弓状核的APUD细胞与AgRP细胞产生
B.垂体前叶内分泌细胞产生作用于下丘脑室旁核
C.下丘脑视上核心与室旁核神经元产生,沿神经垂体束下行至垂体后叶
D.第三脑室旁细胞产生沿神经垂体束下行至垂体后叶
E.侧脑室旁神经细胞产生
C.下丘脑视上核心与室旁核神经元产生,沿神经垂体束下行至垂体后叶
4 5 2011 6. AVP主要受哪些因素的调控? (2.5分) A.渗透压感受器、容量感受器、压力感受器
B.渗透压感受器、神经递质、药物
C.容量感受器、压力感受器与神经递质
D.渗透压感受器、肾小管的反应、喝水的量
E.血容量、渗透压感受器、和体液平衡
A.渗透压感受器、容量感受器、压力感受器
4 5 2011 7. 关于原醛(2.5分) A.难治性高血压均是原醛
B.原醛均表现为高血压、低钾
C.原醛均表现为RAS系统ARR比值增高
D.原醛的病因是由于醛固酮腺瘤所致
E.原醛的尿钾排泄不增高
C.原醛均表现为RAS系统ARR比值增高
4 5 2011 8. 关于原醛的肾性失钾的证据以下哪个是正确的? (2.5分) A.钾的摄入增加,尿钾增加
B.夏季出汗增加,尿钾排泄增加
C.血钾正常,尿钾排泄增加
D.血钾高,尿钾排泄增加
E.血钾低,尿钾排泄增加
E.血钾低,尿钾排泄增加
4 5 2011 9. ARR比值对进一步原醛筛查有意义? (2.5分) A.30倍
B.20倍
C.10倍
D.5倍
E.40倍
A.30倍
4 5 2011 10. 下列哪项为最常见的垂体瘤?(2.5分) A.生长激素瘤
B.泌乳素瘤
C.促肾上腺皮质激素瘤
D.促性腺激素瘤
E.促甲状腺激素瘤
B.泌乳素瘤
4 5 2011 11. 女性,30岁,结婚4年未育,双乳溢乳,垂体发现微腺瘤,首先应行哪项检查?(2.5分) A.ACTH
B.TSH
C.LH
D.PRL
E.GH
D.PRL
4 5 2011 12. 下列哪现不是垂体分泌的激素?(2.5分) A.ACTH
B.TSH
C.LH
D.E
E.GH
D.E
4 5 2011 13. 男性,40岁,头痛,视力下降3个月,查血压140/80mmHg, 左眼颞侧偏盲,血GH 35 g/L (正常<10 g/L)。进一步应行下列哪项检查?(2.5分) A.垂体MRI
B.肌电图
C.头颅X线检查
D.头颅血管造影
E.脑电图
A.垂体MRI
4 5 2011 14. 腺垂体功能减退症最早出现的靶腺功能减退是(2.5分) A.肾上腺皮质功能减退
B.性腺功能减退
C.甲状腺功能减退
D.肾上腺与甲状腺功能减退
E.甲状腺与性腺功能减退
B.性腺功能减退
4 5 2011 15. 关于腺垂体功能减退症患者的治疗,下列哪项不对(2.5分) A.给予左旋甲状腺素50~150μg/d
B.给予泼尼松5.0~7.5mg/d
C.性激素替代,可用人工月经周期
D.感染时糖皮质激素用量可适当加大
E.可放心使用镇静安眠药
E.可放心使用镇静安眠药
4 5 2011 16. 下列哪项对诊断腺垂体功能减退症无意义(2.5分) A.甲状旁腺素测定
B.甲状腺素测定
C.性腺激素测定
D.皮质醇测定
E.泌乳素测定
A.甲状旁腺素测定
4 5 2011 17. 影响PTH分泌最重要的因素是(2.5分) A.血磷浓度
B.血钙浓度
C.血镁浓度
D.血中儿茶酚胺浓度
E.血中前列腺素浓度
B.血钙浓度
4 5 2011 18. 体内调节血钙水平最重要的激素是(2.5分) A.甲状旁腺激素
B.降钙素
C.醛固酮
D.维生素D
E.以上都不是
A.甲状旁腺激素
4 5 2011 19. 降钙素的主要靶器官是(2.5分) A.甲状腺
B.甲状旁腺
C.胃肠道
D.肾脏
E.骨组织
E.骨组织
4 5 2011 20. 血浆中降钙素的主要来源是(2.5分) A.甲状旁腺细胞
B.胰岛D细胞
C.肾上腺皮质网状带
D.甲状腺C细胞
E.肾上腺皮质球状带
D.甲状腺C细胞
4 5 2011 21. 下列哪种激素可促进1,25-(OH)2-VD3形成(2.5分) A.PTH
B.T3
C.T4
D.CT
E.TSH
A.PTH
4 5 2011 22. 影响钙磷代谢的因素下列哪项不是的(2.5分) A.甲状旁腺激素
B.维生素D
C.肾脏功能
D.心脏功能
E.肠道功能
D.心脏功能
4 5 2011 23. 下列哪项不引起高钙血症(2.5分) A.原发性甲状旁腺功能亢进
B.脂质代谢异常
C.肾功能衰竭
D.长期的制动
E.肢端肥大症
B.脂质代谢异常
4 5 2011 24. 低钙血症最具特征的临床表现是(2.5分) A.手足搐搦
B.肌痉挛
C.骨痛、骨折
D.心律不齐、心脏传导阻滞
E.认知能力减退
A.手足搐搦
4 5 2011 25. 纠正低钙搐搦首选(2.5分) A.迅速输注生理盐水
B.静脉注射葡萄糖酸钙
C.口服钙剂和维生素D
D.应用利尿剂
E.应用镇静剂
B.静脉注射葡萄糖酸钙
4 5 2011 26. 低磷血症是指(2.5分) A.指血磷低于0.4mmol/L
B.指血磷低于0.6mmol/L
C.指血磷低于0.7mmol/L
D.指血磷低于1.0mmol/L
E.指血磷低于0.8mmol/L
E.指血磷低于0.8mmol/L
4 5 2011 27. 有关三发性甲旁亢的叙述正确的是(2.5分) A.有三个甲状旁腺腺体原发疾病引起的甲旁亢,叫三发性甲旁亢
B.临床上最常见的甲旁亢是三发性甲旁亢
C.恶性肿瘤转移至三个甲状旁腺引起的甲旁亢叫三发性甲旁亢
D.除了甲状旁腺外,还有另外两个内分泌腺出现病变,临床上以甲旁亢症状为主,叫三发性甲旁亢
E.在继发甲旁亢的基础上,由于腺体受到持久的刺激,自主性分泌过多的PTH,叫三发性甲旁亢
E.在继发甲旁亢的基础上,由于腺体受到持久的刺激,自主性分泌过多的PTH,叫三发性甲旁亢
4 5 2011 28. 女性,42岁,于普查时发现血钙高,无其他异常,亦无明显不适,下一步应做的检查是(2.5分) A.查血清PTH
B.查血FT3、FT4
C.查17羟
D.查血NE
E.查血5-HT
A.查血清PTH
4 5 2011 29. 题干同28,若血清PTH升高,该患者最可能的诊断是(2.5分) A.高钙血症
B.原发性甲旁亢
C.Addison's病
D.继发性甲旁亢
E.甲亢
B.原发性甲旁亢
4 5 2011 30. 题干同28,该种病的治疗原则是(2.5分) A.手术治疗
B.观察
C.药物治疗
D.心理治疗
E.以上都对
A.手术治疗
4 5 2011 31. 皮质醇由肾上腺的什么结构合成和分泌? (2.5分) A.包膜
B.髓质
C.束状带
D.球状带
E.网状带
C.束状带
4 5 2011 32. 下列哪项可能导致继发性肾上腺皮质功能减退? (2.5分) A.肾上腺结核
B.双侧肾上腺切除术
C.肾上腺淀粉样变性
D.席汗综合症
E.恶性肿瘤肾上腺转移
D.席汗综合症
4 5 2011 33. 下列哪项不是慢性原发性肾上腺皮质功能减退的临床表现? (2.5分) A.乏力
B.体位性低血压
C.高钾血症
D.心音低钝
E.面色苍白
E.面色苍白
4 5 2011 34. 下列哪项不能用于鉴别原发性和继发性肾上腺皮质功能减退? (2.5分) A.尿17羟皮质类固醇测定
B.血醛固酮测定
C.皮肤黏膜色素沉着
D.ACTH兴奋试验
E.血ACTH测定
A.尿17羟皮质类固醇测定
4 5 2011 35. 下列哪些情况不需增加糖皮质激素的替代剂量(2.5分) A.分娩
B.局麻
C.创伤
D.感染
E.儿童生长发育迟缓
B.局麻
4 5 2011 36. 肢端肥大症系腺垂体分泌哪种内分泌激素细胞腺瘤或增生引起(2.5分) A.ACTH
B.LH
C.FSH
D.GH
E.TSH
D.GH
4 5 2011 37. 肢端肥大症中最多见于哪一段年龄(2.5分) A.31~40岁
B.21~30岁
C.41~50岁
D.51~60岁
E.61~70岁
A.31~40岁
4 5 2011 38. 肢端肥大症内分泌检查,生长激素一般大于(2.5分) A.5ug/L
B.20ug/L
C.10ug/L
D.15ug/L
E.30ug/L
B.20ug/L
4 5 2011 39. 肢端肥大症内分泌检查,葡萄糖抑制试验血浆GH谷值不被抑制到多少以下(2.5分) A.1ug/L
B.5ug/L
C.10ug/L
D.15ug/L
E.20ug/L
A.1ug/L
4 5 2011 40. 下列哪项疾病需与肢端肥大症鉴别(2.5分) A.糖尿病
B.甲状腺功能亢进
C.手足皮肤骨膜肥厚症
D.皮质醇增多症
E.泌乳素瘤
C.手足皮肤骨膜肥厚症
4 5 2012 1. 降钙素的主要靶器官是(2.5分) A.甲状腺
B.甲状旁腺
C.胃肠道
D.肾脏
E.骨组织
E.骨组织
4 5 2012 2. 下列哪种维生素能显著影响钙的吸收和代谢? (2.5分) A.维生素A
B.维生素B
C.维生素C
D.维生素D
E.维生素E
D.维生素D
4 5 2012 3. 影响PTH分泌最重要的因素是(2.5分) A.血磷浓度
B.血钙浓度
C.血镁浓度
D.血中儿茶酚胺浓度
E.血中前列腺素浓度
B.血钙浓度
4 5 2012 4. Which of the following is not involved in regulating plasma Ca2+ levels? (2.5分) A.Kidneys
B.Skin
C.Liver
D.Lungs
E.Intestine
D.Lungs
4 5 2012 5. I 型(绝经后)骨质疏松是什么型骨质疏松 (2.5分) A.高转换型
B.低转换型
C.平衡型
D.快速型
E.以上都错
A.高转换型
4 5 2012 6. 发生脆性骨折的常见部位为 (2.5分) A.胸腰椎
B.髋部
C.桡尺骨远端
D.肱骨近端
E.以上都是
E.以上都是
4 5 2012 7. 一个60岁妇女,测得骨密度T值为-3.4,全身检查无骨折,该病人诊断为(2.5分) A.正常
B.骨质疏松
C.骨量减少
D.严重骨质疏松
E.以上都错
B.骨质疏松
4 5 2012 8. 下列哪项指标是反映骨吸收敏感性较好的生化指标(2.5分) A.PINP
B.骨钙素
C.PICP
D.CTX
E.以上都是
D.CTX
4 5 2012 9. 影响钙磷代谢的因素下列哪项不是的(2.5分) A.甲状旁腺激素
B.维生素D
C.肾脏功能
D.心脏功能
E.肠道功能
D.心脏功能
4 5 2012 10. 低钙血症最具特征的临床表现是(2.5分) A.手足搐搦
B.肌痉挛
C.骨痛、骨折
D.心律不齐、心脏传导阻滞
E.认知能力减退
A.手足搐搦
4 5 2012 11. 低磷血症是指(2.5分) A.指血磷低于0.4mmol/L
B.指血磷低于0.6mmol/L
C.指血磷低于0.7mmol/L
D.指血磷低于1.0mmol/L
E.指血磷低于0.8mmol/L
E.指血磷低于0.8mmol/L
4 5 2012 12. 纠正高钙血症的治疗措施下列哪项是错误的(2.5分) A.二膦酸盐
B.输入生理盐水
C.速尿
D.双氢克尿噻
E.透析
D.双氢克尿噻
4 5 2012 13. 有关三发性甲旁亢的叙述正确的是(2.5分) A.有三个甲状旁腺腺体原发疾病引起的甲旁亢,叫三发性甲旁亢
B.临床上最常见的甲旁亢是三发性甲旁亢
C.恶性肿瘤转移至三个甲状旁腺引起的甲旁亢叫三发性甲旁亢
D.除了甲状旁腺外,还有另外两个内分泌腺出现病变,临床上以甲旁亢症状为主,叫三发性甲旁亢
E.在继发甲旁亢的基础上,由于腺体受到持久的刺激,自主性分泌过多的PTH,叫三发性甲旁亢
E.在继发甲旁亢的基础上,由于腺体受到持久的刺激,自主性分泌过多的PTH,叫三发性甲旁亢
4 5 2012 14. 抢救高钙危象时需应用(2.5分) A.糖皮质激素
B.酚妥拉明
C.尼群地平
D.大量生理盐水
E.大量10%葡萄糖
D.大量生理盐水
4 5 2012 15. 甲状旁腺功能减退症患者,静脉滴注200UPTH后,尿磷和尿cAMP不增加的是(2.5分) A.假性甲状旁腺功能减退症I型
B.假性甲状旁腺功能减退症II型
C.特发性甲状旁腺功能减退症
D.继发性甲状旁腺功能亢进症
E.低镁血症
A.假性甲状旁腺功能减退症I型
4 5 2012 16. 甲状旁腺素对血钙调节主要通过(2.5分) A.肠和胃
B.肝和胆
C.胰和胆
D.骨、肾和肠
E.脑垂体
D.骨、肾和肠
4 5 2012 17. 血管加压素(ArginineVasopressin,AVP)是由什么.部位?以下哪种细胞产生的? (2.5分) A.下丘脑弓状核的APUD细胞与AgRP细胞产生
B.垂体前叶内分泌细胞产生作用于下丘脑室旁核
C.下丘脑视上核心与室旁核神经元产生,沿神经垂体束下行至垂体后叶
D.第三脑室旁细胞产生沿神经垂体束下行至垂体后叶
E.侧脑室旁神经细胞产生
C.下丘脑视上核心与室旁核神经元产生,沿神经垂体束下行至垂体后叶
4 5 2012 18. 诊断尿崩症的初筛标准? (2.5分) A.尿量大于2500ml/day ,尿比重小于1.005
B.尿量大于4000ml/day,尿比重小于1.005
C.尿量大于4000ml/day ,尿比重小于1.010
D.尿量大于5000ml/day,尿比重小于1.008
E.尿量大于3000ml/day,尿比重大于1.005
B.尿量大于4000ml/day,尿比重小于1.005
4 5 2012 19. 下列哪项为最常见的垂体瘤? (2.5分) A.生长激素瘤
B.泌乳素瘤
C.促肾上腺皮质激素瘤
D.促性腺激素瘤
E.促甲状腺激素瘤
B.泌乳素瘤
4 5 2012 20. 下列哪现不是垂体分泌的激素?(2.5分) A.ACTH
B.TSH
C.LH
D.E
E.GH
D.E
4 5 2012 21. 于垂体瘤下列哪项正确?(2.5分) A.一定有某一激素升高或减少
B.一种腺瘤分泌一种激素
C.TSH瘤常见
D.药物治疗效果好
E.催乳素瘤首选药物治疗
E.催乳素瘤首选药物治疗
4 5 2012 22. 有关垂体微腺瘤下列哪项不正确?(2.5分) A.直径<10mm
B.PRL瘤最常见
C.一般采取经蝶显微外科手术切除腺瘤
D.需开颅经额途径切除腺瘤
E.最后诊断依赖于病理免疫细胞化学分析
D.需开颅经额途径切除腺瘤
4 5 2012 23. 腺垂体功能减退症最早出现的靶腺功能减退是(2.5分) A.肾上腺皮质功能减退
B.性腺功能减退
C.甲状腺功能减退
D.肾上腺与甲状腺功能减退
E.甲状腺与性腺功能减退
B.性腺功能减退
4 5 2012 24. 下列哪项对诊断腺垂体功能减退症无意义(2.5分) A.甲状旁腺素测定
B.甲状腺素测定
C.性腺激素测定
D.皮质醇测定
E.泌乳素测定
A.甲状旁腺素测定
4 5 2012 25. 男性,45岁,鼻咽癌放疗后出现性欲减退、阳痿、怕冷、思睡、思维迟钝、精神淡漠。该病人需考虑下列哪种情况: (2.5分) A.放疗引起甲状腺破坏,至甲状腺功能减退症
B.鼻咽癌复发
C.放疗破坏垂体组织,引起腺垂体功能减退症
D.植物神经功能紊乱
E.前列腺炎
C.放疗破坏垂体组织,引起腺垂体功能减退症
4 5 2012 26. 关于皮质醇增多症下述描述哪项最具诊断价值? (2.5分) A.血压增高
B.骨质疏松
C.葡萄糖耐量减低
D.向心性肥胖
E.明显的低血钾性碱中毒
D.向心性肥胖
4 5 2012 27. Cushing病是(2.5分) A.肾上腺皮质瘤产生大量糖皮质激素
B.垂体瘤分泌大量的ACTH, 引起肾上腺皮质增生
C.长期服用大量的糖皮质激素, 引起向心性肥胖
D.异位ACTH所致肾上腺皮质增生
E.不依赖ACTH的肾上腺瘤结节增生
B.垂体瘤分泌大量的ACTH, 引起肾上腺皮质增生
4 5 2012 28. 关于皮质醇增多症描述中,下列哪一项是错误的(2.5分) A.主要临床表现向心性肥胖, 高血压, 骨质疏松
B.肾上腺皮质增生少见
C.成人多于儿童
D.儿童患者腺癌多见
E.女性男性化明显者提示腺癌
B.肾上腺皮质增生少见
4 5 2012 29. 皮质醇增多症患者中,尿、血中皮质醇均可被大剂量地塞米松抑制试验所抑制,而不能被小剂量地塞米松所抑制,则病因最可能是(2.5分) A.肾上腺皮质腺瘤
B.肾上腺皮质腺癌
C.肾上腺髓质增生
D.肾上腺皮质增生
E.异位ACTH综合征
D.肾上腺皮质增生
4 5 2012 30. 女性,28岁,因摔伤骨折入院,皮肤菲薄,四肢有散在瘀斑。腹下侧可见紫纹,尿17-羟及血皮质醇均高于正常,综合以上临床表现,应首先考虑一下哪种病理生理现象? (2.5分) A.脂代谢障碍
B.蛋白质代谢障碍
C.糖代谢障碍
D.电解质紊乱
E.造血系统及血液改变
B.蛋白质代谢障碍
4 5 2012 31. 下列哪种病不属于肾上腺皮质疾病(2.5分) A.柯兴综合征
B.嗜铬细胞瘤
C.原发性醛固酮增多症
D.先天性11B-羟类固醇脱氢酶缺乏
E.阿狄森病
B.嗜铬细胞瘤
4 5 2012 32. 嗜铬细胞瘤患者的血糖、尿糖变化最可能的是?(2.5分) A.血糖、尿糖正常
B.低血糖
C.一过性血糖增高,尿糖阳性
D.血糖正常,尿糖阳性
E.血糖增高,尿糖阳性
E.血糖增高,尿糖阳性
4 5 2012 33. 嗜铬细胞瘤患者确诊后,下列哪项治疗不正确? (2.5分) A.酚苄明
B.心痛定
C.先用酚苄明,后用心得安
D.先用心得安,后用酚苄明
E.同时合用酚苄明和心得安
D.先用心得安,后用酚苄明
4 5 2012 34. 关于嗜铬细胞瘤哪项是错误的(2.5分) A.约90%嗜铬细胞瘤位于肾上腺髓质
B.80%以上为单侧腺瘤
C.90%为良性
D.儿童几乎不发生
E.可同时发生在肾上腺和肾上腺外组织
D.儿童几乎不发生
4 5 2012 35. 关于原醛的肾性失钾的证据以下哪个是正确的? (2.5分) A.钾的摄 入增加,尿钾增加
B.夏季出汗增加,尿钾排泄增加
C.血钾正常,尿钾排泄增加
D.血钾高,尿钾排泄增加
E.血钾低,尿钾排泄增加
E.血钾低,尿钾排泄增加
4 5 2012 36. 女性, 40岁, 多饮多尿, Bp160/100mmHg, 血pH7.5,血钾2.8mmol/L(2.8mEq/L),下列疾病诊断哪种可能性大(2.5分) A.原发性醛固酮增多症
B.尿崩症
C.糖尿病
D.嗜咯细胞瘤
E.甲亢
A.原发性醛固酮增多症
4 5 2012 37. 肢端肥大症系腺垂体分泌哪种内分泌激素细胞腺瘤或增生引起(2.5分) A.ACTH
B.LH
C.FSH
D.GH
E.TSH
D.GH
4 5 2012 38. 肢端肥大症内分泌检查,葡萄糖抑制试验血浆GH谷值不被抑制到多少以下(2.5分) A.5ug/L
B.12.5ug/L
C.10ug/L
D.15ug/L
E.20ug/L
A.5ug/L
4 5 2012 39. 经治疗后,GH一般控制指随机血GH小于(2.5分) A.5 ug/L
B.6 ug/L
C.8 ug/L
D.10 ug/L
E.12 ug/L
A.5 ug/L
4 5 2012 40. 下列哪种药物不能用于治疗肢端肥大症(2.5分) A.兰曲肽
B.奥曲肽
C.卡麦角林
D.降钙素
E.溴隐亭
D.降钙素
4 5 2013 1. 继发性尿崩症最常见的病因是(2.5分) A.下丘脑-神经垂体部位的肿瘤
B.头部创伤
C.脑部感染性疾病
D.脑血管病变
E.朗格汉斯细胞增生
A.下丘脑-神经垂体部位的肿瘤
4 5 2013 2. 关于禁水试验,错误的是(2.5分) A.禁水期间每2小时排尿一次
B.禁水期间每1小时测体重血压
C.禁水前测体重、血压、尿量、尿比重
D.禁水时间8-12小时
E.禁水时间12-24小时
E.禁水时间12-24小时
4 5 2013 3. 以下哪些药物不能用于治疗尿崩症(2.5分) A.氯磺丙脲
B.垂体后叶素
C.安妥明
D.双氢克尿塞
E.安体舒通
E.安体舒通
4 5 2013 4. 下列哪种激素减少可引起尿崩症(2.5分) A.生长激素
B.甲状旁腺素
C.精氨酸加压素
D.糖皮质激素
E.促肾上腺皮质激素
C.精氨酸加压素
4 5 2013 5. 腺垂体功能减退症最早出现的靶腺功能减退是(2.5分) A.肾上腺皮质功能减退
B.性腺功能减退
C.甲状腺功能减退
D.肾上腺与甲状腺功能减退
E.甲状腺与性腺功能减退
B.性腺功能减退
4 5 2013 6. 下列哪项对诊断腺垂体功能减退症无意义(2.5分) A.甲状旁腺素测定
B.甲状腺素测定
C.性腺激素测定
D.皮质醇测定
E.泌乳素测定
A.甲状旁腺素测定
4 5 2013 7. 垂体危象时,下列何种情况最为多见(2.5分) A.低血糖性昏迷
B.低钾性麻痹
C.谵妄
D.脑梗死
E.高钠高渗性昏迷
A.低血糖性昏迷
4 5 2013 8. 下列哪项为最常见的垂体瘤?(2.5分) A.生长激素瘤
B.泌乳素瘤
C.促肾上腺皮质激素瘤
D.促性腺激素瘤
E.促甲状腺激素瘤
B.泌乳素瘤
4 5 2013 9. 下列哪现不是垂体分泌的激素?(2.5分) A.ACTH
B.TSH
C.LH
D.E
E.GH
D.E
4 5 2013 10. 于垂体瘤下列哪项正确?(2.5分) A.一定有某一激素升高或减少
B.一种腺瘤分泌一种激素
C.TSH瘤常见
D.药物治疗效果好
E.催乳素瘤首选药物治疗
E.催乳素瘤首选药物治疗
4 5 2013 11. 关于催乳素瘤下列哪项不正确?(2.5分) A.为最常见的垂体瘤
B.多见于女性
C.多为微腺瘤
D.溴隐亭治疗效果好
E.停药易复发,故无论何种情况均不宜停药
E.停药易复发,故无论何种情况均不宜停药
4 5 2013 12. 肢端肥大症系腺垂体分泌哪种内分泌激素细胞腺瘤或增生引起(2.5分) A.ACTH
B.LH
C.FSH
D.GH
E.TSH
D.GH
4 5 2013 13. 肢端肥大症内分泌检查,生长激素一般大于(2.5分) A.5ug/L
B.20ug/L
C.10ug/L
D.15ug/L
E.30ug/L
B.20ug/L
4 5 2013 14. 下列哪项疾病需与肢端肥大症鉴别(2.5分) A.糖尿病
B.甲状腺功能亢进
C.手足皮肤骨膜肥厚症
D.皮质醇增多症
E.泌乳素瘤
C.手足皮肤骨膜肥厚症
4 5 2013 15. 经治疗后,GH一般控制指随机血GH小于(2.5分) A.5 ug/L
B.6 ug/L
C.8 ug/L
D.10 ug/L
E.12 ug/L
A.5 ug/L
4 5 2013 16. 下列哪种情况的生长激素瘤患者不宜选择药物治疗(2.5分) A.大腺瘤预计不能切除干净
B.大腺瘤没有压迫症状
C.大腺瘤患者不愿手术
D.大腺瘤有心肺功能不佳不能承受麻醉风险
E.大腺瘤术后有残余
B.大腺瘤没有压迫症状
4 5 2013 17. 原发性醛固酮增多症患者最常出现的症状是(2.5分) A.高血压
B.肌无力及周期性麻痹
C.肢端麻木
D.口渴、多饮、多尿
E.心律失常
A.高血压
4 5 2013 18. 关于原发性醛固酮增多症,最具诊断价值的是(2.5分) A.血ACTH水平
B.24h尿皮质醇水平
C.24h儿茶酚胺水平
D.血醛固酮水平和肾素活性
E.小剂量地塞米松抑制试验
D.血醛固酮水平和肾素活性
4 5 2013 19. 原醛症引起醛固酮分泌增多的病变部位是在 (2.5分) A.下丘脑
B.垂体
C.肾上腺皮质
D.肾上腺髓质
E.以上都不对
C.肾上腺皮质
4 5 2013 20. 原发性醛固酮增多症分型的金标准是(2.5分) A.醛固酮/肾素比值
B.AVS
C.肾上腺CT
D.生理盐水抑制试验
E.以上都不对
B.AVS
4 5 2013 21. 硫酸脱氢表雄酮主要是由下列哪个部位合成和分泌的(2.5分) A.肾脏
B.肾上腺皮质束状带
C.肾上腺皮质网状带
D.肾上腺皮质球状带
E.肾上腺髓质
C.肾上腺皮质网状带
4 5 2013 22. 醛固酮的主要作用是促进肾脏(2.5分) A.近曲小管保钠排钾
B.远曲小管保钠排钾
C.远曲小管保钾排钠
D.近曲小管保钾排钠
E.髓质保钠排钾
B.远曲小管保钠排钾
4 5 2013 23. 肾上腺素是由下列哪个部位分泌的(2.5分) A.肾脏
B.肾上腺皮质束状带
C.肾上腺皮质网状带
D.肾上腺皮质球状带
E.肾上腺髓质
E.肾上腺髓质
4 5 2013 24. 女性,45岁,主因咳嗽发热一周,昏迷半天急诊入院。既往Addison病史。昏迷前纳差、恶心、呕吐数次,为胃内容物。查体:血压80/50mmHg,心率110次/分,律齐,左下肺可闻及湿性啰音,全身皮肤黏膜色素沉着。该患者昏迷最可能的原因是(2.5分) A.低血糖昏迷
B.感染中毒性脑病
C.垂体危象
D.肺性脑病
E.肾上腺皮质功能不全危象
E.肾上腺皮质功能不全危象
4 5 2013 25. 题干同24,为抢救该病人,最需要的化验检查是(2.5分) A.头颅CT
B.心电图
C.血常规
D.血电解质和血糖
E.胸片
D.血电解质和血糖
4 5 2013 26. 影响PTH分泌最重要的因素是(2.5分) A.血磷浓度
B.血钙浓度
C.血镁浓度
D.血中儿茶酚胺浓度
E.血中前列腺素浓度
B.血钙浓度
4 5 2013 27. 降钙素的主要靶器官是(2.5分) A.甲状腺
B.甲状旁腺
C.胃肠道
D.肾脏
E.骨组织
E.骨组织
4 5 2013 28. 下列哪种激素可促进1,25-(OH)2-VD3形成(2.5分) A.PTH
B.T3
C.T4
D.CT
E.TSH
A.PTH
4 5 2013 29. 下列哪项属于甲状旁腺激素的作用?(2.5分) A.抑制骨钙入血
B.使血磷升高
C.抑制远曲小管和集合管对钙的重吸收
D.促进1,25-二羟维生素D3形成
E.促进磷酸盐的重吸收
D.促进1,25-二羟维生素D3形成
4 5 2013 30. 有关三发性甲旁亢的叙述正确的是(2.5分) A.有三个甲状旁腺腺体原发疾病引起的甲旁亢,叫三发性甲旁亢
B.临床上最常见的甲旁亢是三发性甲旁亢
C.恶性肿瘤转移至三个甲状旁腺引起的甲旁亢叫三发性甲旁亢
D.除了甲状旁腺外,还有另外两个内分泌腺出现病变,临床上以甲旁亢症状为主,叫三发性甲旁亢
E.在继发甲旁亢的基础上,由于腺体受到持久的刺激,自主性分泌过多的PTH,叫三发性甲旁亢
E.在继发甲旁亢的基础上,由于腺体受到持久的刺激,自主性分泌过多的PTH,叫三发性甲旁亢
4 5 2013 31. 抢救高钙危象时需应用(2.5分) A.糖皮质激素
B.酚妥拉明
C.尼群地平
D.大量生理盐水
E.大量10%葡萄糖
D.大量生理盐水
4 5 2013 32. 女性患者,因甲状腺肿大出现呼吸困难而行手术切除术,术后患者自觉指端和嘴边有麻木感、刺痛,一遇劳累便出现"抽搐"发作。该患者最有可能的诊断(2.5分) A.癫痫
B.甲状旁腺功能减退症
C.甲状腺功能减退症
D.植物神经功能紊乱
E.神经系统疾病
B.甲状旁腺功能减退症
4 5 2013 33. 题干同32,为明确诊断下一步应做的检查是(2.5分) A.测血PTH和血钙
B.测血磷
C.测血钠
D.测血钾
E.测血镁
A.测血PTH和血钙
4 5 2013 34. 甲状旁腺素对血钙调节主要通过(2.5分) A.肠和胃
B.肝和胆
C.胰和胆
D.骨、肾和肠
E.脑垂体
D.骨、肾和肠
4 5 2013 35. 有关原发性甲旁亢,下列错误的是(2.5分) A.该病主要的生化表现为高血钙、高尿钙,低血磷、低尿磷
B.PTH均增高明显
C.因维生素D充足而多合并佝偻病/骨软化症
D.腺癌患者多于术后诊断明确
E.B超和核素检查是其主要的定位诊断手段
C.因维生素D充足而多合并佝偻病/骨软化症
4 5 2013 36. 影响钙磷代谢的因素下列哪项不是的(2.5分) A.甲状旁腺激素
B.维生素D
C.肾脏功能
D.心脏功能
E.肠道功能
D.心脏功能
4 5 2013 37. 低钙血症最具特征的临床表现是(2.5分) A.手足搐搦
B.肌痉挛
C.骨痛、骨折
D.心律不齐、心脏传导阻滞
E.认知能力减退
A.手足搐搦
4 5 2013 38. 低磷血症是指(2.5分) A.指血磷低于0.4mmol/L
B.指血磷低于0.6mmol/L
C.指血磷低于0.7mmol/L
D.指血磷低于1.0mmol/L
E.指血磷低于0.8mmol/L
E.指血磷低于0.8mmol/L
4 5 2013 39. 下列哪项不是引起高磷血症的病因(2.5分) A.肾功能衰竭
B.甲状旁腺功能减退
C.维生素D缺乏
D.肉芽肿病
E.含磷酸盐泻剂或灌肠液
C.维生素D缺乏
4 5 2013 40. 维生素D中毒可引起高钙、高磷血症,主要的针对病因的治疗措施是(2.5分) A.大量补液
B.使用速尿等快速利尿
C.糖皮质激素的使用
D.降钙素
E.低钙饮食
C.糖皮质激素的使用
4 5 2014 1. 符合下列哪些条件建议检查骨密度 (2.5分) A.女性55岁以上
B.男性60岁以上
C.有骨折史
D.性激素水平低下
E.女性绝经后
D.性激素水平低下
4 5 2014 2. 发生脆性骨折的常见部位为 (2.5分) A.胸腰椎
B.髋部
C.桡尺骨远端
D.肱骨近端
E.以上都是
E.以上都是
4 5 2014 3. 目前国际学术界公认的骨质疏松症诊断的金标准(2.5分) A.超声
B.Q-CT
C.双能X线骨密度仪(DXA)
D.MRI
E.X线摄片
C.双能X线骨密度仪(DXA)
4 5 2014 4. 下列哪个不属于骨质疏松症的治疗药物(2.5分) A.基础补充
B.促进骨形成
C.抑制骨重吸
D.增加矿物质
E.中药
D.增加矿物质
4 5 2014 5. 原发性醛固酮增多症患者最常出现的症状是(2.5分) A.高血压
B.肌无力及周期性麻痹
C.肢端麻木
D.口渴、多饮、多尿
E.心律失常
A.高血压
4 5 2014 6. 关于原发性醛固酮增多症,最具诊断价值的是(2.5分) A.血ACTH水平
B.24h尿皮质醇水平
C.24h儿茶酚胺水平
D.血醛固酮水平和肾素活性
E.小剂量地塞米松抑制试验
D.血醛固酮水平和肾素活性
4 5 2014 7. 原醛伴严重低血钾的患者,在补钾后醛固酮变化是(2.5分) A.增多
B.降低
C.无变化
D.先增后降
E.先降后增
A.增多
4 5 2014 8. 原发性醛固酮增多症临床表现不包括(2.5分) A.多饮、多尿
B.水肿
C.低血钾
D.高血压
E.手足抽搐
B.水肿
4 5 2014 9. 原发性醛固酮增多症分型的金标准是(2.5分) A.醛固酮/肾素比值
B.AVS
C.肾上腺CT
D.生理盐水抑制试验
E.以上都不对
B.AVS
4 5 2014 10. 继发性尿崩症最常见的病因是(2.5分) A.下丘脑-神经垂体部位的肿瘤
B.头部创伤
C.脑部感染性疾病
D.脑血管病变
E.朗格汉斯细胞增生
A.下丘脑-神经垂体部位的肿瘤
4 5 2014 11. 中枢性尿崩症患者控制多尿最适宜的药物是(2.5分) A.垂体加压素水剂
B.油剂鞣酸加压素(长效尿崩停)
C.去氨加压素(弥凝)
D.氢氯噻嗪(双克)
E.氯磺丙脲
C.去氨加压素(弥凝)
4 5 2014 12. 关于禁水试验,错误的是(2.5分) A.禁水期间每2小时排尿一次
B.禁水期间每1小时测体重血压
C.禁水前测体重、血压、尿量、尿比重
D.禁水时间8-12小时
E.禁水时间12-24小时
E.禁水时间12-24小时
4 5 2014 13. 以下哪些药物不能用于治疗尿崩症(2.5分) A.氯磺丙脲
B.垂体后叶素
C.安妥明
D.双氢克尿塞
E.安体舒通
E.安体舒通
4 5 2014 14. Cushing病是(2.5分) A.肾上腺皮质瘤产生大量糖皮质激素
B.垂体瘤分泌大量的ACTH, 引起肾上腺皮质增生
C.长期服用大量的糖皮质激素引起向心性肥胖
D.异位ACTH所致肾上腺皮质增生
E.不依赖ACTH的肾上腺结节增生
B.垂体瘤分泌大量的ACTH, 引起肾上腺皮质增生
4 5 2014 15. 关于库欣综合征,下述描述哪项最具特征性? (2.5分) A.血压增高
B.骨质疏松
C.紫纹
D.低血钾
E.肥胖
C.紫纹
4 5 2014 16. 以下有关Nelson综合征的描述中,哪项是错误的(2.5分) A.多在Cushing病患者行双侧肾上腺切除术后发生
B.全身皮肤粘膜黑色素沉着
C.垂体功能低下
D.血浆ACTH增高。
E.可见垂体瘤
C.垂体功能低下
4 5 2014 17. 嗜铬细胞瘤患者最有特征性的表现是(2.5分) A.阵发性高血压
B.持续性高血压
C.向心性肥胖
D.高血压危象
E.血压忽高忽低,甚至有低血压
E.血压忽高忽低,甚至有低血压
4 5 2014 18. 库欣综合征患者出现下列情况时,应考虑异位激素分泌综合征的可能,除外(2.5分) A.血钾较低
B.皮肤色素沉着更为严重
C.病情进展迅速
D.伴有胸痛、咯血
E.伴有低钠血症
E.伴有低钠血症
4 5 2014 19. 下列哪项为最常见的垂体瘤?(2.5分) A.生长激素瘤
B.泌乳素瘤
C.促肾上腺皮质激素瘤
D.促性腺激素瘤
E.促甲状腺激素瘤
B.泌乳素瘤
4 5 2014 20. 下列哪现不是垂体分泌的激素?(2.5分) A.ACTH
B.TSH
C.LH
D.E
E.GH
D.E
4 5 2014 21. 关于垂体瘤分泌的激素下列哪项不正确?(2.5分) A.生长激素瘤分泌GH
B.催乳素瘤分泌PRL
C.促甲状腺激素瘤分泌TSH
D.促性腺激素瘤分泌FSH/LH
E.促肾上腺皮质激素瘤分泌ACTH、黑素细胞刺激素
A.生长激素瘤分泌GH
4 5 2014 22. 腺垂体功能减退症最早出现的靶腺功能减退是(2.5分) A.肾上腺皮质功能减退
B.性腺功能减退
C.甲状腺功能减退
D.肾上腺与甲状腺功能减退
E.甲状腺与性腺功能减退
B.性腺功能减退
4 5 2014 23. 下列哪项对诊断腺垂体功能减退症无意义(2.5分) A.甲状旁腺素测定
B.甲状腺素测定
C.性腺激素测定
D.皮质醇测定
E.泌乳素测定
A.甲状旁腺素测定
4 5 2014 24. 垂体危象时,下列何种情况最为多见(2.5分) A.低血糖性昏迷
B.低钾性麻痹
C.谵妄
D.脑梗死
E.高钠高渗性昏迷
A.低血糖性昏迷
4 5 2014 25. 影响钙磷代谢的因素下列哪项不是的(2.5分) A.甲状旁腺激素
B.维生素D
C.肾脏功能
D.心脏功能
E.肠道功能
D.心脏功能
4 5 2014 26. 低钙血症最具特征的临床表现是(2.5分) A.手足搐搦
B.肌痉挛
C.骨痛、骨折
D.心律不齐、心脏传导阻滞
E.认知能力减退
A.手足搐搦
4 5 2014 27. 低磷血症是指(2.5分) A.指血磷低于0.4mmol/L
B.指血磷低于0.6mmol/L
C.指血磷低于0.7mmol/L
D.指血磷低于1.0mmol/L
E.指血磷低于0.8mmol/L
E.指血磷低于0.8mmol/L
4 5 2014 28. 维生素D中毒可引起高钙、高磷血症,主要的针对病因的治疗措施是(2.5分) A.大量补液
B.使用速尿等快速利尿
C.糖皮质激素的使用
D.降钙素
E.低钙饮食
C.糖皮质激素的使用
4 5 2014 29. 影响钙磷代谢的因素下列哪项不是的(2.5分) A.甲状旁腺激素
B.维生素D
C.肾脏功能
D.心脏功能
E.肠道功能
D.心脏功能
4 5 2014 30. 低钙血症最具特征的临床表现是(2.5分) A.手足搐搦
B.肌痉挛
C.骨痛、骨折
D.心律不齐、心脏传导阻滞
E.认知能力减退
A.手足搐搦
4 5 2014 31. 低磷血症是指(2.5分) A.指血磷低于0.4mmol/L
B.指血磷低于0.6mmol/L
C.指血磷低于0.7mmol/L
D.指血磷低于1.0mmol/L
E.指血磷低于0.8mmol/L
E.指血磷低于0.8mmol/L
4 5 2014 32. 维生素D中毒可引起高钙、高磷血症,主要的针对病因的治疗措施是(2.5分) A.大量补液
B.使用速尿等快速利尿
C.糖皮质激素的使用
D.降钙素
E.低钙饮食
C.糖皮质激素的使用
4 5 2014 33. Calcitonin and parathyroid hormone both inhibit which of the following?(2.5分) A.Osteoblast activity in bones
B.Osteoclast activity in bones
C.Renal tubular calcium reabsorption
D.Renal tubular phosphate reabsorption
E.Calcium absorption in small intestine
D.Renal tubular phosphate reabsorption
4 5 2014 34. Which of the following increases the rate of excretion of calcium ions by the kidney?(2.5分) A.A decrease in calcitonin concentration in the plasma
B.An increase in phosphate ion concentration in the plasma
C.A decrease in the plasma level of PTH
D.Metabolic alkalosis
E.An increase in the plasma level of GH
C.A decrease in the plasma level of PTH
4 5 2014 35. Which of the following decreases the conversion of 25-hydroxycholecalciferol to 1,25-dihydroxycholecalciferol?(2.5分) A.A diet low in Ca2+
B.Hypocalcemia
C.Hyperparathyroidism
D.Hypophosphatemia
E.Chronic renal failure
E.Chronic renal failure
4 5 2014 36. 硫酸脱氢表雄酮主要是由下列哪个部位合成和分泌的(2.5分) A.肾脏
B.肾上腺皮质束状带
C.肾上腺皮质网状带
D.肾上腺皮质球状带
E.肾上腺髓质
C.肾上腺皮质网状带
4 5 2014 37. 以下不符合继发性肾上腺皮质功能不全的是(2.5分) A.血压偏低
B.皮肤变黑
C.空腹低血糖
D.恶心、纳差、腹胀、体重减轻
E.皮肤变白
B.皮肤变黑
4 5 2014 38. 肾上腺CT发现双侧肾上腺增大,不可能是以下病因(2.5分) A.肾上腺结核
B.肾上腺转移癌
C.肾上腺发育不良
D.先天性肾上腺皮质增生症
E.垂体ACTH腺瘤
C.肾上腺发育不良
4 5 2014 39. 肾上腺素是由下列哪个部位分泌的(2.5分) A.肾脏
B.肾上腺皮质束状带
C.肾上腺皮质网状带
D.肾上腺皮质球状带
E.肾上腺髓质
E.肾上腺髓质
4 5 2014 40. 肢端肥大症内分泌检查,葡萄糖抑制试验血浆GH谷值不被抑制到多少以下(2.5分) A.5ug/L
B.12.5ug/L
C.10ug/L
D.15ug/L
E.20ug/L
A.5ug/L
4 5 2015 1. A 52-year-old man with alcoholism, liver cirrhosis, and chronic fatigue presents with acute back pain and is diagnosed with a vertebral fracture. He also reports tingling and numbness of his skin. Ascites and a mild Trousseau sign are present. Insulin, glucagon, epinephrine, cortisol, growth hormone, thyroid hormone, and thyroid-stimulating hormone are all normal. Insulin-like growth factor 1 and reverse triiodothyronine (rT3) are reduced. Which of the following is most likely increased in this patient? (2.5分) A.Albumin
B.Bone resorption
C.Serum calcium
D.Serum glucose
E.1-25-dihydroxycholecalciferol
B.Bone resorption
4 5 2015 2. Calcitonin and parathyroid hormone both inhibit which of the following(2.5分) A.Osteoblast activity in bones
B.Osteoclast activity in bones
C.Renal tubular calcium reabsorption
D.Renal tubular phosphate reabsorption
E.Calcium absorption in small intestine
D.Renal tubular phosphate reabsorption
4 5 2015 3. Which of the following increases the rate of excretion of calcium ions by the kidney?(2.5分) A.A decrease in calcitonin concentration in the plasma
B.An increase in phosphate ion concentration in the plasma
C.A decrease in the plasma level of PTH
D.Metabolic alkalosis
E.An increase in the plasma level of GH
C.A decrease in the plasma level of PTH
4 5 2015 4. 有关三发性甲旁亢的叙述正确的是(2.5分) A.有三个甲状旁腺腺体原发疾病引起的甲旁亢,叫三发性甲旁亢
B.临床上最常见的甲旁亢是三发性甲旁亢
C.恶性肿瘤转移至三个甲状旁腺引起的甲旁亢叫三发性甲旁亢
D.除了甲状旁腺外,还有另外两个内分泌腺出现病变,临床上以甲旁亢症状为主,叫三发性甲旁亢
E.在继发甲旁亢的基础上,由于腺体受到持久的刺激,自主性分泌过多的PTH,叫三发性甲旁亢
E.在继发甲旁亢的基础上,由于腺体受到持久的刺激,自主性分泌过多的PTH,叫三发性甲旁亢
4 5 2015 5. 抢救高钙危象时需应用(2.5分) A.糖皮质激素
B.酚妥拉明
C.尼群地平
D.大量生理盐水
E.大量10%葡萄糖
D.大量生理盐水
4 5 2015 6. 甲状旁腺功能减退症患者,静脉滴注200UPTH后,尿磷和尿cAMP不增加的是(2.5分) A.假性甲状旁腺功能减退症I型
B.假性甲状旁腺功能减退症II型
C.特发性甲状旁腺功能减退症
D.继发性甲状旁腺功能亢进症
E.低镁血症
A.假性甲状旁腺功能减退症I型
4 5 2015 7. 有关家族性甲状旁腺功能亢进症,下列哪项叙述是错误的?(2.5分) A.家族性甲旁亢占所有甲旁亢的5%以下
B.家族性甲旁亢可表现为多个甲状旁腺增生,特别是MEN1
C.家族性甲旁亢术后易复发
D.家族性甲旁亢可有甲状旁腺恶性肿瘤
E.为防止甲旁亢复发,家族性甲旁亢患者的所有甲状旁腺均需切除
E.为防止甲旁亢复发,家族性甲旁亢患者的所有甲状旁腺均需切除
4 5 2015 8. 肢端肥大症系腺垂体的腺瘤或增生导致哪种泌激素过多分泌引起(2.5分) A.ACTH
B.LH
C.FSH
D.GH
E.TSH
D.GH
4 5 2015 9. 肢端肥大症内分泌检查,葡萄糖抑制试验血浆GH谷值不被抑制到()以下(2.5分) A.5ug/L
B.12.5ug/L
C.10ug/L
D.15ug/L
E.20ug/L
A.5ug/L
4 5 2015 10. 下列哪种药物不能用于治疗肢端肥大症(2.5分) A.兰曲肽
B.奥曲肽
C.卡麦角林
D.降钙素
E.溴隐亭
D.降钙素
4 5 2015 11. 经治疗后GH严格控制,指口服葡萄糖抑制试验GH谷值小于(2.5分) A.3 ug/L
B.2.5 ug/L
C.2 ug/L
D.1.5 ug/L
E.1 ug/L
E.1 ug/L
4 5 2015 12. Cushing病是(2.5分) A.肾上腺皮质瘤产生大量糖皮质激素
B.垂体瘤分泌大量的ACTH, 引起肾上腺皮质增生
C.长期服用大量的糖皮质激素引起向心性肥胖
D.异位ACTH所致肾上腺皮质增生
E.不依赖ACTH的肾上腺结节增生
B.垂体瘤分泌大量的ACTH, 引起肾上腺皮质增生
4 5 2015 13. 关于库欣综合征,下述描述哪项最具特征性?(2.5分) A.血压增高
B.骨质疏松
C.紫纹
D.低血钾
E.肥胖
C.紫纹
4 5 2015 14. 下列哪项不是库欣综合症的常见临床表现?(2.5分) A.骨质疏松
B.糖耐量减退
C.四肢肥胖
D.皮肤紫纹
E.高血压
C.四肢肥胖
4 5 2015 15. 库欣综合征患者中,血、尿皮质醇不能被小剂量地塞米松抑制,但可被大剂量地塞米松抑制,则病因最可能是(2.5分) A.肾上腺皮质腺瘤
B.肾上腺皮质腺癌
C.肾上腺髓质增生
D.Cushing病
E.肾上腺小结节样增生
D.Cushing病
4 5 2015 16. 以下有关Nelson综合征的描述中,哪项是错误的(2.5分) A.多在Cushing病患者行双侧肾上腺切除术后发生
B.全身皮肤粘膜黑色素沉着。
C.垂体功能低下
D.血浆ACTH增高。
E.可见垂体瘤
C.垂体功能低下
4 5 2015 17. 嗜铬细胞瘤患者最有特征性的表现是(2.5分) A.阵发性高血压
B.持续性高血压
C.向心性肥胖
D.高血压危象
E.血压忽高忽低,甚至有低血压
E.血压忽高忽低,甚至有低血压
4 5 2015 18. 对诊断嗜铬细胞瘤最可靠的指标是?(2.5分) A.尿VMA测定
B.血、尿间甲肾上腺测定
C.血浆肾上腺素测定
D.血浆儿茶酚胺测定
E.血浆多巴胺测定
B.血、尿间甲肾上腺测定
4 5 2015 19. 原发性醛固酮增多症患者最常出现的症状是(2.5分) A.高血压
B.肌无力及周期性麻痹
C.肢端麻木
D.口渴、多饮、多尿
E.心律失常
A.高血压
4 5 2015 20. 关于原发性醛固酮增多症,最具诊断价值的是(2.5分) A.血ACTH水平
B.24h尿皮质醇水平
C.24h儿茶酚胺水平
D.血醛固酮水平和肾素活性
E.小剂量地塞米松抑制试验
D.血醛固酮水平和肾素活性
4 5 2015 21. 原醛伴严重低血钾的患者,在补钾后醛固酮变化是(2.5分) A.增多
B.降低
C.无变化
D.先增后降
E.先降后增
A.增多
4 5 2015 22. 继发性尿崩症最常见的病因是(2.5分) A.脑部感染性疾病
B.头部创伤
C.下丘脑-神经垂体部位的肿瘤
D.脑血管病变
E.朗格汉斯细胞增生
C.下丘脑-神经垂体部位的肿瘤
4 5 2015 23. 关于禁水试验,错误的是(2.5分) A.禁水期间每2小时排尿一次
B.禁水期间每1小时测体重血压
C.禁水前测体重、血压、尿量、尿比重
D.禁水时间12-24小时
E.禁水时间8-12小时
D.禁水时间12-24小时
4 5 2015 24. 以下哪些药物不能用于治疗尿崩症(2.5分) A.氯磺丙脲
B.垂体后叶素
C.安妥明
D.双氢克尿塞
E.安体舒通
E.安体舒通
4 5 2015 25. 腺垂体功能减退症最早出现的靶腺功能减退是(2.5分) A.肾上腺皮质功能减退
B.性腺功能减退
C.甲状腺功能减退
D.肾上腺与甲状腺功能减退
E.甲状腺与性腺功能减退
B.性腺功能减退
4 5 2015 26. 关于腺垂体功能减退症患者的治疗,下列哪项不对(2.5分) A.给予左旋甲状腺素50~150μg/d
B.给予泼尼松5.0~7.5mg/d
C.性激素替代,可用人工月经周期
D.感染时糖皮质激素用量可适当加大
E.可放心使用镇静安眠药
E.可放心使用镇静安眠药
4 5 2015 27. 垂体危象时,下列何种情况最为多见(2.5分) A.低血糖性昏迷
B.低钾性麻痹
C.谵妄
D.脑梗死
E.高钠高渗性昏迷
A.低血糖性昏迷
4 5 2015 28. 下列哪项为最常见的垂体瘤?(2.5分) A.生长激素瘤
B.泌乳素瘤
C.促肾上腺皮质激素瘤
D.促性腺激素瘤
E.促甲状腺激素瘤
B.泌乳素瘤
4 5 2015 29. 于垂体瘤下列哪项正确?(2.5分) A.一定有某一激素升高或减少
B.一种腺瘤分泌一种激素
C.TSH瘤常见
D.药物治疗效果好
E.催乳素瘤首选药物治疗
E.催乳素瘤首选药物治疗
4 5 2015 30. 关于催乳素瘤下列哪项不正确?(2.5分) A.为最常见的垂体瘤
B.多见于女性
C.多为微腺瘤
D.溴隐亭治疗效果好
E.停药易复发,故无论何种情况均不宜停药
E.停药易复发,故无论何种情况均不宜停药
4 5 2015 31. 过量糖皮质激素的作用是(2.5分) A.抑制肝脏糖异生
B.抑制外周组织利用葡萄糖
C.抑制蛋白质分解
D.抑制脂肪合成
B.抑制外周组织利用葡萄糖
4 5 2015 32. 醛固酮的主要作用是促进肾脏(2.5分) A.近曲小管保钠排钾
B.远曲小管保钠排钾
C.远曲小管保钾排钠
D.近曲小管保钾排钠
E.髓质保钠排钾
B.远曲小管保钠排钾
4 5 2015 33. 肾上腺X线平片发现肾上腺区钙化影提示(2.5分) A.嗜铬细胞瘤
B.柯兴综合症
C.肾上腺结核
D.原发性醛固增多症
E.垂体ACTH腺瘤
C.肾上腺结核
4 5 2015 34. 肾上腺素是由下列哪个部位分泌的:(2.5分) A.肾脏
B.肾上腺皮质束状带
C.肾上腺皮质网状带
D.肾上腺皮质球状带
E.肾上腺髓质
E.肾上腺髓质
4 5 2015 35. 影响钙磷代谢的因素下列哪项不是的:(2.5分) A.甲状旁腺激素
B.维生素D
C.肾脏功能
D.心脏功能
E.肠道功能
D.心脏功能
4 5 2015 36. 纠正低钙搐搦首选:(2.5分) A.迅速输注生理盐水
B.静脉注射葡萄糖酸钙
C.口服钙剂和维生素D
D.应用利尿剂
E.应用镇静剂
B.静脉注射葡萄糖酸钙
4 5 2015 37. 下列哪项不是引起高磷血症的病因:(2.5分) A.肾功能衰竭
B.甲状旁腺功能减退
C.维生素D缺乏
D.肉芽肿病
E.含磷酸盐泻剂或灌肠液
C.维生素D缺乏
4 5 2015 38. 下列哪些可以诊断绝经后骨质疏松症(2.5分) A.脆性骨折
B.DXA测量的中轴骨骨密度T-值≤-2.0
C.骨密度测量符合低骨量(-2.5<T-值<-1.0)+脆性骨折
D.骨密度测量符合Z-值<-2.0
E.DXA测量的桡骨远端1/3骨密度的T-值≤-2.5
E.DXA测量的桡骨远端1/3骨密度的T-值≤-2.5
4 5 2015 39. 发生脆性骨折的常见部位为(2.5分) A.胸腰椎
B.髋部
C.桡尺骨远端
D.肱骨近端
E.以上都是
E.以上都是
4 5 2015 40. 骨质疏松症与骨组织内哪几种细胞有关(2.5分) A.骨髓前体细胞、破骨细胞、成骨细胞
B.破骨细胞、成骨细胞、骨衬细胞
C.破骨细胞、成骨细胞
D.软骨细胞、成骨细胞
E.成骨细胞、骨髓前体细胞
C.破骨细胞、成骨细胞
4 6 2006 1. 抑制素是由睾丸的哪种细胞分泌的? (4.0分) A.精原细胞
B.精母细胞
C.支持细胞
D.间质细胞
E.精子
C.支持细胞
4 6 2006 2. 输精管可分4部,下列哪一个不属于输精管的分部? (4.0分) A.精索部
B.睾丸部
C.盆部
D.腹股沟管部
E.输精管壶腹部
E.输精管壶腹部
4 6 2006 3. 促进睾丸间质细胞合成与分泌睾酮的激素是(4.0分) A.ACTH
B.TSH
C.FSH
D.LH
E.GnRH
D.LH
4 6 2006 4. 排卵前,血中LH出现高峰的原因是(4.0分) A.血中雌激素和孕激素共同的作用
B.血中雌激素对腺垂体的正反馈作用
C.血中孕激素对腺垂体的正反馈作用
D.FSH的促进作用
E.少量LH本身的短反馈作用
B.血中雌激素对腺垂体的正反馈作用
4 6 2006 5. 下列说法哪一个是错的? (4.0分) A.卵巢的固定装置有卵巢悬韧带、卵巢固有韧带
B.子宫附件是指卵巢和子宫阔韧带
C.阴道前庭是位于两侧小阴唇之间的裂隙
D.卵巢位于盆腔侧壁,髂内外动脉的夹角内
E.前庭大腺开口于阴道前庭
B.子宫附件是指卵巢和子宫阔韧带
4 6 2006 6. 较大剂量缩宫素可用于(3.0分) A.产后止血
B.催产
C.引产
D.泌乳
E.催产,引产
A.产后止血
4 6 2006 7. Which of the following explains the suppression of lactation during pregnancy? (4.0分) A.Blood levels of estrogen and progesterone are high
B.Human placental lactogen levels are too low for milk production to occur
C.The fetal adrenal gland does not produce sufficient estriol
D.Blood prolactin levels are too low for milk production to occur
E.The maternal anterior pituitary is suppressed
A.Blood levels of estrogen and progesterone are high
4 6 2006 8. 子宫圆韧带功能是: (3.0分) A.固定子宫颈
B.限制子宫向两侧移动
C.维持子宫前屈
D.防止子宫向下脱位
E.维持子宫前倾
E.维持子宫前倾
4 6 2006 9. 抑制素的化学性质是(3.0分) A.肽类
B.胺类
C.糖蛋白
D.类固醇
E.蛋白质
C.糖蛋白
4 6 2006 10. 雄激素结合蛋白是由下列哪种细胞产生的? (3.0分) A.睾丸支持细胞
B.肝细胞
C.睾丸生精细胞
D.睾丸间质细胞
E.睾丸毛细血管内皮细胞
A.睾丸支持细胞
4 6 2006 11. 短效和长效口服避孕药主要作用机制是 (3.0分) A.发挥杀灭精子的作用;同时,可形成黏液,阻碍精子运动
B.含有雌激素和孕激素,能协同作用而抑制排卵
C.拮抗孕激素活性,破坏蜕膜,诱发流产
D.主要使子宫内膜发生各种功能和形态变化,阻碍孕卵着床
E.拮抗雌激素受体,抑制或减弱雌激素的作用
B.含有雌激素和孕激素,能协同作用而抑制排卵
4 6 2006 12. 雌二醇的化学性质是(3.0分) A.肽类
B.胺类
C.类固醇
D.固醇类
E.蛋白质
C.类固醇
4 6 2006 13. 下列关于雌激素生理作用的描述,错误的是(3.0分) A.抑制子宫内膜增生、腺体分泌
B.促进阴道上皮细胞增生、角化
C.使输卵管平滑肌活动增强
D.刺激乳腺导管和结缔组织增生
E.促进肾小管对钠和水的重吸收
A.抑制子宫内膜增生、腺体分泌
4 6 2006 14. 男性输精管道不包括:(3.0分) A.精曲小管
B.输精管
C.附睾
D.射精管
E.男性尿道
A.精曲小管
4 6 2006 15. 男性尿道3个扩大是(3.0分) A.舟状窝、尿道膜部、尿道球部
B.尿道外囗、尿道膜部、尿道内囗
C.尿道前列腺部、尿道球部、舟状窝
D.舟状窝、尿道球部、尿道海绵体部
E.尿道内囗、尿道球部、舟状窝
C.尿道前列腺部、尿道球部、舟状窝
4 6 2006 16. Secretion of oxytocin is increased by(3.0分) A.Increased prolactin levels
B.Milk ejection
C.Increased extracellular fluid (ECF) volume
D.Dilation of the cervix
E.Increased serum osmolarity
D.Dilation of the cervix
4 6 2006 17. 过量缩宫素可致子宫平滑肌(3.0分) A.强直性收缩
B.舒张
C.持续性收缩
D.收缩
E.持续性强直收缩
E.持续性强直收缩
4 6 2006 18. 睾酮由睾丸的下列哪种细胞分泌?(3.0分) A.间质细胞
B.精原细胞
C.支持细胞
D.精母细胞
E.精子
A.间质细胞
4 6 2006 19. 下列哪个药物适用于催产和引产 (4.0分) A.Pituitrin(垂体后叶素)
B.Oxytocin(缩宫素)
C.Ergotoxine(麦角毒)
D.Ergometrine(麦角新碱)
E.Ergotamine(麦角胺)
B.Oxytocin(缩宫素)
4 6 2006 20. 孕激素类药物可用于治疗 (3.0分) A.先兆流产
B.消耗性疾病等引起的虚弱
C.再生障碍性贫血
D.诱发流产
E.卵巢功能不全和闭经
A.先兆流产
4 6 2006 21. 有关雄激素类药物特点的叙述,以下哪一项是错误的? (3.0分) A.促进蛋白质合成(同化作用),减少蛋白质分解(异化作用)
B.可用于功能性子宫出血的治疗
C.可抑制骨髓造血功能,使红细胞生成减少
D.女性长期应用后,可出现痤疮等男性化改变
E.有些药物可干扰肝内毛细胆管的排泄功能,引起胆汁淤积性黄疸
C.可抑制骨髓造血功能,使红细胞生成减少
4 6 2006 22. 小剂量缩宫素可用于 (3.0分) A.催产
B.产后止血
C.催产,泌乳
D.催产,引产
E.催产,止血
D.催产,引产
4 6 2006 23. 子宫平滑肌抑制药用于 (3.0分) A.治疗产后出血
B.治疗闭经
C.治疗痛经
D.催产
E.人工流产
C.治疗痛经
4 6 2006 24. 对oxytocin(缩宫素)的叙述哪一项不正确? (4.0分) A.小剂量可加强子宫的节律性收缩
B.大剂量引起子宫强直性收缩
C.其收缩性质与正常分娩相似
D.大剂量适用于催产
E.小剂量适用于催产和引产
D.大剂量适用于催产
4 6 2006 25. 下列有关孕激素作用的叙述,正确的是(3.0分) A.促进子宫内膜发生增生期变化
B.子宫发育
C.促进子宫内膜发生分泌期变化
D.促进并维持女性特征
E.促进子宫收缩
C.促进子宫内膜发生分泌期变化
4 6 2006 26. 育龄期女子基础体温的双相变化与下列哪种激素有关?(4.0分) A.甲状腺激素
B.孕激素
C.雌激素
D.LH
E.FSH
B.孕激素
4 6 2006 27. If a young woman has high plasma levels of T3, cortisol, and renin activity but her blood pressure is only slightly elevated and she has no symptoms or signs of thyrotoxicosis or Cushing's syndrome, the most likely explanation is that(3.0分) A.She has been treated with T3 and cortisol
B.She has been treated with TSH and ACTH
C.She is in the third trimester of pregnancy
D.She has an adrenocortical tumor
E.She has been subjected to chronic stress
C.She is in the third trimester of pregnancy
4 6 2006 28. 雌激素类的药理作用中,不包括 (3.0分) A.增强子宫平滑肌对缩宫素的敏感性
B.能明显促进蛋白质合成,减少蛋白质分解
C.使阴道上皮增生,浅表层细胞角化
D.促进女性性器官的发育和成熟,维持女性第二性征。
E.较大剂量时可抑制下丘脑-垂体系统释放GnRH
B.能明显促进蛋白质合成,减少蛋白质分解
4 6 2006 29. 麦角新碱不具有下述哪种作用(4.0分) A.收缩动脉
B.子宫兴奋
C.收缩静脉
D.大剂量收缩血管内皮细胞
E.阻断α 受体
E.阻断α 受体
4 6 2006 30. 麦角生物碱在临床不能用于(3.0分) A.偏头痛
B.催产,引产
C.加速产后子宫复原
D.治疗子宫出血
E.冬眠合剂的配伍成分
B.催产,引产
4 6 2007 1. 下列关于雌激素生理作用的描述,错误的是(2.0分) A.刺激乳腺导管和结缔组织增生
B.促进阴道上皮细胞增生、角化
C.抑制子宫内膜增生、腺体分泌
D.使输卵管平滑肌活动增强
E.促进肾小管对钠和水的重吸收
C.抑制子宫内膜增生、腺体分泌
4 6 2007 2. 新生儿的睾丸位于(2.0分) A.精索皮下部
B.髂窝
C.腹股沟管
D.腹后壁
E.阴囊
E.阴囊
4 6 2007 3. 男性输精管道不包括:(2.0分) A.输精管
B.附睾
C.精曲小管
D.射精管
E.男性尿道
C.精曲小管
4 6 2007 4. 下列哪项检查能够最终确定病人感染了艾滋病毒: (2.0分) A.HIV抗体筛查试验
B.HIV抗体确诊试验
C.血常规
D.尿常规
B.HIV抗体确诊试验
4 6 2007 5. 子宫阔韧带功能是: (2.0分) A.维持子宫前屈
B.限制子宫向两侧移动
C.防止子宫向下脱位
D.固定子宫颈
E.维持子宫前倾
B.限制子宫向两侧移动
4 6 2007 6. 抑制素对下列哪种腺垂体激素的分泌具有很强的抑制作用? (2.0分) A.TSH
B.LH
C.FSH
D.PRI
E.GH
C.FSH
4 6 2007 7. 人乳头瘤病毒主要感染哪个部位?(2.0分) A.真皮
B.上皮
C.血液
D.结缔组织
E.淋巴
B.上皮
4 6 2007 8. 促进睾丸间质细胞合成与分泌睾酮的激素是(2.0分) A.ACTH
B.FSH
C.LH
D.TSH
E.GnRH
C.LH
4 6 2007 9. 抑制素是由睾丸的哪种细胞分泌的? (2.0分) A.间质细胞
B.支持细胞
C.精母细胞
D.精原细胞
E.精子
B.支持细胞
4 6 2007 10. 扁平湿疣是:(2.0分) A.二期梅毒
B.一期梅毒
C.三期梅毒
D.人类乳头瘤病毒感染
E.衣原体感染
A.二期梅毒
4 6 2007 11. 精子获能的主要部位是(2.0分) A.阴道
B.睾丸曲细精管
C.子宫颈
D.附睾
E.子宫腔
E.子宫腔
4 6 2007 12. 排卵前,血中LH出现高峰的原因是(2.0分) A.FSH的促进作用
B.血中孕激素对腺垂体的正反馈作用
C.血中雌激素对腺垂体的正反馈作用
D.血中雌激素和孕激素共同的作用
E.少量LH本身的短反馈作用
C.血中雌激素对腺垂体的正反馈作用
4 6 2007 13. Which of the following explains the suppression of lactation during pregnancy? (2.0分) A.Blood levels of estrogen and progesterone are high
B.Human placental lactogen levels are too low for milk production to occur
C.The fetal adrenal gland does not produce sufficient estriol
D.Blood prolactin levels are too low for milk production to occur
E.The maternal anterior pituitary is suppressed
A.Blood levels of estrogen and progesterone are high
4 6 2007 14. 黄体酮治疗先兆流产,必须肌肉注射的主要理由是:(2.0分) A.口服吸收缓慢
B.肌肉注射吸收迅速
C.口服给药排泄快
D.肌肉注射能维持较高浓度
E.口服后在胃肠及肝脏迅速破坏
E.口服后在胃肠及肝脏迅速破坏
4 6 2007 15. 下列关于月经的叙述,哪一项是错误的? (2.0分) A.出血期中,GnRH分泌处于较低水平
B.黄体退化,雌激素和孕激素分泌骤减
C.溶酶体释出蛋白水解酶,使组织溶解,内膜剥脱、出血
D.前列腺素F2 释放,子宫内膜血管痉挛
E.出血期中由于雌激素和孕激素的正反馈,FSH和LH分泌增加
E.出血期中由于雌激素和孕激素的正反馈,FSH和LH分泌增加
4 6 2007 16. 雌激素和孕激素作用的相同点是(2.0分) A.使子宫内膜增生变厚
B.促进乳腺导管增生和延长
C.使子宫输卵管平滑肌活动减弱
D.促进阴道上皮细胞角化
E.减少宫颈粘液的分泌
A.使子宫内膜增生变厚
4 6 2007 17. hCG的化学结构和生理作用与下列哪种激素基本相似? (2.0分) A.PRL
B.hCS
C.FSH
D.LH
E.GnRH
D.LH
4 6 2007 18. 关于氯米芬的叙述,下列哪项是正确的?(2.0分) A.可用于卵巢肿的治疗
B.主要用于不孕症的治疗
C.阻断下丘脑的雌激素受体,从而增强雌二醇的负反馈性抑制
D.抑制卵巢雌激素合成,发挥抗雌激素作用
E.激动下丘脑的雌激素受体,从而消除雌二醇的负反馈性抑制
B.主要用于不孕症的治疗
4 6 2007 19. 育龄期女子基础体温的双相变化与下列哪种激素有关? (2.0分) A.甲状腺激素
B.孕激素
C.雌激素
D.LH
E.FSH
B.孕激素
4 6 2007 20. The source of estrogen during the second and third trimesters of pregnancy is the(2.0分) A.Fetal adrenal gland, fetal liver, and placenta
B.Corpus luteum
C.Fetal ovaries
D.Placenta
E.Maternal ovaries and fetal adrenal gland
A.Fetal adrenal gland, fetal liver, and placenta
4 6 2007 21. 能促进支持细胞分泌抑制素的激素是(2.0分) A.LH
B.ADH
C.FSHD
D.ACTH
E.FSH
E.FSH
4 6 2007 22. 当HIV侵入机体后,未进入发病期者被称为: (2.0分) A.窗口期病人
B.艾滋病病毒感染者
C.艾滋病病人
D.艾滋病期病人
B.艾滋病病毒感染者
4 6 2007 23. If a young woman has high plasma levels of T3, cortisol, and renin activity but her blood pressure is only slightly elevated and she has no symptoms or signs of thyrotoxicosis or Cushing's syndrome, the most likely explanation is that(2.0分) A.She has been treated with T3 and cortisol
B.She has been treated with TSH and ACTH
C.She is in the third trimester of pregnancy
D.She has an adrenocortical tumor
E.She has been subjected to chronic stress
C.She is in the third trimester of pregnancy
4 6 2007 24. 关于淋病,以下哪项不对。 (2.0分) A.淋菌对柱状上皮和移行上皮有亲和力
B.淋病的传播途径主要为性交经粘膜感染
C.淋病是由革兰染色阴性的淋病奈氏菌引起的
D.淋病属于性传播疾病
E.宫颈分泌物涂片找到淋球菌,是诊断淋病的金标准
E.宫颈分泌物涂片找到淋球菌,是诊断淋病的金标准
4 6 2007 25. 下列哪项不是急性HIV感染的临床表现: (2.0分) A.淋巴结肿大
B.咽痛
C.发热
D.口腔真菌感染
D.口腔真菌感染
4 6 2007 26. 关于子宫位置的描述,下列哪一条是错的? (2.0分) A.前屈是指子宫底与子宫体之间形成一个向前开放的钝角
B.位于膀胱与直肠之间
C.前倾是指子宫与阴道之间形成一个向前开放的钝角.
D.位于盆腔中央
E.两侧连有输卵管和卵巢
A.前屈是指子宫底与子宫体之间形成一个向前开放的钝角
4 6 2007 27. 抑制排卵避孕药的较常见的不良反应是:(2.0分) A.类早孕反应
B.哺乳妇女乳汁减少
C.闭经
D.子宫不规则出血
E.乳房肿块
D.子宫不规则出血
4 6 2007 28. 艾滋病常见合并症一般不包括:(2.0分) A.持续性或间歇性慢性腹泻
B.复发性单纯疤疹
C.皮肤瘙痒
D.急性肾炎
D.急性肾炎
4 6 2007 29. 下列说法哪一个是错的? (2.0分) A.子宫附件是指卵巢和子宫阔韧带
B.卵巢位于盆腔侧壁,髂内外动脉的夹角内
C.阴道前庭是位于两侧小阴唇之间的裂隙
D.卵巢的固定装置有卵巢悬韧带、卵巢固有韧带
E.前庭大腺开口于阴道前庭
A.子宫附件是指卵巢和子宫阔韧带
4 6 2007 30. 世界卫生组织推荐的诊断艾滋病的主要症状是: (2.0分) A.持续性或间歇性慢性腹泻1个月以上
B.发热1个月以上
C.体重减轻10%以上
D.体重减轻10%以上,发热1个月以上和持续性或间歇性慢性腹泻1个月以上
D.体重减轻10%以上,发热1个月以上和持续性或间歇性慢性腹泻1个月以上
4 6 2007 31. 艾滋病的特异性治疗是:(2.0分) A.抗反转录酶抑制剂治疗
B.高效抗反转录病毒治疗
C.蛋白酶抑制剂治疗
D.融合抑制剂治疗
B.高效抗反转录病毒治疗
4 6 2007 32. 雌激素的临床用途有:(2.0分) A.先兆流产
B.功能性子宫出血
C.消耗性疾病
D.痛经
E.绝经期前的乳腺癌
B.功能性子宫出血
4 6 2007 33. 艾滋病的全称是: (2.0分) A.先天性免疫缺陷综合征
B.获得性免疫缺陷综合征
C.免疫缺陷综合征
D.继发性免疫缺陷综合征
B.获得性免疫缺陷综合征
4 6 2007 34. 月经期,血中激素浓度(2.0分) A.LH先低后高
B.孕激素低
C.雌激素低
D.FSH先低后高
E.以上都是
E.以上都是
4 6 2007 35. 测定血或尿中哪种激素可诊断早期妊娠? (2.0分) A.孕激素
B.雌激素
C.hCG
D.FSH
E.ACTH
C.hCG
4 6 2007 36. 复方炔诺酮片的主要避孕作用机制是:(2.0分) A.抑制子宫内膜的正常增殖,不利于受精卵着床
B.使子宫颈粘液变稠,精子不易进入子宫腔
C.抑制子宫和输卵管活动,改变受精卵运行速度
D.通过负反馈机制抑制排卵
E.抑制卵巢黄体分泌激素
D.通过负反馈机制抑制排卵
4 6 2007 37. 输卵管分四部: 下列哪一个不属于输卵管的分部? (2.0分) A.输卵管伞
B.输卵管子宫部
C.输卵管峡
D.输卵管壶腹
E.输卵管漏斗
A.输卵管伞
4 6 2007 38. 大剂量孕激素的适应症是:(2.0分) A.功能性子宫出血
B.子宫内膜异位症
C.子宫内膜腺癌
D.痛经
E.先兆流产和习惯性流产
C.子宫内膜腺癌
4 6 2007 39. 关于雌激素作用,下列叙述哪项不正确?(2.0分) A.促使子宫内膜由增殖期转为分泌期
B.促进女性性器官发育成熟
C.较大剂量抑制促性腺激素释放激素的分泌
D.提高子宫平滑肌对缩宫素的敏感性
E.抑制乳汁分泌
A.促使子宫内膜由增殖期转为分泌期
4 6 2007 40. 睾酮由睾丸的下列哪种细胞分泌?(2.0分) A.支持细胞
B.精母细胞
C.间质细胞
D.精原细胞
E.精子
C.间质细胞
4 6 2007 41. 下列哪项不是艾滋病的传播途径: (2.0分) A.性传播
B.血液传播
C.母婴垂直传播
D.虫媒传播
D.虫媒传播
4 6 2007 42. 成人感染梅毒,早期未发现症状,4年后也未出现心血管和中枢神经等症状,查血梅毒血清反应为阳性。称__。(2.0分) A.三期梅毒
B.早期隐性梅毒
C.二期梅毒
D.胎传梅毒
E.晚期隐性梅毒
E.晚期隐性梅毒
4 6 2007 43. 新生儿淋菌性眼炎主要的感染途径?(2.0分) A.哺乳
B.产道传染
C.接触被分泌物污染的物品
D.自我接种
E.都不是
B.产道传染
4 6 2007 44. 晚期胎传梅毒的症状多发于__。(2.0分) A.中年期
B.老年期
C.婴儿期
D.儿童及青春期
E.妊娠期
D.儿童及青春期
4 6 2007 45. 卵巢功能低下可选用(2.0分) A.甲基睾丸素
B.黄体酮
C.已烯雌酚
D.强的松龙
E.炔诺酮
C.已烯雌酚
4 6 2007 46. 急性淋病的平均潜伏期是: (2.0分) A.3w
B.3m
C.2w
D.3-5d
E.6m
D.3-5d
4 6 2007 47. 男性,28岁。两周来全身出现散在玫瑰色甲盖大的红斑,累及躯干、四肢掌跖。不痒。肛门附近有半环形排列的湿性丘疹,表面浸渍状。全身淋巴结肿大。应考虑__。 (2.0分) A.二期梅毒
B.多形红斑
C.三期梅毒
D.药物疹
E.念珠菌感染
A.二期梅毒
4 6 2007 48. 尿道内尖锐湿疣最佳治疗方法是:(2.0分) A.微波治疗
B.CO2激光治疗
C.艾拉-光动力治疗
D.液氮冷冻
E.鬼臼毒素治疗
C.艾拉-光动力治疗
4 6 2007 49. 当前我国性传播疾病中居首位的是:(2.0分) A.尖锐湿疣
B.淋病
C.梅毒
D.生殖器疱疹
E.非淋菌性尿道炎
B.淋病
4 6 2007 50. 肛周尖锐湿疣最需要和以下哪种疾病相鉴别?(2.0分) A.扁平湿疣
B.阴茎珍珠样丘疹病
C.肛周湿疹
D.皮脂腺异位
E.痔疮
A.扁平湿疣
4 6 2008 1. 雌激素的临床用途有:(3.0分) A. 痛经
B.功能性子宫出血
C.消耗性疾病
D.先兆流产
E.绝经期前的乳腺癌
B.功能性子宫出血
4 6 2008 2. 黄体酮治疗先兆流产,必须肌肉注射的主要理由是:(3.0分) A.口服吸收缓慢
B.口服给药排泄快
C.肌肉注射吸收迅速
D.肌肉注射能维持较高浓度
E.口服后在胃肠及肝脏迅速破坏
E.口服后在胃肠及肝脏迅速破坏
4 6 2008 3. 关于氯米芬的叙述,下列哪项是正确的?(3.0分) A.抑制卵巢雌激素合成,发挥抗雌激素作用
B.可用于卵巢肿的治疗
C.主要用于不孕症的治疗
D.阻断下丘脑的雌激素受体,从而增强雌二醇的负反馈性抑制
E. 激动下丘脑的雌激素受体,从而消除雌二醇的负反馈性抑制
C.主要用于不孕症的治疗
4 6 2008 4. 抑制排卵避孕药的较常见的不良反应是:(3.0分) A.子宫不规则出血
B.闭经
C.类早孕反应
D.哺乳妇女乳汁减少
E. 乳房肿块
A.子宫不规则出血
4 6 2008 5. 抗着床避孕药的主要优点是:(3.0分) A.不受月经周期的限制
B.每月只需服用1次
C.可替代抑制排卵的避孕药
D. 民居14天以内服用2片即可
E.以上都不是
A.不受月经周期的限制
4 6 2008 6. 复方炔诺酮片的主要避孕作用机制是:(3.0分) A.通过负反馈机制抑制排卵
B. 抑制子宫内膜的正常增殖,不利于受精卵着床
C.使子宫颈粘液变稠,精子不易进入子宫腔
D.抑制子宫和输卵管活动,改变受精卵运行速度
E. 抑制卵巢黄体分泌激素
A.通过负反馈机制抑制排卵
4 6 2008 7. 手术后慢性消耗性疾病病人蛋白质吸收不足时可服用:(3.0分) A.泼尼松龙
B.去氧皮质酮
C.苯丙酸诺龙
D.已烯雌酚
E.炔诺酮
C.苯丙酸诺龙
4 6 2008 8. 治疗再生障碍性贫血可用:(3.0分) A.雌激素类药
B.雄激素类药
C.同化激素类药
D.孕激素类药
E. 盐皮质激素类药
B.雄激素类药
4 6 2008 9. 关于雌激素作用,下列叙述哪项不正确?(3.0分) A.促进女性性器官发育成熟
B.促使子宫内膜由增殖期转为分泌期
C. 较大剂量抑制促性腺激素释放激素的分泌
D. 提高子宫平滑肌对缩宫素的敏感性
E. 抑制乳汁分泌
B.促使子宫内膜由增殖期转为分泌期
4 6 2008 10. 关于雄激素作用,下列叙述哪项不正确?(3.0分) A.促进男性性征和生殖器官的发育
B.抗雌激素作用
C.抑制蛋白质合成
D. 抑制垂体前叶分泌促性腺激素
E.大剂量促进骨髓造血功能
C.抑制蛋白质合成
4 6 2008 11. 艾滋病的全称是: (4.0分) A.免疫缺陷综合征
B.获得性免疫缺陷综合征
C.先天性免疫缺陷综合征
D.继发性免疫缺陷综合征
B.获得性免疫缺陷综合征
4 6 2008 12. HIV感染人体后主要侵犯和破坏哪种细胞: (3.0分) A.红细胞
B.白细胞
C.T淋巴细胞
D.巨噬细胞
C.T淋巴细胞
4 6 2008 13. 艾滋病病毒感染人体后临床症状分期错误的提法是: (3.0分) A.急性感染期
B.病毒携带期
C.无症状感染期
D.艾滋病期
B.病毒携带期
4 6 2008 14. 下列哪项检查能够最终确定病人感染了艾滋病毒: (3.0分) A.血常规
B.HIV抗体确诊试验
C.HIV抗体筛查试验
D.尿常规
B.HIV抗体确诊试验
4 6 2008 15. 艾滋病常见合并症一般不包括: (3.0分) A.复发性单纯疤疹
B.持续性或间歇性慢性腹泻
C.皮肤瘙痒
D.急性肾炎
D.急性肾炎
4 6 2008 16. 艾滋病人体体液含有大量病毒,具有很强传染性的是: (3.0分) A.血液
B.尿液
C.泪液
D.汗液
A.血液
4 6 2008 17. 世界卫生组织推荐的诊断艾滋病的主要症状是:)(3.0分) A.体重减轻10%以上
B.发热1个月以上
C.持续性或间歇性慢性腹泻1个月以上
D.体重减轻10%以上,发热1个月以上和持续性或间歇性慢性腹泻1个月以上
D.体重减轻10%以上,发热1个月以上和持续性或间歇性慢性腹泻1个月以上
4 6 2008 18. 晚期胎传梅毒的症状多发于(3.0分) A.儿童及青春期
B.婴儿期
C.中年期
D.老年期
E.妊娠期
A.儿童及青春期
4 6 2008 19. 哪些皮肤粘膜疹有传染性?(3.0分) A.扁平湿疣
B.滤泡性溃疡
C.树胶样肿
D.扁平苔藓
E.多形红斑
A.扁平湿疣
4 6 2008 20. 梅毒治疗的首选药物是(3.0分) A.螺旋霉素
B.长效青霉素
C.四环素
D.庆大霉素
E.氯霉素
B.长效青霉素
4 6 2008 21. 下列梅毒螺旋体检查方法中哪些是正确的(3.0分) A.暗视野法
B.培养
C.涂片做PAS染色
D.苛性钾法
E.荧光染色
A.暗视野法
4 6 2008 22. 一期梅毒的诊断标准不包括下列哪项 (3.0分) A.有不洁性交史
B.早期有流感症状
C.外生殖器单个无痛性下疳
D.潜伏期3周左右
E. 梅毒血清试验阳性
B.早期有流感症状
4 6 2008 23. 下列不见于二期梅毒表现的是. (3.0分) A.梅毒玫瑰疹
B.扁平湿疣
C.树胶肿
D.粘膜损害
E.梅毒性脱发
C.树胶肿
4 6 2008 24. 关于一期梅毒,哪项是错误的?(3.0分) A.一般患者起病前有不洁性交史
B.潜伏期是 2~4 周左右
C.硬下疳是主要的表现
D.在硬下疳处取材以暗视野检查可见苍白螺旋体
E.梅毒血清试验阴性可排除一期梅毒
E.梅毒血清试验阴性可排除一期梅毒
4 6 2008 25. 梅毒的传播途径不包括: (3.0分) A.胎盘
B.输血
C.呼吸道
D.间接接触
E.产道
C.呼吸道
4 6 2008 26. 治疗梅毒时为避免发生吉海反应加用:(3.0分) A.仙特敏
B.维生素B6
C.西咪替丁
D.泼尼松
E.葡萄糖酸钙
D.泼尼松
4 6 2008 27. 下列描述哪项错误:(3.0分) A.未经治疗的硬下疳都会发展为三期梅毒
B.二期梅毒传染性强
C.梅毒性秃发为不可逆性秃发
D.接吻也可传播梅毒
E.梅毒血清固定没有传染性
C.梅毒性秃发为不可逆性秃发
4 6 2008 28. 经典性病不包括: (3.0分) A.淋病
B.梅毒
C.尖锐湿疣
D.性病性淋巴肉芽肿
E.软下疳
C.尖锐湿疣
4 6 2008 29. 淋病治疗首先药物是:(3.0分) A.青霉素
B.多烯环素
C.环丙沙星
D.氧氟沙星
E.头孢曲松
E.头孢曲松
4 6 2008 30. 有关小儿淋病的描述哪项是不正确的?(3.0分) A.男性患儿多于女性;
B.发病率升高与父母感染淋病有关;
C.治疗首选头孢曲松针;
D.临床上多表现不结膜炎、尿道炎和阴道炎;
E.好发年龄多为0-1岁患儿。
A.男性患儿多于女性;
4 6 2008 31. 有关淋球菌耐药情况的描述,哪项是不正确的?(3.0分) A.大多对青霉素耐药;
B.对头孢曲松敏感性降低,有少数耐药株出现;
C.头孢曲松耐药株出现主要由质粒介导引起;
D.大多对四环素耐药;
E.大多对氟喹诺酮类耐药。
C.头孢曲松耐药株出现主要由质粒介导引起;
4 6 2008 32. 尿道内尖锐湿疣最佳治疗方法是:(3.0分) A.液氮冷冻
B.CO2激光治疗
C.艾拉-光动力治疗
D.微波治疗
E.鬼臼毒素治疗
C.艾拉-光动力治疗
4 6 2008 33. 最常见的高危型HPV是以下哪型(3.0分) A.HPV16、HPV18
B.HPV6、HPV11
C.HPV6、HPV31
D.HPV11、HPV18
E.HPV11、HPV32
A.HPV16、HPV18
4 6 2009 1. 男性尿道的特点,正确的是(2.0分) A.有两个狭窄
B.有三个弯曲
C.膜部最宽
D.耻骨前弯恒定不变
E.尿道前列腺部有射精管开口
E.尿道前列腺部有射精管开口
4 6 2009 2. 女性生殖系统的生殖腺是 (2.0分) A.卵巢
B.输卵管
C.子宫
D.阴道
E.前庭大腺
A.卵巢
4 6 2009 3. 下列答案中属于女性外生殖器的是 (2.0分) A.卵巢
B.子宫
C.尿道
D.女阴
E.阴道
D.女阴
4 6 2009 4. 子宫附件的组成是 (2.0分) A.输卵管和阴道
B.输卵管和前庭大腺
C.输送管道和附属腺体
D.内生殖器中除子宫以外的部分
E.输卵管和卵巢
E.输卵管和卵巢
4 6 2009 5. 一青年女子下腹部剧痛2小时为主诉人院,经查为输卵管宫外孕。输卵管部位的宫外孕最易发生于何处? (2.0分) A.输卵管子宫部
B.输卵管峡
C.输卵管壶腹
D.输卵管漏斗
E.输卵管子宫口附近
C.输卵管壶腹
4 6 2009 6. 睾酮由睾丸的下列哪种细胞分泌? (2.0分) A.间质细胞
B.支持细胞
C.精原细胞
D.精母细胞
E.精子
A.间质细胞
4 6 2009 7. 雄激素结合蛋白是由下列哪种细胞产生的? (2.0分) A.肝细胞
B.睾丸间质细胞
C.睾丸支持细胞
D.睾丸生精细胞
E.睾丸毛细血管内皮细胞
C.睾丸支持细胞
4 6 2009 8. 抑制素是由睾丸的哪种细胞分泌的? (2.0分) A.间质细胞
B.支持细胞
C.精原细胞
D.精母细胞
E.精子
B.支持细胞
4 6 2009 9. 能促进支持细胞分泌抑制素的激素是(2.0分) A.LH
B.ACTH
C.FSHD
D.ADH
E.FSH
E.FSH
4 6 2009 10. 下列关于雌激素生理作用的描述,错误的是(2.0分) A.使输卵管平滑肌活动增强
B.促进阴道上皮细胞增生、角化
C.抑制子宫内膜增生、腺体分泌
D.刺激乳腺导管和结缔组织增生
E.促进肾小管对钠和水的重吸收
C.抑制子宫内膜增生、腺体分泌
4 6 2009 11. 下列有关孕激素作用的叙述,正确的是(2.0分) A.促进子宫内膜发生增生期变化
B.促进子宫内膜发生分泌期变化
C.子宫发育
D.促进并维持女性特征
E.促进子宫收缩
B.促进子宫内膜发生分泌期变化
4 6 2009 12. 雌激素和孕激素作用的相同点是(2.0分) A.促进阴道上皮细胞角化
B.促进乳腺导管增生和延长
C.使子宫输卵管平滑肌活动减弱
D.使子宫内膜增生变厚
E.减少宫颈粘液的分泌
D.使子宫内膜增生变厚
4 6 2009 13. 下列关于月经的叙述,哪一项是错误的? (2.0分) A.出血期中,GnRH分泌处于较低水平
B.黄体退化,雌激素和孕激素分泌骤减
C.前列腺素F2 释放,子宫内膜血管痉挛
D.溶酶体释出蛋白水解酶,使组织溶解,内膜剥脱、出血
E.出血期中由于雌激素和孕激素的正反馈,FSH和LH分泌增加
E.出血期中由于雌激素和孕激素的正反馈,FSH和LH分泌增加
4 6 2009 14. 正常情况下精子与卵子在何处相遇而发生受精? (2.0分) A.输卵管壶腹部
B.输卵管伞部
C.输卵管漏斗部
D.子宫腔
E.子宫颈
A.输卵管壶腹部
4 6 2009 15. hCG的化学结构和生理作用与下列哪种激素基本相似? (2.0分) A.FSH
B.PRL
C.LH
D.hCS
E.GnRH
C.LH
4 6 2009 16. 妊娠黄体的维持是由于(2.0分) A.hCG的作用
B.LH的作用
C.高浓度E和P的作用
D.HCS的作用
E.以上都不是
A.hCG的作用
4 6 2009 17. Which of the following explains the suppression of lactation during pregnancy?(2.0分) A.Blood prolactin levels are too low for milkproduction to occur
B.Human placental lactogen levels are toolow for milk production to occur
C.The fetal adrenal gland does not producesufficient estriol
D.Blood levels of estrogen and progesteroneare high
E.The maternal anterior pituitary is suppressed
D.Blood levels of estrogen and progesteroneare high
4 6 2009 18. The source of estrogen during the secondand third trimesters of pregnancy is the(2.0分) A.Corpus luteum
B.Fetal adrenal gland, fetal liver, and placenta
C.Fetal ovaries
D.Placenta
E.Maternal ovaries and fetal adrenal gland
B.Fetal adrenal gland, fetal liver, and placenta
4 6 2009 19. 人的性器官发育由以下哪些因素决定的(2.0分) A.性染色体决定
B.常染色体决定
C.性激素依赖
D.孕激素依赖
E.脑发育决定
C.性激素依赖
4 6 2009 20. 以下哪些说法不正确(2.0分) A.阴道入口处5厘米左右的一个豆粒状区域
B.位于阴道入口处2.5厘米左右
C.位于G点和子宫颈的中间
D.G点对机械刺激比较敏感
E.G点对摩擦刺激比较敏感
E.G点对摩擦刺激比较敏感
4 6 2009 21. 关于雌激素作用,下列叙述哪项不正确? (2.0分) A.促进女性性器官发育成熟
B.促使子宫内膜由增殖期转为分泌期
C.较大剂量抑制促性腺激素释放激素的分泌
D.提高子宫平滑肌对缩宫素的敏感性
E.抑制乳汁分泌
B.促使子宫内膜由增殖期转为分泌期
4 6 2009 22. 卵巢功能低下可选用: (2.0分) A.已烯雌酚
B.黄体酮
C.甲基睾丸素
D.强的松龙
E.炔诺酮
A.已烯雌酚
4 6 2009 23. 关于氯底酚胺的叙述,下列哪项是正确的? (2.0分) A.抑制卵巢雌激素合成,发挥抗雌激素作用
B.可用于卵巢肿的治疗
C.主要用于不孕症的治疗
D.阻断下丘脑的雌激素受体,从而增强雌二醇的负反馈性抑制
E.激动下丘脑的雌激素受体,从而消除雌二醇的负反馈性抑制
C.主要用于不孕症的治疗
4 6 2009 24. 孕激素类药物可用于治疗 (2.0分) A.先兆流产
B.诱发流产
C.再生障碍性贫血
D.消耗性疾病等引起的虚弱
E.卵巢功能不全和闭经
A.先兆流产
4 6 2009 25. 雌激素类药和孕激素类药均可用于: (2.0分) A.前列腺癌
B.绝经期综合征
C.乳房胀痛
D.晚期乳腺癌
E.痤疮
A.前列腺癌
4 6 2009 26. 手术后慢性消耗性疾病病人蛋白质吸收不足时可服用: (2.0分) A.泼尼松龙
B.去氧皮质酮
C.苯丙酸诺龙
D.已烯雌酚
E.炔诺酮
C.苯丙酸诺龙
4 6 2009 27. 短效和长效口服避孕药主要作用机制是 (2.0分) A.主要使子宫内膜发生各种功能和形态变化,阻碍孕卵着床
B.发挥杀灭精子的作用;同时,可形成黏液,阻碍精子运动
C.含有雌激素和孕激素,能协同作用而抑制排卵
D.拮抗孕激素活性,破坏蜕膜,诱发流产
E.拮抗雌激素受体,抑制或减弱雌激素的作用
C.含有雌激素和孕激素,能协同作用而抑制排卵
4 6 2009 28. 孕激素避孕的主要环节是: (2.0分) A.抑制排卵
B.抗孕卵着床
C.影响子宫收缩
D.影响胎盘功能
E.杀灭精子
B.抗孕卵着床
4 6 2009 29. 下列哪个药物适用于催产和引产 (2.0分) A.麦角毒
B.缩宫素
C.垂体后叶素
D.麦角新碱
E.麦角胺
B.缩宫素
4 6 2009 30. 对缩宫素的叙述哪一项不正确? (2.0分) A.小剂量可加强子宫的节律性收缩
B.其收缩性质与正常分娩相似
C.大剂量引起子宫强直性收缩
D.大剂量适用于催产
E.小剂量适用于催产和引产
D.大剂量适用于催产
4 6 2009 31. 艾滋病的全称是: (2.0分) A.免疫缺陷综合征
B.获得性免疫缺陷综合征
C.先天性免疫缺陷综合征
D.继发性免疫缺陷综合征
B.获得性免疫缺陷综合征
4 6 2009 32. HIV感染人体后主要侵犯和破坏哪种细胞: (2.0分) A.红细胞
B.白细胞
C.T淋巴细胞
D.巨噬细胞
C.T淋巴细胞
4 6 2009 33. 下列哪项不是艾滋病的传播途径: (2.0分) A.性传播
B.母婴垂直传播
C.血液传播
D.虫媒传播
D.虫媒传播
4 6 2009 34. 艾滋病常见合并症一般不包括(2.0分) A.复发性单纯疤疹
B.持续性或间歇性慢性腹泻
C.皮肤瘙痒
D.急性肾炎
D.急性肾炎
4 6 2009 35. 世界卫生组织推荐的诊断艾滋病的主要症状是: (2.0分) A.体重减轻10%以上
B.发热1个月以上
C.持续性或间歇性慢性腹泻1个月以上
D.体重减轻10%以上,发热1个月以上和持续性或间歇性慢性腹泻1个月以上
D.体重减轻10%以上,发热1个月以上和持续性或间歇性慢性腹泻1个月以上
4 6 2009 36. 艾滋病的特异性治疗是:(2.0分) A.高效抗反转录病毒治疗
B.抗反转录酶抑制剂治疗
C.蛋白酶抑制剂治疗
D.融合抑制剂治疗
A.高效抗反转录病毒治疗
4 6 2009 37. 急性淋病的平均潜伏期是:(2.0分) A.3-5d
B.2w
C.3w
D.3m
E.6m
A.3-5d
4 6 2009 38. 以下叙述正确的是: (2.0分) A.淋病奈瑟菌感染只能由性接触传播
B.淋病奈瑟菌为G+菌
C.人类不是淋病奈瑟菌唯一宿主
D.淋病奈瑟菌可产生β-内酰胺酶
E.有毒株无菌毛
D.淋病奈瑟菌可产生β-内酰胺酶
4 6 2009 39. 有关直肠淋病的相关叙述,哪项是不正确的?(2.0分) A.多见于男男同性恋;
B.症状多不严重;
C.肠道菌群复杂,培养阳性率不高;
D.多选用氟喹诺酮类药物治疗
E.临床症状以肛门瘙痒,肛门口有粘液脓性分泌物。
D.多选用氟喹诺酮类药物治疗
4 6 2009 40. 有关淋球菌耐药情况的描述,哪项是不正确的?(2.0分) A.大多对青霉素耐药;
B.对头孢曲松敏感性降低,有少数耐药株出现;
C.头孢曲松耐药株出现主要由质粒介导引起;
D.大多对四环素耐药;
E.大多对氟喹诺酮类耐药。
C.头孢曲松耐药株出现主要由质粒介导引起;
4 6 2009 41. 胎传梅毒是通过以下途径传染(2.0分) A.静脉输液
B.母血经胎盘传入
C.性交
D.托儿所交叉感染
E.营养不良导致传染
B.母血经胎盘传入
4 6 2009 42. 男性,28岁。两周来全身出现散在玫瑰色甲盖大的红斑,累及躯干、四肢掌跖。不痒。肛门附近有半环形排列的湿性丘疹,表面浸渍状。全身淋巴结肿大。应考虑__(2.0分) A.二期梅毒
B.三期梅毒
C.多形红斑
D.药物疹
E.念珠菌感染
A.二期梅毒
4 6 2009 43. 成人感染梅毒,早期未发现症状,4年后也未出现心血管和中枢神经等症状,查血梅毒血清反应为阳性。称__。(2.0分) A.二期梅毒
B.早期隐性梅毒
C.三期梅毒
D.胎传梅毒
E.晚期隐性梅毒
E.晚期隐性梅毒
4 6 2009 44. 先天梅毒下列哪项是错误的(2.0分) A.是经母体胎传
B.大多在妊娠四个月后传给胎儿
C.先天梅毒没有下疳表现
D.先天梅毒也可经父亲传染
E.先天梅毒儿生后即进入二期梅毒感染阶段
D.先天梅毒也可经父亲传染
4 6 2009 45. 梅毒孕妇传给胎儿其传染性最强的是。(2.0分) A.早期潜伏梅毒
B.一、二期梅毒
C.三期梅毒
D.晚期梅毒
E.晚期潜伏梅毒
B.一、二期梅毒
4 6 2009 46. 关于一期梅毒淋巴结炎的叙述,错误的是 (2.0分) A.发生于硬下疳出现1~2周后
B.常累及单侧腹股沟淋巴结或患处附近淋巴结
C.质地较硬,表面常可见红肿或破溃,疼痛明显,且有触痛
D. 消退常需数月
E.淋巴结穿刺检查可见大量苍白螺旋体
C.质地较硬,表面常可见红肿或破溃,疼痛明显,且有触痛
4 6 2009 47. 二期梅毒的骨关节损害最常见的是:(2.0分) A.骨折
B.骨软骨炎
C.关节强直
D.骨髓炎
E.骨膜炎
E.骨膜炎
4 6 2009 48. 梅毒的皮损中下列哪种不含梅毒螺旋体:(2.0分) A.硬下疳
B.扁平湿疣
C.皮肤粘膜斑
D.梅毒性横痃
E.以上都不是
E.以上都不是
4 6 2009 49. 作定量试验,用于观察梅毒疗效、复发及再感染的血清试验是:(2.0分) A.TPHA
B.RPR
C.HIV抗体检测
D.FTA-ABS
E.HSV抗体
B.RPR
4 6 2009 50. 不符合三期梅毒疹特点的是: (2.0分) A.单侧分布
B.破坏性大,愈后留疤痕
C.附近淋巴结不肿大
D.传染性大
E.自觉症状很轻,但客观症状严重
D.传染性大
4 6 2010 1. 关于硬下疳的描述哪项是错误的? (2.0分) A.多为多发;
B.软骨样硬度
C.为一期梅毒的特征性表现
D.无疼痛
E.可自然消退。
A.多为多发;
4 6 2010 2. 梅毒螺旋体的体外生存情况的相关特征哪项是错的? (2.0分) A.在体外不易生存
B.煮沸就很容易杀死
C.酒精也很容易杀死
D.低温环境也容易死亡
E.肥皂水很容易杀死
D.低温环境也容易死亡
4 6 2010 3. 确诊梅毒诊断需用用哪个指标来证实:(2.0分) A.TRUST
B.RPR
C.USR
D.VDRL
E.TPHA
D.VDRL
4 6 2010 4. 孕妇青霉素皮试阳性,那抗梅毒治疗宜用哪种药物?(2.0分) A.四环素
B.红霉素
C.多西环素
D.氟喹诺酮类药
E.氯霉素
B.红霉素
4 6 2010 5. 为避免这种现象的发生,我们通常应用以下哪种药物来预防? (2.0分) A.强的松
B.西替利嗪
C.氯雷他啶
D.青霉素
E.倍他乐克
A.强的松
4 6 2010 6. 神经梅毒要抽脑脊液检查,以下哪种检测项目最敏感?(2.0分) A.USR
B.TP
C.TPHA
D.RPR
E.VDRL
C.TPHA
4 6 2010 7. 急性淋病的平均潜伏期是:(2.0分) A.3-5d
B.2w
C.3w
D.3m
E.6m
A.3-5d
4 6 2010 8. 以下叙述正确的是:(2.0分) A.淋病奈瑟菌感染只能由性接触传播
B.淋病奈瑟菌为G+菌
C.人类不是淋病奈瑟菌唯一宿主
D.淋病奈瑟菌可产生β-内酰胺酶
E.有毒株无菌毛
D.淋病奈瑟菌可产生β-内酰胺酶
4 6 2010 9. 有关直肠淋病的相关叙述,哪项是不正确的?(2.0分) A.多见于男男同性恋;
B.症状多不严重;
C.肠道菌群复杂,培养阳性率不高;
D.多选用氟喹诺酮类药物治疗
E.临床症状以肛门瘙痒,肛门口有粘液脓性分泌物。
D.多选用氟喹诺酮类药物治疗
4 6 2010 10. 有关淋球菌耐药情况的描述,哪项是不正确的?(2.0分) A.大多对青霉素耐药;
B.对头孢曲松敏感性降低,有少数耐药株出现;
C.头孢曲松耐药株出现主要由质粒介导引起;
D.大多对四环素耐药;
E.大多对氟喹诺酮类耐药。
C.头孢曲松耐药株出现主要由质粒介导引起;
4 6 2010 11. 患者,男,21岁。5天前有不洁性交史,2天前开始出现排尿时刺痛,自己检查发现尿道口发红,挤压后有黄白色脓液流出。体检可见尿道口红肿,尿道内有黄色脓液流出。该患者最可能的诊断是:(2.0分) A.非淋菌性尿道炎
B.淋病
C.尖锐湿疣
D.念珠菌性龟头炎
E.梅毒
B.淋病
4 6 2010 12. 该病的确诊方法是(2.0分) A.分泌物病原菌涂片检查和病原菌培养
B.醋酸白试验
C.分泌物PCR核酸检测
D.分泌物病原菌抗原检测
E.梅毒血清学检查
A.分泌物病原菌涂片检查和病原菌培养
4 6 2010 13. 对该患者首选的治疗方案是:(2.0分) A.口服四环素类抗生素
B.长效青霉素素肌肉注射
C.静脉注射头孢曲松钠
D.激光治疗
E.口服抗真菌药物
C.静脉注射头孢曲松钠
4 6 2010 14. 艾滋病的全称是: (2.0分) A.免疫缺陷综合征
B.获得性免疫缺陷综合征
C.先天性免疫缺陷综合征
D.继发性免疫缺陷综合征
B.获得性免疫缺陷综合征
4 6 2010 15. HIV感染人体后主要侵犯和破坏哪种细胞: (2.0分) A.红细胞
B.白细胞
C.T淋巴细胞
D.巨噬细胞
C.T淋巴细胞
4 6 2010 16. 下列哪项不是艾滋病的传播途径: (2.0分) A.性传播
B.母婴垂直传播
C.血液传播
D.虫媒传播
D.虫媒传播
4 6 2010 17. 艾滋病常见合并症一般不包括: (2.0分) A.复发性单纯疤疹
B.持续性或间歇性慢性腹泻
C.皮肤瘙痒
D.急性肾炎
D.急性肾炎
4 6 2010 18. 世界卫生组织推荐的诊断艾滋病的主要症状是: (2.0分) A.体重减轻10%以上
B.发热1个月以上
C.持续性或间歇性慢性腹泻1个月以上
D.体重减轻10%以上,发热1个月以上和持续性或间歇性慢性腹泻1个月以上
D.体重减轻10%以上,发热1个月以上和持续性或间歇性慢性腹泻1个月以上
4 6 2010 19. 尿道内尖锐湿疣最佳治疗方法是:(2.0分) A.液氮冷冻
B.CO2激光治疗
C.艾拉-光动力治疗
D.微波治疗
E.鬼臼毒素治疗
C.艾拉-光动力治疗
4 6 2010 20. 肛周尖锐湿疣最需要和以下哪种疾病相鉴别?(2.0分) A.肛周湿疹
B.皮脂腺异位
C.阴茎珍珠样丘疹病
D.扁平湿疣
E.痔疮
D.扁平湿疣
4 6 2010 21. 人乳头瘤病毒主要感染哪个部位?(2.0分) A.血液
B.上皮
C.真皮
D.结缔组织
E.淋巴
B.上皮
4 6 2010 22. 关于雌激素药理作用,下列叙述哪项不正确? (2.0分) A.促进女性性器官的发育和成熟,维持女性第二性征。
B.能明显促进蛋白质合成,减少蛋白质分解
C.使阴道上皮增生,浅表层细胞角化
D.增强子宫平滑肌对缩宫素的敏感性
E.较大剂量时可抑制下丘脑-垂体系统释放GnRH
B.能明显促进蛋白质合成,减少蛋白质分解
4 6 2010 23. 雌激素可用于治疗哪一疾病? (2.0分) A.痛经
B.先兆流产
C.功能性子宫出血
D.消耗性疾病
E.绝经期前的乳腺癌
C.功能性子宫出血
4 6 2010 24. 氯底酚胺(克罗米酚)属于哪一类型的药物? (2.0分) A.雌激素受体激动剂
B.雌激素受体拮抗剂
C.较弱的雌激素受体激活,中度的雌激素受体拮抗
D.孕激素受体激动剂
E.孕激素受体拮抗剂
C.较弱的雌激素受体激活,中度的雌激素受体拮抗
4 6 2010 25. 孕激素药理作用的叙述中,哪一项不正确? (2.0分) A.促使子宫内膜由增殖期转为分泌期
B.促使乳腺腺泡发育
C.一定剂量可抑制卵巢排卵
D.保钠排钾作用
E.抑制子宫平滑肌的收缩
D.保钠排钾作用
4 6 2010 26. 黄体酮必须肌肉注射,其主要理由是(2.0分) A.口服吸收缓慢
B.口服给药排泄快
C.肌肉注射吸收迅速
D.肌肉注射能维持较高浓度
E.口服后在胃肠及肝脏迅速破坏
E.口服后在胃肠及肝脏迅速破坏
4 6 2010 27. 雄激素药理作用的叙述,哪一项不正确? (2.0分) A.促进男性性征和生殖器官的发育
B.抗雌激素作用
C.抑制垂体前叶分泌促性腺激素
D.抑制蛋白质合成
E.大剂量促进骨髓造血功能
D.抑制蛋白质合成
4 6 2010 28. 哪一类药物可用于治疗再生障碍性贫血? (2.0分) A.雌激素类药
B.雄激素类药
C.同化激素类药
D.孕激素类药
E.盐皮质激素类药
B.雄激素类药
4 6 2010 29. 哪一种短效口服避孕药能抑制排卵? (2.0分) A.苯丙酸诺龙
B.丙酸睾丸素
C.复方炔诺酮
D.炔诺酮
E.炔雌醇
C.复方炔诺酮
4 6 2010 30. 关于探亲避孕药的服药时间,下列哪一项是正确的? (2.0分) A.必须在排卵前
B.必须在排卵后
C.必须在排卵期间
D.必须在月经来潮的第5天
E.月经周期的任何一天
E.月经周期的任何一天
4 6 2010 31. 哪一项关于麦角新碱的叙述是错误的? (2.0分) A.对子宫兴奋作用强而持久
B.对子宫体和子宫颈的兴奋作用无明显差别
C.剂量稍大即引起子宫强直性收缩
D.适用于子宫出血及产后子宫复原
E.临产时的子宫对本药敏感性降低
E.临产时的子宫对本药敏感性降低
4 6 2010 32. 男性尿道最狭窄处为 (2.0分) A.前列腺部
B.膜部
C.尿道外口
D.尿道球部
E.尿道内口
C.尿道外口
4 6 2010 33. 下列结构属于男性附属腺的是(2.0分) A.睾丸
B.附睾
C.精囊腺
D.阴囊
E.阴茎
C.精囊腺
4 6 2010 34. 以下结构不属于精索内容物的是 (2.0分) A.附睾
B.睾丸血管
C.输精管血管
D.鞘韧带
E.输精管
A.附睾
4 6 2010 35. 输卵管最细的部位是 (2.0分) A.输卵管子宫部
B.输卵管峡
C.输卵管壶腹
D.输卵管漏斗
E.以上答案均不对,输卵管粗细较为均匀
B.输卵管峡
4 6 2010 36. 属于女性生殖系统输送管道的是(2.0分) A.卵巢
B.子宫
C.前庭大腺
D.女阴
E.女性尿道
B.子宫
4 6 2010 37. 睾酮由睾丸的下列哪种细胞分泌?(2.0分) A.间质细胞
B.支持细胞
C.精原细胞
D.精母细胞
E.精子
A.间质细胞
4 6 2010 38. 雄激素结合蛋白是由下列哪种细胞产生的? (2.0分) A.肝细胞
B.睾丸间质细胞
C.睾丸支持细胞
D.睾丸生精细胞
E.睾丸毛细血管内皮细胞
C.睾丸支持细胞
4 6 2010 39. 抑制素是由睾丸的哪种细胞分泌的? (2.0分) A.间质细胞
B.支持细胞
C.精原细胞
D.精母细胞
E.精子
B.支持细胞
4 6 2010 40. 能促进支持细胞分泌抑制素的激素是(2.0分) A.LH
B.ACTH
C.FSHD
D.ADH
E.FSH
E.FSH
4 6 2010 41. 下列关于雌激素生理作用的描述,错误的是(2.0分) A.使输卵管平滑肌活动增强
B.促进阴道上皮细胞增生、角化
C.抑制子宫内膜增生、腺体分泌
D.刺激乳腺导管和结缔组织增生
E.促进肾小管对钠和水的重吸收
C.抑制子宫内膜增生、腺体分泌
4 6 2010 42. 下列有关孕激素作用的叙述,正确的是(2.0分) A.促进子宫内膜发生增生期变化
B.促进子宫内膜发生分泌期变化
C.子宫发育
D.促进并维持女性特征
E.促进子宫收缩
B.促进子宫内膜发生分泌期变化
4 6 2010 43. 雌激素和孕激素作用的相同点是(2.0分) A.促进阴道上皮细胞角化
B.促进乳腺导管增生和延长
C.使子宫输卵管平滑肌活动减弱
D.使子宫内膜增生变厚
E.减少宫颈粘液的分泌
D.使子宫内膜增生变厚
4 6 2010 44. 下列关于月经的叙述,哪一项是错误的? (2.0分) A.出血期中,GnRH分泌处于较低水平
B.黄体退化,雌激素和孕激素分泌骤减
C.前列腺素F2 释放,子宫内膜血管痉挛
D.溶酶体释出蛋白水解酶,使组织溶解,内膜剥脱、出血
E.出血期中由于雌激素和孕激素的正反馈,FSH和LH分泌增加
E.出血期中由于雌激素和孕激素的正反馈,FSH和LH分泌增加
4 6 2010 45. 正常情况下精子与卵子在何处相遇而发生受精? (2.0分) A.输卵管壶腹部
B.输卵管伞部
C.输卵管漏斗部
D.子宫腔
E.子宫颈
A.输卵管壶腹部
4 6 2010 46. hCG的化学结构和生理作用与下列哪种激素基本相似?(2.0分) A.FSH
B.PRL
C.LH
D.hCS
E.GnRH
C.LH
4 6 2010 47. 妊娠黄体的维持是由于(2.0分) A.hCG的作用
B.LH的作用
C.高浓度E和P的作用
D.HCS的作用
E.以上都不是
A.hCG的作用
4 6 2010 48. Which of the following explains the suppression of lactation during pregnancy?(2.0分) A.Blood prolactin levels are too low for milk production to occur
B.Human placental lactogen levels are too low for milk production to occur
C.The fetal adrenal gland does not produce sufficient estriol
D.Blood levels of estrogen and progesterone are high
E.The maternal anterior pituitary is suppressed
D.Blood levels of estrogen and progesterone are high
4 6 2010 49. The source of estrogen during the second and third trimesters of pregnancy is the(2.0分) A.Corpus luteum
B.Fetal adrenal gland, fetal liver, and placenta
C.Fetal ovaries
D.Placenta
E.Maternal ovaries and fetal adrenal gland
B.Fetal adrenal gland, fetal liver, and placenta
4 6 2010 50. 女性月经周期中,月经来潮是由于血中的(2.0分) A.E2浓度下降,P浓度升高
B.E2浓度升高,P浓度下降
C.E2和P浓度均下降
D.FSH和LH浓度均下降
E.LH浓度显著升高而形成LH峰
C.E2和P浓度均下降
4 6 2011 1. 关于前列腺,正确的是(2.5分) A.只是一种腺组织
B.呈底在下的栗子形
C.腺管开口于尿道膜部
D.内有尿道通过
E.底部有输精管穿过并开口于尿道
D.内有尿道通过
4 6 2011 2. 精索的被膜中,下列说法正确的是(2.5分) A.精索外筋膜是腹内斜肌腱膜的延续
B.精索内筋膜是腹横肌腱膜的延续
C.提睾肌是腹内斜肌和腹横肌的延续
D.三层被膜均位于阴囊肉膜浅面
E.睾丸和精索被膜截然不同
C.提睾肌是腹内斜肌和腹横肌的延续
4 6 2011 3. 关于男性尿道,正确的是(2.5分) A.分为前列腺部、膜部和海绵体部
B.临床上把膜部和海绵体部称前尿道
C.尿道球部较狭窄
D.前列腺部有输精管开口
E.耻骨下弯凹向下
A.分为前列腺部、膜部和海绵体部
4 6 2011 4. 临床上所说的后尿道是指尿道的(2.5分) A.膜部
B.海绵体部
C.前列腺部
D.球部
E.以上都不是
E.以上都不是
4 6 2011 5. 艾滋病的全称是(2.5分) A.免疫缺陷综合征
B.获得性免疫缺陷综合征
C.先天性免疫缺陷综合征
D.继发性免疫缺陷综合征
B.获得性免疫缺陷综合征
4 6 2011 6. 艾滋病病毒感染人体后临床症状分期错误的提法是(2.5分) A.急性感染期
B.病毒携带期
C.无症状感染期
D.艾滋病期
B.病毒携带期
4 6 2011 7. 下列哪项检查能够最终确定病人感染了艾滋病毒: (2.5分) A.血常规
B.HIV抗体确诊试验
C.HIV抗体筛查试验
D.尿常规
B.HIV抗体确诊试验
4 6 2011 8. 艾滋病的特异性治疗是:(2.5分) A.高效抗反转录病毒治疗
B.抗反转录酶抑制剂治疗
C.蛋白酶抑制剂治疗
D.融合抑制剂治疗
A.高效抗反转录病毒治疗
4 6 2011 9. 尿道内尖锐湿疣最佳治疗方法是:(2.5分) A.液氮冷冻
B.CO2激光治疗
C.艾拉-光动力治疗
D.微波治疗
E.鬼臼毒素治疗
C.艾拉-光动力治疗
4 6 2011 10. 最常见的高危型HPV是以下哪型(2.5分) A.HPV16、HPV18
B.HPV6、HPV11
C.HPV6、HPV31
D.HPV11、HPV18
E.HPV11、HPV32
A.HPV16、HPV18
4 6 2011 11. 急性淋病的平均潜伏期是(2.5分) A.3-5d
B.2w
C.3w
D.3m
E.6m
A.3-5d
4 6 2011 12. 经典性病不包括(2.5分) A.淋病
B.梅毒
C.尖锐湿疣
D.性病性淋巴肉芽肿
E.软下疳
C.尖锐湿疣
4 6 2011 13. 以下叙述正确的是(2.5分) A.淋病奈瑟菌感染只能由性接触传播
B.淋病奈瑟菌为G+菌
C.人类不是淋病奈瑟菌唯一宿主
D.淋病奈瑟菌可产生β-内酰胺酶
E.有毒株无菌毛
D.淋病奈瑟菌可产生β-内酰胺酶
4 6 2011 14. 淋病治疗首先药物是(2.5分) A.青霉素
B.多烯环素
C.环丙沙星
D.氧氟沙星
E.头孢曲松
E.头孢曲松
4 6 2011 15. 胎传梅毒是通过以下途径传染__。(2.5分) A.静脉输液
B.母血经胎盘传入
C.性交
D.托儿所交叉感染
E.营养不良导致传染
B.母血经胎盘传入
4 6 2011 16. 哪些皮肤粘膜疹有传染性?__。(2.5分) A.扁平湿疣
B.滤泡性溃疡
C.树胶样肿
D.扁平苔藓
E.多形红斑
A.扁平湿疣
4 6 2011 17. 先天梅毒下列哪项是错误的__。(2.5分) A.是经母体胎传
B.大多在妊娠四个月后传给胎儿
C.先天梅毒没有下疳表现
D.先天梅毒也可经父亲传染
E.先天梅毒儿生后即进入二期梅毒感染阶段
D.先天梅毒也可经父亲传染
4 6 2011 18. 二期梅毒的骨关节损害最常见的是(2.5分) A.骨折
B.骨软骨炎
C.关节强直
D.骨髓炎
E.骨膜炎
E.骨膜炎
4 6 2011 19. 一期梅毒的诊断标准不包括下列哪项(2.5分) A.有不洁性交史
B.早期有流感症状
C.外生殖器单个无痛性下疳
D.潜伏期3周左右
E.梅毒血清试验阳性
B.早期有流感症状
4 6 2011 20. 梅毒的皮损中下列哪种不含梅毒螺旋体(2.5分) A.硬下疳
B.扁平湿疣
C.皮肤粘膜斑
D.梅毒性横痃
E.以上都不是
E.以上都不是
4 6 2011 21. 作定量试验,用于观察梅毒疗效、复发及再感染的血清试验是(2.5分) A.TPHA
B.RPR
C.HIV抗体检测
D.FTA-ABS
E.HSV抗体
B.RPR
4 6 2011 22. 不符合三期梅毒疹特点的是(2.5分) A.单侧分布
B.破坏性大,愈后留疤痕
C.附近淋巴结不肿大
D.传染性大
E.自觉症状很轻,但客观症状严重
D.传染性大
4 6 2011 23. 睾酮由睾丸的下列哪种细胞分泌? (2.5分) A.间质细胞
B.支持细胞
C.精原细胞
D.精母细胞
E.精子
A.间质细胞
4 6 2011 24. 下列关于睾丸功能调节的叙述,哪一项是错误的? (2.5分) A.FSH对生精过程有启动作用
B.LH刺激间质细胞分泌睾酮
C.睾酮对FSH分泌有负反馈作用
D.抑制素对FSH分泌有负反馈作用
E.生精过程受睾酮和FSH双重控制
C.睾酮对FSH分泌有负反馈作用
4 6 2011 25. 抑制素是由睾丸的哪种细胞分泌的? (2.5分) A.间质细胞
B.支持细胞
C.精原细胞
D.精母细胞
E.精子
B.支持细胞
4 6 2011 26. 雌二醇的化学性质是(2.5分) A.胺类
B.肽类
C.类固醇
D.固醇类
E.蛋白质
C.类固醇
4 6 2011 27. 育龄期女子基础体温的双相变化与下列哪种激素有关? (2.5分) A.雌激素
B.孕激素
C.甲状腺激素
D.LH
E.FSH
B.孕激素
4 6 2011 28. 女性月经周期中,月经来潮是由于血中的(2.5分) A.E2浓度下降,P浓度升高
B.E2浓度升高,P浓度下降
C.E2和P浓度均下降
D.FSH和LH浓度均下降
E.LH浓度显著升高而形成LH峰
C.E2和P浓度均下降
4 6 2011 29. 精子获能的主要部位是(2.5分) A.睾丸曲细精管
B.附睾
C.阴道
D.子宫颈
E.子宫腔
E.子宫腔
4 6 2011 30. hCG的化学结构和生理作用与下列哪种激素基本相似? (2.5分) A.FSH
B.PRL
C.LH
D.hCS
E.GnRH
C.LH
4 6 2012 1. 尿道内尖锐湿疣最佳治疗方法是(2.5分) A.液氮冷冻
B.CO2激光治疗
C.艾拉-光动力治疗
D.微波治疗
E.鬼臼毒素治疗
C.艾拉-光动力治疗
4 6 2012 2. 人乳头瘤病毒主要感染哪个部位? (2.5分) A.血液
B.上皮
C.真皮
D.结缔组织
E.淋巴
B.上皮
4 6 2012 3. 急性淋病的平均潜伏期是 (2.5分) A.3-5d
B.2w
C.3w
D.3m
E.6m
A.3-5d
4 6 2012 4. 以下叙述正确的是(2.5分) A.淋病奈瑟菌感染只能由性接触传播
B.淋病奈瑟菌为G+菌
C.人类不是淋病奈瑟菌唯一宿主
D.淋病奈瑟菌可产生β-内酰胺酶
E.有毒株无菌毛
D.淋病奈瑟菌可产生β-内酰胺酶
4 6 2012 5. 有关小儿淋病的描述哪项是不正确的? (2.5分) A.男性患儿多于女性;
B.发病率升高与父母感染淋病有关;
C.治疗首选头孢曲松针;
D.临床上多表现为结膜炎、尿道炎和阴道炎;
E.好发年龄多为0-1岁患儿。
A.男性患儿多于女性;
4 6 2012 6. 有关直肠淋病的相关叙述,哪项是不正确的? (2.5分) A.多见于男男同性恋;
B.症状多不严重;
C.肠道菌群复杂,培养阳性率不高;
D.多选用氟喹诺酮类药物治疗
E.临床症状以肛门瘙痒,肛门口有粘液脓性分泌物。
D.多选用氟喹诺酮类药物治疗
4 6 2012 7. 关于硬下疳的描述哪项是错误的? (2.5分) A.多为多发;
B.软骨样硬度
C.为一期梅毒的特征性表现
D.无疼痛
E.可自然消退
A.多为多发;
4 6 2012 8. 梅毒螺旋体的体外生存情况的相关特征哪项是错的? (2.5分) A.在体外不易生存
B.煮沸就很容易杀死
C.酒精也很容易杀死
D.低温环境也容易死亡
E.肥皂水很容易杀死
D.低温环境也容易死亡
4 6 2012 9. 下例哪项不是梅毒螺旋体的特征?(2.5分) A.小而纤细的螺旋状微生物
B.运动方式有旋转式,蛇行式和伸缩式
C.普通显微镜下不易发现梅毒螺旋体
D.银染色阴性
E.人是唯一宿主
D.银染色阴性
4 6 2012 10. 孕妇青霉素皮试阳性,那抗梅毒治疗宜用哪种药物? (2.5分) A.四环素
B.红霉素
C.多西环素
D.氟喹诺酮类药
E.氯霉素
B.红霉素
4 6 2012 11. 为避免这种现象的发生,我们通常应用以下哪种药物来预防?(2.5分) A.强的松
B.西替利嗪
C.氯雷他啶
D.青霉素
E.倍他乐克
A.强的松
4 6 2012 12. 艾滋病的全称是(2.5分) A.免疫缺陷综合征
B.获得性免疫缺陷综合征
C.先天性免疫缺陷综合征
D.继发性免疫缺陷综合征
E.以上均错
B.获得性免疫缺陷综合征
4 6 2012 13. 下列哪项不是急性HIV感染的临床表现(2.5分) A.发热
B.咽痛
C.淋巴结肿大
D.口腔真菌感染
E.以上均是
D.口腔真菌感染
4 6 2012 14. 下列哪项检查能够最终确定病人感染了艾滋病毒(2.5分) A.血常规
B.HIV抗体确诊试验
C.HIV抗体筛查试验
D.尿常规
E.以上均是
B.HIV抗体确诊试验
4 6 2012 15. 艾滋病人体体液含有大量病毒,具有很强传染性的是(2.5分) A.血液
B.尿液
C.泪液
D.汗液
E.以上均是
A.血液
4 6 2012 16. HIV感染人体后主要侵犯和破坏哪种细胞(2.5分) A.红细胞
B.白细胞
C.T淋巴细胞
D.巨噬细胞
E.以上均是
C.T淋巴细胞
4 6 2012 17. 关于前列腺,正确的是(2.5分) A.只是一种腺组织
B.呈底在下的栗子形
C.腺管开口于尿道膜部
D.内有尿道通过
E.底部有输精管穿过并开口于尿道
D.内有尿道通过
4 6 2012 18. 关于男性尿道,正确的是(2.5分) A.分为前列腺部、膜部和海绵体部
B.临床上把膜部和海绵体部称前尿道
C.尿道球部较狭窄
D.前列腺部有输精管开口
E.耻骨下弯凹向下
A.分为前列腺部、膜部和海绵体部
4 6 2012 19. 男性,9岁,因阴囊肿大就诊。经查阴囊透光试验阳性,表示某结构内有液体积存,该结构是(2.5分) A.睾丸固有鞘膜腔
B.附睾
C.睾丸
D.精索
E.输精管
A.睾丸固有鞘膜腔
4 6 2012 20. 关于输卵管四个部分说法正确的是 (2.5分) A.输卵管峡最细
B.输卵管结扎多在输卵管壶腹部进行
C.输卵管壶腹部是最常见的卵子受精场所
D.输卵管外侧2/3为输卵管漏斗
E.输卵管壶腹部在输卵管漏斗的外侧
C.输卵管壶腹部是最常见的卵子受精场所
4 6 2012 21. 子宫属于腹膜间位器官,有部分表面未被腹膜覆盖,从哪个部位行剖宫取胎术能避免腹膜腔感染和术后粘连(2.5分) A.子宫体
B.子宫峡
C.子宫底
D.子宫颈阴道部
E.子宫腔
B.子宫峡
4 6 2012 22. 关于雌激素药理作用,下列叙述哪项不正确? (2.5分) A.促进女性性器官的发育和成熟,维持女性第二性征。
B.能明显促进蛋白质合成,减少蛋白质分解
C.使阴道上皮增生,浅表层细胞角化
D.增强子宫平滑肌对缩宫素的敏感性
E.较大剂量时可抑制下丘脑-垂体系统释放GnRH
B.能明显促进蛋白质合成,减少蛋白质分解
4 6 2012 23. 雌激素可用于治疗哪一疾病? (2.5分) A.痛经
B.先兆流产
C.功能性子宫出血
D.消耗性疾病
E.绝经期前的乳腺癌
C.功能性子宫出血
4 6 2012 24. 氯米芬(克罗米酚)属于哪一类型的药物(2.5分) A.雌激素受体激动剂
B.雌激素受体拮抗剂
C.较弱的雌激素受体激活,中度的雌激素受体拮抗
D.孕激素受体激动剂
E.孕激素受体拮抗剂
C.较弱的雌激素受体激活,中度的雌激素受体拮抗
4 6 2012 25. 孕激素药理作用的叙述中,哪一项不正确? (2.5分) A.促使子宫内膜由增殖期转为分泌期
B.促使乳腺腺泡发育
C.一定剂量可抑制卵巢排卵
D.保钠排钾作用
E.抑制子宫平滑肌的收缩
D.保钠排钾作用
4 6 2012 26. 下列关于米非司酮说法错误的是(2.5分) A.具有孕激素样活性
B.具有抑制排卵作用
C.具有抗着床作用
D.可用于终止早期妊娠
E.可用于紧急避孕
A.具有孕激素样活性
4 6 2012 27. 服用哪一药物可治疗手术后慢性消耗性疾病病人的蛋白质吸收不足? (2.5分) A.泼尼松龙
B.去氧皮质酮
C.苯丙酸诺龙
D.已烯雌酚
E.炔诺酮
C.苯丙酸诺龙
4 6 2012 28. 下列哪种属于短效口服避孕药? (2.5分) A.苯丙酸诺龙
B.丙酸睾丸素
C.复方炔诺酮
D.炔诺酮
E.炔雌醇
C.复方炔诺酮
4 6 2012 29. 下列哪一药物适用于催产和引产? (2.5分) A.麦角毒
B.缩宫素
C.垂体后叶素
D.麦角新碱
E.麦角胺
B.缩宫素
4 6 2012 30. 睾酮由睾丸的下列哪种细胞分泌? (2.5分) A.间质细胞
B.支持细胞
C.精原细胞
D.精母细胞
E.精子
A.间质细胞
4 6 2012 31. 雄激素结合蛋白是由下列哪种细胞产生的? (2.5分) A.肝细胞
B.睾丸间质细胞
C.睾丸支持细胞
D.睾丸生精细胞
E.睾丸毛细血管内皮细胞
C.睾丸支持细胞
4 6 2012 32. 抑制素是由睾丸的哪种细胞分泌的? (2.5分) A.间质细胞
B.支持细胞
C.精原细胞
D.精母细胞
E.精子
B.支持细胞
4 6 2012 33. 能促进支持细胞分泌抑制素的激素是(2.5分) A.LH
B.ACTH
C.FSHD
D.ADH
E.FSH
E.FSH
4 6 2012 34. 排卵前,血中LH出现高峰的原因是(2.5分) A.血中孕激素对腺垂体的正反馈作用
B.血中雌激素对腺垂体的正反馈作用
C.血中雌激素和孕激素共同的作用
D.FSH的促进作用
E.少量LH本身的短反馈作用
B.血中雌激素对腺垂体的正反馈作用
4 6 2012 35. 雌激素和孕激素作用的相同点是(2.5分) A.促进阴道上皮细胞角化
B.促进乳腺导管增生和延长
C.使子宫输卵管平滑肌活动减弱
D.使子宫内膜增生变厚
E.减少宫颈粘液的分泌
D.使子宫内膜增生变厚
4 6 2012 36. 精子获能的主要部位是(2.5分) A.睾丸曲细精管
B.附睾
C.阴道
D.子宫颈
E.子宫腔
E.子宫腔
4 6 2012 37. 测定血或尿中哪种激素可诊断早期妊娠? (2.5分) A.孕激素
B.雌激素
C.FSH
D.hCG
E.ACTH
D.hCG
4 6 2012 38. 妊娠黄体的维持是由于(2.5分) A.hCG的作用
B.LH的作用
C.高浓度E和P的作用
D.HCS的作用
E.以上都不是
A.hCG的作用
4 6 2012 39. Secretion of oxytocin is increased by(2.5分) A.Milk ejection
B.Dilation of the cervix
C.Increased prolactin levels
D.Increased extracellular fluid (ECF) volume
E.Increased serum osmolarity
B.Dilation of the cervix
4 6 2012 40. Which of the following is not a male secondary sexual characteristic? (2.5分) A.A beard
B.An increased incidence of' acne
C.A deep voice
D.Increased fat in the buttocks
E.An enlarged penis
D.Increased fat in the buttocks
4 6 2013 1. 雄激素结合蛋白是由下列哪种细胞产生的? (2.5分) A.肝细胞
B.睾丸间质细胞
C.睾丸支持细胞
D.睾丸生精细胞
E.睾丸毛细血管内皮细胞
C.睾丸支持细胞
4 6 2013 2. 抑制素是由睾丸的哪种细胞分泌的? (2.5分) A.间质细胞
B.支持细胞
C.精原细胞
D.精母细胞
E.精子
B.支持细胞
4 6 2013 3. 能促进支持细胞分泌抑制素的激素是(2.5分) A.LH
B.ACTH
C.FSHD
D.ADH
E.FSH
E.FSH
4 6 2013 4. 下列关于雌激素生理作用的描述,错误的是(2.5分) A.使输卵管平滑肌活动增强
B.促进阴道上皮细胞增生.角化
C.抑制子宫内膜增生.腺体分泌
D.刺激乳腺导管和结缔组织增生
E.促进肾小管对钠和水的重吸收
C.抑制子宫内膜增生.腺体分泌
4 6 2013 5. 下列有关孕激素作用的叙述,正确的是(2.5分) A.促进子宫内膜发生增生期变化
B.促进子宫内膜发生分泌期变化
C.子宫发育
D.促进并维持女性特征
E.促进子宫收缩
B.促进子宫内膜发生分泌期变化
4 6 2013 6. 雌激素和孕激素作用的相同点是(2.5分) A.促进阴道上皮细胞角化
B.促进乳腺导管增生和延长
C.使子宫输卵管平滑肌活动减弱
D.使子宫内膜增生变厚
E.减少宫颈粘液的分泌
D.使子宫内膜增生变厚
4 6 2013 7. 下列关于月经的叙述,哪一项是错误的? (2.5分) A.出血期中,GnRH分泌处于较低水平
B.黄体退化,雌激素和孕激素分泌骤减
C.前列腺素F2?释放,子宫内膜血管痉挛
D.溶酶体释出蛋白水解酶,使组织溶解,内膜剥脱.出血
E.出血期中由于雌激素和孕激素的正反馈,FSH和LH分泌增加
E.出血期中由于雌激素和孕激素的正反馈,FSH和LH分泌增加
4 6 2013 8. 正常情况下精子与卵子在何处相遇而发生受精? (2.5分) A.输卵管壶腹部
B.输卵管伞部
C.输卵管漏斗部
D.子宫腔
E.子宫颈
A.输卵管壶腹部
4 6 2013 9. hCG的化学结构和生理作用与下列哪种激素基本相似? (2.5分) A.FSH
B.PRL
C.LH
D.hCS
E.GnRH
C.LH
4 6 2013 10. 妊娠黄体的维持是由于(2.5分) A.hCG的作用
B.LH的作用
C.高浓度E和P的作用
D.HCS的作用
E.以上都不是
A.hCG的作用
4 6 2013 11. Secretion of oxytocin is increased by(2.5分) A.Milk ejection
B.Dilation of the cervix
C.Increased prolactin levels
D.Increased extracellular fluid (ECF) volume
E.Increased serum osmolarity
B.Dilation of the cervix
4 6 2013 12. Which of the following is not a male secondary sexual characteristic? (2.5分) A.A beard
B.An increased incidence of' acne
C.A deep voice
D.Increased fat in the buttocks
E.An enlarged penis
D.Increased fat in the buttocks
4 6 2013 13. 尿道内尖锐湿疣最佳治疗方法是(2.5分) A.液氮冷冻
B.CO2激光治疗
C.艾拉-光动力治疗
D.微波治疗
E.鬼臼毒素治疗
C.艾拉-光动力治疗
4 6 2013 14. 肛周尖锐湿疣最需要和以下哪种疾病相鉴别?(2.5分) A.肛周湿疹
B.皮脂腺异位
C.阴茎珍珠样丘疹病
D.扁平湿疣
E.痔疮
D.扁平湿疣
4 6 2013 15. 急性淋病的平均潜伏期是(2.5分) A.3-5d
B.2w
C.3w
D.3m
E.6m
A.3-5d
4 6 2013 16. 以下叙述正确的是(2.5分) A.淋病奈瑟菌感染只能由性接触传播
B.淋病奈瑟菌为G+菌
C.人类不是淋病奈瑟菌唯一宿主
D.淋病奈瑟菌可产生β-内酰胺酶
E.有毒株无菌毛
D.淋病奈瑟菌可产生β-内酰胺酶
4 6 2013 17. 有关直肠淋病的相关叙述,哪项是不正确的?(2.5分) A.多见于男男同性恋;
B.症状多不严重;
C.肠道菌群复杂,培养阳性率不高;
D.多选用氟喹诺酮类药物治疗
E.临床症状以肛门瘙痒,肛门口有粘液脓性分泌物。
D.多选用氟喹诺酮类药物治疗
4 6 2013 18. 有关淋球菌耐药情况的描述,哪项是不正确的?(2.5分) A.大多对青霉素耐药;
B.对头孢曲松敏感性降低,有少数耐药株出现;
C.头孢曲松耐药株出现主要由质粒介导引起;
D.大多对四环素耐药;
E.大多对氟喹诺酮类耐药。
C.头孢曲松耐药株出现主要由质粒介导引起;
4 6 2013 19. 该病的确诊方法是(2.5分) A.分泌物病原菌涂片检查和病原菌培养
B.醋酸白试验
C.分泌物PCR核酸检测
D.分泌物病原菌抗原检测
E.梅毒血清学检查
A.分泌物病原菌涂片检查和病原菌培养
4 6 2013 20. 关于硬下疳的描述哪项是错误的?(2.5分) A.多为多发;
B.软骨样硬度
C.为一期梅毒的特征性表现
D.无疼痛
E.可自然消退。
A.多为多发;
4 6 2013 21. 梅毒螺旋体的体外生存情况的相关特征哪项是错的?(2.5分) A.在体外不易生存
B.煮沸就很容易杀死
C.酒精也很容易杀死
D.低温环境也容易死亡
E.肥皂水很容易杀死
D.低温环境也容易死亡
4 6 2013 22. 梅毒可以引起脱发,临床上多表现为(2.5分) A.虫蚀样脱发
B.脂溢性脱发
C.斑秃样脱发
D.束状发
E.休止期脱发
A.虫蚀样脱发
4 6 2013 23. 孕妇青霉素皮试阳性,那抗梅毒治疗宜用哪种药物?(2.5分) A.四环素
B.红霉素
C.多西环素
D.氟喹诺酮类药
E.氯霉素
B.红霉素
4 6 2013 24. 心血管梅毒最常侵犯哪个位置?(2.5分) A.二尖瓣
B.三尖瓣
C.主动脉
D.冠状动脉
E.心肌
C.主动脉
4 6 2013 25. 关于前列腺,正确的是(2.5分) A.只是一种腺组织
B.呈底在下的栗子形
C.腺管开口于尿道膜部
D.内有尿道通过
E.底部有输精管穿过并开口于尿道
D.内有尿道通过
4 6 2013 26. 关于男性尿道,正确的是(2.5分) A.分为前列腺部.膜部和海绵体部
B.临床上把膜部和海绵体部称前尿道
C.尿道球部较狭窄
D.前列腺部有输精管开口
E.耻骨下弯凹向下
A.分为前列腺部.膜部和海绵体部
4 6 2013 27. 男性,9岁,因阴囊肿大就诊。经查阴囊透光试验阳性,表示某结构内有液体积存,该结构是(2.5分) A.睾丸固有鞘膜腔
B.附睾
C.睾丸
D.精索
E.输精管
A.睾丸固有鞘膜腔
4 6 2013 28. 关于输卵管四个部分说法正确的是(2.5分) A.输卵管峡最细
B.输卵管结扎多在输卵管壶腹部进行
C.输卵管壶腹部是最常见的卵子受精场所
D.输卵管外侧2/3为输卵管漏斗
E.输卵管壶腹部在输卵管漏斗的外侧
C.输卵管壶腹部是最常见的卵子受精场所
4 6 2013 29. 子宫属于腹膜间位器官,有部分表面未被腹膜覆盖,从哪个部位行剖宫取胎术能避免腹膜腔感染和术后粘连(2.5分) A.子宫体
B.子宫峡
C.子宫底
D.子宫颈阴道部
E.子宫腔
B.子宫峡
4 6 2013 30. In combination oral contraceptives, the progestin is added primarily to(2.5分) A.ensure prompt withdrawal
B.decrease cervical mucous
C.inhibit ovarian estrogen metabolism
D.increase fallopian tube contractility
E.none of above
E.none of above
4 6 2013 31. Which is used to treat infertility in women(2.5分) A.human menopausal gonadotropin(HMG)
B.human chorionic gonadotropin(HCG)
C.either human menopausal gonadotropin (HMG ) or human chorionic gonadotropin(HCG)
D.human menopausal gonadotropin(HMG) plus human chorionic gonadotropin
E.estrogen
D.human menopausal gonadotropin(HMG) plus human chorionic gonadotropin
4 6 2013 32. Which of the following drugs is a progestin that commonly used in compound recipe of oral contraceptives? (2.5分) A.mestranol
B.diethylstilbestrol
C.norethindrone
D.metandienone
E.estradiol
C.norethindrone
4 6 2013 33. Orally- active androgenic steroid is(2.5分) A.methyltestosterone
B.diethylstilbestrol
C.clomiphene
D.ethinylestradiol
E.none of the above
A.methyltestosterone
4 6 2013 34. Which of the following is a nature estrogen(2.5分) A.diethylstilbestrolum
B.estradiol
C.quinestrol
D.ethinylestradiol
E.estradiol valerate
B.estradiol
4 6 2013 35. 艾滋病的全称是(2.5分) A.免疫缺陷综合征
B.获得性免疫缺陷综合征
C.先天性免疫缺陷综合征
D.继发性免疫缺陷综合征
B.获得性免疫缺陷综合征
4 6 2013 36. 艾滋病病毒感染人体后临床症状分期错误的提法是(2.5分) A.急性感染期
B.病毒携带期
C.无症状感染期
D.艾滋病期
B.病毒携带期
4 6 2013 37. 下列哪项检查能够最终确定病人感染了艾滋病毒(2.5分) A.血常规
B.HIV抗体确诊试验
C.HIV抗体筛查试验
D.尿常规
B.HIV抗体确诊试验
4 6 2013 38. 艾滋病人体体液含有大量病毒,具有很强传染性的是(2.5分) A.血液
B.尿液
C.泪液
D.汗液
A.血液
4 6 2013 39. 艾滋病的特异性治疗是(2.5分) A.高效抗反转录病毒治疗
B.抗反转录酶抑制剂治疗
C.蛋白酶抑制剂治疗
D.融合抑制剂治疗
A.高效抗反转录病毒治疗
4 6 2013 40. 睾酮由睾丸的下列哪种细胞分泌? (2.5分) A.间质细胞
B.支持细胞
C.精原细胞
D.精母细胞
E.精子
A.间质细胞
4 6 2014 1. 尿道内尖锐湿疣最佳治疗方法是(2.5分) A.液氮冷冻
B.CO2激光治疗
C.艾拉-光动力治疗
D.微波治疗
E.鬼臼毒素治疗
C.艾拉-光动力治疗
4 6 2014 2. 最常见的高危型HPV是以下哪型(2.5分) A.HPV16、HPV18
B.HPV6、HPV11
C.HPV6、HPV31
D.HPV11、HPV18
E.HPV11、HPV32
A.HPV16、HPV18
4 6 2014 3. 急性淋病的平均潜伏期是(2.5分) A.3-5天
B.2周
C.3周
D.3月
E.6月
A.3-5天
4 6 2014 4. 以下叙述正确的是(2.5分) A.淋病奈瑟菌感染只能由性接触传播
B.淋病奈瑟菌为G+菌
C.人类不是淋病奈瑟菌唯一宿主
D.淋病奈瑟菌可产生β-内酰胺酶
E.有毒株无菌毛
D.淋病奈瑟菌可产生β-内酰胺酶
4 6 2014 5. 淋病治疗首先药物是(2.5分) A.青霉素
B.多烯环素
C.环丙沙星
D.氧氟沙星
E.头孢曲松
E.头孢曲松
4 6 2014 6. 有关小儿淋病的描述哪项是不正确的? (2.5分) A.男性患儿多于女性;
B.发病率升高与父母感染淋病有关;
C.治疗首选头孢曲松针;
D.临床上多表现为结膜炎、尿道炎和阴道炎;
E.好发年龄多为0-1岁患儿。
A.男性患儿多于女性;
4 6 2014 7. 关于硬下疳的描述哪项是错误的? (2.5分) A.多为多发;
B.软骨样硬度
C.为一期梅毒的特征性表现
D.无疼痛
E.可自然消退
A.多为多发;
4 6 2014 8. 梅毒螺旋体的体外生存情况的相关特征哪项是错的? (2.5分) A.在体外不易生存
B.煮沸就很容易杀死
C.酒精也很容易杀死
D.低温环境也容易死亡
E.肥皂水很容易杀死
D.低温环境也容易死亡
4 6 2014 9. HIV感染人体后主要侵犯和破坏哪种细胞(2.5分) A.红细胞
B.白细胞
C.T淋巴细胞
D.巨噬细胞
C.T淋巴细胞
4 6 2014 10. 二期梅毒疹表现多种多样,下列哪项不是二期梅毒疹的特征? (2.5分) A.玫瑰疹
B.扁平湿疣
C.口腔会出现糜烂
D.斑秃
E.皮肤脓疱样毛囊性丘疹
D.斑秃
4 6 2014 11. 心血管梅毒最常侵犯哪个位置? (2.5分) A.二尖瓣
B.三尖瓣
C.主动脉
D.冠状动脉
E.心肌
C.主动脉
4 6 2014 12. 梅毒可以引起脱发,临床上多表现为(2.5分) A.虫蚀样脱发
B.脂溢性脱发
C.斑秃样脱发
D.束状发
E.休止期脱发
A.虫蚀样脱发
4 6 2014 13. 关于前列腺,正确的是(2.5分) A.只是一种腺组织
B.呈底在下的栗子形
C.腺管开口于尿道膜部
D.内有尿道通过
E.底部有输精管穿过并开口于尿道
D.内有尿道通过
4 6 2014 14. 关于男性尿道,正确的是(2.5分) A.分为前列腺部、膜部和海绵体部
B.临床上把膜部和海绵体部称前尿道
C.尿道球部较狭窄
D.前列腺部有输精管开口
E.耻骨下弯凹向下
A.分为前列腺部、膜部和海绵体部
4 6 2014 15. 男性,29岁,育有两女,按要求进行节育手术,选择的部位在输精管的(2.5分) A.睾丸部
B.精索部
C.腹股沟管部
D.盆部
E.任何一部
B.精索部
4 6 2014 16. 子宫属于腹膜间位器官,有部分表面未被腹膜覆盖,从哪个部位行剖宫取胎术能避免腹膜腔感染和术后粘连(2.5分) A.子宫体
B.子宫峡
C.子宫底
D.子宫颈阴道部
E.子宫腔
B.子宫峡
4 6 2014 17. 男性性器官的发育主要依赖(2.5分) A.基因
B.雌激素
C.睾酮
D.孕激素
E.ACTH
C.睾酮
4 6 2014 18. 人类的性别由()决定(2.5分) A.性器官
B.常染色体
C.性染色体
D.雌激素
E.雄激素
C.性染色体
4 6 2014 19. 最容易激发性反应的感觉器官是(2.5分) A.听觉
B.触觉
C.痛觉
D.温度觉
E.深感觉
B.触觉
4 6 2014 20. In combination oral contraceptives, the progestin is added primarily to(2.5分) A.ensure prompt withdrawal
B.decrease cervical mucous
C.inhibit ovarian estrogen metabolism
D.increase fallopian tube contractility
E.none of above
E.none of above
4 6 2014 21. Which is used to treat infertility in women(2.5分) A.human menopausal gonadotropin(HMG)
B.human chorionic gonadotropin(HCG)
C.either human menopausal gonadotropin (HMG ) or human chorionic gonadotropin(HCG)
D.human menopausal gonadotropin(HMG) plus human chorionic gonadotropin
E.estrogen
D.human menopausal gonadotropin(HMG) plus human chorionic gonadotropin
4 6 2014 22. Which of the following drugs is a progestin that commonly used in compound recipe of oral contraceptives? (2.5分) A.mestranol
B.diethylstilbestrol
C.norethindrone
D.metandienone
E.estradiol
C.norethindrone
4 6 2014 23. Orally- active androgenic steroid is(2.5分) A.methyltestosterone
B.diethylstilbestrol
C.clomiphene
D.ethinylestradiol
E.none of the above
A.methyltestosterone
4 6 2014 24. Which of the following is a nature estrogen(2.5分) A.diethylstilbestrolum
B.estradiol
C.quinestrol
D.ethinylestradiol
E.estradiol valerate
B.estradiol
4 6 2014 25. The main action of progesterone in the contraceptive drugs is to(2.5分) A.affect the normality of tubal
B.affect the uterine contraction
C.affect the fertilization
D.affect the ovulation
E.affect the placental function
D.affect the ovulation
4 6 2014 26. 下列属于孕激素作用的是(2.5分) A.刺激阴道上皮细胞分化
B.促进输卵管平滑肌收缩
C.促进子宫颈分泌稀薄的粘液
D.促进子宫内膜腺体增生、分泌
E.促进子宫平滑肌收缩
D.促进子宫内膜腺体增生、分泌
4 6 2014 27. Which of the following organs can produce androgens?(2.5分) A.testes
B.ovaries
C.adrenal cortex
D.Both testes and ovaries are correct.
E.All of the choices are correct.
E.All of the choices are correct.
4 6 2014 28. 体内精子储存在(2.5分) A.睾丸
B.前列腺
C.精囊腺
D.附睾和输精管
E.尿道球腺
D.附睾和输精管
4 6 2014 29. 关于男性的生精过程描述正确的是(2.5分) A.受垂体激素调控
B.从出生时开始
C.当年老后会完全终止
D.整个过程持续大约30天
E.生精小管里形成单倍体精子后具有受精能力
A.受垂体激素调控
4 6 2014 30. 调控阴茎勃起不需要(2.5分) A.中枢系统的调控和唤起
B.龟头机械感受器的刺激
C.交感神经的兴奋
D.血管壁内NO的作用
E.勃起后对静脉的压迫
C.交感神经的兴奋
4 6 2014 31. 雄激素不能够促进(2.5分) A.性冲动
B.肌肉含量
C.骨密度
D.红细胞数
E.GnRH的分泌
E.GnRH的分泌
4 6 2014 32. 以下哪种激素在排卵期前一天会诱导LH迅速增高? (2.5分) A.孕酮
B.雌激素
C.睾酮
D.FSH
E.GnRH
B.雌激素
4 6 2014 33. 下列关于雌激素生理作用的叙述,正确的是(2.5分) A.抑制输卵管运动
B.促进子宫内膜腺体增生和分泌
C.促进阴道上皮细胞增生和角化
D.促进乳腺发育并引起泌乳
E.促进脂肪组织分布于脏器周围
C.促进阴道上皮细胞增生和角化
4 6 2014 34. 以下关于受精过程错误的是(2.5分) A.受精后,卵子才能完成第二次减数分裂
B.受精后,卵子释放第二极体
C.受精前精子需要经过获能
D.输卵管蠕动能够帮助精子向内游动
E.受精时,顶体反应溶解透明带后,精子钻入卵子内
E.受精时,顶体反应溶解透明带后,精子钻入卵子内
4 6 2014 35. Menopause (2.5分) A.is the cessation of menstrual cycles.
B.results in low plasma levels of estrogens.
C.occurs as a result of cessation of gonadotropin secretion.
D.Both is the cessation of menstrual cycles and results in low plasma levels of estrogens are correct.
E.All of the choices are correct.
D.Both is the cessation of menstrual cycles and results in low plasma levels of estrogens are correct.
4 6 2014 36. 雌二醇由下列哪种物质衍生而来(2.5分) A.肽
B.多糖
C.甘油三酯
D.胆固醇
E.氨基化合物
D.胆固醇
4 6 2014 37. 一孕妇在分娩时被给予激素处理以帮助宫颈张开,这一激素生理状态下是下列哪一组织器官分泌的? (2.5分) A.肾上腺皮质
B.垂体前叶
C.下丘脑
D.乳腺
E.垂体后叶
E.垂体后叶
4 6 2014 38. 下列哪种激素是排卵期基础体温升高的原因? (2.5分) A.雌激素
B.人绒毛膜促性腺激素(HCG)
C.黄体生成素(LH)
D.孕酮
E.催乳素
D.孕酮
4 6 2014 39. 以下对于促卵泡激素(FSH)描述正确的是(2.5分) A.是一类固醇激素
B.雄性没有太多FSH,它对雄性生殖系统的功能没有太多作用
C.由卵巢或睾丸分泌
D.能刺激生殖细胞的成熟
E.对于GnRH的释放有正反馈作用而无负反馈作用
D.能刺激生殖细胞的成熟
4 6 2014 40. 下列哪项不是急性HIV感染的临床表现( )(2.5分) A.发热
B.咽痛
C.淋巴结肿大
D.口腔真菌感染
D.口腔真菌感染
4 6 2015 1. 艾滋病的全称是(2.5分) A.免疫缺陷综合征
B.获得性免疫缺陷综合征
C.先天性免疫缺陷综合征
D.继发性免疫缺陷综合征
B.获得性免疫缺陷综合征
4 6 2015 2. HIV感染人体后主要侵犯和破坏哪种细胞(2.5分) A.红细胞
B.白细胞
C.T淋巴细胞
D.巨噬细胞
C.T淋巴细胞
4 6 2015 3. 下列哪项检查能够最终确定病人感染了艾滋病毒(2.5分) A.血常规
B.HIV抗体确诊试验
C.HIV抗体筛查试验
D.尿常规
B.HIV抗体确诊试验
4 6 2015 4. 尿道内尖锐湿疣最佳治疗方法是(2.5分) A.液氮冷冻
B.CO2激光治疗
C.艾拉-光动力治疗
D.微波治疗
E.鬼臼毒素治疗
C.艾拉-光动力治疗
4 6 2015 5. 肛周尖锐湿疣最需要和以下哪种疾病相鉴别?(2.5分) A.肛周湿疹
B.皮脂腺异位
C.阴茎珍珠样丘疹病
D.扁平湿疣
E.痔疮
D.扁平湿疣
4 6 2015 6. 急性淋病的平均潜伏期是:(2.5分) A.3-5d
B.2w
C.3w
D.3m
E.6m
A.3-5d
4 6 2015 7. 淋病治疗首先药物是(2.5分) A.青霉素
B.多烯环素
C.环丙沙星
D.氧氟沙星
E.头孢曲松
E.头孢曲松
4 6 2015 8. 有关小儿淋病的描述哪项是不正确的(2.5分) A.男性患儿多于女性;
B.发病率升高与父母感染淋病有关;
C.治疗首选头孢曲松针;
D.临床上多表现为结膜炎、尿道炎和阴道炎;
E.好发年龄多为0-1岁患儿。
A.男性患儿多于女性;
4 6 2015 9. 有关淋球菌耐药情况的描述,哪项是不正确的?(2.5分) A.大多对青霉素耐药;
B.对头孢曲松敏感性降低,有少数耐药株出现;
C.头孢曲松耐药株出现主要由质粒介导引起;
D.大多对四环素耐药;
E.大多对氟喹诺酮类耐药。
C.头孢曲松耐药株出现主要由质粒介导引起;
4 6 2015 10. 精索的被膜中,下列说法正确的是(2.5分) A.精索外筋膜是腹内斜肌腱膜的延续
B.精索内筋膜是腹横肌腱膜的延续
C.提睾肌是腹内斜肌和腹横肌的延续
D.三层被膜均位于阴囊肉膜浅面
E.睾丸和精索被膜截然不同
C.提睾肌是腹内斜肌和腹横肌的延续
4 6 2015 11. 男性,9岁,因阴囊肿大就诊。经查阴囊透光试验阳性,表示某结构内有液体积存,该结构是(2.5分) A.睾丸固有鞘膜腔
B.附睾
C.睾丸
D.精索
E.输精管
A.睾丸固有鞘膜腔
4 6 2015 12. 一青年女子下腹部剧痛2小时为主诉人院,经查为输卵管宫外孕。输卵管部位的宫外孕最易发生于何处? (2.5分) A.输卵管子宫部
B.输卵管峡
C.输卵管壶腹
D.输卵管漏斗
E.输卵管子宫口附近
B.输卵管峡
4 6 2015 13. In combination oral contraceptives, the progestin is added primarily to(2.5分) A.ensure prompt withdrawal
B.decrease cervical mucous
C.inhibit ovarian estrogen metabolism
D.increase fallopian tube contractility
E.none of above
E.none of above
4 6 2015 14. Which is used to treat infertility in women(2.5分) A.human menopausal gonadotropin(HMG)
B.human chorionic gonadotropin(HCG)
C.either human menopausal gonadotropin (HMG ) or human chorionic gonadotropin(HCG)
D.human menopausal gonadotropin(HMG) plus human chorionic gonadotropin
E.estrogen
D.human menopausal gonadotropin(HMG) plus human chorionic gonadotropin
4 6 2015 15. Which of the following is commonly used as a progestin in combined oral contraceptives?(2.5分) A.mestranol
B.diethylstilbestrol
C.norethynodrel
D.medroxyprogesterone
E.none of the above
E.none of the above
4 6 2015 16. Orally- active androgenic steroid is(2.5分) A.methyltestosterone
B.diethylstilbestrol
C.clomiphene
D.ethinylestradiol
E.none of the above
A.methyltestosterone
4 6 2015 17. Which one of the following is a synthetic estrogen used in oral contraceptives?(2.5分) A.mestranol
B.norgestrel
C.clomiphene
D.estradiol
E.norethindrone
A.mestranol
4 6 2015 18. Which of the following is used for the treatment of aplastic anemia(2.5分) A.ACTH
B.estradiol
C.progesterone
D.androgen
E.throid hormone
D.androgen
4 6 2015 19. The main action of progesterone in the contraceptive drugs is to(2.5分) A.affect the normality of tubal
B.affect the uterine contraction
C.affect the fertilization
D.affect the ovulation
E.affect the placental function
D.affect the ovulation
4 6 2015 20. Which disease in postmenopausal women can't be effectively treated by supplementation of estrogen? (2.5分) A.Osteoporosis
B.Alzheimer disease
C.Postmenopausal syndrome
D.Ovarian cancer
E.Functional uterine bleeding
D.Ovarian cancer
4 6 2015 21. 哪些皮肤粘膜疹有传染性?(2.5分) A.扁平湿疣
B.滤泡性溃疡
C.树胶样肿
D.扁平苔藓
E.多形红斑
A.扁平湿疣
4 6 2015 22. 梅毒治疗的首选药物是(2.5分) A.螺旋霉素
B.长效青霉素
C.四环素
D.庆大霉素
E.氯霉素
B.长效青霉素
4 6 2015 23. 下列梅毒螺旋体检查方法中哪些是正确的(2.5分) A.暗视野法
B.培养
C.涂片做PAS染色
D.苛性钾法
E.荧光染色
A.暗视野法
4 6 2015 24. 一期梅毒的诊断标准不包括下列哪项(2.5分) A.有不洁性交史
B.早期有流感症状
C.外生殖器单个无痛性下疳
D.潜伏期3周左右
E.梅毒血清试验阳性
B.早期有流感症状
4 6 2015 25. 下列不见于二期梅毒表现的是(2.5分) A.梅毒玫瑰疹
B.扁平湿疣
C.树胶肿
D.粘膜损害
E.梅毒性脱发
C.树胶肿
4 6 2015 26. 关于一期梅毒,哪项是错误的?(2.5分) A.一般患者起病前有不洁性交史
B.潜伏期是 2~4 周左右
C.硬下疳是主要的表现
D.在硬下疳处取材以暗视野检查可见苍白螺旋体
E.梅毒血清试验阴性可排除一期梅毒
E.梅毒血清试验阴性可排除一期梅毒
4 6 2015 27. 作定量试验,用于观察梅毒疗效、复发及再感染的血清试验是(2.5分) A.TPHA
B.RPR
C.HIV抗体检测
D.FTA-ABS
E.HSV抗体
B.RPR
4 6 2015 28. 不符合三期梅毒疹特点的是(2.5分) A.单侧分布
B.破坏性大,愈后留疤痕
C.附近淋巴结不肿大
D.传染性大
E.自觉症状很轻,但客观症状严重
D.传染性大
4 6 2015 29. 下列描述哪项错误(2.5分) A.未经治疗的硬下疳都会发展为三期梅毒
B.二期梅毒传染性强
C.梅毒性秃发为不可逆性秃发
D.接吻也可传播梅毒
E.梅毒血清固定没有传染性
C.梅毒性秃发为不可逆性秃发
4 6 2015 30. Which of the following organs can produce androgens?(2.5分) A.testes
B.ovaries
C.adrenal cortex
D.Both testes and ovaries are correct.
E.All of the choices are correct.
E.All of the choices are correct.
4 6 2015 31. 体内精子储存在(2.5分) A.睾丸
B.前列腺
C.精囊腺
D.附睾和输精管
E.尿道球腺
D.附睾和输精管
4 6 2015 32. 关于成熟精子描述不正确的是(2.5分) A.头部主要由顶体和细胞核
B.中部主要的细胞器是线粒体
C.主要的细胞器是细胞核和内质网
D.长长的尾部主要用于游动
E.直线游动能力是其活性的检测指标之一
C.主要的细胞器是细胞核和内质网
4 6 2015 33. 关于男性的生精过程描述正确的是(2.5分) A.受垂体激素调控
B.从出生时开始
C.当年老后会完全终止
D.整个过程持续大约30天
E.生精小管里形成单倍体精子后具有受精能力
A.受垂体激素调控
4 6 2015 34. 调控阴茎勃起不需要(2.5分) A.中枢系统的调控和唤起
B.龟头机械感受器的刺激
C.交感神经的兴奋
D.血管壁内NO的作用
E.勃起后对静脉的压迫
C.交感神经的兴奋
4 6 2015 35. 对孕酮生理作用阐述错误的是?(2.5分) A.降低体温
B.使子宫内膜适于胚胎着床
C.诱发粘厚的宫颈粘液
D.抑制阴道上皮细胞增殖
E.激活乳腺增长
A.降低体温
4 6 2015 36. 在孕期,以下哪种激素刺激黄体的持续存在?(2.5分) A.孕酮
B.雌激素
C.FSH
D.LH
E.hCG
E.hCG
4 6 2015 37. 妊娠后期孕妇血中雌激素和孕激素处于高水平的原因是(2.5分) A.下丘脑促垂体区分泌活动加强
B.腺垂休分泌活动加强
C.卵巢分泌活动加强
D.胎盘分泌活动加强
E.胆固醇被加速用来合成激素
D.胎盘分泌活动加强
4 6 2015 38. Menopause(2.5分) A.is the cessation of menstrual cycles.
B.results in low plasma levels of estrogens.
C.occurs as a result of cessation of gonadotropin secretion.
D.Both is the cessation of menstrual cycles and results in low plasma levels of estrogens are correct.
E.All of the choices are correct.
D.Both is the cessation of menstrual cycles and results in low plasma levels of estrogens are correct.
4 6 2015 39. 下列哪种激素是排卵期基础体温升高的原因?(2.5分) A.雌激素
B.人绒毛膜促性腺激素(HCG)
C.黄体生成素(LH)
D.孕酮
E.催乳素
D.孕酮
4 6 2015 40. Beginning on the first day of the menstrual cycle, the order of events in the uterus is(2.5分) A.proliferative, secretory, menstrual phases.
B.secretory, proliferative, menstrual phases.
C.menstrual, secretory, proliferative phases.
D.menstrual, proliferative, secretory phases.
E.secretory, menstrual, proliferative phases.
D.menstrual, proliferative, secretory phases.
4 7 2006 1. 老年骨质疏松宜选用 (2.0分) A.黄体酮
B.甲基睾酮
C.苯丙酸诺龙
D.氯米芬
E.炔诺酮
C.苯丙酸诺龙
4 7 2006 2. 下列哪类人群不属于艾滋病毒感染的高危人群(2.0分) A.静脉吸毒者
B.性乱交者
C.同性恋者
D.无偿献血人员
D.无偿献血人员
4 7 2006 3. 勃起功能障碍的治疗手段不包括下列哪项?(2.0分) A.心理治疗
B.包皮环切术
C.药物治疗
D.激素替代
E.阴茎假体植入
B.包皮环切术
4 7 2006 4. 乳腺癌早期诊断的主要手段?(2.0分) A.定期的专科体检
B.频繁的自我检查。
C.定期乳腺B超±乳腺钼靶
C.定期乳腺B超±乳腺钼靶
4 7 2006 5. HIV感染人体后主要侵犯和破坏哪种细胞(2.0分) A.红细胞
B.T淋巴细胞
C.白细胞
D.巨噬细胞
B.T淋巴细胞
4 7 2006 6. 当HIV侵入机体后,未进入发病期者被称为(2.0分) A.艾滋病病毒感染者
B.艾滋病病人
C.窗口期病人
D.艾滋病期病人
A.艾滋病病毒感染者
4 7 2006 7. 关于梅毒下疳的叙述,错误的是(2.0分) A.损害常为多发
B.为圆形或椭圆形、境界清楚的溃疡
C.无疼痛与压痛
D.触诊有软骨样硬度
E.不经治疗3~8周自然消失
A.损害常为多发
4 7 2006 8. HPV主要通过哪种途径感染(2.0分) A.直接接触
B.血液传播
C.呼吸道
D.消化道
E.蚊虫叮咬
A.直接接触
4 7 2006 9. 心血管梅毒一般不会出现下列哪种表现(2.0分) A.主动脉瘤
B.主动脉关闭不全
C.冠状动脉狭窄
D.主动脉炎
E.二尖瓣关闭不全
E.二尖瓣关闭不全
4 7 2006 10. 患者在一次不洁性生活后出现尿急,尿痛。尿道口流脓。本患者最可能的诊断(2.0分) A.尿道念珠菌感染
B.生殖道衣原体感染
C.淋病
D.细菌性尿道炎
C.淋病
4 7 2006 11. 急性乳腺炎常见细菌(2.0分) A.表皮葡萄球菌
B.金黄色葡萄球菌或链球菌
C.牙龈二氧化碳噬纤维菌
B.金黄色葡萄球菌或链球菌
4 7 2006 12. 梅毒治疗的首选药物是 (2.0分) A.多西环素
B.长效青霉素
C.阿奇霉素
D.头孢三嗪
E.复方新诺明
B.长效青霉素
4 7 2006 13. 下列不见于二期梅毒表现的是(2.0分) A.树胶肿
B.扁平湿疣
C.梅毒疹
D.粘膜损害
A.树胶肿
4 7 2006 14. 以下不是吉-海反应的表现是(2.0分) A.原发疾病加重
B.寒战,发热
C.主动脉破裂
D.RPR滴度急剧上升
D.RPR滴度急剧上升
4 7 2006 15. 淋病的治愈标准为(2.0分) A.A与C均正确
B.尿液常规检查阴性
C.在治疗结束后第4天及第8天涂片培养二次均阴性
D.症状体征全部消失
E.A、B与C均正确
E.A、B与C均正确
4 7 2006 16. 尖锐湿疣的病原体(2.0分) A.人乳头状瘤病毒
B.传染性软疣病毒
C.疱疹病毒
D.痘病毒
A.人乳头状瘤病毒
4 7 2006 17. 艾滋病病毒感染人体后临床症状分期错误的提法是(2.0分) A.无症状感染期
B.病毒携带期
C.急性感染期
D.艾滋病期
B.病毒携带期
4 7 2006 18. 西地那非(万艾可)治疗勃起功能障碍的作用机制是因为它是一种(2.0分) A.β受体激动剂
B.α肾上腺受体拮抗剂
C.多巴胺受体激动剂
D.PDE5抑制剂
E.一氧化氮供体
D.PDE5抑制剂
4 7 2006 19. 关于三期梅毒共同点的叙述,错误的是(2.0分) A.愈后留有萎缩性瘢痕,面部皮肤毁容
B.自觉症状很轻,但客观症状严重
C.损害数目多、分布对称
D.损害内梅毒螺旋体少,传染性弱或无传染性
E.梅毒血清试验阳性率低
C.损害数目多、分布对称
4 7 2006 20. 下列哪项不是睾丸肿瘤的血清标记物(2.0分) A.PSA
B.AFP
C.HCG
D.LDH
E.PALP
A.PSA
4 7 2006 21. 以下属于高危型的HPV是(2.0分) A.HPV11
B.HPV16
C.HPV6
D.HPV2
B.HPV16
4 7 2006 22. 以下哪项不是软下疳的临床特点(2.0分) A.溃疡多发
B.主要通过性接触传播
C.无痛
D.男性多见
C.无痛
4 7 2006 23. 下列哪项不属于良性前列腺增生的特点?(2.0分) A.常发生于前列腺周围区
B.增生的腺体有两层细胞构成
C.发病与雄激素水平有关
D.不易恶变为前列腺癌
A.常发生于前列腺周围区
4 7 2006 24. 晚期梅毒病人除皮肤黏膜损害外,发病率最高的是 (2.0分) A.骨梅毒
B.神经梅毒
C.眼梅毒
D.心血管梅毒
E.口腔梅毒
A.骨梅毒
4 7 2006 25. 前列腺上皮内瘤变(PIN)和浸润性前列腺癌最主要区别是前者(2.0分) A.无相应的基因水平改变
B.异型上皮细胞外有一层基底细胞
C.细胞的异型程度较小
D.累及范围较小
B.异型上皮细胞外有一层基底细胞
4 7 2006 26. 有关精原细胞瘤形态特点哪项不正确? (2.0分) A.瘤细胞形成小叶状结构
B.部分病例可伴有滋养细胞分化
C.间质常有淋巴细胞浸润
D.瘤细胞间分界不清
D.瘤细胞间分界不清
4 7 2006 27. 前列腺癌Gleason分级中最常见的生长方式是(2.0分) A.Gleason 2级
B.Gleason 1级
C.Gleason 3级
D.Gleason 4级
E.Gleason 5级
C.Gleason 3级
4 7 2006 28. 炎性乳腺癌的预后?(2.0分) A.预后中等
B.不易转移,预后良好
C.极易远处转移而极差
C.极易远处转移而极差
4 7 2006 29. 雄激素的临床用途有(2.0分) A.习惯性流产
B.痛经止痛
C.人工周期
D.不孕症
E.功能性子宫出血
E.功能性子宫出血
4 7 2006 30. 乳腺癌的局部治疗方法?(2.0分) A.内分泌治疗和曲妥珠单抗靶向治疗
B.化疗
C.外科手术、放疗
C.外科手术、放疗
4 7 2006 31. 睾丸肿瘤的多学科治疗不包括下列哪个选项(2.0分) A.放疗
B.根治性睾丸切除
C.化疗
D.腹股沟淋巴结清扫
E.腹膜后淋巴结清扫
D.腹股沟淋巴结清扫
4 7 2006 32. 下列哪种病理类型的睾丸肿瘤不是生殖细胞来源(2.0分) A.精原细胞瘤
B.绒癌
C.移行细胞癌
D.畸胎瘤
E.胚胎癌
C.移行细胞癌
4 7 2006 33. 新生儿淋菌性眼炎主要的感染途径是(2.0分) A.接触被分泌物污染的物品
B.产道传染
C.自我接种
D.哺乳
B.产道传染
4 7 2006 34. 下列哪项不是睾丸肿瘤的病因(2.0分) A.睾丸外伤后血运障碍
B.睾丸鞘膜积液
C.隐睾
D.长期毒物接触,如氧化锌
E.病毒感染
B.睾丸鞘膜积液
4 7 2006 35. 世界卫生组织推荐的诊断艾滋病的主要症状是(2.0分) A.持续性或间歇性慢性腹泻1个月以上
B.发热1个月以上
C.体重减轻10%以上
D.体重减轻10%以上,发热1个月以上和持续性或间歇性慢性腹泻1个月以上
D.体重减轻10%以上,发热1个月以上和持续性或间歇性慢性腹泻1个月以上
4 7 2006 36. 阴茎海绵体注射治疗阴茎勃起功能障碍的药物不包括 (2.0分) A.罂粟碱
B.前列腺素E1
C.西地那非
D.酚妥拉明
E.血管肠肽
C.西地那非
4 7 2006 37. 艾滋病人体体液含有大量病毒,具有很强传染性的是(2.0分) A.尿液
B.血液
C.泪液
D.汗液
B.血液
4 7 2006 38. 患者在一次不洁性生活后出现尿急,尿痛。尿道口流脓。首先选择的检查是(2.0分) A.细菌的PCR 检测
B.细菌培养
C.细菌涂片革兰氏染色
D.以上都不是
C.细菌涂片革兰氏染色
4 7 2006 39. 关于一期梅毒淋巴结炎的叙述,错误的是(2.0分) A.常累及单侧腹股沟淋巴结或患处附近淋巴结
B.发生于硬下疳出现1~2周后
C.质地较硬,表面常可见红肿或破溃,疼痛明显,且有触痛
D.消退常需数月
E.淋巴结穿刺检查可见大量苍白螺旋体
C.质地较硬,表面常可见红肿或破溃,疼痛明显,且有触痛
4 7 2006 40. 前列腺癌血道转移最常见的器官是(2.0分) A.肝
B.骨
C.肺
D.脑
E.肾
B.骨
4 7 2006 41. 浆细胞性乳腺炎与乳腺癌的鉴别(2.0分) A.活检病理
B.乳腺有无窦道或瘘管形成
C.乳腺有无红肿热痛等临床特征
A.活检病理
4 7 2006 42. 乳腺增生症外科干预的指征是什么?(2.0分) A.乳腺疼痛明显
B.乳腺专科查体发现乳腺呈结节状
C.怀疑乳腺恶性肿瘤(BI-RADS ≥4分)
C.怀疑乳腺恶性肿瘤(BI-RADS ≥4分)
4 7 2006 43. 急性乳腺炎常见于哪些人群?(2.0分) A.非初产妇,多次哺乳的妇女
B.产后2-6周哺乳期妇女,初产妇多见
C.免疫力低下的妇女,如糖尿病患者等
B.产后2-6周哺乳期妇女,初产妇多见
4 7 2006 44. 哪种患者禁用雄激素(2.0分) A.乳腺癌
B.贫血
C.胃十二指肠溃疡
D.前列腺癌
E.急慢性肝炎
D.前列腺癌
4 7 2006 45. 急性乳腺炎的治疗?(2.0分) A.抗感染治疗,如有脓肿形成则行脓肿切开引流术
B.定期复查
C.保守治疗即可
A.抗感染治疗,如有脓肿形成则行脓肿切开引流术
4 7 2006 46. 肝功能不良者服用性激素最易发生(2.0分) A.消化不良
B.胆汁淤积性黄疸
C.病毒性肝炎
D.肝良性腺瘤
E.肝硬化
B.胆汁淤积性黄疸
4 7 2006 47. 夜间阴茎胀大试验的作用是(2.0分) A.判断勃起功能障碍的程度
B.测定阴茎的粗细和长度
C.测定阴茎勃起的硬度
D.鉴别动脉性还是静脉性勃起功能障碍
E.鉴别心理性还是器质性勃起功能障碍
E.鉴别心理性还是器质性勃起功能障碍
4 7 2006 48. 睾丸功能不全宜选用(2.0分) A.美雄酮
B.炔诺酮
C.甲基睾酮
D.黄体酮
E.甲地孕酮
C.甲基睾酮
4 7 2006 49. 下列哪项不属于疣状癌的的特点?(2.0分) A.形态与尖锐湿疣相似
B.仅局部浸润,极少转移
C.发病与HPV感染无关
D.为高分化鳞癌
C.发病与HPV感染无关
4 7 2006 50. 乳腺癌化疗的最常用一线药物?(2.0分) A.紫杉类,如紫杉醇、多烯紫杉醇等
B.蒽环类,如阿霉素、表阿霉素等
C.铂类,如卡铂、顺铂等。
B.蒽环类,如阿霉素、表阿霉素等
4 7 2007 1. 治疗男性勃起功能障碍(ED)的药物,包括: (2.5分) A.血管紧张素环化酶(ACE)抑制剂
B.各种增高cAMP浓度的药物
C.磷酸二酯酶5(PDE5)抑制剂
D.各种前列腺素及其制剂
E.花生四烯酸环氧酶2(COX2)抑制剂
C.磷酸二酯酶5(PDE5)抑制剂
4 7 2007 2. 睾丸肿瘤的多学科治疗不包括下列哪个选项(2.5分) A.放疗
B.腹股沟淋巴结清扫
C.根治性睾丸切除
D.化疗
E.腹膜后淋巴结清扫
B.腹股沟淋巴结清扫
4 7 2007 3. Sildenafil(西地那非)主要用于治疗 (2.5分) A.男性性欲低下
B.早泄(PE,提前射精)
C.肺动脉高压症(PAH)
D.男性勃起功能障碍(ED)
E.ED和PAH
E.ED和PAH
4 7 2007 4. 前列腺增生症对人体最大的危害在于: (2.5分) A.常引起尿路结石
B.常引起排尿障碍
C.常引起内分泌紊乱
D.易引起癌变
E.以上都不是
A.常引起尿路结石
4 7 2007 5. 乳腺最常见的良性肿瘤是(2.5分) A.腺瘤
B.纤维瘤
C.导管内乳头状瘤
D.纤维腺瘤
D.纤维腺瘤
4 7 2007 6. 37、Which one is incorrect about breast fibroadenomas? (2.5分) A.Fibroadenomas are benign solid tumors composed of stromal and epithelial elements
B.They are rarely seen as new masses in women after the age of 40 or 45
C.Fibroadenoma is the most common tumor in women younger than 30 years
D.On excision, fibroadenomas are poor-encapsulated masses that may detach easily from surrounding breast tissue
E.Mammography is of little help in distinguishing between cysts and fibroadenomas
D.On excision, fibroadenomas are poor-encapsulated masses that may detach easily from surrounding breast tissue
4 7 2007 7. 阴茎海绵体注射治疗阴茎勃起功能障碍的药物不包括 ___?(2.5分) A.前列腺素E1
B.酚妥拉明
C.西地那非
D.罂粟碱
E.血管肠肽
C.西地那非
4 7 2007 8. 关于精原细胞瘤的叙述,下列哪项是错误的: (2.5分) A.隐睾症者其发病率高
B.起源于原始生殖细胞
C.瘤细胞由单一性细胞组成
D.恶性程度很高,易远处转移
E.对放射治疗敏感
D.恶性程度很高,易远处转移
4 7 2007 9. 与典型睾丸精原细胞瘤相应的卵巢肿瘤是: (2.5分) A.畸胎瘤
B.卵泡膜细胞瘤
C.无性细胞瘤
D.颗粒细胞瘤
E.内胚窦瘤
C.无性细胞瘤
4 7 2007 10. 属于男生殖系统非特异性感染的是: (2.5分) A.附睾结核
B.淋病
C.尖锐湿疣
D.急性前列腺炎
D.急性前列腺炎
4 7 2007 11. 乳腺疾病最常用的检查是?(2.5分) A.钼靶+MRI
B.乳腺B超±乳腺钼靶
C.乳腺磁共振检查
D.乳腺红外线检查
E.肿瘤标志物检测
B.乳腺B超±乳腺钼靶
4 7 2007 12. 下列哪项不是睾丸肿瘤的病因(2.5分) A.长期毒物接触,如氧化锌
B.睾丸鞘膜积液
C.睾丸外伤后血运障碍
D.隐睾
E.病毒感染
B.睾丸鞘膜积液
4 7 2007 13. 乳腺增生症外科干预的指征是:(2.5分) A.乳腺专科查体发现乳腺呈结节状
B.乳腺疼痛明显
C.考虑恶变可能(BI-RADS ≥4分)
D.有乳头溢液症状
E.单侧乳腺结节
C.考虑恶变可能(BI-RADS ≥4分)
4 7 2007 14. 下列哪项不属于睾丸肿瘤的临床表现(2.5分) A.精索静脉曲张
B.无痛性睾丸肿块
C.下腹部或腹股沟区钝痛
D.男性乳房发育
E.疼痛性睾丸肿块伴局部红肿
A.精索静脉曲张
4 7 2007 15. 以下不属于男生殖系统先天畸形的是:(2.5分) A.隐睾
B.尿道下裂
C.尿道狭窄
D.隐匿性阴茎
C.尿道狭窄
4 7 2007 16. 以下哪项不是阴茎癌的癌前病变(2.5分) A.阴茎角化过度
B.阴茎粘膜白斑
C.阴茎珍珠样丘疹
D.阴茎乳突状瘤
E.干燥性闭塞性龟头炎
C.阴茎珍珠样丘疹
4 7 2007 17. 下列哪项不是勃起功能障碍的发病因素?(2.5分) A.输精管结扎
B.糖尿病
C.高血压
D.精神心理疾病
E.吸烟
A.输精管结扎
4 7 2007 18. 阴茎勃起过程中最关键的神经递质是____?(2.5分) A.多巴胺
B.乙酰胆碱
C.γ-氨基丁酸
D.一氧化氮
E.肾上腺素
D.一氧化氮
4 7 2007 19. 夜间阴茎胀大试验的作用是____?(2.5分) A.测定阴茎勃起的硬度
B.判断勃起功能障碍的程度
C.测定阴茎的粗细和长度
D.鉴别动脉性还是静脉性勃起功能障碍
E.鉴别心理性还是器质性勃起功能障碍
E.鉴别心理性还是器质性勃起功能障碍
4 7 2007 20. 勃起功能障碍的治疗手段不包括下列哪项?(2.5分) A.药物治疗
B.心理治疗
C.包皮环切术
D.激素替代
E.阴茎假体植入
C.包皮环切术
4 7 2007 21. 前列腺增生的病变特点是:(2.5分) A.前列腺腺体肥大
B.前列腺平滑肌增生
C.前列腺间质纤维组织增生
D.前列腺腺体增生
E.前列腺腺体、平滑肌和纤维组织均增生
E.前列腺腺体、平滑肌和纤维组织均增生
4 7 2007 22. 下列关于浆细胞性乳腺炎错误的是:(2.5分) A.其治疗主要以抗炎治疗和手术完整切除病灶为主
B.目前大多数学者认为其是无菌性化学性炎症
C.浆细胞性乳腺的肿块易误诊为癌
D.经产妇绝经期前后好发
E.该病自然病程多为2-4周
E.该病自然病程多为2-4周
4 7 2007 23. 阴茎癌的危险因素有(2.5分) A.尖锐湿疣
B.包茎
C.包皮过长
D.HPV感染
E.以上都是
E.以上都是
4 7 2007 24. 以下不属于生殖系统器官的是:(2.5分) A.睾丸
B.膀胱
C.前列腺
D.精囊
B.膀胱
4 7 2007 25. 关于阴茎癌的叙述,错误的是: (2.5分) A.大多为分化差的鳞状细胞癌
B.常有包茎史
C.常发生于40岁左右
D.肿瘤初起位于阴茎头部
E.早期即可发生双侧腹股沟淋巴结转移
A.大多为分化差的鳞状细胞癌
4 7 2007 26. 与乳腺癌发生有关的病变是(2.5分) A.硬化性腺病
B.乳腺纤维腺瘤
C.囊肿伴大汗腺化生
D.乳腺纤维囊性变伴导管上皮增生
D.乳腺纤维囊性变伴导管上皮增生
4 7 2007 27. 急性乳腺炎常见于哪些人群?(2.5分) A.非初产妇,多次哺乳的妇女
B.孕晚期妇女
C.产后2-6周哺乳期妇女,初产妇多见
D.青春期女性
E.免疫力低下的妇女,如糖尿病患者等
C.产后2-6周哺乳期妇女,初产妇多见
4 7 2007 28. 浆细胞性乳腺炎慢性期的临床特点?(2.5分) A.乳腺触及波动感的肿块但尚无皮损
B.窦道或瘘管的形成
C.乳头溢脓伴局部红肿
D.局部皮肤有桔皮征出现
E.乳头血性溢液
B.窦道或瘘管的形成
4 7 2007 29. 关于急性乳腺炎的规范治疗下列正确的是? (2.5分) A.发病初期即行手术清创
B.局部理疗
C.抗感染治疗,如有脓肿形成则行脓肿切开引流术
D.定期复查
E.清热解毒中成药治疗
C.抗感染治疗,如有脓肿形成则行脓肿切开引流术
4 7 2007 30. 诊断高分化前列腺癌最可靠的证据是: (2.5分) A.包膜浸润
B.血管浸润
C.淋巴管浸润
D.周围神经浸润
E.以上都是
A.包膜浸润
4 7 2007 31. 乳腺癌淋巴转移时首先转移到(2.5分) A.乳内淋巴结
B.锁骨上淋巴结
C.同侧腋窝淋巴结
D.锁骨下淋巴结
C.同侧腋窝淋巴结
4 7 2007 32. 前列腺癌最常见于前列腺的: (2.5分) A.双侧叶
B.中性
C.前叶
D.后叶
E.顶叶
D.后叶
4 7 2007 33. Inflammatory breast cancer has the characteristics except:(2.5分) A.It is clinically manifested as erythema, edema, and warmth of the breast as a result of lymphatic obstruction
B.A palpable mass is required for the diagnosis
C.Peau d'orange is the term used to describe the orange peel appearance of the skin
D.Axillary nodal metastases are common
E.There is a significant risk for distant metastases
B.A palpable mass is required for the diagnosis
4 7 2007 34. About Paget's disease, which one is incorrect? (2.5分) A.A skin specimen containing Paget cells secures the diagnosis
B.There is a low risk for distant metastases
C.It is characterized clinically by nipple erythema and irritation, which may progress to nipple crusting and ulceration
D.Paget's disease accounts for 1% or less of breast malignancies
E.Paget's Disease is usually seen in young women
E.Paget's Disease is usually seen in young women
4 7 2007 35. The most reliable method to determine the nature of breast lumps is (2.5分) A.Breast MRI
B.Mammography
C.Ultrasonography
D.Discharge smear
E.Biopsy
E.Biopsy
4 7 2007 36. Which one is the indication of using adjuvant target therapy for breast cancer? (2.5分) A.Her-2 (-)
B.CerbB2 (+)
C.CerbB2 (++)
D.CerbB2 (-)
E.Her-2 (+)
E.Her-2 (+)
4 7 2007 37. 下列乳腺癌预后差的是(2.5分) A.粘液癌
B.小叶癌
C.小管癌
D.浸润性导管癌
D.浸润性导管癌
4 7 2007 38. 勃起功能国际问卷(IIEF-5)的主要作用是___?(2.5分) A.寻找勃起功能障碍的病因
B.鉴别动脉性还是静脉性勃起功能障碍
C.鉴别心理性还是器质性勃起功能障碍
D.指导勃起功能障碍的治疗
E.判断勃起功能障碍的程度
E.判断勃起功能障碍的程度
4 7 2007 39. 关于乳腺癌预后,下列叙述不正确的是(2.5分) A.PR、ER均阳性者预后差
B.无淋巴结转移者预后好于有淋巴结转移者
C.乳腺粘液癌预后比浸润性导管癌好
D.Her-2阳性者预后差
A.PR、ER均阳性者预后差
4 7 2007 40. 鉴别前列腺原发癌和转移癌最有价值的免疫组化方法是检测: (2.5分) A.癌胚抗原
B.特异性酸性磷酸酶和前列腺特异性抗原
C.抑癌基因p53
D.碱性磷酸酶
E.血管源性生长因子受体
B.特异性酸性磷酸酶和前列腺特异性抗原
4 7 2008 1. 以下不属于生殖系统器官的是:(2.0分) A.膀胱
B.睾丸
C.前列腺
D.精囊
A.膀胱
4 7 2008 2. 属于男生殖系统非特异性感染的是: (2.0分) A.附睾结核
B.尖锐湿疣
C.淋病
D.急性前列腺炎
D.急性前列腺炎
4 7 2008 3. 以下不属于男生殖系统先天畸形的是:(2.0分) A.隐睾
B.尿道狭窄
C.尿道下裂
D.隐匿性阴茎
B.尿道狭窄
4 7 2008 4. 以下哪个是男生殖系统良性肿瘤: (2.0分) A.阴囊Paget氏病
B.精索横纹肌肉瘤
C.精原细胞瘤
D.阴茎血管瘤
D.阴茎血管瘤
4 7 2008 5. 下列哪一个是天然雄激素: (2.0分) A.Testosterone(睾酮)
B.Methyltestosterone(甲睾酮)
C.Testosterone propionate(丙酸睾丸素)
D.cyproterone(环丙孕酮)
E.Sidenafil(西地那非)
A.Testosterone(睾酮)
4 7 2008 6. 下列哪一个不属于同化激素(2.0分) A.Nandrolone phenpropionate(苯丙酸诺龙)
B.Sidenafil(西地那非)
C.Danazol(达那唑)
D.Stanozolol(司坦唑醇)
E.Metandienone(去氢甲睾酮)
B.Sidenafil(西地那非)
4 7 2008 7. 雄激素类药物不能用于(2.0分) A.孕妇、新生儿
B.妇科疾病
C.前列腺癌患者
D.再生障碍性贫血
E.A和C
E.A和C
4 7 2008 8. Sildenafil(西地那非)主要用于治疗(2.0分) A.男性勃起功能障碍(ED)
B.早泄(PE,提前射精)
C.肺动脉高压症(PAH)
D.男性性欲低下
E.ED和PAH
E.ED和PAH
4 7 2008 9. 治疗男性早泄(PE)的口服药物包括(2.0分) A.各种三环类抗抑郁药
B.抗躁狂药
C.选择性去甲肾上腺素再摄取抑制药
D.选择性5-羟色胺再摄取抑制剂
E.雄激素类药物
D.选择性5-羟色胺再摄取抑制剂
4 7 2008 10. 阴茎勃起过程中最关键的神经递质是(2.0分) A.多巴胺
B.乙酰胆碱
C.一氧化氮
D.γ-氨基丁酸
E.肾上腺素
C.一氧化氮
4 7 2008 11. 勃起功能国际问卷(IIEF-5)的主要作用是(2.0分) A.寻找勃起功能障碍的病因
B.鉴别心理性还是器质性勃起功能障碍
C.鉴别动脉性还是静脉性勃起功能障碍
D.指导勃起功能障碍的治疗
E.判断勃起功能障碍的程度
E.判断勃起功能障碍的程度
4 7 2008 12. 勃起功能障碍的治疗手段不包括下列哪项?(2.0分) A.心理治疗
B.药物治疗
C.包皮环切术
D.激素替代
E.阴茎假体植入
C.包皮环切术
4 7 2008 13. 阴茎海绵体注射治疗阴茎勃起功能障碍的药物不包括 (2.0分) A.前列腺素E1
B.罂粟碱
C.西地那非
D.酚妥拉明
E.血管肠肽
C.西地那非
4 7 2008 14. 下列哪项不是睾丸肿瘤的血清标记物(2.0分) A.HCG
B. AFP
C. PSA
D. LDH
E. PALP
C. PSA
4 7 2008 15. 睾丸肿瘤的多学科治疗不包括下列哪个选项(2.0分) A. 根治性睾丸切除
B.放疗
C. 化疗
D.腹股沟淋巴结清扫
E.腹膜后淋巴结清扫
D.腹股沟淋巴结清扫
4 7 2008 16. 下列哪项不属于睾丸肿瘤的临床表现(2.0分) A.无痛性睾丸肿块
B.精索静脉曲张
C.下腹部或腹股沟区钝痛
D.男性乳房发育
E.疼痛性睾丸肿块伴局部红肿
B.精索静脉曲张
4 7 2008 17. 阴茎癌最常见的病理类型是(2.0分) A.腺癌
B.移行细胞癌
C. 鳞状细胞癌
D.基底细胞癌
E.黑色素瘤
C. 鳞状细胞癌
4 7 2008 18. 阴茎癌的前哨淋巴结位于(2.0分) A.盆腔
B.腹膜后
C.腹股沟区
D. 腋窝
E.腹主动脉旁
C.腹股沟区
4 7 2008 19. WHO不育症定义:无保护性生活____不怀孕。(2.0分) A.1年
B.2年
C.3年
D.半年
E.18个月
A.1年
4 7 2008 20. 男性生殖生理中,LH的主要靶细胞是:(2.0分) A.Germ cell
B.Leydigcell
C.Sertoli cell
D.Syncytiotrophoblast
E.Spermatogonia
B.Leydigcell
4 7 2008 21. 下列五个选项中,哪个是最常见导致男性不育的因素(2.0分) A.酒精成瘾
B.大麻成瘾
C.精索静脉曲张
D.Klinefelter Syndrome
E.前列腺炎
C.精索静脉曲张
4 7 2008 22. 克氏综合征患者,精夜分析提示无精症,睾丸活检发现曲细精管内少量精子,患者有强烈的生育意愿,最佳后续选择是:(2.0分) A. ICSI
B.IUI
C.IVF-ET
D.激素替代治疗
E.Vasovasostomy
A. ICSI
4 7 2008 23. 正常精液的精子密度应该:(2.0分) A. >10*106/ml
B. >20*106/ml
C.>50*106/ml
D.>100*106/ml
E.>200*106/ml
B. >20*106/ml
4 7 2008 24. 性反应周期不包括以下哪一项(2.0分) A.性欲望
B.性兴奋
C.性高潮
D..射精
E.性不应期
E.性不应期
4 7 2008 25. 以下哪项不是性功能障碍的直接生物学因素(2.0分) A.糖尿病
B. 苯二氮卓药物依赖
C.酒精依赖
D.冠心病
E. 百忧解(抗抑郁剂)
D.冠心病
4 7 2008 26. 以下哪项不是性功能障碍的心理社会因素(2.0分) A.焦虑
B.抑郁
C. 神经性厌食
D. 恐惧
E.失眠
E.失眠
4 7 2008 27. 哪项不是导致性共障碍的非适应性认知 (2.0分) A.婚前性行为是错误的
B.手淫是肮脏的
C. 婚姻性行为是为传宗接代
D.男性在任何情况下都应该能够勃起
E.女性在性行为不会疼痛
E.女性在性行为不会疼痛
4 7 2008 28. 乳腺最常见的良性肿瘤是(2.0分) A.导管内乳头状瘤
B.纤维瘤
C.腺瘤
D.纤维腺瘤
D.纤维腺瘤
4 7 2008 29. 乳腺癌的橘皮样外观是由于(2.0分) A.癌组织阻塞真皮内淋巴管而引起皮肤水肿
B.癌组织侵犯表皮引起局部多发浅表溃疡
C.癌组织引起真皮纤维组织增生和收缩
D.癌组织阻塞真皮内导管导致局部导管扩张
A.癌组织阻塞真皮内淋巴管而引起皮肤水肿
4 7 2008 30. 关于乳腺癌预后,下列叙述不正确的是(2.0分) A.无淋巴结转移者预后好于有淋巴结转移者
B.PR、ER均阳性者预后差
C.乳腺粘液癌预后比浸润性导管癌好
D.Her-2阳性者预后差
B.PR、ER均阳性者预后差
4 7 2008 31. 与乳腺癌发生有关的病变是(2.0分) A.囊肿伴大汗腺化生
B.乳腺纤维腺瘤
C.硬化性腺病
D.乳腺纤维囊性变伴导管上皮增生
D.乳腺纤维囊性变伴导管上皮增生
4 7 2008 32. 下列关于乳腺增生症治疗错误的是:(2.0分) A.确诊的乳腺增生症不需外科治疗
B.药物不能根治,只能缓解部分病人的部分症状
C.乳腺增生症是良性疾病,一旦确诊无需随访复查。
D.手术治疗的主要目的是活检以明确诊断、排除恶变
E.疼痛明显患者可考虑三苯氧胺治疗
C.乳腺增生症是良性疾病,一旦确诊无需随访复查。
4 7 2008 33. 下列关于浆细胞性乳腺炎错误的是:(2.0分) A.目前大多数学者认为其是无菌性化学性炎症
B.经产妇绝经期前后好发
C.其治疗主要以抗炎治疗和手术完整切除病灶为主
D.浆细胞性乳腺的肿块易误诊为癌
E.该病自然病程多为2-4周
E.该病自然病程多为2-4周
4 7 2008 34. 引发急性乳腺炎的常见细菌是?(2.0分) A.铜绿假单胞菌
B.金黄色葡萄球菌或链球菌
C.阴沟肠杆菌
D.表皮葡萄球菌
E.牙龈二氧化碳噬纤维菌
B.金黄色葡萄球菌或链球菌
4 7 2008 35. 浆细胞性乳腺炎慢性期的临床特点?(2.0分) A.窦道或瘘管的形成
B.乳腺触及波动感的肿块但尚无皮损
C.乳头溢脓伴局部红肿
D.局部皮肤有桔皮征出现
E.乳头血性溢液
A.窦道或瘘管的形成
4 7 2008 36. 乳腺疾病最常用的检查是?(2.0分) A.乳腺红外线检查
B.乳腺B超±乳腺钼靶
C.乳腺磁共振检查
D.钼靶+MRI
E.肿瘤标志物检测
B.乳腺B超±乳腺钼靶
4 7 2008 37. About sentinel lymph node biopsy of breast cancer, which one is incorrect? (2.0分) A.The technique of sentinel node biopsy was developed with the goal of reducing the surgical procedure
B.In sentinel node mapping, tracer particles injected into breast tissue, either at the site of the tumor or in the subareolar area
C.The sentinel node is identified as a blue or radioactive node (or both)
D.If pathologic analysis of the sentinel node is negative, the likelihood of other nodes being involved is sufficiently low
E.Morbidity rates are substantially lower with sentinel node biopsy than with axillary dissection
A.The technique of sentinel node biopsy was developed with the goal of reducing the surgical procedure
4 7 2008 38. About Paget's disease, which one is incorrect? (2.0分) A.Paget's disease accounts for 1% or less of breast malignancies
B.It is characterized clinically by nipple erythema and irritation, which may progress to nipple crusting and ulceration
C.A skin specimen containing Paget cells secures the diagnosis
D.There is a low risk for distant metastases
E.Paget's Disease is usually seen in young women
E.Paget's Disease is usually seen in young women
4 7 2008 39. The most reliablemethodtodeterminethenatureofbreast lumps is(2.0分) A.Ultrasonography
B.Mammography
C.Breast MRI
D.Discharge smear
E.Biopsy
E.Biopsy
4 7 2008 40. Which one is incorrect about breast fibroadenomas? (2.0分) A.Fibroadenomas are benign solid tumors composed of stromal and epithelial elements
B.Fibroadenoma is the most common tumor in women younger than 30 years
C.They are rarely seen as new masses in women after the age of 40 or 45
D.On excision, fibroadenomas are poor-encapsulated masses that may detach easily from surrounding breast tissue
E.Mammography is of little help in distinguishing between cysts and fibroadenomas
D.On excision, fibroadenomas are poor-encapsulated masses that may detach easily from surrounding breast tissue
4 7 2008 41. 下列哪一个不属于抗雄激素类药物(2.0分) A.Cyproterone(环丙孕酮)
B.Sidenafil(西地那非)
C.Ketoconazole(酮康唑)
D.Abiraterone(阿比特龙)
E.Finasteride(非那雄胺)
B.Sidenafil(西地那非)
4 7 2008 42. 性功能障碍的心理评估有许多,以下哪项不属于(2.0分) A. 临床面谈
B.自陈问卷
C. 心理生理评估
D. 超声检查
E.躯体检查
D. 超声检查
4 7 2008 43. 下列乳腺癌预后差的是(2.0分) A.小管癌
B.小叶癌
C.粘液癌
D.浸润性导管癌
D.浸润性导管癌
4 7 2008 44. Which one is the indication of using adjuvant target therapy for breast cancer? (2.0分) A.CerbB2 (-)
B.CerbB2 (+)
C.CerbB2 (++)
D.Her-2 (-)
E.Her-2 (+)
E.Her-2 (+)
4 7 2008 45. 夜间阴茎胀大试验的作用是(2.0分) A.测定阴茎的粗细和长度
B.判断勃起功能障碍的程度
C.测定阴茎勃起的硬度
D.鉴别动脉性还是静脉性勃起功能障碍
E.鉴别心理性还是器质性勃起功能障碍
E.鉴别心理性还是器质性勃起功能障碍
4 7 2008 46. 西地那非(万艾可)治疗勃起功能障碍的作用机制是因为它是一种(2.0分) A.多巴胺受体激动剂
B.α肾上腺受体拮抗剂
C.β受体激动剂
D.PDE5抑制剂
E.一氧化氮供体
D.PDE5抑制剂
4 7 2008 47. 男性生殖生理中,FSH的主要靶细胞是:(2.0分) A. Germ cell
B.Leydigcell
C.Sertoli cell
D.Syncytiotrophoblast
E.Spermatogonia
C.Sertoli cell
4 7 2008 48. 精子的成熟周期是:(2.0分) A.1月
B.1周
C.3~5天
D.28-32天
E.70~75天
E.70~75天
4 7 2008 49. 下列五个选项中,哪个不是睾丸源性的不育因素:(2.0分) A. Klinefelter Syndrome
B.KallmannSyndrome
C.Sertoli-cell-only Syndrome
D. Noonan Syndrome
E.Varicocele
B.KallmannSyndrome
4 7 2008 50. 急性乳腺炎常见于哪些人群?(2.0分) A.青春期女性
B.孕晚期妇女
C.产后2-6周哺乳期妇女,初产妇多见
D.非初产妇,多次哺乳的妇女
E.免疫力低下的妇女,如糖尿病患者等
C.产后2-6周哺乳期妇女,初产妇多见
4 7 2009 1. 乳腺橘皮样外观最常见于 (2.5分) A.浸润性小叶癌
B.浸润性导管癌
C.小叶原位癌
D.典型髓样癌
E.导管内原位癌
B.浸润性导管癌
4 7 2009 2. 在我国,乳腺癌占女性恶性肿瘤的 (2.5分) A.第一位
B.第二位
C.第三位
D.第四位
E.第五位
B.第二位
4 7 2009 3. 发生于外上象限的乳腺癌发生淋巴道转移时最常累及的淋巴结是 (2.5分) A.同侧腋窝淋巴结
B.锁骨下淋巴结
C.胸肌间淋巴结组
D.乳内静脉淋巴结
E. 对侧腋窝淋巴结
A.同侧腋窝淋巴结
4 7 2009 4. 属于癌前病变的乳腺疾病为 (2.5分) A.乳腺组织增生
B.硬化性腺病
C.增生性纤维囊性变伴异型增生
D.乳腺导管上皮大汗腺化生
E.纤维腺瘤
C.增生性纤维囊性变伴异型增生
4 7 2009 5. WHO不育症定义:无避孕性生活_______不怀孕。(2.5分) A.1年
B.2年
C.3年
D.半年
E.18个月
A.1年
4 7 2009 6. 精子的成熟周期是:(2.5分) A.1月
B.1周
C.3~5天
D.28-32天
E.70~75天
C.3~5天
4 7 2009 7. 克氏综合征患者,精夜分析提示无精症,睾丸活检发现曲细精管内少量精子,患者有强烈 的生育意愿,最佳后续选择是:(2.5分) A.ICSI
B. IUI
C.IVF-ET
D.激素替代治疗
E. Vasovasostomy
A.ICSI
4 7 2009 8. 正常精液的精子密度应该:(2.5分) A.>10*106/ml
B. >20*106/ml
C.>50*106/ml
D.>100*106/ml
E.>200*106/ml
B. >20*106/ml
4 7 2009 9. 下列哪项不是睾丸肿瘤的血清标记物(2.5分) A.HCG
B. AFP
C. PSA
D. LDH
E.PALP
C. PSA
4 7 2009 10. 睾丸肿瘤的多学科治疗不包括下列哪个选项(2.5分) A. 根治性睾丸切除
B.放疗
C. 化疗
D.腹股沟淋巴结清扫
E.腹膜后淋巴结清扫
D.腹股沟淋巴结清扫
4 7 2009 11. 下列哪项不属于睾丸肿瘤的临床表现(2.5分) A.无痛性睾丸肿块
B. 精索静脉曲张
C.下腹部或腹股沟区钝痛
D.男性乳房发育
E.疼痛性睾丸肿块伴局部红肿
B. 精索静脉曲张
4 7 2009 12. 阴茎癌最常见的病理类型是 (2.5分) A.腺癌
B.移行细胞癌
C. 鳞状细胞癌
D. 基底细胞癌
E.黑色素瘤
C. 鳞状细胞癌
4 7 2009 13. 阴茎癌的前哨淋巴结位于(2.5分) A.盆腔
B.腹膜后
C.腹股沟区
D. 腋窝
E.腹主动脉旁
C.腹股沟区
4 7 2009 14. 勃起功能障碍(ED)的药物治疗目的主要在于 (2.5分) A.增强性欲冲动
B.增强性快感
C.延迟射精时间
D.延长性交消退时间
E.增加阴茎海绵窦内充血,以达到和保持满意的阴茎勃起
E.增加阴茎海绵窦内充血,以达到和保持满意的阴茎勃起
4 7 2009 15. 下列哪一类药物是治疗ED的一线药物? (2.5分) A.育亨宾(yohimbine)
B.局部应用前列腺素E1
C.PDE5抑制药
D.AChE抑制药
E.PLA2抑制药
C.PDE5抑制药
4 7 2009 16. 西地那非(sildenafil)的作用机制是(2.5分) A.抑制cGMP降解,进而降低平滑肌细胞内[Ca2+]
B.阻滞钙通道,降低平滑肌细胞内[Ca2+]
C.激活AC活性,增加平滑肌细胞内cAMP含量
D.抑制AC活性,降低平滑肌细胞内cAMP含量
E.激活eNOS,进而增加平滑肌细胞内cGMP含量
A.抑制cGMP降解,进而降低平滑肌细胞内[Ca2+]
4 7 2009 17. 治疗早泄(PE)的药物包括(2.5分) A.育亨宾(yohimbine)
B.丙咪嗪(imipramine)
C.普鲁卡因(procaine)
D.伐地那非(vardenafil)
E.以上均不是
E.以上均不是
4 7 2009 18. 前列腺增生的病变特点是: (2.5分) A.前列腺腺体肥大
B.前列腺腺体增生
C.前列腺间质纤维组织增生
D.前列腺平滑肌增生
E.前列腺腺体、平滑肌和纤维组织均增生
E.前列腺腺体、平滑肌和纤维组织均增生
4 7 2009 19. 最常见的前列腺癌是: (2.5分) A.粘液癌
B.移行细胞癌
C..鳞状细胞癌
D.腺癌
E.透明细胞癌
D.腺癌
4 7 2009 20. 关于阴茎癌的叙述,错误的是: (2.5分) A.常发生于40岁左右
B.常有包茎史
C.大多为分化差的鳞状细胞癌
D.肿瘤初起位于阴茎头部 E.
E.早期即可发生双侧腹股沟淋巴结转移
C.大多为分化差的鳞状细胞癌
4 7 2009 21. 前列腺增生症对人体最大的危害在于: (2.5分) A.易引起癌变
B.常引起尿路结石
C.常引起排尿障碍
D.常引起内分泌紊乱
E..以上都不是
B.常引起尿路结石
4 7 2009 22. 乳腺增生症外科干预的指征是:(2.5分) A.乳腺专科查体发现乳腺呈结节状
B.乳腺疼痛明显
C.有乳头溢液症状
D.考虑恶变可能(BI-RADS ≥4分)
E.单侧乳腺结节
D.考虑恶变可能(BI-RADS ≥4分)
4 7 2009 23. 急性乳腺炎常见于哪些人群?(2.5分) A.青春期女性
B.孕晚期妇女
C.产后2-6周哺乳期妇女,初产妇多见
D.非初产妇,多次哺乳的妇女
E.免疫力低下的妇女,如糖尿病患者等
C.产后2-6周哺乳期妇女,初产妇多见
4 7 2009 24. 关于急性乳腺炎的规范治疗下列正确的是? (2.5分) A.定期复查
B.抗感染治疗,如有脓肿形成则行脓肿切开引流术
C.发病初期即行手术清创
D.局部理疗
E.清热解毒中成药治疗
B.抗感染治疗,如有脓肿形成则行脓肿切开引流术
4 7 2009 25. 乳腺疾病最常用的检查是?(2.5分) A.乳腺红外线检查
B.乳腺B超±乳腺钼靶
C.乳腺磁共振检查
D.钼靶+MRI
E.肿瘤标志物检测
B.乳腺B超±乳腺钼靶
4 7 2009 26. 关于男性乳房肥大症(男乳发育),下列错误的是:(2.5分) A.与体内雌激素相对过多、雄激素相对不足有关
B.青春期与老年男性好发
C.好发于正在进行抗雄激素治疗的前列腺癌患者
D.需与男性乳腺癌相鉴别
E.青春期男乳发育通常建议手术治疗
E.青春期男乳发育通常建议手术治疗
4 7 2009 27. 性反应周期不包括以下哪一项(2.5分) A.性欲望
B.性兴奋
C.性高潮
D.射精
E.性不应期
E.性不应期
4 7 2009 28. 以下哪项不是性功能障碍的直接生物学因素(2.5分) A.糖尿病
B.苯二氮卓药物依赖
C.酒精依赖
D. 冠心病
E. 百忧解(抗抑郁剂)
D. 冠心病
4 7 2009 29. 以下哪项不是性功能障碍的心理社会因素 (2.5分) A.焦虑
B. 抑郁
C.神经性厌食
D. 恐惧
E.失眠
E.失眠
4 7 2009 30. 哪项不是导致性共障碍的非适应性认知 (2.5分) A.婚前性行为是错误的
B. 手淫是肮脏的
C. 婚姻性行为是为传宗接代
D.男性在任何情况下都应该能够勃起
E.女性在性行为不会疼痛
E.女性在性行为不会疼痛
4 7 2009 31. About sentinel lymph node biopsy of breast cancer, which one is incorrect? (2.5分) A.The technique of sentinel node biopsy was developed with the goal of reducing the surgical procedure
B.In sentinel node mapping, tracer particles injected into breast tissue, either at the site of the tumor or in the subareolar area
C.The sentinel node is identified as a blue or radioactive node (or both)
D.If pathologic analysis of the sentinel node is negative, the likelihood of other nodes being involved is sufficiently low
E.Morbidity rates are substantially lower with sentinel node biopsy than with axillary dissection
A.The technique of sentinel node biopsy was developed with the goal of reducing the surgical procedure
4 7 2009 32. About Paget's disease, which one is incorrect? (2.5分) A.Paget's disease accounts for 1% or less of breast malignancies
B.It is characterized clinically by nipple erythema and irritation, which may progress to nipple crusting and ulceration
C.A skin specimen containing Paget cells secures the diagnosis
D.There is a low risk for distant metastases
E.Paget's Disease is usually seen in young women
E.Paget's Disease is usually seen in young women
4 7 2009 33. The most reliable method to determine the nature of breast lumps is(2.5分) A. Ultrasonography
B. Mammography
C. Breast MRI
D.Discharge smear
E. Biopsy
E. Biopsy
4 7 2009 34. Which one is incorrect about breast fibroadenomas?(2.5分) A.Fibroadenomas are benign solid tumors composed of stromal and epithelial elements
B.Fibroadenoma is the most common tumor in women younger than 30 years
C.They are rarely seen as new masses in women after the age of 40 or 45
D.On excision, fibroadenomas are poor-encapsulated masses that may detach easily from surrounding breast tissue
E.Mammography is of little help in distinguishing between cysts and fibroadenomas
D.On excision, fibroadenomas are poor-encapsulated masses that may detach easily from surrounding breast tissue
4 7 2009 35. 以下不属于生殖系统器官的是:(2.5分) A.膀胱
B.睾丸
C.前列腺
D.精囊
A.膀胱
4 7 2009 36. 属于男生殖系统非特异性感染的是: (2.5分) A.附睾结核
B.尖锐湿疣
C.淋病
D.急性前列腺炎
D.急性前列腺炎
4 7 2009 37. 以下不属于男生殖系统先天畸形的是(2.5分) A.隐睾
B.尿道狭窄
C.尿道下裂
D.隐匿性阴茎
B.尿道狭窄
4 7 2009 38. 以下哪个是男生殖系统良性肿瘤: (2.5分) A.阴囊Paget氏病
B.精索横纹肌肉瘤
C.精原细胞瘤
D.阴茎血管瘤
D.阴茎血管瘤
4 7 2009 39. 下列哪种病理类型的睾丸肿瘤不是生殖细胞来源(2.5分) A. 绒癌
B. 畸胎瘤
C. 精原细胞瘤
D. 移行细胞癌
E. 胚胎癌
D. 移行细胞癌
4 7 2009 40. 前列腺癌最常见于前列腺的: (2.5分) A.前叶
B.中性
C.双侧叶
D.后叶
E.顶叶
D.后叶
4 7 2010 1. WHO不育症定义:无避孕性生活___不怀孕。(2.0分) A.1年
B.2年
C.3年
D.半年
E.18个月
A.1年
4 7 2010 2. 男性生殖生理中,FSH的主要靶细胞是(2.0分) A.Germ cell
B.Leydig cell
C.Sertoli cell
D.Syncytiotrophoblast
E.Spermatogonia
B.Leydig cell
4 7 2010 3. 精子的成熟周期是(2.0分) A.1月
B.1周
C.3~5天
D.28-32天
E.70~75天
C.3~5天
4 7 2010 4. 克氏综合征患者,精夜分析提示无精症,睾丸活检发现曲细精管内少量精子,患者有强烈 的生育意愿,最佳后续选择是:(2.0分) A.ICSI
B.IUI
C.IVF-ET
D.激素替代治疗
E.Vasovasostomy
A.ICSI
4 7 2010 5. 正常精液的精子密度应该(2.0分) A.>10*106/ml
B.>20*106/ml
C.>50*106/ml
D.>100*106/ml
E.>200*106/ml
B.>20*106/ml
4 7 2010 6. 乳腺橘皮样外观最常见于 (2.0分) A.浸润性小叶癌
B.浸润性导管癌
C.小叶原位癌
D.典型髓样癌
E.导管内原位癌
B.浸润性导管癌
4 7 2010 7. 在我国,乳腺癌占女性恶性肿瘤的 (2.0分) A.第一位
B.第二位
C.第三位
D.第四位
E.第五位
B.第二位
4 7 2010 8. 发生于外上象限的乳腺癌发生淋巴道转移时最常累及的淋巴结是 (2.0分) A.同侧腋窝淋巴结
B.锁骨下淋巴结
C.胸肌间淋巴结组
D.乳内静脉淋巴结
E.对侧腋窝淋巴结
A.同侧腋窝淋巴结
4 7 2010 9. 属于癌前病变的乳腺疾病为 (2.0分) A.乳腺组织增生
B.硬化性腺病
C.增生性纤维囊性变伴异型增生
D.乳腺导管上皮大汗腺化生
E.纤维腺瘤
C.增生性纤维囊性变伴异型增生
4 7 2010 10. 药物治疗勃起功能障碍(ED)的主要目的在于 (2.0分) A.增强性欲冲动
B.增强性快感
C.延迟射精时间
D.延长性交消退时间
E.增加阴茎海绵窦内充血,以达到和保持满意的阴茎勃起
E.增加阴茎海绵窦内充血,以达到和保持满意的阴茎勃起
4 7 2010 11. 下列哪一类药物是治疗ED的一线药物 (2.0分) A.育亨宾(yohimbine)
B.局部应用前列腺素E1
C.PDE5抑制药
D.AChE抑制药
E.PLA2抑制药
C.PDE5抑制药
4 7 2010 12. 西地那非(sildenafil)的作用机制是 (2.0分) A.抑制cGMP降解,进而降低平滑肌细胞内[Ca2+]
B.阻滞钙通道,降低平滑肌细胞内[Ca2+]
C.激活AC活性,增加平滑肌细胞内cAMP含量
D.抑制AC活性,降低平滑肌细胞内cAMP含量
E.激活eNOS,进而增加平滑肌细胞内cGMP含量
A.抑制cGMP降解,进而降低平滑肌细胞内[Ca2+]
4 7 2010 13. 育亨宾(yohimbine)属于哪一类药物? (2.0分) A.前列腺素类药物
B.三环类抗抑郁药
C.PDE5抑制药
D. 2肾上腺素受体拮抗药
E. 1肾上腺素受体拮抗药
D. 2肾上腺素受体拮抗药
4 7 2010 14. 治疗早泄(PE)作用较强,已经得到临床证明的药物是 (2.0分) A.舍曲林(sertraline)
B.氟西汀(fluoxetine)
C.达泊西汀(dapoxetine)
D.帕罗西丁(paroxetine)
E.氯丙咪嗪(clomipramine)
C.达泊西汀(dapoxetine)
4 7 2010 15. About sentinel lymph node biopsy of breast cancer, which one is incorrect? (2.0分) A.The technique of sentinel node biopsy was developed with the goal of reducing the surgical procedure
B.In sentinel node mapping, tracer particles injected into breast tissue, either at the site of the tumor or in the subareolar area
C.The sentinel node is identified as a blue or radioactive node (or both)
D.If pathologic analysis of the sentinel node is negative, the likelihood of other nodes being involved is sufficiently low
E.Morbidity rates are substantially lower with sentinel node biopsy than with axillary dissection
A.The technique of sentinel node biopsy was developed with the goal of reducing the surgical procedure
4 7 2010 16. About Paget's disease, which one is incorrect? (2.0分) A.Paget's disease accounts for 1% or less of breast malignancies
B.It is characterized clinically by nipple erythema and irritation, which may progress to nipple crusting and ulceration
C.A skin specimen containing Paget cells secures the diagnosis
D.There is a low risk for distant metastases
E.Paget's Disease is usually seen in young women
E.Paget's Disease is usually seen in young women
4 7 2010 17. The most reliable method to determine the nature of breast lumps is (2.0分) A.Ultrasonography
B.Mammography
C.Breast MRI
D.Discharge smear
E.Biopsy
E.Biopsy
4 7 2010 18. Which one is incorrect about breast fibroadenomas? (2.0分) A.Fibroadenomas are benign solid tumors composed of stromal and epithelial elements
B.Fibroadenoma is the most common tumor in women younger than 30 years
C.They are rarely seen as new masses in women after the age of 40 or 45
D.On excision, fibroadenomas are poor-encapsulated masses that may detach easily from surrounding breast tissue
E.Mammography is of little help in distinguishing between cysts and fibroadenomas
D.On excision, fibroadenomas are poor-encapsulated masses that may detach easily from surrounding breast tissue
4 7 2010 19. 乳腺癌辅助治疗中,赫赛汀靶向治疗的标准疗程是(2.0分) A.6月
B.1年
C.2年
D.视患者的危险因素决定,6-12月
E.视患者的危险因素决定,12-24月
B.1年
4 7 2010 20. 下列哪项不是勃起功能障碍的发病因素?(2.0分) A.糖尿病
B.高血压
C.精神心理疾病
D.输精管结扎
E.吸烟
D.输精管结扎
4 7 2010 21. 下列哪项不会引起器质性勃起功能障碍?(2.0分) A.夫妻感情不和
B.脊柱外伤
C.阴茎硬结症
D.动脉粥样硬化
E.骨盆骨折
A.夫妻感情不和
4 7 2010 22. 夜间阴茎胀大试验的作用是(2.0分) A.测定阴茎的粗细和长度
B.判断勃起功能障碍的程度
C.测定阴茎勃起的硬度
D.鉴别动脉性还是静脉性勃起功能障碍
E.鉴别心理性还是器质性勃起功能障碍
E.鉴别心理性还是器质性勃起功能障碍
4 7 2010 23. 西地那非(万艾可)治疗勃起功能障碍的作用机制是因为它是一种(2.0分) A.多巴胺受体激动剂
B.α肾上腺受体拮抗剂
C.β受体激动剂
D.PDE5抑制剂
E.一氧化氮供体
D.PDE5抑制剂
4 7 2010 24. 以下不属于生殖系统器官的是:(2.0分) A.膀胱
B.睾丸
C.前列腺
D.精囊
A.膀胱
4 7 2010 25. 以下不属于男生殖系统先天畸形的是(2.0分) A.隐睾
B.尿道狭窄
C.尿道下裂
D.隐匿性阴茎
B.尿道狭窄
4 7 2010 26. 性反应周期不包括以下哪一项)(2.0分) A.性欲望
B.性兴奋
C.性高潮
D.射精
E.性不应期
E.性不应期
4 7 2010 27. 以下哪项不是性功能障碍的直接生物学因素 (2.0分) A.糖尿病
B. 苯二氮卓药物依赖
C.酒精依赖
D.冠心病
E.百忧解(抗抑郁剂)
D.冠心病
4 7 2010 28. 以下哪项不是性功能障碍的心理社会因素(2.0分) A.焦虑
B.抑郁
C.神经性厌食
D. 恐惧
E. 失眠
E. 失眠
4 7 2010 29. 哪项不是导致性共障碍的非适应性认知(2.0分) A.婚前性行为是错误的
B.手淫是肮脏的
C.婚姻性行为是为传宗接代
D.男性在任何情况下都应该能够勃起
E.女性在性行为不会疼痛
E.女性在性行为不会疼痛
4 7 2010 30. 早泄的生物和心理因素不包括(2.0分) A.神经素质
B. 躯体疾病
C. 躯体损伤
D.药物副作用
E.婚姻不和睦
E.婚姻不和睦
4 7 2010 31. 下列关于乳腺增生症治疗错误的是:(2.0分) A.确诊的乳腺增生症不需外科治疗
B.药物不能根治,只能缓解部分病人的部分症状
C.乳腺增生症是良性疾病,一旦确诊无需随访复查。
D.手术治疗的主要目的是活检以明确诊断、排除恶变
E.疼痛明显患者可考虑三苯氧胺治疗
C.乳腺增生症是良性疾病,一旦确诊无需随访复查。
4 7 2010 32. 下列关于导管周围乳腺炎错误的是:(2.0分) A.目前大多数学者认为其是无菌性化学性炎症
B.经产妇绝经期前后好发
C.其治疗主要以抗炎治疗和手术完整切除病灶为主
D.又称浆细胞性乳腺炎,易误诊为癌
E.该病自然病程多为2-4周
E.该病自然病程多为2-4周
4 7 2010 33. 引发急性乳腺炎的常见细菌是?(2.0分) A.铜绿假单胞菌
B.金黄色葡萄球菌或链球菌
C.阴沟肠杆菌
D.表皮葡萄球菌
E.牙龈二氧化碳噬纤维菌
B.金黄色葡萄球菌或链球菌
4 7 2010 34. 导管周围乳腺炎慢性期的临床特点是什么?(2.0分) A.窦道或瘘管的形成
B.乳腺触及有波动感的肿块但尚无皮损
C.乳头溢脓伴局部红肿
D.局部皮肤有桔皮征出现
E.乳头血性溢液
A.窦道或瘘管的形成
4 7 2010 35. 关于乳腺增生症,下列哪一项不是临床需关注避免漏诊肿瘤的情况?(2.0分) A.病变局限于单侧
B.肿块大小与月经周期无关联的
C.青春期曾有乳腺纤维腺瘤手术史
D.绝经后仍有明显的腺体增生
E.既往乳房手术病理有上皮增生活跃及不典型增生
C.青春期曾有乳腺纤维腺瘤手术史
4 7 2010 36. 关于男性乳房肥大症(男乳发育),下列错误的是:(2.0分) A.青春期及老年男性好发
B.其发病机理是体内雌激素相对过多、雄激素相对过少所致
C.肝肾基础疾病及药物、保健品均有可能是其病因
D.青春期患者一般推荐手术治疗
E.影响外观或考虑恶变可能者可行手术治疗
D.青春期患者一般推荐手术治疗
4 7 2010 37. 阴茎勃起过程中最关键的神经递质是(2.0分) A.多巴胺
B.乙酰胆碱
C.一氧化氮
D.γ-氨基丁酸
E.肾上腺素
C.一氧化氮
4 7 2010 38. 勃起功能障碍的治疗手段不包括下列哪项?(2.0分) A.心理治疗
B.药物治疗
C.包皮环切术
D.激素替代
E.阴茎假体植入
C.包皮环切术
4 7 2010 39. 以下哪项是属于性欲障碍(2.0分) A.性厌恶障碍
B.勃起障碍
C.性交困难
D.阴道痉挛
E.冷阴
A.性厌恶障碍
4 7 2010 40. 性功能障碍的心理评估有许多,以下哪项不属于(2.0分) A.临床面谈
B.自陈问卷
C.心理生理评估
D. 超声检查
E.躯体检查
D. 超声检查
4 7 2010 41. 治疗早泄(PE)的药物包括 (2.0分) A.育亨宾(yohimbine)
B.丙咪嗪(imipramine)
C.普鲁卡因(procaine)
D.伐地那非(vardenafil)
E.以上均不是
E.以上均不是
4 7 2010 42. 利多卡因(lidocaine)治疗早泄(PE)的给药方法是(2.0分) A.口服
B.静脉注射
C.肌内注射
D.阴茎表面
E.海绵窦内注射
D.阴茎表面
4 7 2010 43. Breast cancer is frequently seen in (2.0分) A.Upper-outer quadrant
B.Upper-inner quadrant
C.Lower-outer quadrant
D.Lower-inner quadrant
E.Nipple-areola area
A.Upper-outer quadrant
4 7 2010 44. Which one is the indication of using adjuvant target therapy for breast cancer? (2.0分) A.CerbB2 (-)
B.CerbB2 (+)
C.CerbB2 (++)
D.Her-2 (-)
E.Her-2 (+)
E.Her-2 (+)
4 7 2010 45. 乳腺癌辅助治疗原则中,下列哪项是错误的(2.0分) A.激素受体阳性的患者原则上都需要内分泌治疗
B.HER2过表达、肿块>1cm的患者原则上都需要靶向治疗
C.淋巴结转移的患者原则上都需要放疗
D.肿瘤>1cm的患者,原则上都需要化疗
E.淋巴结转移的患者原则上都需要化疗
D.肿瘤>1cm的患者,原则上都需要化疗
4 7 2010 46. 急性乳腺炎常见于哪些人群?(2.0分) A.青春期女性
B.孕晚期妇女
C.产后2-6周哺乳期妇女,初产妇多见
D.非初产妇,多次哺乳的妇女
E.免疫力低下的妇女,如糖尿病患者等
C.产后2-6周哺乳期妇女,初产妇多见
4 7 2010 47. 乳腺增生症外科干预的指征是:(2.0分) A.乳腺专科查体发现乳腺呈结节状
B.乳腺疼痛明显
C.有乳头溢液症状
D.考虑恶变可能(BI-RADS ≥4分)
E.单侧乳腺结节
D.考虑恶变可能(BI-RADS ≥4分)
4 7 2010 48. 有关精液分析参考值标准,下列哪项描述最准确(2.0分) A. 最新版本是WHO第三版
B.最新版本是WHO第四版
C.最新版本且最常用是WHO第五版
D.最新版本是第五版,目前多沿用第四版
E.最新版本是WHO第六版
D.最新版本是第五版,目前多沿用第四版
4 7 2010 49. 下列五个选项中,哪个不是睾丸源性的不育因素(2.0分) A.Klinefelter Syndrome
B.Kallmann Syndrome
C.Sertoli-cell-only Syndrome
D. Noonan Syndrome
E.Varicocele
B.Kallmann Syndrome
4 7 2010 50. 患者精液分析提示无精症,血FSH、LH、睾酮正常范围,最有诊断价值的检查是(2.0分) A. 输精管造影
B.睾丸活检
C.染色体检查
D.蝶鞍CT
E.家系调查
B.睾丸活检
4 7 2011 1. WHO不育症定义:无保护性生活____不怀孕。(2.5分) A.1年
B.2年
C.3年
D.半年
E.18个月
A.1年
4 7 2011 2. 男性生殖生理中,FSH的主要靶细胞是(2.5分) A.Germ cell
B.Leydig cell
C.Sertoli cell
D.Syncytiotrophoblast
E.Spermatogonia
C.Sertoli cell
4 7 2011 3. 男性生殖生理中,LH的主要靶细胞是(2.5分) A.Germ cell
B.Leydig cell
C.Sertoli cell
D.Syncytiotrophoblast
E.Spermatogonia
B.Leydig cell
4 7 2011 4. 精子的成熟周期是(2.5分) A.1月
B.1周
C.3~5天
D.28-32天
E.70~75天
E.70~75天
4 7 2011 5. 下列五个选项中,哪个是最常见导致男性不育的因素(2.5分) A.酒精成瘾
B.大麻成瘾
C.精索静脉曲张
D.Klinefelter Syndrome
E.前列腺炎
C.精索静脉曲张
4 7 2011 6. Which factor usually is not associated with ED:(2.5分) A.Body mass index (BMI)
B.Exercise
C.Diabetes
D.Circumcision
E.Radical prostatectomy
D.Circumcision
4 7 2011 7. The hormonal ED is not caused by:(2.5分) A.Hypogonadism
B.Hyperprolactinemia
C.Hyper- and hypothyroidism
D.Hyper- and hypocortisolism
E.Hyperlipidaemia
E.Hyperlipidaemia
4 7 2011 8. Which one is not common adverse events of the PDE5 inhibitors (2.5分) A.Headache
B.Flushing
C.Dyspepsia
D.Nasal congestion
E.Constipation
E.Constipation
4 7 2011 9. Which statement on Sildenafil is incorrect:(2.5分) A.Sildenafil was launched in 1998 and was the first PDE5I available on the market.
B.The recommended starting dose is 50 mg and should be adapted according to the patient'sresponse and side effects.
C.Efficacy may be maintained for up to 12 h
D.Nitrates are not contraindicated with Sildenafil
E.One of common adverse events of Sildenafill is headache
D.Nitrates are not contraindicated with Sildenafil
4 7 2011 10. Which one is not the Specialised diagnostic tests for ED(2.5分) A.Serum glucose
B.Nocturnal penile tumescence and rigidity test
C.Intracavernous injection test
D.Duplex ultrasound of the penis
E.Arteriography and dynamic infusion cavernosometry or cavernosography
A.Serum glucose
4 7 2011 11. The most reliable method to determine the nature of breast lumps is (2.5分) A.Ultrasonography
B.Mammography
C.Breast MRI
D.Discharge smear
E.Biopsy
E.Biopsy
4 7 2011 12. Which one is the indication of using adjuvant target therapy for breast cancer? (2.5分) A.CerbB2 (-)
B.CerbB2 (+)
C.CerbB2 (++)
D.Her-2 (-)
E.Her-2 (+)
E.Her-2 (+)
4 7 2011 13. 目前对于高危的、激素受体阳性的、绝经前乳腺癌患者,一般推荐内分泌治疗方案是? (2.5分) A.口服芳香化酶抑制剂5年
B.口服芳香化酶抑制剂10年
C.口服他莫昔芬5年
D.口服他莫昔芬10年
E.口服他莫昔芬15年
D.口服他莫昔芬10年
4 7 2011 14. 乳腺癌辅助治疗中,赫赛汀靶向治疗的标准疗程是(2.5分) A.6月
B.1年
C.2年
D.视患者的危险因素决定,6-12月
E.视患者的危险因素决定,12-24月
B.1年
4 7 2011 15. 乳腺癌辅助治疗原则中,下列哪项是错误的(2.5分) A.激素受体阳性的患者原则上都需要内分泌治疗
B.HER2过表达、肿块>1cm的患者原则上都需要靶向治疗
C.淋巴结转移的患者原则上都需要放疗
D.肿瘤>1cm的患者,原则上都需要化疗
E.淋巴结转移的患者原则上都需要化疗
D.肿瘤>1cm的患者,原则上都需要化疗
4 7 2011 16. 药物治疗勃起功能障碍(ED)的主要目的在于 (2.5分) A.增强性欲冲动
B.增强性快感
C.延迟射精时间
D.延长性交消退时间
E.增加阴茎海绵窦内充血,以达到和保持满意的阴茎勃起
E.增加阴茎海绵窦内充血,以达到和保持满意的阴茎勃起
4 7 2011 17. 下列哪一类药物是治疗ED的一线药物? (2.5分) A.育亨宾(yohimbine)
B.局部应用前列腺素E1
C.PDE5抑制药
D.AChE抑制药
E.PLA2抑制药
C.PDE5抑制药
4 7 2011 18. 前列腺素E1治疗ED的常用给药途径为 (2.5分) A.口服
B.静脉注射
C.肌内注射
D.海绵窦内注射
E.阴茎表面应用
D.海绵窦内注射
4 7 2011 19. 治疗早泄(PE)的药物包括 (2.5分) A.育亨宾(yohimbine)
B.丙咪嗪(imipramine)
C.普鲁卡因(procaine)
D.伐地那非(vardenafil)
E.以上均不是
E.以上均不是
4 7 2011 20. 西地那非(sildenafil)的特点有(2.5分) A.是一种PDE5抑制剂
B.可用于治疗ED
C.可用于治疗肺动脉高压
D.可口服给药
E.以上均是
E.以上均是
4 7 2011 21. 前列腺癌最常见于前列腺的(2.5分) A.前叶
B.中性
C.双侧叶
D.后叶
E.顶叶
D.后叶
4 7 2011 22. 诊断高分化前列腺癌最可靠的证据是 (2.5分) A.包膜浸润
B.淋巴管浸润
C.血管浸润
D.周围神经浸润
E.以上都是
A.包膜浸润
4 7 2011 23. 睾丸最常见的肿瘤是 (2.5分) A.胚胎性癌
B.精原细胞瘤
C.畸胎瘤
D.绒毛膜癌
E.内胚窦瘤
B.精原细胞瘤
4 7 2011 24. 前列腺增生症对人体最大的危害在于 (2.5分) A.易引起癌变
B.常引起尿路结石
C.常引起排尿障碍
D.常引起内分泌紊乱
E.以上都不是
B.常引起尿路结石
4 7 2011 25. 乳腺橘皮样外观最常见于(2.5分) A.浸润性小叶癌
B.浸润性导管癌
C.小叶原位癌
D.典型髓样癌
E.导管内原位癌
B.浸润性导管癌
4 7 2011 26. 80%以上乳腺癌来源于(2.5分) A.非增生性纤维囊性变
B.乳腺导管上皮
C.腺泡上皮
D.小叶间质
E.乳腺纤维腺瘤
B.乳腺导管上皮
4 7 2011 27. 不符合乳腺癌的描述是(2.5分) A.肿瘤灰白、质硬
B.呈浸润性生长
C.最易发生血道转移
D.皮肤可呈桔皮样改变
E.可出现乳头回缩现象
C.最易发生血道转移
4 7 2011 28. 下列关于乳腺癌的描述,正确的是(2.5分) A.导管内原位癌常发生于大中导管
B.粉刺癌术后不易复发
C.浸润性小叶癌预后较好
D.并非所有的导管内原位癌都会发展为浸润癌
E.乳腺不会发生黏液癌
D.并非所有的导管内原位癌都会发展为浸润癌
4 7 2011 29. 乳腺浸润性小叶癌多发生于(2.5分) A.青春期
B.哺乳期
C.生育期
D.绝经期
E.老年期
E.老年期
4 7 2011 30. 乳腺最常见的良性瘤是(2.5分) A.脂肪瘤
B.腺瘤
C.纤维瘤
D.纤维腺瘤
E.导管内乳头状瘤
D.纤维腺瘤
4 7 2011 31. 最常见的前列腺癌是(2.5分) A.粘液癌
B.移行细胞癌
C.鳞状细胞癌
D.腺癌
E.透明细胞癌
D.腺癌
4 7 2011 32. 关于精原细胞瘤的叙述,下列哪项是错误的 (2.5分) A.起源于原始生殖细胞
B.隐睾症者其发病率高
C.瘤细胞由单一性细胞组成
D.恶性程度很高,易远处转移
E.对放射治疗敏感
D.恶性程度很高,易远处转移
4 7 2011 33. 性反应周期不包括以下哪一项(2.5分) A.性欲望
B.性兴奋
C.性高潮
D.射精
E.性不应期
E.性不应期
4 7 2011 34. 以下哪项是属于性欲障碍(2.5分) A.性厌恶障碍
B.勃起障碍
C.性交困难
D.阴道痉挛
E.冷阴
A.性厌恶障碍
4 7 2011 35. 以下哪项不是性功能障碍的直接生物学因素(2.5分) A.糖尿病
B.苯二氮卓药物依赖
C.酒精依赖
D.冠心病
E.百忧解(抗抑郁剂)
D.冠心病
4 7 2011 36. 以下哪项不是性功能障碍的间接生物学因素(2.5分) A.年龄大
B.吸烟
C.消化性溃疡
D.疼痛
E.关节炎
C.消化性溃疡
4 7 2011 37. 以下哪项不是性功能障碍的心理社会因素 (2.5分) A.焦虑
B.抑郁
C.神经性厌食
D.恐惧
E.失眠
E.失眠
4 7 2011 38. 属于男生殖系统非特异性感染的是(2.5分) A.附睾结核
B.尖锐湿疣
C.淋病
D.急性前列腺炎
D.急性前列腺炎
4 7 2011 39. 以下不属于男生殖系统先天畸形的是(2.5分) A.隐睾
B.尿道狭窄
C.尿道下裂
D.隐匿性阴茎
B.尿道狭窄
4 7 2011 40. 以下哪个是男生殖系统良性肿瘤(2.5分) A.阴囊Paget氏病
B.精索横纹肌肉瘤
C.精原细胞瘤
D.阴茎血管瘤
C.精原细胞瘤
4 7 2012 1. WHO不育症定义:无保护性生活_______不怀孕(2.5分) A.1年
B.2年
C.3年
D.半年
E.18个月
A.1年
4 7 2012 2. 男性生殖生理中,FSH的主要靶细胞是(2.5分) A.Germ cell
B.Leydig cell
C.Sertoli cell
D.Syncytiotrophoblast
E.Spermatogonia
C.Sertoli cell
4 7 2012 3. 精子的成熟周期是(2.5分) A.1月
B.1周
C.3~5天
D.28-32天
E.70~75天
E.70~75天
4 7 2012 4. 下述药物中,哪个对男性生育功能危害最小(2.5分) A.西咪替丁
B.美沙酮
C.安体舒通
D.雌二醇氮芥
E.抗坏血酸
E.抗坏血酸
4 7 2012 5. 正常精液的精子密度应该(2.5分) A.>10*106/ml
B. >20*106/ml
C.>50*106/ml
D.>100*106/ml
E.>200*106/ml
B. >20*106/ml
4 7 2012 6. 下列五个选项中,哪个不是睾丸源性的不育因素(2.5分) A.Klinefelter Syndrome
B.Kallmann Syndrome
C.Sertoli-cell-only Syndrome
D.Noonan Syndrome
E.Varicocele
B.Kallmann Syndrome
4 7 2012 7. Which factor usually is not associated with ED(2.5分) A.Body mass index (BMI)
B.Exercise
C.Diabetes
D.Circumcision
E.Radical prostatectomy
D.Circumcision
4 7 2012 8. The hormonal ED is not caused by(2.5分) A.Hypogonadism
B.Hyperprolactinemia
C.Hyper- and hypothyroidism
D.Hyper- and hypocortisolism
E.Hyperlipidaemia
E.Hyperlipidaemia
4 7 2012 9. Which one is not common adverse events of the PDE5 inhibitors(2.5分) A.Headache
B.Flushing
C.Dyspepsia
D.Nasal congestion
E.Constipation
E.Constipation
4 7 2012 10. Which statement is incorrect?(2.5分) A.Erection is a neuro-vasculo-tissular phenomenon under hormonal control
B.Lifestyle modification (intensive exercise and decrease in BMI) can improve erectile function.
C.ED is uncommon after robotic assisted radical prostatectomy.
D.ED is common after external radiotherapy and brachytherapy.
E.ED shares risk factors with cardiovascular disease.
C.ED is uncommon after robotic assisted radical prostatectomy.
4 7 2012 11. Which one is not the Specialised diagnostic tests for ED(2.5分) A.Serum glucose
B.Nocturnal penile tumescence and rigidity test
C.Intracavernous injection test
D.Duplex ultrasound of the penis
E.Arteriography and dynamic infusion cavernosometry or cavernosography
A.Serum glucose
4 7 2012 12. Which statement on ED is not correct:(2.5分) A.PDE5Is are first-line therapy.
B.Intracavernous injection is second-line therapy.
C.Penile transplantation is third-line therapy.
D.When a curable cause of ED is found, it must be treated first.
E.Lifestyle changes and risk factor modification must precede or accompany ED treatment.
C.Penile transplantation is third-line therapy.
4 7 2012 13. 乳腺癌辅助治疗原则中,下列哪项是错误的(2.5分) A.激素受体阳性的患者原则上都需要内分泌治疗
B.HER2过表达、肿块>1cm的患者原则上都需要靶向治疗
C.淋巴结转移的患者原则上都需要放疗
D.肿瘤>1cm的患者,原则上都需要化疗
E.淋巴结转移的患者原则上都需要化疗
D.肿瘤>1cm的患者,原则上都需要化疗
4 7 2012 14. 乳腺癌辅助治疗中,赫赛汀靶向治疗的标准疗程是(2.5分) A.6月
B.1年
C.2年
D.视患者的危险因素决定,6-12月
E.视患者的危险因素决定,12-24月
B.1年
4 7 2012 15. 目前对于高危的、激素受体阳性的、绝经前乳腺癌患者,一般推荐内分泌治疗方案是?(2.5分) A.口服芳香化酶抑制剂5年
B.口服芳香化酶抑制剂10年
C.口服他莫昔芬5年
D.口服他莫昔芬10年
E.口服他莫昔芬15年
D.口服他莫昔芬10年
4 7 2012 16. Breast cancer is frequently seen in (2.5分) A.Upper-outer quadrant
B.Upper-inner quadrant
C.Lower-outer quadrant
D.Lower-inner quadrant
E.Nipple-areola area
A.Upper-outer quadrant
4 7 2012 17. The most reliable method to determine the nature of breast lumps is (2.5分) A.Ultrasonography
B.Mammography
C.Breast MRI
D.Discharge smear
E.Biopsy
E.Biopsy
4 7 2012 18. Which one is the indication of using adjuvant target therapy for breast cancer? (2.5分) A.CerbB2 (-)
B.CerbB2 (+)
C.CerbB2 (++)
D.Her-2 (-)
E.Her-2 (+)
E.Her-2 (+)
4 7 2012 19. 育亨宾(yohimbine)属于哪一类药物? (2.5分) A.前列腺素类药物
B.三环类抗抑郁药
C.PDE5抑制药
D.2肾上腺素受体拮抗药
E.1肾上腺素受体拮抗药
D.2肾上腺素受体拮抗药
4 7 2012 20. 治疗早泄(PE)的药物包括 (2.5分) A.育亨宾(yohimbine)
B.丙咪嗪(imipramine)
C.普鲁卡因(procaine)
D.伐地那非(vardenafil)
E.以上均不是
E.以上均不是
4 7 2012 21. 治疗早泄(PE)作用较强,已经得到临床证明的药物是 (2.5分) A.舍曲林(sertraline)
B.氟西汀(fluoxetine)
C.达泊西汀(dapoxetine)
D.帕罗西丁(paroxetine)
E.氯丙咪嗪(clomipramine)
C.达泊西汀(dapoxetine)
4 7 2012 22. 药物治疗勃起功能障碍(ED)的主要目的在于 (2.5分) A.增强性欲冲动
B.增强性快感
C.延迟射精时间
D.延长性交消退时间
E.增加阴茎海绵窦内充血,以达到和保持满意的阴茎勃起
E.增加阴茎海绵窦内充血,以达到和保持满意的阴茎勃起
4 7 2012 23. 治疗勃起功能障碍(ED)的药物不包括 (2.5分) A.前列腺素类药物
B.PDE5抑制药
C.三环类抗抑郁药
D.2肾上腺素受体拮抗药(育亨宾)
E.对于继发性ED,治疗原发病的药物
C.三环类抗抑郁药
4 7 2012 24. 下列哪一类药物是治疗ED的一线药物? (2.5分) A.育亨宾(yohimbine)
B.局部应用前列腺素E1
C.PDE5抑制药
D.AChE抑制药
E.PLA2抑制药
C.PDE5抑制药
4 7 2012 25. 睾丸最常见的肿瘤是(2.5分) A.精原细胞瘤
B.胚胎性癌
C.畸胎瘤
D.绒毛膜癌
E.内胚窦瘤
A.精原细胞瘤
4 7 2012 26. 前列腺增生症的主要危害在于(2.5分) A.易引起癌变
B.常引起性功能障碍
C.常引起排尿障碍
D.常引起内分泌紊乱
E.以上都不是
C.常引起排尿障碍
4 7 2012 27. 前列腺癌中最常见的是(2.5分) A.腺癌
B.小细胞癌
C.神经内分泌癌
D.鳞状细胞癌
E.移行细胞癌(尿路上皮癌)
A.腺癌
4 7 2012 28. 前列腺癌的早期病灶多见于前列腺的(2.5分) A.前叶
B.中叶
C.外周带
D.侧叶
E.无确定部位
C.外周带
4 7 2012 29. 下列肿瘤中预后最差的是(2.5分) A.典型的精原细胞瘤
B.精母细胞型精原细胞瘤
C.胚胎性癌
D.前列腺癌
E.畸胎癌
C.胚胎性癌
4 7 2012 30. 下列肿瘤中,除哪项外均来自睾丸生殖细胞 (2.5分) A.精原细胞瘤
B.内胚窦瘤
C.畸胎瘤
D.支持细胞瘤
E.绒毛膜癌
D.支持细胞瘤
4 7 2012 31. 睾丸最常见的生殖细胞肿瘤是(2.5分) A.精原细胞瘤
B.畸胎瘤
C.胚胎癌
D.绒癌
E.卵黄囊瘤
A.精原细胞瘤
4 7 2012 32. 睾丸肿瘤中HCG100%升高的是(2.5分) A.精原细胞瘤
B.畸胎瘤
C.胚胎癌
D.绒癌
E.卵黄囊瘤
D.绒癌
4 7 2012 33. 阴茎癌最常见的病理类型是(2.5分) A.鳞状细胞癌
B.基底细胞癌
C.腺癌
D.移行细胞癌
E.以上均是
A.鳞状细胞癌
4 7 2012 34. 阴茎癌TNM分期中肿瘤侵犯尿道海绵体或阴茎海绵体是(2.5分) A.Ta期
B.T1期
C.T2期
D.T3期
E.T4期
C.T2期
4 7 2012 35. 乳腺橘皮样外观最常见于 (2.5分) A.小叶原位癌
B.典型髓样癌
C.导管内原位癌
D.浸润性导管癌
E.浸润性小叶癌
D.浸润性导管癌
4 7 2012 36. 乳腺最常见的良性肿瘤是 (2.5分) A.脂肪瘤
B.腺瘤
C.导管内乳头状瘤
D.纤维瘤
E.纤维腺瘤
E.纤维腺瘤
4 7 2012 37. 乳腺粉刺癌一般指(2.5分) A.髓样癌
B.小叶癌
C.硬癌
D.胶样癌
E.高级别导管内癌
E.高级别导管内癌
4 7 2012 38. 乳腺癌的发病因素,不包括(2.5分) A.外伤
B.家族史
C.雌激素水平高
D.吸烟
E.不育
A.外伤
4 7 2012 39. 提示乳腺癌预后不良的因素不包括(2.5分) A.原发灶>5cm
B.肿瘤侵及皮肤
C.雌、孕激素受体表达阳性
D.HER2基因扩增
E.淋巴结转移
C.雌、孕激素受体表达阳性
4 7 2012 40. 乳腺癌患者有HER2基因扩增时,可接受(2.5分) A.内分泌治疗
B.放射治疗
C.化疗治疗
D.靶向治疗
E.生物治疗
D.靶向治疗
4 7 2013 1. 患者男性,17岁。因凌晨醒来左侧阴囊疼痛2小时就诊,体检:左侧阴囊肿大,左睾丸触痛明显,睾丸附睾触及不清,阴囊抬高后疼痛加剧。诊断上,首先需考虑(2.5分) A.急性附睾炎
B.睾丸扭转
C.睾丸附件扭转
D.输尿管结石
E.睾丸鞘膜积液
B.睾丸扭转
4 7 2013 2. 精索静脉曲张分级正确的是(2.5分) A.精索静脉曲张1度:外观未见阴囊曲张静脉,触诊时未及明显异常,嘱用力屏气后可及曲张静脉团块。
B.精索静脉曲张1度:外观可见阴囊曲张静脉,嘱用力屏气后可及曲张静脉团块。
C.精索静脉曲张2度:外观可见阴囊曲张静脉,触诊时明显异常。
D.精索静脉曲张2度:外观未见阴囊曲张静脉,触诊时未见明显异常,嘱用力屏气后可及曲张静脉团块。
E.精索静脉曲张3度:外观未见阴囊曲张静脉,触诊时可及明显曲张静脉团块。
A.精索静脉曲张1度:外观未见阴囊曲张静脉,触诊时未及明显异常,嘱用力屏气后可及曲张静脉团块。
4 7 2013 3. 下列透光试验阳性的是(2.5分) A.阴囊疝
B.鞘膜积液
C.阴囊血肿
D.阴囊结核
E.阴囊结石
B.鞘膜积液
4 7 2013 4. 阴茎癌早期转移部位是(2.5分) A.肺
B.肝
C.骨
D.腹股沟淋巴结
E.腹膜后淋巴结
D.腹股沟淋巴结
4 7 2013 5. 睾丸癌最常见的原因是(2.5分) A.隐睾或睾丸未降
B.损伤
C.母亲在妊娠应用外源性雌激素过多
D.感染
E.家族遗传因素
A.隐睾或睾丸未降
4 7 2013 6. 阴茎白膜断裂体检的最重要的特征是(2.5分) A.阴茎皮下血肿,但阴茎仍能伸直
B.阴茎向白膜断裂处对侧弯曲
C.阴茎向白膜断裂处同侧弯曲
D.阴茎向上弯曲
E.阴茎向下弯曲
B.阴茎向白膜断裂处对侧弯曲
4 7 2013 7. Which factor usually is not associated with ED:(2.5分) A.Body mass index (BMI)
B.Exercise
C.Diabetes
D.Circumcision
E.Radical prostatectomy
D.Circumcision
4 7 2013 8. Which statement is incorrect?(2.5分) A.Erection is a neuro-vasculo-tissular phenomenon under hormonal control
B.Lifestyle modification (intensive exercise and decrease in BMI) can improve erectile function.
C.ED is uncommon after robotic assisted radical prostatectomy.
D.ED is common after external radiotherapy and brachytherapy.
E.ED shares risk factors with cardiovascular disease.
C.ED is uncommon after robotic assisted radical prostatectomy.
4 7 2013 9. The hormonal ED is not caused by(2.5分) A.Hypogonadism
B.Hyperprolactinemia
C.Hyper- and hypothyroidism
D.Hyper- and hypocortisolism
E.Hyperlipidaemia
E.Hyperlipidaemia
4 7 2013 10. Which one is not the Specialised diagnostic tests for ED(2.5分) A.Serum glucose
B.Nocturnal penile tumescence and rigidity test
C.Intracavernous injection test
D.Duplex ultrasound of the penis
E.Arteriography and dynamic infusion cavernosometry or cavernosography
A.Serum glucose
4 7 2013 11. Which one is not common adverse events of the PDE5 inhibitors(2.5分) A.Headache
B.Flushing
C.Dyspepsia
D.Nasal congestion
E.Constipation
E.Constipation
4 7 2013 12. Which statement on ED is not correct(2.5分) A.PDE5Is are first-line therapy.
B.Intracavernous injection is second-line therapy.
C.Penile transplantation is third-line therapy.
D.When a curable cause of ED is found, it must be treated first.
E.Lifestyle changes and risk factor modification must precede or accompany ED treatment.
C.Penile transplantation is third-line therapy.
4 7 2013 13. Breast cancer is frequently seen in(2.5分) A.Upper-outer quadrant
B.Upper-inner quadrant
C.Lower-outer quadrant
D.Lower-inner quadrant
E.Nipple-areola area
A.Upper-outer quadrant
4 7 2013 14. The most reliable method to determine the nature of breast lumps is(2.5分) A.Ultrasonography
B.Mammography
C.Breast MRI
D.Discharge smear
E.Biopsy
E.Biopsy
4 7 2013 15. Which one is the indication of using adjuvant target therapy for breast cancer? (2.5分) A.CerbB2 (-)
B.CerbB2 (+)
C.CerbB2 (++)
D.Her-2 (-)
E.Her-2 (+)
E.Her-2 (+)
4 7 2013 16. Which one is incorrect about breast fibroadenomas? (2.5分) A.Fibroadenomas are benign solid tumors composed of stromal and epithelial elements
B.Fibroadenoma is the most common tumor in women younger than 30 years
C.They are rarely seen as new masses in women after the age of 40 or 45
D.On excision, fibroadenomas are poor-encapsulated masses that may detach easily from surrounding breast tissue
E.Mammography is of little help in distinguishing between cysts and fibroadenomas
D.On excision, fibroadenomas are poor-encapsulated masses that may detach easily from surrounding breast tissue
4 7 2013 17. 目前对于高危的、激素受体阳性的、绝经前乳腺癌患者,一般推荐内分泌治疗方案是? (2.5分) A.口服芳香化酶抑制剂5年
B.口服芳香化酶抑制剂10年
C.口服他莫昔芬5年
D.口服他莫昔芬10年
E.口服他莫昔芬15年
D.口服他莫昔芬10年
4 7 2013 18. 乳腺癌辅助治疗中,赫赛汀靶向治疗的标准疗程是(2.5分) A.6月
B.1年
C.2年
D.视患者的危险因素决定,6-12月
E.视患者的危险因素决定,12-24月
B.1年
4 7 2013 19. 乳腺癌辅助治疗原则中,下列哪项是错误的(2.5分) A.激素受体阳性的患者原则上都需要内分泌治疗
B.HER2过表达、肿块>1cm的患者原则上都需要靶向治疗
C.淋巴结转移的患者原则上都需要放疗
D.肿瘤>1cm的患者,原则上都需要化疗
E.淋巴结转移的患者原则上都需要化疗
D.肿瘤>1cm的患者,原则上都需要化疗
4 7 2013 20. 前列腺增生的病变特点是(2.5分) A.前列腺腺体肥大
B.前列腺腺体增生
C.前列腺间质纤维组织增生
D.前列腺平滑肌增生
E.前列腺腺体、平滑肌和纤维组织均增生
E.前列腺腺体、平滑肌和纤维组织均增生
4 7 2013 21. 前列腺癌最常见于前列腺的(2.5分) A.前叶
B.中叶
C.双侧叶
D.后叶
E.顶叶
D.后叶
4 7 2013 22. 最常见的前列腺癌是(2.5分) A.粘液癌
B.移行细胞癌
C.鳞状细胞癌
D.腺癌
E.透明细胞癌
D.腺癌
4 7 2013 23. 诊断高分化前列腺癌最可靠的证据是(2.5分) A.包膜浸润
B.淋巴管浸润
C.血管浸润
D.周围神经浸润
E.以上都是
A.包膜浸润
4 7 2013 24. 关于阴茎癌的叙述,错误的是(2.5分) A.常发生于40岁左右
B.常有包茎史
C.大多为分化差的鳞状细胞癌
D.肿瘤初起位于阴茎头部
E.早期即可发生双侧腹股沟淋巴结转移
C.大多为分化差的鳞状细胞癌
4 7 2013 25. 睾丸最常见的肿瘤是(2.5分) A.胚胎性癌
B.精原细胞瘤
C.畸胎瘤
D.绒毛膜癌
E.内胚窦瘤
B.精原细胞瘤
4 7 2013 26. 乳腺癌的癌前病变是(2.5分) A.乳腺纤维腺瘤
B.增生性纤维囊性变伴不典型增生
C.硬化性腺病
D.乳腺导管上皮大汗腺样化生
E.乳腺结构不良
B.增生性纤维囊性变伴不典型增生
4 7 2013 27. 乳腺癌的起源部位主要是(2.5分) A.导管上皮
B.腺泡上皮
C.真皮汗腺上皮
D.纤维囊性乳腺病
E.纤维腺瘤
A.导管上皮
4 7 2013 28. 乳腺癌最常见的发生部位是(2.5分) A.乳头部
B.外下象限
C.内上象限
D.内下象限
E.外上象限
E.外上象限
4 7 2013 29. 下列乳腺癌中,属于实体癌的是(2.5分) A.硬癌
B.小叶癌
C.导管癌
D.粘液癌
E.炎症样癌
A.硬癌
4 7 2013 30. 较常累及双侧乳腺的乳腺癌为(2.5分) A.浸润性导管癌
B.浸润性小叶癌
C.管内癌
D.乳头状癌
E.粘液癌
B.浸润性小叶癌
4 7 2013 31. 乳腺癌中常见的类型是(2.5分) A..导管内癌
B.浸润性导管癌
C.小叶原位癌
D.浸润性小叶癌
E.典型髓样癌
B.浸润性导管癌
4 7 2013 32. 乳腺最常见的良性肿瘤是(2.5分) A.脂肪瘤
B.乳腺纤维囊性变
C.导管内乳头状瘤
D.纤维瘤
E.纤维腺瘤
E.纤维腺瘤
4 7 2013 33. WHO不育症定义:无保护性生活_____不怀孕。(2.5分) A.1年
B.2年
C.3年
D.半年
E.18个月
A.1年
4 7 2013 34. 男性生殖生理中,FSH的主要靶细胞是(2.5分) A.Germ cell
B.Leydig cell
C.Sertoli cell
D.Syncytiotrophoblast
E.Spermatogonia
C.Sertoli cell
4 7 2013 35. 精子的成熟周期是(2.5分) A.1月
B.1周
C.3~5天
D.28-32天
E.70~75天
E.70~75天
4 7 2013 36. 下列五个选项中,哪个是最常见导致男性不育的因素(2.5分) A.酒精成瘾
B.大麻成瘾
C.精索静脉曲张
D.Klinefelter Syndrome
E.前列腺炎
C.精索静脉曲张
4 7 2013 37. 下述药物中,哪个对男性生育功能危害最小(2.5分) A.西咪替丁
B.美沙酮
C.安体舒通
D.雌二醇氮芥
E.抗坏血酸
E.抗坏血酸
4 7 2013 38. 患者精液分析提示无精症,血FSH、LH、睾酮正常范围,最有诊断价值的检查是(2.5分) A.输精管造影
B.睾丸活检
C.染色体检查
D.蝶鞍CT
E.家系调查
B.睾丸活检
4 7 2013 39. 男性生殖生理中,LH的主要靶细胞是(2.5分) A.Germ cell
B.Leydig cell
C.Sertoli cell
D.Syncytiotrophoblast
E.Spermatogonia
B.Leydig cell
4 7 2013 40. 睾丸癌早期转移部位是(2.5分) A.肺
B.肝
C.骨
D.腹股沟淋巴结
E.腹膜后淋巴结
E.腹膜后淋巴结
4 7 2014 1. Which factor usually is not associated with ED(2.5分) A.Body mass index (BMI)
B.Exercise
C.Diabetes
D.Circumcision
E.Radical prostatectomy
D.Circumcision
4 7 2014 2. Which statement is incorrect? (2.5分) A.Erection is a neuro-vasculo-tissular phenomenon under hormonal control
B.Lifestyle modification (intensive exercise and decrease in BMI) can improve erectile function.
C.ED is uncommon after robotic assisted radical prostatectomy.
D.ED is common after external radiotherapy and brachytherapy.
E.ED shares risk factors with cardiovascular disease.
C.ED is uncommon after robotic assisted radical prostatectomy.
4 7 2014 3. The hormonal ED is not caused by(2.5分) A.Hypogonadism
B.Hyperprolactinemia
C.Hyper- and hypothyroidism
D.Hyper- and hypocortisolism
E.Hyperlipidaemia
E.Hyperlipidaemia
4 7 2014 4. Which one is not the Specialised diagnostic tests for ED (2.5分) A.Serum glucose
B.Nocturnal penile tumescence and rigidity test
C.Intracavernous injection test
D.Duplex ultrasound of the penis
E.Arteriography and dynamic infusion cavernosometry or cavernosography
A.Serum glucose
4 7 2014 5. Which one is not common adverse events of the PDE5 inhibitors(2.5分) A.Headache
B.Flushing
C.Dyspepsia
D.Nasal congestion
E.Constipation
E.Constipation
4 7 2014 6. About Paget's disease, which one is incorrect?(2.5分) A.Paget's disease accounts for 1% or less of breast malignancies
B.It is characterized clinically by nipple erythema and irritation, which may progress to nipple crusting and ulceration
C.A skin specimen containing Paget cells secures the diagnosis
D.There is a low risk for distant metastases
E.Paget's Disease is usually seen in young women
E.Paget's Disease is usually seen in young women
4 7 2014 7. Breast cancer is frequently seen in(2.5分) A.Upper-outer quadrant
B.Upper-inner quadrant
C.Lower-outer quadrant
D.Lower-inner quadrant
E.Nipple-areola area
A.Upper-outer quadrant
4 7 2014 8. The most reliablemethodtodeterminethenatureofbreast lumps is(2.5分) A.Ultrasonography
B.Mammography
C.Breast MRI
D.Discharge smear
E.Biopsy
E.Biopsy
4 7 2014 9. Which one is the indication of using adjuvant target therapy for breast cancer? (2.5分) A.CerbB2 (-)
B.CerbB2 (+)
C.CerbB2 (++)
D.Her-2 (-)
E.Her-2 (+)
E.Her-2 (+)
4 7 2014 10. 乳腺癌辅助治疗中,赫赛汀靶向治疗的标准疗程是(2.5分) A.6月
B.1年
C.2年
D.视患者的危险因素决定,6-12月
E.视患者的危险因素决定,12-24月
B.1年
4 7 2014 11. 导致老年男性排尿困难最常见的原因是(2.5分) A.肾癌
B.肾结石
C.膀胱癌
D.良性前列腺增生症
E.前列腺癌
D.良性前列腺增生症
4 7 2014 12. 下列哪项不是前列腺增生的特点(2.5分) A.无包膜
B.质韧,结节状
C.极少发生癌变
D.切面可见蜂窝状腔隙
E.与周围正常前列腺组织界限清
E.与周围正常前列腺组织界限清
4 7 2014 13. 下列哪项不是前列腺癌的特点(2.5分) A.无包膜
B.质硬,沙砾状
C.肉眼常可辨认,并可肛诊检查
D.高分化腺癌多见
E.与周围正常前列腺组织界限不清
C.肉眼常可辨认,并可肛诊检查
4 7 2014 14. 前列腺癌最常见的血道转移部位是(2.5分) A.肝脏
B.骨髓
C.肺脏
D.脾脏
E.大脑
B.骨髓
4 7 2014 15. 精原细胞瘤属于(2.5分) A.生殖细胞源性肿瘤
B.上皮源性肿瘤
C.间叶源性肿瘤
D.神经内分泌肿瘤
E.淋巴造血肿瘤
A.生殖细胞源性肿瘤
4 7 2014 16. WHO不育症定义:无保护性生活_______不怀孕。(2.5分) A.1年
B.2年
C.3年
D.半年
E.18个月
A.1年
4 7 2014 17. 男性生殖生理中,FSH的主要靶细胞是(2.5分) A.Germ cell
B.Leydig cell
C.Sertoli cell
D.Syncytiotrophoblast
E.Spermatogonia
C.Sertoli cell
4 7 2014 18. 精子的成熟周期是(2.5分) A.1月
B.1周
C.3~5天
D.28-32天
E.70~75天
E.70~75天
4 7 2014 19. 下列五个选项中,哪个是最常见导致男性不育的因素(2.5分) A.酒精成瘾
B.大麻成瘾
C.精索静脉曲张
D.Klinefelter Syndrome
E.前列腺炎
C.精索静脉曲张
4 7 2014 20. 下述药物中,哪个对男性生育功能危害最小(2.5分) A.西咪替丁
B.美沙酮
C.安体舒通
D.雌二醇氮芥
E.抗坏血酸
E.抗坏血酸
4 7 2014 21. 哪一类药物治疗不会影响男性性功能的是(2.5分) A.Selective serotonin reuptake inhibitors (SSRIs)
B.Anti-hypertensive agents
C.Anti-androgen agents
D.Monoamine oxidase inhibitors
E.Antiatherosclerotic agents
E.Antiatherosclerotic agents
4 7 2014 22. 关于PDE5选择性抑制剂治疗ED收效的原理不正确的是(2.5分) A.increase the intracellularcAMP levels
B.increase the intracellular cGMP levels
C.decrease the intracellular Ca2+ levels
D.increase the intracellular K+ levels
E.increase the intracellular NO levels
A.increase the intracellularcAMP levels
4 7 2014 23. 关于Yohimbine治疗ED收效的原理是(2.5分) A.block the α1 receptor
B.block the α2 receptor
C.block the β1 receptor
D.block the β2 receptor
E.None of above
B.block the α2 receptor
4 7 2014 24. 选择性PDE5抑制剂可用于以下的治疗,叙述不正确的是(2.5分) A.高原性肺水肿
B.肺动脉高压
C.脊髓外伤所致的ED
D.糖尿病性ED
E.上述都是
E.上述都是
4 7 2014 25. 一般不出现乳房内包块的乳腺癌是(2.5分) A.导管内癌
B.浸润性导管癌
C.小叶原位癌
D.浸润小叶癌
E.伴有髓样特征的癌
C.小叶原位癌
4 7 2014 26. 乳腺癌主要起源于(2.5分) A.导管上皮
B.腺泡上皮
C.纤维腺瘤
D.纤维囊性乳腺病
E.终末导管小叶单元上皮
E.终末导管小叶单元上皮
4 7 2014 27. 不符合乳腺纤维囊性病的描述是(2.5分) A.与雌激素分泌过多有关
B.囊肿形成
C.纤维增生
D.腺泡和导管增生
E.导管乳头状瘤
E.导管乳头状瘤
4 7 2014 28. 乳腺橘皮样外观最常见于(2.5分) A.粉刺癌
B.小叶原位癌
C.导管原位癌
D.浸润性导管癌
E.浸润性小叶癌
D.浸润性导管癌
4 7 2014 29. 乳腺癌分子分型中对Herceptin靶向治疗常有效的是哪一型(2.5分) A.管腔型A
B.管腔型B
C.HER过表达型
D.基底样型
E.HER阴性乳腺癌
C.HER过表达型
4 7 2014 30. 湿疹样癌,常起源于(2.5分) A.外上象限
B.内上象限
C.乳头内的大乳管
D.中央区
E.整个乳房内
C.乳头内的大乳管
4 7 2014 31. 患者男性,17岁。因凌晨醒来左侧阴囊疼痛2小时就诊,体检:左侧阴囊肿大,左睾丸触痛明显,睾丸附睾触及不清,阴囊抬高后疼痛加剧。诊断上,首先需考虑(2.5分) A.急性附睾炎
B.睾丸扭转
C.睾丸附件扭转
D.输尿管结石
E.睾丸鞘膜积液
B.睾丸扭转
4 7 2014 32. 患者,男,5岁。发现左侧阴囊可复性肿大4年。PE:立位时左侧阴囊肿大,平卧位消失,透光试验阳性。下列诊断正确的是(2.5分) A.睾丸鞘膜积液
B.精索鞘膜积液
C.睾丸精索鞘膜积液
D.交通性鞘膜积液
E.腹股沟斜疝
D.交通性鞘膜积液
4 7 2014 33. 附睾结核体格检查的最典型特征是(2.5分) A.附睾尾部结节
B.附睾头部结节
C.睾丸附睾分界不清
D.睾丸附睾均可及结节
E.精索可及串珠样结节
E.精索可及串珠样结节
4 7 2014 34. 下列透光试验阳性的是(2.5分) A.阴囊疝
B.鞘膜积液
C.阴囊血肿
D.阴囊结核
E.阴囊结石
B.鞘膜积液
4 7 2014 35. 淋病时,尿道分泌物常见为: (2.5分) A.粘液性分泌物
B.黄色脓性分泌物
C.血性分泌物
D.白色脓性分泌物
E.尿道出血
B.黄色脓性分泌物
4 7 2014 36. 下列五个选项中,哪个不是睾丸源性的不育因素(2.5分) A.Klinefelter Syndrome
B.Kallmann Syndrome
C.Sertoli-cell-only Syndrome
D.Noonan Syndrome
E.Varicocele
B.Kallmann Syndrome
4 7 2014 37. 以下不属于生殖系统器官的是(2.5分) A.膀胱
B.睾丸
C.前列腺
D.精囊
A.膀胱
4 7 2014 38. 属于男生殖系统非特异性感染的是(2.5分) A.附睾结核
B.尖锐湿疣
C.淋病
D.急性前列腺炎
D.急性前列腺炎
4 7 2014 39. 以下不属于男生殖系统先天畸形的是(2.5分) A.隐睾
B.尿道狭窄
C.尿道下裂
D.隐匿性阴茎
B.尿道狭窄
4 7 2014 40. 以下哪个是男生殖系统良性肿瘤(2.5分) A.阴囊Paget氏病
B.精索横纹肌肉瘤
C.精原细胞瘤
D.阴茎血管瘤
C.精原细胞瘤
4 7 2015 1. WHO不育症定义:无保护性生活_______不怀孕。(2.5分) A.1年
B.2年
C.3年
D.半年
E.18个月
A.1年
4 7 2015 2. 男性生殖生理中,FSH的主要靶细胞是:(2.5分) A.Germ cell
B.Leydig cell
C.Sertoli cell
D.Syncytiotrophoblast
E.Spermatogonia
C.Sertoli cell
4 7 2015 3. 精子的成熟周期是:(2.5分) A.1月
B.1周
C.3~5天
D.28-32天
E.70~75天
E.70~75天
4 7 2015 4. 下列五个选项中,哪个是最常见导致男性不育的因素(2.5分) A.酒精成瘾
B.大麻成瘾
C.精索静脉曲张
D.Klinefelter Syndrome
E.前列腺炎
C.精索静脉曲张
4 7 2015 5. 克氏综合征患者,精夜分析提示无精症,睾丸活检发现曲细精管内少量精子,患者有强烈的生育意愿,最佳后续选择是:(2.5分) A.ICSI
B.IUI
C.IVF-ET
D.激素替代治疗
E.Vasovasostomy
A.ICSI
4 7 2015 6. 下述药物中,哪个对男性生育功能危害最小:(2.5分) A.西咪替丁
B.美沙酮
C.安体舒通
D.雌二醇氮芥
E.抗坏血酸
E.抗坏血酸
4 7 2015 7. Which factor usually is not associated with ED:(2.5分) A.Body mass index (BMI)
B.Exercise
C.Diabetes
D.Circumcision
E.Radical prostatectomy
D.Circumcision
4 7 2015 8. Which statement is incorrect?(2.5分) A.Erection is a neuro-vasculo-tissular phenomenon under hormonal control
B.Lifestyle modification (intensive exercise and decrease in BMI) can improve erectile function.
C.ED is uncommon after robotic assisted radical prostatectomy.
D.ED is common after external radiotherapy and brachytherapy.
E.ED shares risk factors with cardiovascular disease.
C.ED is uncommon after robotic assisted radical prostatectomy.
4 7 2015 9. The hormonal ED is not caused by:(2.5分) A.Hypogonadism
B.Hyperprolactinemia
C.Hyper- and hypothyroidism
D.Hyper- and hypocortisolism
E.Hyperlipidaemia
E.Hyperlipidaemia
4 7 2015 10. Which one is not common adverse events of the PDE5 inhibitors(2.5分) A.Headache
B.Flushing
C.Dyspepsia
D.Nasal congestion
E.Constipation
E.Constipation
4 7 2015 11. Which statement on Sildenafilis incorrect:(2.5分) A.Sildenafil was launched in 1998 and was the first PDE5I available on the market.
B.The recommended starting dose is 50 mg and should be adapted according to the patient'sresponse and side effects.
C.Efficacy may be maintained for up to 12 h
D.Nitrates are not contraindicated with Sildenafil
E.One of common adverse events of Sildenafill is headache
D.Nitrates are not contraindicated with Sildenafil
4 7 2015 12. 国际勃起功能评分(IIEF-5)不包括下列哪项(2.5分) A.对阴茎勃起及维持的信心如何
B.尝试性交有多少次感到满足
C.性交时保持阴茎勃起至性交完毕有多大困难
D.阴茎进入阴道后有多少次能维持阴茎勃起
E.阴茎在阴道内维持勃起多少时间射精
E.阴茎在阴道内维持勃起多少时间射精
4 7 2015 13. About Paget's disease, which one is incorrect? (2.5分) A.Paget's disease accounts for 1% or less of breast malignancies
B.It is characterized clinically by nipple erythema and irritation, which may progress to nipple crusting and ulceration
C.A skin specimen containing Paget cells secures the diagnosis
D.There is a low risk for distant metastases
E.Paget's Disease is usually seen in young women
E.Paget's Disease is usually seen in young women
4 7 2015 14. The most reliablemethodtodeterminethenatureofbreast lumps is (2.5分) A.Ultrasonography
B.Mammography
C.Breast MRI
D.Discharge smear
E.Biopsy
E.Biopsy
4 7 2015 15. Which one is the indication of using adjuvant target therapy for breast cancer? (2.5分) A.CerbB2 (-)
B.CerbB2 (+)
C.CerbB2 (++)
D.Her-2 (-)
E.Her-2 (+)
E.Her-2 (+)
4 7 2015 16. 目前对于高危的、激素受体阳性的、绝经前乳腺癌患者,一般推荐内分泌治疗方案是(2.5分) A.口服芳香化酶抑制剂5年
B.口服芳香化酶抑制剂10年
C.口服他莫昔芬5年
D.口服他莫昔芬10年
E.口服他莫昔芬15年
D.口服他莫昔芬10年
4 7 2015 17. 乳腺癌辅助治疗中,赫赛汀靶向治疗的标准疗程是 (2.5分) A.6月
B.1年
C.2年
D.视患者的危险因素决定,6-12月
E.视患者的危险因素决定,12-24月
B.1年
4 7 2015 18. 乳腺癌辅助治疗原则中,下列哪项是错误的 (2.5分) A.激素受体阳性的患者原则上都需要内分泌治疗
B.HER2过表达、肿块>1cm的患者原则上都需要靶向治疗
C.淋巴结转移的患者原则上都需要放疗
D.肿瘤>1cm的患者,原则上都需要化疗
E.淋巴结转移的患者原则上都需要化疗
D.肿瘤>1cm的患者,原则上都需要化疗
4 7 2015 19. 关于前列腺炎下列哪项是错误的(2.5分) A.常常继发于膀胱或泌尿道感染
B.病原多为金黄色葡萄球菌
C.急性炎症以中性粒细胞浸润为主
D.可发生结节病
E.前列腺可肿大或触痛
B.病原多为金黄色葡萄球菌
4 7 2015 20. 下列关于前列腺增生症的镜下特点哪项是错误的(2.5分) A.主要由腺体、纤维、平滑肌增生为主
B.腺腔内可见淀粉样小体
C.上皮增生呈乳头状或皱褶形成
D.主要由腺体、炎细胞、平滑肌增生为主
E.上皮化生常位于梗死灶周围
A.主要由腺体、纤维、平滑肌增生为主
4 7 2015 21. 下列哪项不是前列腺增生的特点(2.5分) A.无包膜
B.质韧,结节状
C.极少发生癌变
D.切面可见蜂窝状腔隙
E.与周围正常前列腺组织界限清
E.与周围正常前列腺组织界限清
4 7 2015 22. 下列哪项不是前列腺癌的特点(2.5分) A.无包膜
B.质硬,沙砾状
C.肉眼常可辨认,并可肛诊检查
D.高分化腺癌多见
E.与周围正常前列腺组织界限不清
C.肉眼常可辨认,并可肛诊检查
4 7 2015 23. 疣状癌与尖锐湿疣的区别是:(2.5分) A.两者大体特点不同
B.两者组织学结构不同
C.疣状癌可发生出血
D.疣状癌可发生浸润或转移
E.疣状癌可局部变硬
D.疣状癌可发生浸润或转移
4 7 2015 24. 男性乳腺发育症最显著的特点是:(2.5分) A.纤维组织增生
B.导管上皮乳头状增生
C.乳腺小叶形成
D.脂肪组织增生
E.乳晕下纽扣状结节形成
B.导管上皮乳头状增生
4 7 2015 25. 患者男,21岁,左侧阴囊胀大,伴坠胀感、隐痛半年余来院就诊。PE:立位检查示左侧阴囊明显松弛下垂,阴囊根部可触及蚯蚓样团块,平卧后团块逐渐消失。左侧睾丸及附睾触及无殊。患者首先诊断:(2.5分) A.精索静脉曲张
B.睾丸鞘膜积液
C.交通性鞘膜积液
D.精索鞘膜积液
E.腹股沟斜疝
A.精索静脉曲张
4 7 2015 26. 男,18岁,左侧阴囊坠胀感2月。B超:左侧精索静脉区可见迂曲扩张管样结构,增加腹压后内径增宽,最宽处0.4cm, CDFI:其内可见静脉血流信号。造成该病最有可能的病因是:(2.5分) A.腹膜后肿瘤
B.左侧精索静脉呈直角注入左肾静脉
C.左肾肿瘤
D.左肾静脉癌栓
E.乙状结肠肿瘤
B.左侧精索静脉呈直角注入左肾静脉
4 7 2015 27. 患者男,40岁。发现右侧阴囊肿物渐行性增大6月。查体:右侧睾丸肿大,质硬,与睾丸界限不清,平躺后肿块无明显变化。用手托起沉重感,透光试验阴性。应首先考虑:(2.5分) A.睾丸肿瘤
B.睾丸扭转
C.附睾炎
D.睾丸鞘膜积液
E.腹股沟斜疝
A.睾丸肿瘤
4 7 2015 28. 患者男,36岁。洗澡时发现左侧阴囊肿块就诊。查体:左侧睾丸肿大,质硬,沉重感。血HCG及AFP均正常范围。B超示左睾丸实质性占位。临床诊断睾丸肿瘤。睾丸肿瘤分型中最常见的是:(2.5分) A.精原细胞瘤
B.间质细胞瘤
C.绒毛膜癌
D.支持细胞瘤
E.卵黄囊瘤
A.精原细胞瘤
4 7 2015 29. 患者男,27岁。发现右侧阴囊肿块渐进性增大半年就诊,10年前曾有隐睾下降病史,B超示右睾丸实性占位,大小约2cm左右,首先考虑睾丸肿瘤。如需进一步检查血肿瘤标志物,临床常用的是:(2.5分) A.Ca199,AFP
B.CEA,HCG
C.PSA,AFP
D.Ca125,CEA
E.AFP,HCG
E.AFP,HCG
4 7 2015 30. 患者,男性,16岁,左阴囊疼痛2小时,体检:左侧阴囊肿大,左睾丸附睾触及不清,左侧精索肿大明显,阴囊抬高后疼痛加重,首先需考虑的诊断是(2.5分) A.精索鞘膜积液
B.腹股沟疝
C.精索囊肿
D.睾丸扭转
E.精索静脉曲张
D.睾丸扭转
4 7 2015 31. 平滑肌收缩是ED发生的关键原因之一,叙述不正确的是(2.5分) A.VIP能扩张上述平滑肌使阴茎勃起
B.NO供体扩张上述平滑肌使阴茎勃起
C.RhoA/ROCK抑制剂扩张上述平滑肌使阴茎勃起
D.乙酰胆碱扩张上述平滑肌使阴茎勃起
E.上述都不是
E.上述都不是
4 7 2015 32. 关于选择性PDE抑制剂叙述不正确的是(2.5分) A.抑制PDE5活性最强的是Vardenafil
B.作用时间最长的是Tadalafil
C.Sidenafil是第一个用于临床的选择性PDE5抑制剂
D.PDE3和PDE4抑制剂也可以产生阴茎勃起的效应
E.上述都不是
E.上述都不是
4 7 2015 33. 关于选择性PDE5抑制剂叙述不正确的是(2.5分) A.常禁止与硝酸酯类抗心绞痛药物合用
B.常禁止用于最近有中风、心血管事件发生的病人
C.视觉障碍常发生于Vardenafil使用者而常不发生于Sidenafil使用者
D.可能诱发心肌埂塞、室性心律失常、或突然的失聪
E.上述都不是
C.视觉障碍常发生于Vardenafil使用者而常不发生于Sidenafil使用者
4 7 2015 34. 选择性PDE5抑制剂可用于以下的治疗,叙述不正确的是(2.5分) A.高原性肺水肿
B.肺动脉高压
C.脊髓外伤所致的ED
D.糖尿病性ED
E.上述都是
E.上述都是
4 7 2015 35. 一般不出现乳房内包块的乳腺癌是(2.5分) A.导管内癌
B.浸润性导管癌
C.小叶原位癌
D.浸润小叶癌
E.伴有髓样特征的癌
C.小叶原位癌
4 7 2015 36. 乳腺癌主要起源于(2.5分) A.导管上皮
B.腺泡上皮
C.纤维腺瘤
D.纤维囊性乳腺病
E.终末导管小叶单元上皮
E.终末导管小叶单元上皮
4 7 2015 37. 不符合乳腺纤维囊性病的描述是(2.5分) A.与雌激素分泌过多有关
B.囊肿形成
C.纤维增生
D.腺泡和导管增生
E.导管乳头状瘤
E.导管乳头状瘤
4 7 2015 38. 乳腺橘皮样外观最常见于(2.5分) A.粉刺癌
B.小叶原位癌
C.导管原位癌
D.浸润性导管癌
E.浸润性小叶癌
D.浸润性导管癌
4 7 2015 39. 乳腺癌分子分型中对Herceptin靶向治疗常有效的是哪一型:(2.5分) A.管腔型A
B.管腔型B
C.HER过表达型
D.基底样型
E.HER阴性乳腺癌
C.HER过表达型
4 7 2015 40. 湿疹样癌,常起源于(2.5分) A.外上象限
B.内上象限
C.乳头内的大乳管
D.中央区
E.整个乳房内
C.乳头内的大乳管
4 8 2006 1. 下列哪项不是艾滋病的传播途径: (2.0分) A.母婴垂直传播
B.性传播
C.血液传播
D.虫媒传播
D.虫媒传播
4 8 2006 2. which of the following is the definition of dysfunctional uterine bleeding? (2.0分) A.irregular bleeding with pain
B.Any irregular bleeding
C.irregular bleeding causing functional disability
D.rregular bleeding in the absence of anatomic lesions
D.rregular bleeding in the absence of anatomic lesions
4 8 2006 3. 目前我国开展婚检的主要目的是:(2.0分) A.检查准备结婚的双方是否存在传染病。
B.检查准备结婚的双方是否患有严重的精神性疾病。
C.检查准备结婚的双方是否患有影响生育的严重疾病
D.检查准备结婚的双方是否患有严重遗传性疾病
E.以上都是
E.以上都是
4 8 2006 4. 女孩青春期出现的最早征兆是?(2.0分) A.乳房出现硬结
B.月经来潮
C.出现阴毛和腋毛
D.身高突增
A.乳房出现硬结
4 8 2006 5. A 14-year-old girl complains of irregular vaginal bleeding. Her general examination and pelvic organs are normal. Of the following, the most likely cause of anovulatory bleeding (dysfunctional uterine bleeding [DUB]) in this patient is which of the following? (2.0分) A.congenital adrenal hyperplasia (CAH)
B.hypothyroidism
C.pituitary adenoma
D.polycystic ovary syndrome (PCOS)
E.hypothalamic immaturity
E.hypothalamic immaturity
4 8 2006 6. 孕产妇死亡率统计指标中哪项说法是不正确的?(2.0分) A.孕产妇死亡率指标计算:分子是当地某一时段的孕产妇死亡人数,分母是当地同一时段所有的孕产妇数。
B.孕产妇死亡指标计算通常以10万分娩量作为基数。
C.孕产妇死亡率的指标是反应该国家或地区经济医疗水平的指标。
D.孕产妇死亡率较高的地区死亡产科因素的死亡构成比较高,死亡率低的地区以非产科因素构成比较高。
A.孕产妇死亡率指标计算:分子是当地某一时段的孕产妇死亡人数,分母是当地同一时段所有的孕产妇数。
4 8 2006 7. 关于促进母乳喂养成功的措施中哪些是错误的?(2.0分) A.孩子一出生尽早和母亲皮肤接触,吸吮母亲乳头。
B.正常新生儿4个月内除母乳外不需要给孩子添加任何辅助食物甚至水。
C.产后24小时和母亲同室
D.孩子出生后养成定时哺乳的习惯。
D.孩子出生后养成定时哺乳的习惯。
4 8 2006 8. 雌激素类的药理作用中,不包括(2.0分) A.增强子宫平滑肌对缩宫素的敏感性
B.能明显促进蛋白质合成,减少蛋白质分解
C.使阴道上皮增生,浅表层细胞角化
D.促进女性性器官的发育和成熟,维持女性第二性征。
E.较大剂量时可抑制下丘脑-垂体系统释放GnRH
B.能明显促进蛋白质合成,减少蛋白质分解
4 8 2006 9. 下列哪个药物适用于催产和引产 (2.0分) A.垂体后叶素
B.缩宫素
C.麦角毒
D.麦角新碱
E.麦角胺
B.缩宫素
4 8 2006 10. 关注妇女保健的最重要意义? (2.0分) A.妇女从生理上体弱多病
B.妇女人口多
C.妇女健康关系到子代健康;妇女是脆弱人群(社会地位、经济、教育、就业、医疗);妇女在生理解剖结构上与男性不同,在青春期、围婚期、孕期、分娩期、产褥期、哺乳期、更年期有特殊的生理和病理问题。
D.妇女社会地位低,大多数没有医疗保障
C.妇女健康关系到子代健康;妇女是脆弱人群(社会地位、经济、教育、就业、医疗);妇女在生理解剖结构上与男性不同,在青春期、围婚期、孕期、分娩期、产褥期、哺乳期、更年期有特殊的生理和病理问题。
4 8 2006 11. 人群对艾滋病病毒的易感性正确的描述是:(2.0分) A.只有高危人群易感
B.种族、性别与易感性有关
C.人群对HIV普遍易感
D.只有性传播疾病患者易感
C.人群对HIV普遍易感
4 8 2006 12. 孕激素类药物可用于治疗 (2.0分) A.再生障碍性贫血
B.消耗性疾病等引起的虚弱
C.诱发流产
D.先兆流产
E.卵巢功能不全和闭经
D.先兆流产
4 8 2006 13. 下列哪项不适合选择体外受精胚胎移植手术? (2.0分) A.双侧输卵管切除术后
B.免疫因素所致不孕
C.双侧输卵管阻塞
D.子宫内膜异位症
E.结核性子宫内膜炎
E.结核性子宫内膜炎
4 8 2006 14. A 44-year-old woman (gravida 5, para 5) comes in complaining that she has noticed a bulge protruding out of her vagina. Her other medical problems include hypertension treated with medication, diabetes mellitus, and alcoholism. She stands at work as a grocery clerk. She has a family history of genital prolapse. On examination,you notice a uterine prolapse, cystocele,and rectocele. Which of the following is her major risk factor for her pelvic support disorder? (2.0分) A.hypertension
B.childbirth
C.diabetes mellitus
D.positive family history
E.environmental factors-job
B.childbirth
4 8 2006 15. 女性,30岁,婚后3年未孕,月经规则,近2年出现进行性痛经,曾行输卵管通液检查,显示通畅,妇科检查,子宫正常大小,后位,不活动,后壁有触痛性小结节,左附件可及4cm×3cm×3cm包块,不活动,有压痛,为进一步确诊,应首选何种检查(2.0分) A.输卵管碘油造影
B.子宫内膜病理检查
C.腹腔镜检查
D.基础体温测定
E.剖腹探查
C.腹腔镜检查
4 8 2006 16. A patient with hypogonadotropic hypogonadism desires ovulation. What is the initial treatment of choice? (2.0分) A.bromocriptine mesylate
B.hMG therapy
C.low-dose estrogen therapy
D.cyclic progesterone
E.clomiphene citrate
B.hMG therapy
4 8 2006 17. 精子在进入宫腔后,能存活(2.0分) A.24小时
B.12小时
C.6小时
D.2~3天
E.3~5天
D.2~3天
4 8 2006 18. 排卵后在体内卵子的寿命是(2.0分) A.2~3天
B.12小时
C.24小时
D.6小时
E.3~5天
C.24小时
4 8 2006 19. A 22-year-old woman with amenorrhea of 6 weeks' duration undergoes surgery for acute appendicitis. At the time of surgery a 3-cm semisolid left ovarian cyst is discovered. It is vascular and appears to contain a blood-filled central cavity. A serum pregnancy test is positive. Of the following, what is the most appropriate next step in this patient's management? (2.0分) A.oophorectomy
B.salpingo-oophorectomy
C.ovarian wedge resection
D.ovarian cystectomy
E.no additional therapy indicated
E.no additional therapy indicated
4 8 2006 20. A 28-year-old woman asks, "What percentage of reproductive-age couples are unable to conceive after 1 year of coitus without contraception?"How can you correctly respond? (2.0分) A.10 to 20%
B.1 to2%
C.30 to 40%
D.50 to 60%
E.greater than 60%
A.10 to 20%
4 8 2006 21. 下列关于CIN的叙述,哪项不正确?(2.0分) A.CIN病变上皮细胞核分裂增加,可以出现病理性核分裂
B.CIN I级不属于癌前病变
C.CIN以及宫颈癌与HPV-6、HPV-11感染关系密切
D.宫颈鳞一般由CIN发展而来
C.CIN以及宫颈癌与HPV-6、HPV-11感染关系密切
4 8 2006 22. A 21-year-old diabetic athletic woman on a lowdose oral contraceptive comes to your clinic with irregular menses and galactorrhea. On examination, galactorrhea is confirmed, with fat globules seen microscopically. She currently takes metoclopramide (Reglan) for delayed gastric emptying. A random serum prolactin level is 65 ng/mL. Which of the following is most likely responsible for her hyperprolactinemia? (2.0分) A.oral contraceptive
B.pregnancy
C.metoclopramide
D.pituitary adenoma
E.exercis
C.metoclopramide
4 8 2006 23. 下列哪项不属于辅助生育技术(2.0分) A.IVF/ET
B.AIH
C.TCT
D.PGD
E.ICSI
C.TCT
4 8 2006 24. In addition to history and physical examination, which of the following help to differentiate anatomical causes of abnormal uterine bleeding from dysfunctional uterine bleeding? (2.0分) A.Basal body temperatrue chart
B.Endometrial biopsy
C.Daily FSH levels
D.Daily prolactin levels
B.Endometrial biopsy
4 8 2006 25. A 26-year-old woman presents with hirsutism and irregular menses. Her mother, who is diabetic, had similar complaints prior to menopause. On physical exam, this patient is noted to have terminal hair on her chin and a gray-brown velvety discoloration on the back of her neck. This lesion is acanthosis nigricans. What is the most likely diagnosis in this patient? (2.0分) A.Hyperinsulinemia
B.Adrenal tumor
C.Late onset congenital adrenal hyperplasia
D.Ovarian androgen excess
E.Polycystic ovarian syndrome
A.Hyperinsulinemia
4 8 2006 26. A 56-year-old woman complains that she is "sitting on a ball." She says constipation is a significant problem for her and that sometimes she needs to push stool out of her rectum by inserting a finger in the vagina and pressing on a bulge. On further examination, which of the following will you most likely find? (2.0分) A.complete uterine prolapse
B.rectocele
C.enterocele
D.cystocele
E.hemorrhoid
B.rectocele
4 8 2006 27. what is the most common cause of pathologic amenorrhea? (2.0分) A.sherman syndrome
B.disruption of the hypothalamic-pituitary axis
C.Outflow obstruction
D.kallman syndrome
A.sherman syndrome
4 8 2006 28. 下列有关葡萄胎的描述,哪项不正确?(2.0分) A.绒毛水肿呈葡萄状
B.患者血尿hCG水平显著升高
C.发生与染色体异常有关
D.重要特征是滋养层细胞增生
E.完全性葡萄胎大多发展为侵袭性葡萄胎或绒癌
E.完全性葡萄胎大多发展为侵袭性葡萄胎或绒癌
4 8 2006 29. A 34-year-old woman comes to you for a chief complaint of hirsutism. She states that this has been present since menarche, but has gotten worse in the past two years. Her menses have become more irregular, now every 28-45 days apart. She states that she quit smoking and gained approximately thirty pounds in the past three years. Her mother is obese, diabetic and has hirsutism. The patient has acanthosis nigricans present on the back of her neck. There is no hair seen on her chin, but she shaves every few days. She also has a significant amount hair present on the back of her hand and on her forearms. TSH, prolactin, 17-hydroxyprogesterone and DHEAS are normal. Testosterone is mildly elevated. Which of the following is the most likely etiology of her hirsutism?(2.0分) A.Ovarian neoplasm
B.Polycystic ovarian syndrome
C.Diabetes
D.Cushing's syndrome
E.Adrenal tumor
B.Polycystic ovarian syndrome
4 8 2006 30. A 49-year-old parous woman comes in complaining that over the last several years it feels as though "her organs are progressively falling out her vagina." Along with this, she complains of losing urine with coughing, occasional urgency,and sometimes a feeling of incomplete emptying of her bladder with voiding. On further examination,which of the following will you likely find?(2.0分) A.rectocele
B.cystocele
C.enterocele
D.complete uterine prolapse
E.urinary tract infection (UTI)
B.cystocele
4 8 2006 31. A 16-year-old girl has not experienced menarche. Examination shows absence of breast development and small but otherwise normal female pelvic organs. Which of the following diagnostic tests is most useful in determining the etiology of the amenorrhea? (2.0分) A.serum follicle-stimulating hormone (FSH)
B.serum estradiol
C.magnetic resonance imaging (MRI) of the head
D.serum testosterone
E.ovarian biopsy
A.serum follicle-stimulating hormone (FSH)
4 8 2006 32. 研究妇女保健工作的最主要目的?(2.0分) A.使妇女能获得特殊关注
B.减少并控制危害妇女健康的疾病;保障妇女生殖健康
C.妇女病能得到免费治疗
D.能实施我国的计划生育措施。
B.减少并控制危害妇女健康的疾病;保障妇女生殖健康
4 8 2006 33. A 32-year-old woman comes to the office due to the inability to conceive for last two years. She reports having been on oral contraceptives for 8 years prior to stopping them a year ago to attempt pregnancy. She had menarche at age 14 and has had irregular cycles about every 3 months until she started birth control pills, which made her cycles regular. In the last year, she has had about 5 cycles in total; her last menstrual period was 6 weeks ago. She is otherwise in good health and has not had any surgeries. She has no history of abnormal Pap smears or sexually transmitted diseases. Her only medication is a multivitamin. Her husband of 4 years is 35-years-old, and has a 10-year-old son from a previous marriage. She is 5'4" tall and weighs 165 pounds. On general appearance, she seems to be hirsute on the face and the abdomen. The rest of her exam is otherwise normal. Which of the following is most likely to help identify the underlying cause of this woman's infertility? (2.0分) A.Ratio of luteinizing hormone to follicle stimulating hormone (LH/FSH)
B.Testosterone levels
C.Dehydroepiandrosterone Sulfate (DHEAS) levels
D.Thyroid function tests
E.Prolactin levels
B.Testosterone levels
4 8 2006 34. You are counseling a 30-year-old woman who wants to become pregnant. Which of the following is the most accurate method for her to time intercourse? (2.0分) A.profuse, thin, acellular cervical mucus
B.thermogenic shift in basal body temperature (BBT)
C.urinary luteinizing hormone (LH) kit testing
D.serum progesterone level
E.mittelschmerz
C.urinary luteinizing hormone (LH) kit testing
4 8 2006 35. An 18-year-old woman comes to your clinic with irregular cycles since menarche and mild hirsutism. She is not interested in pregnancy or contraception. Her serum TSH, prolactin, and dehydroepiandrosterone sulfate (DHEAS) levels are normal, with a slightly elevated serum testosterone level of 80 ng/dL. Which of the following is the most appropriate next step for this patient? (2.0分) A.clomiphene citrate
B.GnRH stimulation test
C.oral contraceptive treatment
D.endometrial biopsy
E.bromocriptine
C.oral contraceptive treatment
4 8 2006 36. A 15-year-old girl is seen in the emergency department. She has a sudden onset of heavy vaginal bleeding. She has noted irregular, painless vaginal bleeding of 6 months' duration. Her past medical history is unremarkable, and she is not sexually active. Physical and pelvic examinations are normal, but blood is coming through the cervical os. A serum pregnancy test is negative, and hematocrit is 37% (normal, 35 to 45%). Of the following, which is the best course of immediate action? (2.0分) A.observation
B.nonsteroidal anti-inflammatory therapy
C.progesterone therapy
D.estrogen therapy
E.D&C
D.estrogen therapy
4 8 2006 37. Fecal incontinence may be related to which of the following?(2.0分) A.innervation of the pelvic floor and the anal sphincters
B.interplay between the pubococcygeus muscle and rectum
C.normal colonic transit time
D.nulliparity
E.urinary retention
A.innervation of the pelvic floor and the anal sphincters
4 8 2006 38. 患者,24岁,结婚3年不孕,月经周期24天,经期正常,经量多,测基础体温曲线高温相为8天。本例考虑为(2.0分) A.属正常月经周期
B.黄体功能不全
C.子宫内膜脱落不全
D.无排卵型功能失调性子宫出血
E.排卵型月经过多
B.黄体功能不全
4 8 2006 39. 子宫颈癌最常发生于(2.0分) A.宫颈鳞状上皮
B.宫颈管粘膜上皮
C.宫内膜化生的鳞状上皮
D.宫颈鳞状上皮和柱状上皮交界处
E.子宫颈腺体
D.宫颈鳞状上皮和柱状上皮交界处
4 8 2006 40. A 25-year-old woman suffers a severe intrapartum hemorrhage. Which of the following symptoms is evidence of pituitary infarction? (2.0分) A.lactation failure
B.diarrhea
C.easy bruisability
D.infrequent urination
E.perspiration
A.lactation failure
4 8 2006 41. A 32-year-old woman comes to the office due to the inability to conceive for last two years. She reports having been on oral contraceptives for 8 years prior to stopping them a year ago to attempt pregnancy. She had menarche at age 14 and had irregular cycles about every 3 months apart until started birth control pills, which made her cycles regular. In the last year, she has had about 5 cycles in total; her last menstrual period was 6 weeks ago. She is otherwise in good health and has not had any surgeries. She has no history of abnormal Pap smears or sexually transmitted diseases. Her only medication is a multivitamin. Her husband of 4 years is 35-years-old, and has a 10-year-old son from a previous marriage. She is 5'4" tall and weighs 165 pounds. On general appearance, she seems to be hirsute on the face and the abdomen. The rest of her exam is otherwise normal. In addition to weight loss and starting Metformin, what is the most appropriate treatment for this patient's infertility problem? (2.0分) A.Laparoscopy and ovarian biopsy
B.Ovulation induction agents such as Clomiphene Citrate
C.In vitro fertilization with ICSI (Intracytoplasmic sperm injection)
D.Intrauterine insemination
E.In vitro fertilization without ICSI
B.Ovulation induction agents such as Clomiphene Citrate
4 8 2006 42. 艾滋病的全称是: (2.0分) A.先天性免疫缺陷综合征
B.获得性免疫缺陷综合征
C.免疫缺陷综合征
D.继发性免疫缺陷综合征
B.获得性免疫缺陷综合征
4 8 2006 43. 下列哪项不是急性HIV感染的临床表现: (2.0分) A.咽痛
B.发热
C.淋巴结肿大
D.口腔真菌感染
D.口腔真菌感染
4 8 2006 44. 生殖道衣原体感染主要的病原体是 (2.0分) A.衣原体
B.支原体
C.螺旋体
D.病毒
A.衣原体
4 8 2006 45. 女性不孕因素中最常见的病因是(2.0分) A.无排卵
B.子宫颈糜烂
C.子宫内膜异位症
D.输卵管因素
E.子宫肌瘤
D.输卵管因素
4 8 2006 46. 下列有关梅毒螺旋体的特点正确的是 (2.0分) A.离开人体不易生存
B.寒冷条件下不能存活
C.煮沸、干燥和普通消毒剂等很难杀灭
D.属于需氧微生物
A.离开人体不易生存
4 8 2006 47. 治疗淋病的首选药物是(2.0分) A.四环素
B.头孢三嗪
C.阿奇霉素
D.青霉素
E.复方新诺明
B.头孢三嗪
4 8 2006 48. A 42-year-old G2P2 woman presents with very profuse menstrual bleeding. Her menses occurs every 28 days. She noted that the bleeding became worse after the birth of her last child and it has increased in severity over the last two years. She has a history of 2 Cesarean sections. She is currently taking combination oral contraceptives, which were started six months ago by her family physician. On pelvic examination, the uterus is normal in size and no adnexal masses are noted. Her hemoglobin is 11.9 mg/dl, hematocrit 35.1%. Pelvic sonography reveals a normal appearing uterus and a normal appearing endometrial echo. The ovaries appear normal without adnexal masses. Endometrial biopsy revealed a secretory endometrium. The patient is unhappy with the current therapy and wants a new and more definitive treatment. Which of the following would be the best long-term treatment?(2.0分) A.Leuprolide acetate
B.Dilatation and Curettage
C.Laparoscopic myomectomy
D.Endometrial ablation
E.Cyclic Progestins
D.Endometrial ablation
4 8 2006 49. 淋病发生并发症的常见原因是(2.0分) A.性交
B.酗酒
C.治疗不当
D.抽烟
D.抽烟
4 8 2006 50. 湿疹样乳腺癌的外科治疗?(2.0分) A.乳腺癌改良根治术
B.单纯乳腺切除手术
C.乳腺癌根治术
B.单纯乳腺切除手术
4 8 2007 1. 以下哪种不是诱发排卵的药物是(2.0分) A.黄体生成激素释放激素(LHRH)
B.三苯氧胺
C.绝经促性素(hMG)
D.克罗米酚(CC)
E.溴隐亭
B.三苯氧胺
4 8 2007 2. 研究妇女保健工作的最主要目的?(2.0分) A.妇女病能得到免费治疗
B.使妇女能获得特殊关注
C.减少并控制危害妇女健康的疾病;保障妇女生殖健康
D.能实施我国的计划生育措施。
C.减少并控制危害妇女健康的疾病;保障妇女生殖健康
4 8 2007 3. A 26-year-old women taking antibiotics for cystitis complains of itching ,burning ,and a yellowish vaginal discharge. Which of the following is the best therapy ? (2.0分) A.Erythromycin
B.Metronidazole
C.Fluconazole
D.Hydrocortisone
E.Clindamycin
C.Fluconazole
4 8 2007 4. A 16-year-old G0 woman presents to the emergency department with a two day history of “belly pain.” She is sexually active with a new partner and is not using any form of contraception. Temperature is 39°C. On exam, she has lower abdominal tenderness and guarding. On pelvic exam, she has diffuse tenderness over the uterus and bilaterally. Beta hCG <5. What is the most likely diagnosis in this patient? (2.0分) A.Ectopic pregnancy
B.Intrauterine pregnancy
C.Acute Cystitis
D.Appendicitis
E.Acute salpingitis
E.Acute salpingitis
4 8 2007 5. 我国计算围产儿死亡率的定义中哪个说法是正确的?(2.0分) A.分母为与分子同一时间段的产妇数。
B.分子是孕24周后所分娩的所有死胎,死产和7天内的新生儿死亡数。
C.分子是孕28周后所分娩的所有死胎,死产和7天内的新生儿死亡数。
D.分母为与分子同一时间段的活产数。
C.分子是孕28周后所分娩的所有死胎,死产和7天内的新生儿死亡数。
4 8 2007 6. 关于流行病学,下列哪种说法正确(3.0分) A.从个体的角度研究疾病和健康状况及其影响因素
B.只研究慢性病的危险因素
C.只研究传染病的流行特征和防治措施
D.研究人群中疾病和健康状况的分布及其影响因素
D.研究人群中疾病和健康状况的分布及其影响因素
4 8 2007 7. A 15-year-old girl is seen in the emergency department. She has a sudden onset of heavy vaginal bleeding. She has noted irregular, painless vaginal bleeding of 6 months' duration. Her past medical history is unremarkable, and she is not sexually active. Physical and pelvic examinations are normal, but blood is coming through the cervical os. A serum pregnancy test is negative, and hematocrit is 37% (normal, 35 to 45%). Of the following, which is the best course of immediate action? (2.0分) A.observation
B.progesterone therapy
C.estrogen therapy
D.nonsteroidal anti-inflammatory therapy
E.D&C
C.estrogen therapy
4 8 2007 8. 在设计病例对照研究时,对照的选择最好是: (2.0分) A.从该地区未患该病的全人群中选对照
B.从医院的其他病人中选对照
C.从不同人群中选择多组对照
D.从病人的亲属,同事中选对照
A.从该地区未患该病的全人群中选对照
4 8 2007 9. 据统计正常情况下,婚后2年初孕率为(2.0分) A.60%
B.90%
C.80%
D.70%
E.95%
E.95%
4 8 2007 10. 一项乳房癌与高脂肪饮食关系的前瞻性队列研究中,发现高脂肪饮食妇女乳房癌发病率48.0/10万,非高脂肪饮食者为25.4/10万,其相对危险度为: (2.0分) A.48
B.22.6/10万
C.1.89
D.0.0048
C.1.89
4 8 2007 11. Hirsutism, ovarian androgen excess, and elevated serum LH levels are associated with which of the following? (2.0分) A.PCOS
B.Cushing's syndrome
C.adrenal tumor
D.11-beta-hydroxylase deficiency
E.arrhenoblastoma
A.PCOS
4 8 2007 12. 青年女性,闭经3个月,阴道不规律出血,血块中夹有水泡。检查发现子宫体积大,阴道壁有暗紫色结节、出血、坏死。 最大可能是(2.0分) A.恶性葡萄胎
B.葡萄胎
C.宫外孕
D.葡萄状肉瘤
E.绒毛膜癌
A.恶性葡萄胎
4 8 2007 13. 关于子宫颈鳞癌发生发展过程下列哪一项是正确的(3.0分) A.早期浸润癌-原位癌-浸润癌
B.原位癌-早期浸润癌-浸润癌
C.上皮增生-原位癌-浸润癌
D.上皮不典型增生-早期浸润癌-浸润癌
E.上皮不典型增生-原位癌-早期浸润癌-浸润癌
E.上皮不典型增生-原位癌-早期浸润癌-浸润癌
4 8 2007 14. 孕激素类药物可用于治疗 (2.0分) A.再生障碍性贫血
B.诱发流产
C.先兆流产
D.消耗性疾病等引起的虚弱
E. 卵巢功能不全和闭经
C.先兆流产
4 8 2007 15. 产时妇女保健的关键是(2.0分) A.要住院分娩
B.要做到防感染、防滞产、防产伤、防出血、防窒息
C.要新法接生
D.高危孕妇提前住院
B.要做到防感染、防滞产、防产伤、防出血、防窒息
4 8 2007 16. A 28-year-old patient complains of amenorrhea after dilation and curettage (D&C) for postpartum bleeding. Of the following, which is the most likely diagnosis? (2.0分) A.Kallmann syndrome
B.Sheehan's syndrome
C.gonadal dysgenesis
D.Mayer-Rokitansky-Kiister-Hauser syndrome
E.Asherman's syndrome
E.Asherman's syndrome
4 8 2007 17. 发病率指标来自(2.0分) A.对门诊病人的调查
B.对住院病人的调查
C.对社区人群的调查
D.对所有病人的调查
C.对社区人群的调查
4 8 2007 18. A 32-year-old G3P1 woman presents to your office for an annual examination. During the course of your discussion, the patient mentions that she has had an annoying vulvar irritation and slight itch for months. She sometimes notices an odor, but does not experience a specific discharge. She is sexually active with the same male partner for the past 6 months and they occasionally use condoms. On examination, she has a grayish, frothy discharge within the vaginal vault. You perform a wet prep of her vaginal secretions which shows flagellated organisms. Which of the following is the most likely diagnosis in this patient? (2.0分) A.Bacterial vaginosis
B.Trichomonas
C.Yeast, Non-Candida albicans
D.Recent intercourse
E.Chlamydia
B.Trichomonas
4 8 2007 19. 绒毛膜癌与侵蚀性葡萄胎的区别是(2.0分) A.间质的含量
B.肿瘤细胞的异型性
C.有无血管
D.有无绒毛
E.有无转移
D.有无绒毛
4 8 2007 20. 我国的孕产妇系统保健内容包括哪些内容 (2.0分) A.孕期至少5次的产前检查
B.住院分娩,产后访视3次
C.孕12周内进行早孕检查
D.及早诊断妊娠
E.以上都是
E.以上都是
4 8 2007 21. A 30-year-old women is brought to the emergency department with fever and a blood pressure of 70/40mmHg.she is presumed to be in septic shock.which of the following is a fundamental principle for treatment?(2.0分) A.Oral fluid resuscitation
B.Intravenous normal saline
C.Await blood culture results prior to initiation of antibiotic therapy
D.Plasmapheresis
E.Blood transfusion
B.Intravenous normal saline
4 8 2007 22. 上述患者最佳治疗方案(2.0分) A.IVF-ET
B.宫腔镜
C.输卵管通液
D.腹腔镜
E.以上都不是
D.腹腔镜
4 8 2007 23. 恶性葡萄胎与良性葡萄胎的主要区别是(2.0分) A.可见水肿绒毛
B.绒毛消失
C.滋养细胞增生
D.绒毛侵犯子宫壁深部肌层
E.绒毛间质血管消失
D.绒毛侵犯子宫壁深部肌层
4 8 2007 24. 女性不孕最常见的因素是(2.0分) A.宫颈因素
B.输卵管因素
C.子宫因素
D.不排卵
E.阴道因素
B.输卵管因素
4 8 2007 25. 关于精液正常值下述哪项是错误的(2.0分) A.精子数>4000万/ml
B.活动数>32%
C.pH为7.2~7.5
D.精液量为1.5ml~6ml,平均2~4ml
E.异常精子数<85%
E.异常精子数<85%
4 8 2007 26. A 27-year-old woman complains of a fishy odor and a vaginal discharge.Speculum examination reveals an erythematous vagina and punctations of the cervix.Which of the following is the most likely diagnosis?(2.0分) A.Candidal vaginitis
B.Human papilloma virus
C.Bacterial vaginosis
D.Trichomonal vaginitis
E.Herpes simple virus
D.Trichomonal vaginitis
4 8 2007 27. 女性,30岁,原发不孕3年,月经5~6/20+~50+天,量中等,无痛经,妇科检查未发现特殊症状。进一步检查应首选(2.0分) A.X线腹部平片
B.性交后试验
C.输卵管通液
D.B型超声
E.基础内分泌测定和排卵监测
E.基础内分泌测定和排卵监测
4 8 2007 28. 女性,33岁。孕3产0,最后妊娠行人工流产术终止,至今已5年,未采取任何避孕措施,妇科检查:宫体正常大小,双侧附件区增厚,无明显压痛。此患者最可能的诊断是(2.0分) A.继发不孕
B.绝对不孕+附件炎
C.继发不孕+附件炎
D.原发不孕
E.原发不孕+附件炎
C.继发不孕+附件炎
4 8 2007 29. A 25-year-old woman experiences galactorrhea and amenorrhea of 8 weeks' duration with irregular vaginal bleeding. Which of the following serum assays should initially be performed? (2.0分) A.hCG
B.prolactin
C.progesterone
D.FSH
E.luteinizing hormone (LH)
A.hCG
4 8 2007 30. 上述患者最终行HSG, 图片如下图所示,根据图片,你认为其诊断为下述哪种(2.0分) A.子宫肌瘤
B.宫腔粘连
C.卵巢子宫内膜异位症
D.输卵管积液
E.以上都不是
D.输卵管积液
4 8 2007 31. A 19-year-old woman and her boyfriend wish to use condoms as a barrier contraceptive method. This couple should be advised that the most common reason for failure (pregnancy or STD transmission) is which of the following? (2.0分) A.breakage
B.inconsistent use
C.use without concombinate use of a spermicide
D.leakage caused by spermicidal creams
E.spill of condom contents upon withdrawal
B.inconsistent use
4 8 2007 32. A 56-year-old woman complains that she is "sitting on a ball." She says constipation is a significant problem for her and that sometimes she needs to push stool out of her rectum by inserting a finger in the vagina and pressing on a bulge. On further examination, which of the following will you most likely find? (2.0分) A.complete uterine prolapse
B.rectocele
C.enterocele
D.cystocele
E.hemorrhoid
B.rectocele
4 8 2007 33. what is the most common cause of pathologic amenorrhea? (2.0分) A.disruption of the hypothalamic-pituitary axis
B.Outflow obstruction
C.asherman syndrome
D.kallman syndrome
C.asherman syndrome
4 8 2007 34. The administration of mifepristone results in which of the following? (2.0分) A.menses when given during the follicular phase
B.delayed menses when given during the midluteal phase
C.abortion when given in early pregnancy
D.resistance to prostaglandin inhibitors
E.induction of progesterone receptors in the endometrium
C.abortion when given in early pregnancy
4 8 2007 35. A 20-year-old woman complains of 6-weeks amenorrhea with pregnancy test positive. The ultrasound shows an intrauterine gestation sac 1.5cm. The patient does not want to keep the baby. Which method would you advised? (2.0分) A.prostaglandins
B.intra-amniotic installation agents
C.high-dose oxytocin
D.Mifepristone(RU486) +Misoprostol
E.hysteroscopy
D.Mifepristone(RU486) +Misoprostol
4 8 2007 36. 关注妇女保健的最重要意义? (2.0分) A.妇女健康关系到子代健康;妇女是脆弱人群(社会地位、经济、教育、就业、医疗);妇女在生理解剖结构上与男性不同,在青春期、围婚期、孕期、分娩期、产褥期、哺乳期、更年期有特殊的生理和病理问题。
B.妇女从生理上体弱多病
C.妇女人口多
D.妇女社会地位低,大多数没有医疗保障
A.妇女健康关系到子代健康;妇女是脆弱人群(社会地位、经济、教育、就业、医疗);妇女在生理解剖结构上与男性不同,在青春期、围婚期、孕期、分娩期、产褥期、哺乳期、更年期有特殊的生理和病理问题。
4 8 2007 37. 在病例对照研究中,匹配是指: (2.0分) A.在安排病例和对照时,使两者的研究因素相一致的方法
B.在安排病例和对照组时,使两者的某些特征或变量相一致的方法
C.在安排病例和对照时,使两者的所有特征或变量相一致的方法
D.病例组的研究因素的数量与对照组完全一致
B.在安排病例和对照组时,使两者的某些特征或变量相一致的方法
4 8 2007 38. The following methods are NOT hormonal contraception except:(2.0分) A.nature family planning
B.diaphragm
C.spermicidal
D.condom
E.emergency contraception with progestin
E.emergency contraception with progestin
4 8 2007 39. In addition to history and physical examination, which of the following help to differentiate anatomical causes of abnormal uterine bleeding from dysfunctional uterine bleeding?(2.0分) A.Daily FSH levels
B.Basal body temperatrue chart
C.Endometrial biopsy
D.Daily prolactin levels
C.Endometrial biopsy
4 8 2007 40. A 25-year-old sexually active woman complains of a "fishy" smelling gray-white vaginal discharge. You examine this on wet mount and see epithelial cells with clusters of bacteria obscuring their borders. The vaginal pH is 5.5.This infection has been most closely implicated in which of the following complications of pregnancy? (2.0分) A.preterm birth.
B.intrauterine growth restriction
C.congenital cataracts
D.placenta previa
E.preeclampsia
A.preterm birth.
4 8 2007 41. A 23-year-old woman with irregular menses complains of facial hair increasing in amount over several years. She is sexually active but does not wish to conceive. Examination demonstrates hirsutism, obesity, and hyperpigmentation of the neck and axillae. The ovaries are bilaterally enlarged and cystic. A serum testosterone value is 1.2 ng/mL (normal, <0.8 ng/mL). Serum levels of DHEAS, 17-hydroxyprogesterone (17-OHP), and prolactin are normal. Of the following, which is the best single therapeutic agent for this patient? (2.0分) A.glucocorticoids
B.oral contraceptives (OCs)
C.clomiphene citrate
D.antiandrogens
E.GnRH analogue
B.oral contraceptives (OCs)
4 8 2007 42. Luteectomy before 42 days' gestation is most likely to result in which of the following? (2.0分) A.spontaneous miscarriage
B.prolonged (postdates) gestation
C.reduction of basal body temperature (BBT)
D.masculinization of a female fetus
E.no demonstrable effect
A.spontaneous miscarriage
4 8 2007 43. A 22-year-old woman is seen for vaginal discharge.The presumptive diagnosis is bacterial vainosis.Which of the following most likely is present?(2.0分) A.pH<4.5
B.Positive Kleihauer-Betke test
C.Predominance of anaerobes
D.Heterogenous vaginal discharge
E.Negative clue cell
C.Predominance of anaerobes
4 8 2007 44. A 23-year-old patient is considering contraceptive methods but is devoutly religious and will not accept a method that may "cause an abortion." The primary mechanism by which IUD prevent pregnancy is which of the following? (2.0分) A.preventing fertilization
B.creating chronic endometritis
C.inhibiting ovulation
D.altering tuba1 motility
E.destroying sperm
A.preventing fertilization
4 8 2007 45. A 25-year-old woman suffers a severe intrapartum hemorrhage. Which of the following symptoms is evidence of pituitary infarction? (2.0分) A.easy bruisability
B.diarrhea
C.infrequent urination
D.lactation failure
E.perspiration
D.lactation failure
4 8 2007 46. Kegel exercises were designed to do which of the following? (2.0分) A.strengthen the abdominal muscles after childbirth
B.increase the blood flow to the perineum to speed the healing of an episiotomy
C.prevent denervation of pelvic muscles after childbirth
D.improve the tone of the muscles surrounding the bladder base and proximal bladder neck
E.decrease the muscle atrophy associated with aging
D.improve the tone of the muscles surrounding the bladder base and proximal bladder neck
4 8 2007 47. A 32-year-old G0 woman comes to your office because she has been unable to conceive for one year. She has regular menses every 27-34 days. She is currently in a mutually monogamous relationship, reports menarche at the age of 13, with coitarche at the age of 15; she has had 7 sexual partners, including her husband of 3 years. The patient reports that she has had an abnormal Pap smear about 4 years ago, but things have been normal since. She also states that, in her early twenties, she had problems with “female infections” and was on “lots of antibiotics,” but was never hospitalized. She denies any pain with intercourse and she denies any intermenstrual bleeding. Her husband has fathered 2 children from a previous marriage. What is the most likely underlying cause of infertility in this patient? (2.0分) A.Tubal disease
B.Ovulatory dysfunction
C.Male factor infertility
D.Cervical stenosis
E.Endometriosis
A.Tubal disease
4 8 2007 48. A 44-year-old woman (gravida 5, para 5) comes in complaining that she has noticed a bulge protruding out of her vagina. Her other medical problems include hypertension treated with medication, diabetes mellitus, and alcoholism. She stands at work as a grocery clerk. She has a family history of genital prolapse. On examination,you notice a uterine prolapse, cystocele,and rectocele. Which of the following is her major risk factor for her pelvic support disorder? (2.0分) A.positive family history
B.hypertension
C.diabetes mellitus
D.childbirth
E.environmental factors-job
D.childbirth
4 8 2007 49. 下列哪个药物适用于催产和引产 (2.0分) A.垂体后叶素
B.缩宫素
C.麦角毒
D.麦角新碱
E.麦角胺
B.缩宫素
4 8 2008 1. 绒毛膜癌与侵蚀性葡萄胎的区别是(2.0分) A.间质的含量
B.有无血管
C.肿瘤细胞的异型性
D.有无绒毛
E.有无转移
D.有无绒毛
4 8 2008 2. 青年女性,闭经3个月,阴道不规律出血,血块中夹有水泡。检查发现子宫体积大,阴道壁有暗紫色结节、出血、坏死。 最大可能是 (2.0分) A.宫外孕
B.葡萄状肉瘤
C.葡萄胎
D.恶性葡萄胎
E.绒毛膜癌
D.恶性葡萄胎
4 8 2008 3. 宫颈早期浸润癌是指 (2.0分) A.癌浸润深度不超过基底膜下5mm
B.癌浸润深度超过基底膜5mm
C.癌浸润深度不超过基底膜下3cm
D.癌浸润深度超过基底膜1mm
E.癌浸润深度在基底膜下0。5~1cm
A.癌浸润深度不超过基底膜下5mm
4 8 2008 4. 诊断早期宫颈癌最可靠的依据是(2.0分) A.有接触性出血史
B.阴道镜检查
C.盆腔检查
D.宫颈细胞学检查
E.宫颈病理切片检查
E.宫颈病理切片检查
4 8 2008 5. 关注妇女保健的最重要意义? (2.0分) A.妇女从生理上体弱多病
B.妇女健康关系到子代健康;;妇女是脆弱人群(社会地位、经济、教育、就业、医疗);妇女在生理解剖结构上与男性不同,在青春期、围婚期、孕期、分娩期、产褥期、哺乳期、更年期有特殊的生理和病理问题。
C.妇女人口多
D.妇女社会地位低,大多数没有医疗保障
B.妇女健康关系到子代健康;;妇女是脆弱人群(社会地位、经济、教育、就业、医疗);妇女在生理解剖结构上与男性不同,在青春期、围婚期、孕期、分娩期、产褥期、哺乳期、更年期有特殊的生理和病理问题。
4 8 2008 6. 目前我国开展婚检的主要目的是(2.0分) A.检查准备结婚的双方是否存在传染病。
B.检查准备结婚的双方是否患有严重的精神性疾病。
C.检查准备结婚的双方是否患有严重遗传性疾病
D.检查准备结婚的双方是否患有影响生育的严重疾病
E.以上都是
E.以上都是
4 8 2008 7. 我国的孕产妇系统保健内容包括哪些内容 (2.0分) A.及早诊断妊娠
B.孕12周内进行早孕检查
C.孕期至少5次的产前检查
D.住院分娩,产后访视3次
E.以上都是
E.以上都是
4 8 2008 8. 女孩青春期出现的最早征兆是?(2.0分) A.月经来潮
B.乳房出现硬结
C.出现阴毛和腋毛
D.身高突增
B.乳房出现硬结
4 8 2008 9. 孕产妇死亡率统计指标中哪项说法是不正确的?(2.0分) A.孕产妇死亡率的指标是反应该国家或地区经济医疗水平的指标。
B.孕产妇死亡指标计算通常以10万分娩量作为基数。
C.孕产妇死亡率指标计算:分子是当地某一时段的孕产妇死亡人数,分母是当地同一时段所有的孕产妇数。
D.孕产妇死亡率较高的地区死亡产科因素的死亡构成比较高,死亡率低的地区以非产科因素构成比较高。
C.孕产妇死亡率指标计算:分子是当地某一时段的孕产妇死亡人数,分母是当地同一时段所有的孕产妇数。
4 8 2008 10. 关于流行病学,下列哪种说法正确 (2.0分) A.从个体的角度研究疾病和健康状况及其影响因素
B.只研究传染病的流行特征和防治措施
C.只研究慢性病的危险因素
D.研究人群中疾病和健康状况的分布及其影响因素
D.研究人群中疾病和健康状况的分布及其影响因素
4 8 2008 11. 患病率指标来自(2.0分) A.现况调查资料
B.门诊病例资料
C.住院病例资料
D.专科医院病例资料
A.现况调查资料
4 8 2008 12. 在设计病例对照研究时,对照的选择最好是: (2.0分) A.从医院的其他病人中选对照
B.从该地区未患该病的全人群中选对照
C.从不同人群中选择多组对照
D.从病人的亲属,同事中选对照
B.从该地区未患该病的全人群中选对照
4 8 2008 13. 在病例对照研究中,比值比(OR)的含义是: (2.0分) A.病例组的发病率与对照组的发病率之比
B.病例组的暴露比值与对照组的暴露比值的比
C.对照组的暴露比值除以病例组的暴露比值
D.病例组的暴露比值与对照组的暴露比值之差
B.病例组的暴露比值与对照组的暴露比值的比
4 8 2008 14. 一项乳房癌与高脂肪饮食关系的前瞻性队列研究中,发现高脂肪饮食妇女乳房癌发病率48.0/10万,非高脂肪饮食者为25.4/10万,其相对危险度为: (2.0分) A.1.89
B.22.6/10万
C.48
D.0.0048
A.1.89
4 8 2008 15. A 19-year-old woman and her boyfriend wish to use condoms as a barrier contraceptive method. This couple should be advised that the most common reason for failure (pregnancy or STD transmission) is which of the following? (2.0分) A.breakage
B.inconsistent use
C.leakage caused by spermicidal creams
D.use without concombinate use of a spermicide
E.spill of condom contents upon withdrawal
B.inconsistent use
4 8 2008 16. A 23-year-old patient is considering contraceptive methods but is devoutly religious and will not accept a method that may "cause an abortion." The primary mechanism by which IUD prevent pregnancy is which of the following? (2.0分) A.creating chronic endometritis
B.preventing fertilization
C.inhibiting ovulation
D.altering tuba1 motility
E.destroying sperm
B.preventing fertilization
4 8 2008 17. Reducing the estrogen content of OCs results in an decreasing in the rate of which of the following? (2.0分) A.pregnancy
B.breakthrough bleeding (BTB)
C.thromboembolic complications
D.insulin resistance
E.premenstrual symptoms
C.thromboembolic complications
4 8 2008 18. The following methods are NOT hormonal contraception except: (2.0分) A.condom
B.diaphragm
C.spermicidal
D.nature family planning
E.emergency contraception with progestin
E.emergency contraception with progestin
4 8 2008 19. A 17-year-old woman with a history of ectopic pregnancy presents for contraceptive contraception. Which of the following contraceptive methods would be relatively or absolutely contraindicated? (2.0分) A.OCs
B.progestin-only pill (the minipill)
C.male condoms
D.cervical cap
E.IUD
E.IUD
4 8 2008 20. A 20-year-old woman complains of 6-weeks amenorrhea with pregnancy test positive. The ultrasound shows an intrauterine gestation sac 1.5cm. The patient does not want to keep the baby. Which method would you advised? (2.0分) A.high-dose oxytocin
B.intra-amniotic installation agents
C.prostaglandins
D.Mifepristone(RU486) +Misoprostol
E.hysteroscopy
D.Mifepristone(RU486) +Misoprostol
4 8 2008 21. A 27-year-old G0 presents to the clinic because of concerns that she has not been able to get pregnant for the last 3 months. She married a year ago and was using condoms for contraception, which she stopped 3 months ago when she decided to start a family. She is in good health and her only medication is a prenatal vitamin. Her periods are regular every 28 days with normal flow; her last period was 2 weeks ago. She has no history of sexually transmitted infections and no abnormal Pap smears. Her husband is also healthy with no medical problems. She is 5'4" tall and weighs 130 pounds. Her examination, including a pelvic exam, is completely normal. What is the most appropriate next step in the management of this patient? (2.0分) A.Reassurance and observation
B. Order a hysterosalpingogram
C.Order a pelvic ultrasound
D. Recommend husband gets a semen analysis
E.Order blood LH and FSH levels
A.Reassurance and observation
4 8 2008 22. A 45-year-old G3P3 woman comes to the office because she has been unable to conceive for the last two years. She is healthy and has 3 children ages 10, 12 and 14, whom she conceived with her husband. She used a copper IUD after the birth of her last child and had it removed two years ago, hoping to have another child. She has no history of sexually transmitted infections or abnormal Pap smears. Her cycles are regular every 28 to 35 days. She is not taking any medications. She has been married for the last 16 years, and her husband is 52-years-old and in good health. Her physical examination, including a pelvic exam, is completely normal. What is the most appropriate next step in the management of this patient? (2.0分) A.Perform a hysteroscopy
B. Order a hysterosalpingogram
C. Order a Clomid challenge test
D. Order a semen analysis
E.Basal body temperatures for 6 months
C. Order a Clomid challenge test
4 8 2008 23. 23. A 27-year-old G0 presents to the clinic because of concerns that she has not been able to get pregnant for the last year. She has been married for two years ago and was using birth control pills for contraception. She stopped using birth control pills when she decided to start a family one year ago. She is in good health and her only medication is a prenatal vitamin. Her periods are regular, every 28 days, with normal flow; her last period was 2 weeks ago. She has no history of sexually transmitted infections and no abnormal Pap smears. Her husband is also healthy with no medical problems. She is 5'4" tall and weighs 130 pounds. Her examination, including a pelvic exam, is completely normal. Laboratory results show normal thyroid function tests and normal prolactin level. What is the most appropriate next step in the management of this patient? (2.0分) A.Reassurance and observation
B.Perform a pelvic ultrasound
C.Order a hysterosalpingogram
D.Order a semen analysis
E.Recommend a diagnostic laparoscopy
D.Order a semen analysis
4 8 2008 24. A 31-year-old infertility patient with regular ovulatory menstrual cycles has begun therapy with clomiphene citrate. Before she starts therapy,what information should you provide her regarding the medication? (2.0分) A.Typically, the timing of ovulation is increased by a week.
B.Approximately 40% of patients will respond to clomiphene citrate with increased endometrial thickness.
C.The risk of multiple gestation is 25%.
D.Clomiphene citrate improves the fecundity rate principally through its effect on the endometrial lining.
E.Risk and side effects of clomiphene citrate include nausea, hot flushes,weight gain, and mood swings.
E.Risk and side effects of clomiphene citrate include nausea, hot flushes,weight gain, and mood swings.
4 8 2008 25. A patient with hypogonadotropic hypogonadism desires ovulation. What is the initial treatment of choice? (2.0分) A.low-dose estrogen therapy
B.hMG therapy
C.bromocriptine mesylate
D.cyclic progesterone
E.clomiphene citrate
B.hMG therapy
4 8 2008 26. A 31-year-old patient is preparing to start in vitro fertilization (IVF) because of obstructed fallopian tubes. On hysterosalpingogram (HSG), it is noted that she has large dilated hydrosalpinges present bilaterally. What should be the next step? (2.0分) A.The patient should begin her IVF treatment cycle.
B.The patient should repeat the HSG to confirm the result.
C.The patient should not be offered the opportunity to have IVF.
D.Bilateral tubal ligation should be done prior to starting IVF.
E.Her hydrosalpinges should be drained via transvaginal aspiration prior to starting IVF.
D.Bilateral tubal ligation should be done prior to starting IVF.
4 8 2008 27. Infertility is a common symptom of a luteal-phase defect. What is another common symptom? (2.0分) A.vaginal dryness
B.spontaneous miscarriage
C.tubal occlusion
D.breast tenderness
E.ovarian enlargement
B.spontaneous miscarriage
4 8 2008 28. Which of the following semen parameters would not be evaluated in a semen analysis?(2.0分) A.sperm motility
B.sperm concentration
C.sperm morphology
D.sperm penetration
E.eminal fluid viscosity
D.sperm penetration
4 8 2008 29. A patient has a profuse, thin, acellular cervical mucus with a high degree of stretchability and a palm-leaf crystallization pattern upon drying. Which of the following situations is compatible with this finding? (2.0分) A.the secretory phase of the menstrual cycle
B.anovulation
C.on combination birth control pills
D.being postmenopausal
E.second trimester of pregnancy
B.anovulation
4 8 2008 30. A28-year-old woman is seen for her first obstetrical visit. Her last menstrual period (LMP) was 8 weeks ago. Her history is significant for infertility due to chronic salpingitis and she required in vitro fertilization (IVF) with multiple embryo transfer. A serum pregnancy test is positive. A transabdominal ultrasound shows an enlarged uterus containing five viable fetuses. You advise her that the optimal outcome can be achieved only with which of the following? (2.0分) A.close supervision
B.embryo reduction
C.intramuscular prostaglandin
D.progestin therapy
E.termination of the pregnancy
B.embryo reduction
4 8 2008 31. A 27-year-old woman complains of a fishy odor and a vaginal discharge.Speculum examination reveals an erythematous vagina and punctations of the cervix.Which of the following is the most likely diagnosis?(2.0分) A.Candidal vaginitis
B.Trichomonal vaginitis
C.Bacterial vaginosis
D.Human papilloma virus
E.Herpes simple virus
B.Trichomonal vaginitis
4 8 2008 32. A 45-year-old Laotian woman is visiting her daughter. She comes to your office complaining of frequent intermenstrual bleeding for years.You examine her and feel that her pelvis is "firmly fixed," with little mobility of the organs. You perform an endometrial biopsy.The pathology report returns stating that "frequent giant cells, caseous necrosis, and granuloma formation" are seen. Which of the following is the most likely cause of this women's condition? (2.0分) A.syphilis
B.toxoplasmosis
C.tuberculosis
D.Neisseria gonorrhea
E.L. monocytogenes
C.tuberculosis
4 8 2008 33. A 26-year-old women taking antibiotics for cystitis complains of itching ,burning ,and a yellowish vaginal discharge. Which of the following is the best therapy ? (2.0分) A.Metronidazole
B.Erythromycin
C.Fluconazole
D.Hydrocortisone
E.Clindamycin
C.Fluconazole
4 8 2008 34. A 17-year-old G0 sexually active woman presents to the emergency room with pelvic pain that began within the last day. She reports menarche at the age of 15 and coitarche soon thereafter. She has had 4 male partners, including her new boyfriend of a few weeks. Her blood pressure is 100/60, pulse 100, and T 40℃. On speculum examination, you note a foul-smelling mucopurulent discharge from her cervical os and she has significant tenderness with manipulation of her uterus. What is the next best step in the management of this patient?(2.0分) A.Outpatient treatment with oral broad spectrum antibiotics
B.Outpatient treatment with intramuscular and oral broad spectrum antibiotics
C.Inpatient treatment, intravenous antibiotics, and dilatation and curettage
D.Inpatient treatment, laparoscopy with pelvic lavage
E.Inpatient treatment and intravenous antibiotics
E.Inpatient treatment and intravenous antibiotics
4 8 2008 35. A 16-year-old G0 woman presents to the emergency department with a two day history of "belly pain." She is sexually active with a new partner and is not using any form of contraception. Temperature is 39°C. On exam, she has lower abdominal tenderness and guarding. On pelvic exam, she has diffuse tenderness over the uterus and bilaterally. Beta hCG <5. What is the most likely diagnosis in this patient? (2.0分) A.Ectopic pregnancy
B.Appendicitis
C.Acute Cystitis
D.Intrauterine pregnancy
E.Acute salpingitis
E.Acute salpingitis
4 8 2008 36. An 18-year-old woman undergoes laparoscopy for acute abdomen Erythematous fallopian tubes are noted.Cultures of the purulent drainage most likely would reveal which of the following?(2.0分) A.Multiple organisms
B.Neisseria gonorrhoeae
C.Chlamydia tratromatis
D.Peptostreptococcus species
E.Treponema pallidum
A.Multiple organisms
4 8 2008 37. Which of the following is a risk factor for developing PID?(2.0分) A.IUD use
B.Candida vaginitis
C.Oral contraceptive agents
D.Depot medroxyprogesterone acetate
E.use vaginal spermicides
A.IUD use
4 8 2008 38. A 30-year-old women is brought to the emergency department with fever and a blood pressure of 70/40mmHg.she is presumed to be in septic shock.which of the following is a fundamental principle for treatment?(2.0分) A.Intravenous normal saline
B.Plasmapheresis
C.Oral fluid resuscitation
D.Await blood culture results prior to initiation of antibiotic therapy
E.Blood transfusion
A.Intravenous normal saline
4 8 2008 39. Hirsutism, ovarian androgen excess, and elevated serum LH levels are associated with which of the following? (2.0分) A.11-beta-hydroxylase deficiency
B.Cushing's syndrome
C.adrenal tumor
D.PCOS
E.arrhenoblastoma
D.PCOS
4 8 2008 40. A 34-year-old woman comes to you for a chief complaint of hirsutism. She states that this has been present since menarche, but has gotten worse in the past two years. Her menses have become more irregular, now every 28-45 days apart. She states that she quit smoking and gained approximately thirty pounds in the past three years. Her mother is obese, diabetic and has hirsutism. The patient has acanthosis nigricans present on the back of her neck. There is no hair seen on her chin, but she shaves every few days. She also has a significant amount hair present on the back of her hand and on her forearms. TSH, prolactin, 17-hydroxyprogesterone and DHEAS are normal. Testosterone is mildly elevated. Which of the following is the most likely etiology of her hirsutism?(2.0分) A.Polycystic ovarian syndrome
B.Ovarian neoplasm
C.Diabetes
D.Cushing's syndrome
E.Adrenal tumor
A.Polycystic ovarian syndrome
4 8 2008 41. A 32-year-old woman comes to the office due to the inability to conceive for last two years. She reports having been on oral contraceptives for 8 years prior to stopping them a year ago to attempt pregnancy. She had menarche at age 14 and had irregular cycles about every 3 months apart until started birth control pills, which made her cycles regular. In the last year, she has had about 5 cycles in total; her last menstrual period was 6 weeks ago. She is otherwise in good health and has not had any surgeries. She has no history of abnormal Pap smears or sexually transmitted diseases. Her only medication is a multivitamin. Her husband of 4 years is 35-years-old, and has a 10-year-old son from a previous marriage. She is 5'4" tall and weighs 165 pounds. On general appearance, she seems to be hirsute on the face and the abdomen. The rest of her exam is otherwise normal. In addition to weight loss and starting Metformin, what is the most appropriate treatment for this patient's infertility problem? (2.0分) A.Laparoscopy and ovarian biopsy
B.Ovulation induction agents such as Clomiphene Citrate
C.Intrauterine insemination
D.In vitro fertilization with ICSI (Intracytoplasmic sperm injection)
E.In vitro fertilization without ICSI
B.Ovulation induction agents such as Clomiphene Citrate
4 8 2008 42. A 26-year-old woman presents with hirsutism and irregular menses. Her mother, who is diabetic, had similar complaints prior to menopause. On physical exam, this patient is noted to have terminal hair on her chin and a gray-brown velvety discoloration on the back of her neck. This lesion is acanthosis nigricans. What is the most likely diagnosis in this patient? (2.0分) A.Hyperinsulinemia
B. Ovarian androgen excess
C. Late onset congenital adrenal hyperplasia
D. Adrenal tumor
E. Polycystic ovarian syndrome
A.Hyperinsulinemia
4 8 2008 43. A 24-year-old woman complains of bothersome hirsutism and skipping periods.She does not have evidence of voice changes,hair loss,or clitoromegaly.Pelvic examination does not reveal adnexal masses.serum DHEA-S,testosterone,and 17-hydroxyprogesterone levels are normal.LH/FSH ratio is 2:1.Which of the following is the most likely diagnosis? (2.0分) A.Polycystic ovarian syndrome
B.Familial hirsutism
C.Ovarian tumor
D.Adrenal tumor
E. Cushing's syndrome
A.Polycystic ovarian syndrome
4 8 2008 44. A 26-year-old woman comes to the office due to irregular menses since menarche, worsening for the last 6 months. The patient has noted increasing hair growth on her chin and most recently hair growth on her chest, requiring that she shave periodically. No one in her family has hirsutism. On exam, you also notice acne on her chin, acanthosis nigricans and temporal balding. Her serum testosterone is elevated. You diagnose hyperthecosis. Which of the following might also be associated with this condition? (2.0分) A.Good response to oral contraceptive therapy
B.Insulin sensitivity
C.Atrophic changes of external genitalia
D.Deepening of the voice
E.Infertility easily treated with ovulation induction
D.Deepening of the voice
4 8 2008 45. what is the most common cause of pathologic amenorrhea? (2.0分) A.Outflow obstruction
B.disruption of the hypothalamic-pituitary axis
C.asherman syndrome
D.kallman syndrome
C.asherman syndrome
4 8 2008 46. A 22-year-old woman with amenorrhea of 6 weeks' duration undergoes surgery for acute appendicitis. At the time of surgery a 3-cm semisolid left ovarian cyst is discovered. It is vascular and appears to contain a blood-filled central cavity. A serum pregnancy test is positive. Of the following, what is the most appropriate next step in this patient's management? (2.0分) A.ovarian cystectomy
B.ovarian wedge resection
C.oophorectomy
D.salpingo-oophorectomy
E.no additional therapy indicated
E.no additional therapy indicated
4 8 2008 47. A 16-year-old girl has not experienced menarche. Examination shows absence of breast development and small but otherwise normal female pelvic organs. Which of the following diagnostic tests is most useful in determining the etiology of the amenorrhea? (2.0分) A.serum follicle-stimulating hormone (FSH)
B.serum estradiol
C.serum testosterone
D.magnetic resonance imaging (MRI) of the head
E.ovarian biopsy
A.serum follicle-stimulating hormone (FSH)
4 8 2008 48. A 28-year-old patient complains of amenorrhea after dilation and curettage (D&C) for postpartum bleeding. Of the following, which is the most likely diagnosis? (2.0分) A.gonadal dysgenesis
B.Sheehan's syndrome
C.Kallmann syndrome
D.Mayer-Rokitansky-Kiister-Hauser syndrome
E.Asherman's syndrome
E.Asherman's syndrome
4 8 2008 49. Luteectomy before 42 days' gestation is most likely to result in which of the following? (2.0分) A.prolonged (postdates) gestation
B.spontaneous miscarriage
C.reduction of basal body temperature (BBT)
D.masculinization of a female fetus
E.no demonstrable effect
B.spontaneous miscarriage
4 8 2008 50. 最常见的功血为 (2.0分) A.黄体功能不足
B.子宫内膜脱落不全
C.排卵期出血
D.排卵型月经过多
E.无排卵性功血
E.无排卵性功血
4 8 2009 1. 关注妇女保健的最重要意义? (2.0分) A.妇女从生理上体弱多病
B.妇女健康关系到子代健康;;妇女是脆弱人群(社会地位、经济、教育、就业、医疗);妇女在生理解剖结构上与男性不同,在青春期、围婚期、孕期、分娩期、产褥期、哺乳期、更年期有特殊的生理和病理问题。
C.妇女人口多
D.妇女社会地位低,大多数没有医疗保障
B.妇女健康关系到子代健康;;妇女是脆弱人群(社会地位、经济、教育、就业、医疗);妇女在生理解剖结构上与男性不同,在青春期、围婚期、孕期、分娩期、产褥期、哺乳期、更年期有特殊的生理和病理问题。
4 8 2009 2. 目前我国开展婚检的主要目的是:(2.0分) A.检查准备结婚的双方是否存在传染病。
B.检查准备结婚的双方是否患有严重的精神性疾病。
C.检查准备结婚的双方是否患有严重遗传性疾病
D.检查准备结婚的双方是否患有影响生育的严重疾病
E.以上都是
E.以上都是
4 8 2009 3. 我国的孕产妇系统保健内容包括哪些内容 (2.0分) A.及早诊断妊娠
B.孕12周内进行早孕检查
C.孕期至少5次的产前检查
D.住院分娩,产后访视3次
E.以上都是
E.以上都是
4 8 2009 4. 女孩青春期出现的最早征兆是? (2.0分) A.月经来潮
B.乳房出现硬结
C.出现阴毛和腋毛
D.身高突增
B.乳房出现硬结
4 8 2009 5. 孕产妇死亡率统计指标中哪项说法是不正确的? (2.0分) A.孕产妇死亡率的指标是反应该国家或地区经济医疗水平的指标。
B.孕产妇死亡指标计算通常以10万分娩量作为基数。
C.孕产妇死亡率指标计算:分子是当地某一时段的孕产妇死亡人数,分母是当地同一时段所有的孕产妇数。
D.孕产妇死亡率较高的地区死亡产科因素的死亡构成比较高,死亡率低的地区以非产科因素构成比较高。
C.孕产妇死亡率指标计算:分子是当地某一时段的孕产妇死亡人数,分母是当地同一时段所有的孕产妇数。
4 8 2009 6. 流行病学的研究范围是(2.0分) A.传染病
B. 疾病和健康状况
C.传染病和地方病
D.传染病和非传染病
B. 疾病和健康状况
4 8 2009 7. 发病率指标来自(2.0分) A.对住院病人的调查
B.对门诊病人的调查
C.对社区人群的调查
D.对所有病人的调查
C.对社区人群的调查
4 8 2009 8. 在抽样调查中,下列哪种抽样方法的抽样误差最大? (2.0分) A.单纯随机抽样
B.系统抽样
C.分层抽样
D.整群抽样
D.整群抽样
4 8 2009 9. 在病例对照研究中,匹配是指(2.0分) A.在安排病例和对照组时,使两者的某些特征或变量相一致的方法
B.在安排病例和对照时,使两者的研究因素相一致的方法
C.在安排病例和对照时,使两者的所有特征或变量相一致的方法
D.病例组的研究因素的数量与对照组完全一致
A.在安排病例和对照组时,使两者的某些特征或变量相一致的方法
4 8 2009 10. 队列研究的最大优点是(2.0分) A.对较多的人进行较长时间的随访
B.发生偏倚的机会少
C.较直接地验证因素与疾病的因果关系
D.研究的结果常能代表全人群
C.较直接地验证因素与疾病的因果关系
4 8 2009 11. 绒毛膜癌与侵蚀性葡萄胎的区别是 (2.0分) A.间质的含量
B.有无血管
C.肿瘤细胞的异型性
D.有无绒毛
E.有无转移
D.有无绒毛
4 8 2009 12. 青年女性,闭经3个月,阴道不规律出血,血块中夹有水泡。检查发现子宫体积大,阴道壁有暗紫色结节、出血、坏死。最大可能是(2.0分) A.宫外孕
B.葡萄状肉瘤
C.葡萄胎
D.恶性葡萄胎
E.绒毛膜癌
D.恶性葡萄胎
4 8 2009 13. 宫颈早期浸润癌是指(2.0分) A.癌浸润深度不超过基底膜下5mm
B.癌浸润深度超过基底膜5mm
C.癌浸润深度不超过基底膜下3cm
D.癌浸润深度超过基底膜1mm
E.癌浸润深度在基底膜下0。5~1cm
A.癌浸润深度不超过基底膜下5mm
4 8 2009 14. 诊断早期宫颈癌最可靠的依据是(2.0分) A.有接触性出血史
B.阴道镜检查
C.盆腔检查
D.宫颈细胞学检查
E.宫颈病理切片检查
E.宫颈病理切片检查
4 8 2009 15. 排卵后,由于孕激素对体温中枢的作用,基础体温可持续上升(2.0分) A.16~18天
B.12~14天
C.10~12天
D.8~10天
E.6~8天
B.12~14天
4 8 2009 16. 与精子产生无关的因素是(2.0分) A.双侧隐睾引起曲细精管萎缩
B.慢性中毒(吸烟、酗酒)
C.过度精神紧张
D.幼年腮腺炎并发睾丸炎
E.输精管结核
E.输精管结核
4 8 2009 17. 下列哪项不属于辅助生育技术(2.0分) A.LEEP
B.AID
C.AIH
D.IVF-ET
E.GIFT
A.LEEP
4 8 2009 18. 女性,28岁。孕1产0,性生活正常,同居3年未避孕未孕,月经5~6天/28~30天,妇科检查:宫颈光滑,宫体大小正常,宫旁左侧及后方有粘连及压痛,右侧附件可及,进一步处理首选(2.0分) A.人工周期
B.全身抗炎治疗
C.氯底酚胺
D.输卵管通液
E.子宫输卵管造影
B.全身抗炎治疗
4 8 2009 19. 女性,32岁,孕0产0,婚后不孕3年,月经3~5/28~30天,妇科检查未发现特殊症状。进一步检查首选(2.0分) A.丈夫精液常规检查
B.输卵管通液
C.子宫输卵管造影
D.宫腔镜
E.腹腔镜
A.丈夫精液常规检查
4 8 2009 20. A 44-year-old woman (gravida 5, para 5) comes in complaining that she has noticed a bulge protruding out of her vagina. Her other medical problems include hypertension treated with medication, diabetes mellitus, and alcoholism. She stands at work as a grocery clerk. She has a family history of genital prolapse. On examination,you notice a uterine prolapse, cystocele,and rectocele. Which of the following is her major risk factor for her pelvic support disorder? (2.0分) A.childbirth
B.hypertension
C.diabetes mellitus
D.positive family history
E.environmental factors-job
A.childbirth
4 8 2009 21. A 56-year-old woman complains that she is "sitting on a ball." She says constipation is a significant problem for her and that sometimes she needs to push stool out of her rectum by inserting a finger in the vagina and pressing on a bulge. On further examination, which of the following will you most likely find? (2.0分) A.cystocele
B.rectocele
C.enterocele
D.complete uterine prolapse
E.hemorrhoid
B.rectocele
4 8 2009 22. Kegel exercises were designed to do which of the following? (2.0分) A.strengthen the abdominal muscles after childbirth
B.increase the blood flow to the perineum to speed the healing of an episiotomy
C. improve the tone of the muscles surrounding the bladder base and proximal bladder neck
D.prevent denervation of pelvic muscles after childbirth
E.decrease the muscle atrophy associated with aging
C. improve the tone of the muscles surrounding the bladder base and proximal bladder neck
4 8 2009 23. A 52-year-old postmenopausal woman complains of urinary frequency, urgency, and urge incontinence. She is otherwise healthy. Which of the following is included in the behavioral treatment you recommend? (2.0分) A.relaxation techniques
B.anticholinergic medication
C.voiding every hour during the daytime
D.bladder retraining
E.incontinence pad testing
D.bladder retraining
4 8 2009 24. Gynecologic fistulas may be commonly related to which of the following? (2.0分) A.urologic surgery
B.total abdominal hysterectomy
C.general surgery
D.colorectalsurgry
E.vascularsurgry
B.total abdominal hysterectomy
4 8 2009 25. 一位80岁的妇女,孕5产4,主诉阴道口肿物突出20年,有二次急产史,产后未得到良好的休息。有高血压史20年,正规服用降压药,合并糖尿病,抽烟每天10支。近一月感便秘。检查发现子宫颈位于处女膜外5cm。该妇女子宫脱垂的主要原因是什么? (2.0分) A.分娩对盆底的损伤
B.高血压病
C.糖尿病
D.绝经后的雌激素下降
E.便秘
A.分娩对盆底的损伤
4 8 2009 26. 一位76岁的妇女,孕3产3,第一胎为难产,绝经30年,有阴道口肿物15年,近二天不能排尿和排便,检查发现整个子宫、阴道前壁和阴道后壁完全掉出在阴道口外。你该诊断何种疾病? (2.0分) A.一度子宫脱垂
B.二度子宫脱垂
C.三度子宫脱垂
D.四度盆腔器官脱垂
E.子宫内翻
D.四度盆腔器官脱垂
4 8 2009 27. 哪位妇女最有可能已经绝经? (2.0分) A.35岁的妇女,体重75kg,停经3个月
B.45岁妇女,最近服用减肥药,体重从原来的70kg减至50kg,2个月未来月经
C.50岁妇女,没来月经13个月
D.55岁妇女,停经11个月
E.21岁女性,从未来过月经
C.50岁妇女,没来月经13个月
4 8 2009 28. 下列哪一项不属于绝经的危害?(2.0分) A.潮热、潮红、睡眠障碍、抑郁
B.直肠癌
C.生殖道萎缩
D.骨质疏松
E.冠心病发生率增加
B.直肠癌
4 8 2009 29. 雌激素类药物的药理作用中,不包括 (2.0分) A.促进女性性器官的发育和成熟,维持女性第二性征。
B.能明显促进蛋白质合成,减少蛋白质分解
C.使阴道上皮增生,浅表层细胞角化
D.增强子宫平滑肌对缩宫素的敏感性
E.较大剂量时可抑制下丘脑-垂体系统释放GnRH
B.能明显促进蛋白质合成,减少蛋白质分解
4 8 2009 30. 下列哪个药物适用于催产和引产 (2.0分) A.麦角毒
B.缩宫素
C.垂体后叶素
D.麦角新碱
E.麦角胺
B.缩宫素
4 8 2009 31. 对缩宫素的叙述哪一项不正确?(2.0分) A.小剂量可加强子宫的节律性收缩
B.其收缩性质与正常分娩相似
C.大剂量引起子宫强直性收缩
D.大剂量适用于催产
E.小剂量适用于催产和引产
D.大剂量适用于催产
4 8 2009 32. A 25-year-old sexually active woman complainsof a "fishy" smelling gray-white vaginaldischarge. You examine this on wet mount andsee epithelial cells with clusters of bacteriaobscuring their borders. The vaginal pH is 5.5.This infection has been most closely implicatedin which of the following complications ofpregnancy? (2.0分) A.intrauterine growth restriction
B.preterm birth.
C.congenital cataracts
D.placenta previa
E.preeclampsia
B.preterm birth.
4 8 2009 33. A patient is found to have bilaterally equal adnexal pain; cervical motion tenderness; direct abdominal tenderness; temperature, 39℃; and WBC, 12,000/mL. Which of the following is the most likely diagnosis? (2.0分) A.ectopic pregnancy
B.pelvic inflammatory disease (PID)
C.ruptured corpus luteum cyst of the ovary
D.endometriosis
E.endometritis
B.pelvic inflammatory disease (PID)
4 8 2009 34. A 32-year-old G3P1 woman presents to your office for an annual examination. During the course of your discussion, the patient mentions that she has had an annoying vulvar irritation and slight itch for months. She sometimes notices an odor, but does not experience a specific discharge. She is sexually active with the same male partner for the past 6 months and they occasionally use condoms. On examination, she has a grayish, frothy discharge within the vaginal vault. You perform a wet prep of her vaginal secretions which shows flagellated organisms. Which of the following is the most likely diagnosis in this patient?(2.0分) A.Yeast, Non-Candida albicans
B.Trichomonas
C.Bacterial vaginosis
D.Recent intercourse
E.Chlamydia
B.Trichomonas
4 8 2009 35. A 36-year-old G0 woman presents to the emergency department accompanied by her female partner. The patient is febrile and notes severe belly pain. She states that this pain began 2-3 days ago and was associated with diarrhea as well as some nausea. It has gotten progressively worse and she has now developed a fever. Neither her partner, nor other close contacts report any type of viral illness. She had her appendix removed as a teenager. On examination, her temperature is 38.5°C, her abdomen is tender with mild guarding and rebound, and she has an elevated white count. On pelvic examination, she is exquisitely tender, such that you cannot complete your exam. Pelvic ultrasound demonstrates bilateral 3-4 cm complex masses. What is the most likely underlying pathogenesis of her illness?(2.0分) A.Diverticulitis
B.A sexually transmitted infection
C.Reactivation of an old infection
D.Ascending infection
E.Pyelonephritis
D.Ascending infection
4 8 2009 36. A 32-year-old G0 woman comes to your office because she has been unable to conceive for one year. She has regular menses every 27-34 days. She is currently in a mutually monogamous relationship, reports menarche at the age of 13, with coitarche at the age of 15; she has had 7 sexual partners, including her husband of 3 years. The patient reports that she has had an abnormal Pap smear about 4 years ago, but things have been normal since. She also states that, in her early twenties, she had problems with "female infections" and was on "lots of antibiotics," but was never hospitalized. She denies any pain with intercourse and she denies any intermenstrual bleeding. Her husband has fathered 2 children from a previous marriage. What is the most likely underlying cause of infertility in this patient?(2.0分) A.Male factor infertility
B.Cervical stenosis
C.Ovulatory dysfunction
D.Tubal disease
E.Endometriosis
D.Tubal disease
4 8 2009 37. Which of the following is the most accurate method for diagnosing acute salpingitis?(2.0分) A.Clinical criteria
B.Sonography
C.Computed tomographic scan
D.Laparoscopy
E.Blood culture
D.Laparoscopy
4 8 2009 38. Which of the following infections is most likely to be associated with tubal disease?(2.0分) A.Trichomonasvaginalis
B.Bacteroides species
C.Chlamydia trachomatis
D.Proteus species
E.Treponemapallidum
C.Chlamydia trachomatis
4 8 2009 39. A 22-year-old woman is seen for vaginal discharge.The presumptive diagnosis is bacterial vainosis.Which of the following most likely is present?(2.0分) A.pH<4.5
B.Heterogenous vaginal discharge
C.Predominance of anaerobes
D.Positive Kleihauer-Betke test
E.Negative clue cell
C.Predominance of anaerobes
4 8 2009 40. A 21-year-old diabetic athletic woman on a lowdoseoral contraceptive comes to your clinic withirregular menses and galactorrhea. On examination,galactorrhea is confirmed, with fat globulesseen microscopically. She currently takesmetoclopramide (Reglan) for delayed gastricemptying. A random serum prolactin level is65 ng/mL. Which of the following is most likelyresponsible for her hyperprolactinemia?(2.0分) A.metoclopramide
B.pregnancy
C.oral contraceptive
D.pituitary adenoma
E.exercis
A.metoclopramide
4 8 2009 41. A 25-year-old woman suffers a severe intrapartumhemorrhage. Which of the followingsymptoms is evidence of pituitary infarction?(2.0分) A.infrequent urination
B.diarrhea
C.easybruisability
D.lactation failure
E.perspiration
D.lactation failure
4 8 2009 42. An 18-year-old patient has not experiencedmenarche. Examination shows normal breastdevelopment and absence of a uterus. Which ofthe following diagnostic tests is most useful indetermining the etiology of the amenorrhea?(2.0分) A.serum FSH
B.serum estradiol
C.serum testosterone
D.MRI of the head
E.ovarian biopsy
C.serum testosterone
4 8 2009 43. A 25-year-old healthy woman complains ofbreast tenderness and amenorrhea of 6 weeks'duration. She uses condoms for birth controland does not take any medication. Examinationdemonstrates a whitish breast discharge withmilk-containing fat droplets on microscopicexamination. A pregnancy test is negative, anda serum TSH level is normal. A serum prolactinis 80 ng/mL (normal, <20 ng/mL). Of the following,the next step in the management of thispatient should be to obtain radiologic assessmentof which structure(s)? (2.0分) A.kidneys
B.lumbar spine
C.sella turcica
D.chest
E.pelvic organs
C.sella turcica
4 8 2009 44. A 19-year-old woman and her boyfriend wishto use condoms as a barrier contraceptivemethod. This couple should be advised thatthe most common reason for failure (pregnancyor STD transmission) is which of the following? (2.0分) A.breakage
B.inconsistent use
C.leakage caused by spermicidal creams
D.use without concombinate use of aspermicide
E.spill of condom contents uponwithdrawal
B.inconsistent use
4 8 2009 45. A 23-year-old patient is considering contraceptivemethods but is devoutly religious and willnot accept a method that may "cause an abortion."The primary mechanism by which IUD prevent pregnancy is which of the following? (2.0分) A.creating chronic endometritis
B.preventing fertilization
C.inhibiting ovulation
D.altering tuba1 motility
E.destroying sperm
B.preventing fertilization
4 8 2009 46. Reducing the estrogen content of OCs results inandecreasing in the rate of which of the following?)(2.0分) A.pregnancy
B.breakthrough bleeding (BTB)
C.thromboembolic complications
D.insulin resistance
E.premenstrual symptoms
C.thromboembolic complications
4 8 2009 47. The following methods are NOT hormonal contraception except: (2.0分) A.condom
B.diaphragm
C.spermicidal
D.nature family planning
E.emergency contraception with progestin
E.emergency contraception with progestin
4 8 2009 48. A 17-year-old woman with a history of ectopic pregnancy presents for contraceptive contraception. Which of the following contraceptive methods would be relatively or absolutelycontraindicated? (2.0分) A.OCs
B.progestin-only pill (the minipill)
C.male condoms
D.cervical cap
E.IUD
E.IUD
4 8 2009 49. 最常见的功血为 (2.0分) A.黄体功能不足
B.子宫内膜脱落不全
C.排卵期出血
D.排卵型月经过多
E.无排卵性功血
E.无排卵性功血
4 8 2009 50. 患者女,26岁。停经60天后阴道出血11天。检查:子宫正常大小,质软,宫颈黏液见典型羊齿叶状结晶。应考虑为 (2.0分) A.先兆流产
B.卵巢性闭经
C.异位妊娠
D.无排卵性功血
E.黄体萎缩不全所致的出血
D.无排卵性功血
4 8 2010 1. 绒毛膜癌与侵蚀性葡萄胎的区别是 (2.0分) A.间质的含量
B.有无血管
C.肿瘤细胞的异型性
D.有无绒毛
E.有无转移
D.有无绒毛
4 8 2010 2. 青年女性,闭经3个月,阴道不规律出血,血块中夹有水泡。检查发现子宫体积大,阴道壁有暗紫色结节、出血、坏死。 最大可能是(2.0分) A.宫外孕
B.葡萄状肉瘤
C.葡萄胎
D.恶性葡萄胎
E.绒毛膜癌
D.恶性葡萄胎
4 8 2010 3. 宫颈早期浸润癌是指 (2.0分) A.癌浸润深度不超过基底膜下5mm
B.癌浸润深度超过基底膜5mm
C.癌浸润深度不超过基底膜下3cm
D.癌浸润深度超过基底膜1mm
E.癌浸润深度在基底膜下0.5~1cm
A.癌浸润深度不超过基底膜下5mm
4 8 2010 4. 诊断早期宫颈癌最可靠的依据是(2.0分) A.有接触性出血史
B.阴道镜检查
C.盆腔检查
D.宫颈细胞学检查
E.宫颈病理切片检查
E.宫颈病理切片检查
4 8 2010 5. A 44-year-old woman (gravida 5, para 5) comes in complaining that she has noticed a bulge protruding out of her vagina. Her other medical problems include hypertension treated with medication, diabetes mellitus, and alcoholism. She stands at work as a grocery clerk. She has a family history of genital prolapse. On examination,you notice a uterine prolapse, cystocele,and rectocele. Which of the following is her major risk factor for her pelvic support disorder? (2.0分) A.childbirth
B.hypertension
C.diabetes mellitus
D.positive family history
E.environmental factors-job
A.childbirth
4 8 2010 6. A 56-year-old woman complains that she is "sitting on a ball." She says constipation is a significant problem for her and that sometimes she needs to push stool out of her rectum by inserting a finger in the vagina and pressing on a bulge. On further examination, which of the following will you most likely find? (2.0分) A.cystocele
B.rectocele
C.enterocele
D.complete uterine prolapse
E.hemorrhoid
B.rectocele
4 8 2010 7. Kegel exercises were designed to do which of the following? (2.0分) A.strengthen the abdominal muscles after childbirth
B.increase the blood flow to the perineum to speed the healing of an episiotomy
C.improve the tone of the muscles surrounding the bladder base and proximal bladder neck
D.prevent denervation of pelvic muscles after childbirth
E.decrease the muscle atrophy associated with aging
C.improve the tone of the muscles surrounding the bladder base and proximal bladder neck
4 8 2010 8. A 52-year-old postmenopausal woman complains of urinary frequency, urgency, and urge incontinence. She is otherwise healthy. Which of the following is included in the behavioral treatment you recommend? (2.0分) A.relaxation techniques
B.anticholinergic medication
C.voiding every hour during the daytime
D.bladder retraining
E.incontinence pad testing
D.bladder retraining
4 8 2010 9. Gynecologic fistulas may be commonly related to which of the following? (2.0分) A.urologic surgery
B.total abdominal hysterectomy
C.general surgery
D.colorectal surgry
E.vascular surgry
B.total abdominal hysterectomy
4 8 2010 10. 一位80岁的妇女,孕5产4,主诉阴道口肿物突出20年,有二次急产史,产后未得到良好的休息。有高血压史20年,正规服用降压药,合并糖尿病,抽烟每天10支。近一月感便秘。检查发现子宫颈位于处女膜外5cm。该妇女子宫脱垂的主要原因是什么? (2.0分) A.分娩对盆底的损伤
B.高血压病
C.糖尿病
D.绝经后的雌激素下降
E.便秘
A.分娩对盆底的损伤
4 8 2010 11. 一位76岁的妇女,孕3产3,第一胎为难产,绝经30年,有阴道口肿物15年,近二天不能排尿和排便,检查发现整个子宫、阴道前壁和阴道后壁完全掉出在阴道口外。你该诊断何种疾病? (2.0分) A.一度子宫脱垂
B.二度子宫脱垂
C.三度子宫脱垂
D.四度盆腔器官脱垂
E.子宫内翻
D.四度盆腔器官脱垂
4 8 2010 12. 哪位妇女最有可能已经绝经? (2.0分) A.35岁的妇女,体重75kg,停经3个月
B.45岁妇女,最近服用减肥药,体重从原来的70kg减至50kg,2个月未来月经
C.50岁妇女,没来月经13个月
D.55岁妇女,停经11个月
E.21岁女性,从未来过月经
C.50岁妇女,没来月经13个月
4 8 2010 13. 下列哪一项不属于绝经的危害?(2.0分) A.潮热、潮红、睡眠障碍、抑郁
B.直肠癌
C.生殖道萎缩
D.骨质疏松
E.冠心病发生率增加
B.直肠癌
4 8 2010 14. 最常见的功血为(2.0分) A.黄体功能不足
B.子宫内膜脱落不全
C.排卵期出血
D.排卵型月经过多
E.无排卵性功血
E.无排卵性功血
4 8 2010 15. 患者女,18岁。平素月经不规则,本次停经3月后阴道大量出血11天。B超检查:子宫偏小,内膜厚4mm,双侧卵巢多囊样改变。应考虑为 (2.0分) A.先兆流产
B.卵巢性闭经
C.子宫内膜息肉
D.无排卵性功血
E.黄体萎缩不全所致的出血
D.无排卵性功血
4 8 2010 16. 下列哪项用孕激素治疗有效:(2.0分) A.结核性子宫内膜炎
B.子宫内膜萎缩
C.子宫内膜对雌激素不起反应者
D.子宫内膜已受雌激素影响者
E.垂体性闭经
D.子宫内膜已受雌激素影响者
4 8 2010 17. 题干同第6题,该病人的下步治疗应为(2.0分) A.月经前半期给雌激素
B.月经后第5天给克罗米芬
C.下次月经前8-10天给孕激素
D.雌孕激素序贯
E.雌孕激素联合
C.下次月经前8-10天给孕激素
4 8 2010 18. 题干同第8题,对此患者,下步处置应为:(2.0分) A.止血芳酸
B.大量雌激素
C.大量雄激素
D.诊刮
E.大量孕激素
D.诊刮
4 8 2010 19. 雌激素类药物的药理作用中,不包括(2.0分) A.促进女性性器官的发育和成熟,维持女性第二性征。
B.能明显促进蛋白质合成,减少蛋白质分解
C.使阴道上皮增生,浅表层细胞角化
D.增强子宫平滑肌对缩宫素的敏感性
E.较大剂量时可抑制下丘脑-垂体系统释放GnRH
B.能明显促进蛋白质合成,减少蛋白质分解
4 8 2010 20. 下列哪个药物适用于催产和引产 (2.0分) A.麦角毒
B.缩宫素
C.垂体后叶素
D.麦角新碱
E.麦角胺
B.缩宫素
4 8 2010 21. 对缩宫素的叙述哪一项不正确?(2.0分) A.小剂量可加强子宫的节律性收缩
B.其收缩性质与正常分娩相似
C.大剂量引起子宫强直性收缩
D.大剂量适用于催产
E.小剂量适用于催产和引产
D.大剂量适用于催产
4 8 2010 22. A 27-year-old woman complains of a fishy odor and a vaginal discharge.Speculum examination reveals an erythematous vagina and punctations of the cervix.Which of the following is the most likely diagnosis?(2.0分) A.Candidal vaginitis
B.Trichomonal vaginitis
C.Bacterial vaginosis
D.Human papilloma virus
E.Herpes simple virus
B.Trichomonal vaginitis
4 8 2010 23. A 45-year-old Laotian woman is visiting her daughter. She comes to your office complaining of frequent intermenstrual bleeding for years.You examine her and feel that her pelvis is "firmly fixed," with little mobility of the organs. You perform an endometrial biopsy.The pathology report returns stating that "frequent giant cells, caseous necrosis, and granuloma formation" are seen. Which of the following is the most likely cause of this women's condition? (2.0分) A.syphilis
B.toxoplasmosis
C.tuberculosis
D.Neisseria gonorrhea
E.L. monocytogenes
C.tuberculosis
4 8 2010 24. A 26-year-old women taking antibiotics for cystitis complains of itching ,burning ,and a yellowish vaginal discharge. Which of the following is the best therapy ? (2.0分) A.Metronidazole
B.Erythromycin
C.Fluconazole
D.Hydrocortisone
E.Clindamycin
C.Fluconazole
4 8 2010 25. A 17-year-old G0 sexually active woman presents to the emergency room with pelvic pain that began within the last day. She reports menarche at the age of 15 and coitarche soon thereafter. She has had 4 male partners, including her new boyfriend of a few weeks. Her blood pressure is 100/60, pulse 100, and T 40℃. On speculum examination, you note a foul-smelling mucopurulent discharge from her cervical os and she has significant tenderness with manipulation of her uterus. What is the next best step in the management of this patient? (2.0分) A.Outpatient treatment with oral broad spectrum antibiotics
B.Outpatient treatment with intramuscular and oral broad spectrum antibiotics
C.Inpatient treatment, intravenous antibiotics, and dilatation and curettage
D. Inpatient treatment, laparoscopy with pelvic lavage
E.Inpatient treatment and intravenous antibiotics
E.Inpatient treatment and intravenous antibiotics
4 8 2010 26. A 16-year-old G0 woman presents to the emergency department with a two day history of "belly pain." She is sexually active with a new partner and is not using any form of contraception. Temperature is 39°C. On exam, she has lower abdominal tenderness and guarding. On pelvic exam, she has diffuse tenderness over the uterus and bilaterally. Beta hCG <5. What is the most likely diagnosis in this patient? (2.0分) A.Ectopic pregnancy
B. Appendicitis
C.Acute Cystitis
D.Intrauterine pregnancy
E. Acute salpingitis
E. Acute salpingitis
4 8 2010 27. An 18-year-old woman undergoes laparoscopy for acute abdomen Erythematous fallopian tubes are noted.Cultures of the purulent drainage most likely would reveal which of the following?(2.0分) A.Multiple organisms
B.Neisseria gonorrhoeae
C.Chlamydia tratromatis
D.Peptostreptococcus species
E.Treponema pallidum
A.Multiple organisms
4 8 2010 28. Which of the following is a risk factor for developing PID?(2.0分) A.IUD use
B.Candida vaginitis
C.Oral contraceptive agents
D.Depot medroxyprogesterone acetate
E.use vaginal spermicides
A.IUD use
4 8 2010 29. A 30-year-old women is brought to the emergency department with fever and a blood pressure of 70/40mmHg.she is presumed to be in septic shock.which of the following is a fundamental principle for treatment?(2.0分) A.Intravenous normal saline
B.Plasmapheresis
C.Oral fluid resuscitation
D.Await blood culture results prior to initiation of antibiotic therapy
E.Blood transfusion
A.Intravenous normal saline
4 8 2010 30. 关注妇女保健的最重要意义? (2.0分) A.妇女从生理上体弱多病
B.妇女健康关系到子代健康;妇女是脆弱人群(社会地位、经济、教育、就业、医疗);妇女在生理解剖结构上与男性不同,在青春期、围婚期、孕期、分娩期、产褥期、哺乳期、更年期有特殊的生理和病理问题。
C.妇女人口多
D.妇女社会地位低,大多数没有医疗保障
B.妇女健康关系到子代健康;妇女是脆弱人群(社会地位、经济、教育、就业、医疗);妇女在生理解剖结构上与男性不同,在青春期、围婚期、孕期、分娩期、产褥期、哺乳期、更年期有特殊的生理和病理问题。
4 8 2010 31. 目前我国开展婚检的主要目的是:(2.0分) A.检查准备结婚的双方是否存在传染病。
B.检查准备结婚的双方是否患有严重的精神性疾病。
C.检查准备结婚的双方是否患有严重遗传性疾病
D.检查准备结婚的双方是否患有影响生育的严重疾病
E.以上都是
E.以上都是
4 8 2010 32. 我国的孕产妇系统保健内容包括哪些内容 (2.0分) A.及早诊断妊娠
B.孕12周内进行早孕检查
C.孕期至少5次的产前检查
D.住院分娩,产后访视3次
E.以上都是
E.以上都是
4 8 2010 33. 女孩青春期出现的最早征兆是?(2.0分) A.月经来潮
B.乳房出现硬结
C.出现阴毛和腋毛
D.身高突增
B.乳房出现硬结
4 8 2010 34. 孕产妇死亡率统计指标中哪项说法是不正确的?(2.0分) A.孕产妇死亡率的指标是反应该国家或地区经济医疗水平的指标。
B.孕产妇死亡指标计算通常以10万分娩量作为基数。
C.孕产妇死亡率指标计算:分子是当地某一时段的孕产妇死亡人数,分母是当地同一时段所有的孕产妇数。
D.孕产妇死亡率较高的地区死亡产科因素的死亡构成比较高,死亡率低的地区以非产科因素构成比较高。
C.孕产妇死亡率指标计算:分子是当地某一时段的孕产妇死亡人数,分母是当地同一时段所有的孕产妇数。
4 8 2010 35. 关于流行病学,下列哪种说法正确 (2.0分) A.从个体的角度研究疾病和健康状况及其影响因素
B.只研究传染病的流行特征和防治措施
C.只研究慢性病的危险因素
D.研究人群中疾病和健康状况的分布及其影响因素
D.研究人群中疾病和健康状况的分布及其影响因素
4 8 2010 36. 患病率指标来自(2.0分) A.现况调查资料
B.门诊病例资料
C.住院病例资料
D.专科医院病例资料
A.现况调查资料
4 8 2010 37. 在设计病例对照研究时,对照的选择最好是: (2.0分) A.从医院的其他病人中选对照
B.从该地区未患该病的全人群中选对照
C.从不同人群中选择多组对照
D.从病人的亲属,同事中选对照
B.从该地区未患该病的全人群中选对照
4 8 2010 38. 在病例对照研究中,比值比(OR)的含义是: (2.0分) A.病例组的发病率与对照组的发病率之比
B.病例组的暴露比值与对照组的暴露比值的比
C.对照组的暴露比值除以病例组的暴露比值
D.病例组的暴露比值与对照组的暴露比值之差
B.病例组的暴露比值与对照组的暴露比值的比
4 8 2010 39. 一项乳房癌与高脂肪饮食关系的前瞻性队列研究中,发现高脂肪饮食妇女乳房癌发病率48.0/10万,非高脂肪饮食者为25.4/10万,其相对危险度为: (2.0分) A.1.89
B.22.6/10万
C.48
D.0.0048
A.1.89
4 8 2010 40. A 19-year-old woman and her boyfriend wish to use condoms as a barrier contraceptive method. This couple should be advised that the most common reason for failure (pregnancy or STD transmission) is which of the following? (2.0分) A.breakage
B.inconsistent use
C.leakage caused by spermicidal creams
D.use without concombinate use of a spermicide
E.spill of condom contents upon withdrawal
B.inconsistent use
4 8 2010 41. A 23-year-old patient is considering contraceptive methods but is devoutly religious and will not accept a method that may "cause an abortion." The primary mechanism by which IUD prevent pregnancy is which of the following? (2.0分) A.creating chronic endometritis
B.preventing fertilization
C.inhibiting ovulation
D.altering tuba1 motility
E.destroying sperm
B.preventing fertilization
4 8 2010 42. Reducing the estrogen content of OCs results in an decreasing in the rate of which of the following? (2.0分) A.pregnancy
B.breakthrough bleeding (BTB)
C.thromboembolic complications
D.insulin resistance
E.premenstrual symptoms
C.thromboembolic complications
4 8 2010 43. The following methods are NOT hormonal contraception except: (2.0分) A.condom
B.diaphragm
C.spermicidal
D.nature family planning
E.emergency contraception with progestin
E.emergency contraception with progestin
4 8 2010 44. A 17-year-old woman with a history of ectopic pregnancy presents for contraceptive contraception. Which of the following contraceptive methods would be relatively or absolutely contraindicated? (2.0分) A.OCs
B.progestin-only pill (the minipill)
C.male condoms
D.cervical cap
E.IUD
E.IUD
4 8 2010 45. 据统计正常情况下,婚后2年初孕率为(2.0分) A.0.6
B.0.7
C.0.8
D.0.9
E.0.95
E.0.95
4 8 2010 46. 女性不孕最常见的因素是(2.0分) A.不排卵
B.输卵管因素
C.子宫因素
D.宫颈因素
E.阴道因素
B.输卵管因素
4 8 2010 47. 关于精液正常值下述哪项是错误的(2.0分) A.精液量为1.5ml~6ml,平均2~4ml
B.pH为7.2~7.5
C.精子数>4000万/ml
D.活动数>32%
E.异常精子数<85%
E.异常精子数<85%
4 8 2010 48. 以下哪种不是诱发排卵的药物是(2.0分) A.克罗米酚(CC)
B.三苯氧胺
C.绝经促性素(hMG)
D.黄体生成激素释放激素(LHRH)
E.溴隐亭
B.三苯氧胺
4 8 2010 49. 女性,30岁,原发不孕3年,月经5~6/20+~50+天,量中等,无痛经,妇科检查未发现特殊症状。进一步检查应首选(2.0分) A.输卵管通液
B.B型超声
C.性交后试验
D.X线腹部平片
E.基础内分泌测定和排卵监测
E.基础内分泌测定和排卵监测
4 8 2010 50. 女性,33岁。孕3产0,最后妊娠行人工流产术终止,至今已5年,未采取任何避孕措施,妇科检查:宫体正常大小,双侧附件区增厚,无明显压痛。此患者最可能的诊断是(2.0分) A.继发不孕+附件炎
B.绝对不孕+附件炎
C.继发不孕
D.原发不孕
E.原发不孕+附件炎
A.继发不孕+附件炎
4 8 2011 1. 关注妇女保健的最重要意义? (2.5分) A.妇女从生理上体弱多病
B.妇女健康关系到子代健康;;妇女是脆弱人群(社会地位、经济、教育、就业、医疗);妇女在生理解剖结构上与男性不同,在青春期、围婚期、孕期、分娩期、产褥期、哺乳期、更年期有特殊的生理和病理问题。
C.妇女人口多
D.妇女社会地位低,大多数没有医疗保障
B.妇女健康关系到子代健康;;妇女是脆弱人群(社会地位、经济、教育、就业、医疗);妇女在生理解剖结构上与男性不同,在青春期、围婚期、孕期、分娩期、产褥期、哺乳期、更年期有特殊的生理和病理问题。
4 8 2011 2. 目前我国开展婚检的主要目的是(2.5分) A.检查准备结婚的双方是否存在传染病。
B.检查准备结婚的双方是否患有严重的精神性疾病。
C.检查准备结婚的双方是否患有严重遗传性疾病
D.检查准备结婚的双方是否患有影响生育的严重疾病
E.以上都是
E.以上都是
4 8 2011 3. 我国的孕产妇系统保健内容包括哪些内容 (2.5分) A.及早诊断妊娠
B.孕12周内进行早孕检查
C.孕期至少5次的产前检查
D.住院分娩,产后访视3次
E.以上都是
E.以上都是
4 8 2011 4. 女孩青春期出现的最早征兆是?(2.5分) A.月经来潮
B.乳房出现硬结
C.出现阴毛和腋毛
D.身高突增
B.乳房出现硬结
4 8 2011 5. 孕产妇死亡率统计指标中哪项说法是不正确的?(2.5分) A.孕产妇死亡率的指标是反应该国家或地区经济医疗水平的指标。
B.孕产妇死亡指标计算通常以10万分娩量作为基数。
C.孕产妇死亡率指标计算:分子是当地某一时段的孕产妇死亡人数,分母是当地同一时段所有的孕产妇数。
D.孕产妇死亡率较高的地区死亡产科因素的死亡构成比较高,死亡率低的地区以非产科因素构成比较高。
C.孕产妇死亡率指标计算:分子是当地某一时段的孕产妇死亡人数,分母是当地同一时段所有的孕产妇数。
4 8 2011 6. 关于流行病学,下列哪种说法正确(2.5分) A.从个体的角度研究疾病和健康状况及其影响因素
B.只研究传染病的流行特征和防治措施
C.只研究慢性病的危险因素
D.研究人群中疾病和健康状况的分布及其影响因素
D.研究人群中疾病和健康状况的分布及其影响因素
4 8 2011 7. 患病率指标来自(2.5分) A.现况调查资料
B.门诊病例资料
C.住院病例资料
D.专科医院病例资料
A.现况调查资料
4 8 2011 8. 在设计病例对照研究时,对照的选择最好是: (2.5分) A.从医院的其他病人中选对照
B.从该地区未患该病的全人群中选对照
C.从不同人群中选择多组对照
D.从病人的亲属,同事中选对照
B.从该地区未患该病的全人群中选对照
4 8 2011 9. 在病例对照研究中,比值比(OR)的含义是(2.5分) A.病例组的发病率与对照组的发病率之比
B.病例组的暴露比值与对照组的暴露比值的比
C.对照组的暴露比值除以病例组的暴露比值
D.病例组的暴露比值与对照组的暴露比值之差
B.病例组的暴露比值与对照组的暴露比值的比
4 8 2011 10. 一项乳房癌与高脂肪饮食关系的前瞻性队列研究中,发现高脂肪饮食妇女乳房癌发病率48.0/10万,非高脂肪饮食者为25.4/10万,其相对危险度为(2.5分) A.1.89
B.22.6/10万
C.48
D.0.0048
A.1.89
4 8 2011 11. what is the most common cause of pathologic amenorrhea? (2.5分) A.Outflow obstruction
B.disruption of the hypothalamic-pituitary axis
C.asherman syndrome
D.kallman syndrome
C.asherman syndrome
4 8 2011 12. A 22-year-old woman with amenorrhea of 6 weeks' duration undergoes surgery for acute appendicitis. At the time of surgery a 3-cm semisolid left ovarian cyst is discovered. It is vascular and appears to contain a blood-filled central cavity. A serum pregnancy test is positive. Of the following, what is the most appropriate next step in this patient's management? (2.5分) A.ovarian cystectomy
B.ovarian wedge resection
C.oophorectomy
D.salpingo-oophorectomy
E.no additional therapy indicated
E.no additional therapy indicated
4 8 2011 13. A 16-year-old girl has not experienced menarche. Examination shows absence of breast development and small but otherwise normal female pelvic organs. Which of the following diagnostic tests is most useful in determining the etiology of the amenorrhea? (2.5分) A.serum follicle-stimulating hormone (FSH)
B.serum estradiol
C.serum testosterone
D.magnetic resonance imaging (MRI) of the head
E.ovarian biopsy
A.serum follicle-stimulating hormone (FSH)
4 8 2011 14. A 28-year-old patient complains of amenorrhea after dilation and curettage (D&C) for postpartum bleeding. Of the following, which is the most likely diagnosis? (2.5分) A.gonadal dysgenesis
B.Sheehan's syndrome
C.Kallmann syndrome
D.Mayer-Rokitansky-Kiister-Hauser syndrome
E.Asherman's syndrome
E.Asherman's syndrome
4 8 2011 15. Luteectomy before 42 days' gestation is most likely to result in which of the following? (2.5分) A.prolonged (postdates) gestation
B.spontaneous miscarriage
C.reduction of basal body temperature (BBT)
D.masculinization of a female fetus
E.no demonstrable effect
B.spontaneous miscarriage
4 8 2011 16. Hirsutism, ovarian androgen excess, and elevated serum LH levels are associated with which of the following? (D)(2.5分) A.11-beta-hydroxylase deficiency
B.Cushing's syndrome
C.adrenal tumor
D.PCOS
E.arrhenoblastoma
D.PCOS
4 8 2011 17. A 23-year-old woman with irregular menses complains of facial hair increasing in amount over several years. She is sexually active but does not wish to conceive. Examination demonstrates hirsutism, obesity, and hyperpigmentation of the neck and axillae. The ovaries are bilaterally enlarged and cystic. A serum testosterone value is 1.2 ng/mL (normal, <0.8 ng/mL). Serum levels of DHEAS, 17-hydroxyprogesterone (17-OHP), and prolactin are normal. Of the following, which is the best single therapeutic agent for this patient? (2.5分) A.oral contraceptives (OCs)
B.glucocorticoids
C.clomiphene citrate
D.antiandrogens
E.GnRH analogue
A.oral contraceptives (OCs)
4 8 2011 18. A 32-year-old woman comes to the office due to the inability to conceive for last two years. She reports having been on oral contraceptives for 8 years prior to stopping them a year ago to attempt pregnancy. She had menarche at age 14 and has had irregular cycles about every 3 months until she started birth control pills, which made her cycles regular. In the last year, she has had about 5 cycles in total; her last menstrual period was 6 weeks ago. She is otherwise in good health and has not had any surgeries. She has no history of abnormal Pap smears or sexually transmitted diseases. Her only medication is a multivitamin. Her husband of 4 years is 35-years-old, and has a 10-year-old son from a previous marriage. She is 5'4" tall and weighs 165 pounds. On general appearance, she seems to be hirsute on the face and the abdomen. The rest of her exam is otherwise normal. Which of the following is most likely to help identify the underlying cause of this woman's infertility? (2.5分) A.Dehydroepiandrosterone Sulfate (DHEAS) levels
B.Testosterone levels
C. Ratio of luteinizing hormone to follicle stimulating hormone (LH/FSH)
D.Thyroid function tests
E.Prolactin levels
B.Testosterone levels
4 8 2011 19. An 18-year-old woman comes to your clinic with irregular cycles since menarche and mild hirsutism. She is not interested in pregnancy or contraception. Her serum TSH, prolactin, and dehydroepiandrosterone sulfate (DHEAS) levels are normal, with a slightly elevated serum testosterone level of 80 ng/dL. Which of the following is the most appropriate next step for this patient? (2.5分) A.oral contraceptive treatment
B.endometrial biopsy
C.GnRH stimulation test
D.clomiphene citrate
E.bromocriptine
A.oral contraceptive treatment
4 8 2011 20. A 21-year-old woman comes to the office because of acne, irregular menses and hirsutism. She initially was evaluated 6 months ago. At that time, she was diagnosed with idiopathic hirsutism. She was started on oral contraceptive pills to improve her symptoms. Menstrual periods now occur every month, but her hirsutism has not significantly improved. Which of the following would be an appropriate treatment for hirsutism, in addition to the oral contraceptives?(2.5分) A.Spironolactone
B.Lupron
C.Danazol
D.Depo-Provera
E.Steroids
A.Spironolactone
4 8 2011 21. A 35-year-old Asian woman presents with irregular menses and hirsutism of 3 months duration. The patient has no family history of hirsutism. On exam, the patient was noted to have terminal hair growth on her chest and recently had electrolysis to remove similar hair on her chin. Her total testosterone is 76 ng /dl (normal) and her DHEAS is 1500?g/dl (elevated). Which of the following is the most likely diagnosis in this patient? (2.5分) A.PCOS
B.Ovarian tumor
C.Cushing's syndrome
D.Adrenal tumor
E.Idiopathic
D.Adrenal tumor
4 8 2011 22. 子宫颈慢性炎的病变不包括(2.5分) A.子宫息肉
B.子宫颈单纯性糜烂
C.子宫颈上皮内肿瘤
D.子宫颈鳞状上皮化生
E.子宫颈腺囊肿
C.子宫颈上皮内肿瘤
4 8 2011 23. 子宫颈癌的癌前病变是(2.5分) A.宫颈息肉
B.腺体潴留性囊肿
C.宫颈上皮鳞状化生
D.宫颈上皮不典型增生
E.宫颈肥厚
D.宫颈上皮不典型增生
4 8 2011 24. 子宫内膜增生症时,在何种类型中能发现细胞异型性 (2.5分) A.单纯型
B.复杂型
C.非典型性
D.实性
E.囊性
C.非典型性
4 8 2011 25. 子宫颈鳞状上皮内病变,是指(2.5分) A.上皮细胞增生并具有异型性
B.上皮细胞变成癌细胞
C.上皮细胞增厚、钉突延长
D.上皮细胞完全失去分化能力
E.子宫颈柱状上皮被鳞状上皮代替
A.上皮细胞增生并具有异型性
4 8 2011 26. 宫颈早期浸润癌是指(2.5分) A.癌穿过基底膜,深度在1mm以内
B.癌穿过基底膜,深度在2mm以内
C.癌穿过基底膜,深度在3mm以内
D.癌穿过基底膜,深度在4mm以内
E.癌穿过基底膜,深度在5mm以内
E.癌穿过基底膜,深度在5mm以内
4 8 2011 27. 最常见的功血为(2.5分) A.黄体功能不足
B.子宫内膜脱落不全
C.排卵期出血
D.排卵型月经过多
E.无排卵性功血
E.无排卵性功血
4 8 2011 28. 患者女,26岁。停经60天后阴道出血11天。检查:子宫正常大小,质软,宫颈黏液见典型羊齿叶状结晶。应考虑为(2.5分) A.先兆流产
B.卵巢性闭经
C.异位妊娠
D.无排卵性功血
E.黄体萎缩不全所致的出血
D.无排卵性功血
4 8 2011 29. 青春期功血大出血使用大剂量雌 激素止血后,雌激素减量的原则(2.5分) A.止血后每天减1/2量至维持量
B.止血后每天减1/3量至维持量
C.止血后每2天减1/3量至维持量
D.止血后每2天减1/2量至维持量
E.止血后每3天减1/3量至维持量
E.止血后每3天减1/3量至维持量
4 8 2011 30. 35岁,已生育2女孩,近半年来月经不调,8-12/26天,基础体温双相,月经第6天刮出宫内膜病理为:仍可见分泌性内膜,应考虑什么诊断?(2.5分) A.更年期月经紊乱
B.不全流产
C.黄体功能不足
D.黄体萎缩不全
E.无排卵型功血
D.黄体萎缩不全
4 8 2011 31. 女,53岁,近2-3年月经不调,表现为周期延长,经量增多且淋漓不净。此次停经3个月,阴道出血10余天,量多,给予诊刮止血,刮出物组织学检查为子宫内膜简单型增生过长。其诊断考虑为(2.5分) A.无排卵型功血
B.黄体功能不足
C.子宫内膜不规则脱落
D.子宫内膜炎
E.排卵型功血
A.无排卵型功血
4 8 2011 32. A 44-year-old woman (gravida 5, para 5) comes in complaining that she has noticed a bulge protruding out of her vagina. Her other medical problems include hypertension treated with medication, diabetes mellitus, and alcoholism. She stands at work as a grocery clerk. She has a family history of genital prolapse. On examination,you notice a uterine prolapse, cystocele,and rectocele. Which of the following is her major risk factor for her pelvic support disorder? (2.5分) A.childbirth
B.hypertension
C.diabetes mellitus
D.positive family history
E.environmental factors-job
A.childbirth
4 8 2011 33. A 56-year-old woman complains that she is "sitting on a ball." She says constipation is a significant problem for her and that sometimes she needs to push stool out of her rectum by inserting a finger in the vagina and pressing on a bulge. On further examination, which of the following will you most likely find? (2.5分) A.cystocele
B.rectocele
C.enterocele
D.complete uterine prolapse
E.hemorrhoid
B.rectocele
4 8 2011 34. Kegel exercises were designed to do which of the following?(2.5分) A.strengthen the abdominal muscles after childbirth
B.increase the blood flow to the perineum to speed the healing of an episiotomy
C.improve the tone of the muscles surrounding the bladder base and proximal bladder neck
D.prevent denervation of pelvic muscles after childbirth
E.decrease the muscle atrophy associated with aging
C.improve the tone of the muscles surrounding the bladder base and proximal bladder neck
4 8 2011 35. A 52-year-old postmenopausal woman complains of urinary frequency, urgency, and urge incontinence. She is otherwise healthy. Which of the following is included in the behavioral treatment you recommend? (2.5分) A.relaxation techniques
B.anticholinergic medication
C.voiding every hour during the daytime
D.bladder retraining
E.incontinence pad testing
D.bladder retraining
4 8 2011 36. Gynecologic fistulas may be commonly related to which of the following? (2.5分) A.urologic surgery
B.total abdominal hysterectomy
C.general surgery
D.colorectal surgry
E.vascular surgry
B.total abdominal hysterectomy
4 8 2011 37. 一位80岁的妇女,孕5产4,主诉阴道口肿物突出20年,有二次急产史,产后未得到良好的休息。有高血压史20年,正规服用降压药,合并糖尿病,抽烟每天10支。近一月感便秘。检查发现子宫颈位于处女膜外5cm。该妇女子宫脱垂的主要原因是什么? (2.5分) A.分娩对盆底的损伤
B.高血压病
C.糖尿病
D.绝经后的雌激素下降
E.便秘
A.分娩对盆底的损伤
4 8 2011 38. 一位76岁的妇女,孕3产3,第一胎为难产,绝经30年,有阴道口肿物15年,近二天不能排尿和排便,检查发现整个子宫、阴道前壁和阴道后壁完全掉出在阴道口外。你该诊断何种疾病? (2.5分) A.一度子宫脱垂
B.二度子宫脱垂
C.三度子宫脱垂
D.四度盆腔器官脱垂
E.子宫内翻
D.四度盆腔器官脱垂
4 8 2011 39. 哪位妇女最有可能已经绝经? (2.5分) A.35岁的妇女,体重75kg,停经3个月
B.45岁妇女,最近服用减肥药,体重从原来的70kg减至50kg,2个月未来月经
C.50岁妇女,没来月经13个月
D.55岁妇女,停经11个月
E.21岁女性,从未来过月经
C.50岁妇女,没来月经13个月
4 8 2011 40. 下列哪一项不属于绝经的危害?(2.5分) A.潮热、潮红、睡眠障碍、抑郁
B.直肠癌
C.生殖道萎缩
D.骨质疏松
E.冠心病发生率增加
B.直肠癌
4 8 2012 1. A 90-year-old woman comes to your office complaining that she feels as though she is "sitting on a ball." On examination, you find that the vagina is essentially turned inside out, and the entire uterus lies outside the vaginal introitus. This condition is known as which of the following? (2.5分) A.first-degree prolapse
B.second-degree prolapse
C.third-degree prolapse
D.fourth-degree prolapse or procidentia
E.vaginal evisceration
D.fourth-degree prolapse or procidentia
4 8 2012 2. A 52-year-old postmenopausal woman complains of urinary frequency, urgency, and urge incontinence. She is otherwise healthy. Which of the following is included in the conservative treatment you recommend? (2.5分) A.relaxation techniques
B.anticholinergic medication
C.voiding every hour during the daytime
D.vaginal estrogen use
E.incontinence pad testing
D.vaginal estrogen use
4 8 2012 3. Of the following, which is the most common cause of rectovaginal fistula? (2.5分) A.obstetrical delivery
B.irradiation to the pelvis
C.carcinoma
D.hemorrhoidectomy
E.Crohn's disease
A.obstetrical delivery
4 8 2012 4. 一位60岁的妇女,孕3产2, 2年前感觉有肿物自阴道内向下掉出,休息后好转,近10天感到尿频、尿急和尿不尽,有时甚至需要用手上推肿物才能排尿。当你行妇科检查时,最有可能发现的异常疾病是什么? (2.5分) A.膀胱脱垂
B.直肠脱垂
C.子宫完全脱垂
D.尿路感染
E.阴道顶脱垂
A.膀胱脱垂
4 8 2012 5. 一位妇女56岁,因子宫颈原位癌,10天行子宫全切术,术后留置导尿3天,一直伴有低热,2天前患者诉说阴道内有少量液体流出,今天早晨起持续不断有液体自阴道流出。她最有可能发生了何种手术并发症? (2.5分) A.压力性尿失禁
B.尿瘘
C.尿路感染
D.阴道顶愈合不良
E.盆腔炎
B.尿瘘
4 8 2012 6. 绝经的根本原因是什么?(2.5分) A.卵巢功能的衰竭
B.手术切除了子宫
C.下丘脑-垂体-卵巢轴功能失调
D.手术切除一侧卵巢
E.垂体功能减退
A.卵巢功能的衰竭
4 8 2012 7. 下列哪项不属于HRT的适应证?(2.5分) A.心悸、潮热
B.骨质疏松,年龄小于60岁
C.生殖道萎缩
D.严重肝功能损害
E.月经紊乱伴睡眠障碍
D.严重肝功能损害
4 8 2012 8. 目前我国政府开展的改善妇女健康状况的重大公共卫生项目包括 (2.5分) A.免费婚前医学检查
B.孕前及早孕期间免费补充叶酸
C.农村妇女可以享受住院分娩补助
D.有乙肝,梅毒和艾滋病感染的孕妇可以免费得到母婴传播阻断服务
E.以上都是
E.以上都是
4 8 2012 9. 目前我国开展婚检的主要目的是(2.5分) A.检查准备结婚的双方是否存在传染病。
B.检查准备结婚的双方是否患有严重的精神性疾病。
C.检查准备结婚的双方是否患有严重遗传性疾病
D.检查准备结婚的双方是否患有影响生育的严重疾病
E.以上都是
E.以上都是
4 8 2012 10. 我国的孕产妇系统保健内容包括哪些内容 (2.5分) A.及早诊断妊娠
B.孕12周内进行早孕检查
C.孕期至少5次的产前检查
D.住院分娩,产后访视3次
E.以上都是
E.以上都是
4 8 2012 11. 关于促进母乳喂养成功的措施中哪些是错误的? (2.5分) A.孩子一出生尽早和母亲皮肤接触,吸吮母亲乳头。
B.产后24小时和母亲同室
C.正常新生儿4个月内除母乳外不需要给孩子添加任何辅助食物甚至水。
D.孩子出生后养成定时哺乳的习惯。
E.A和B
D.孩子出生后养成定时哺乳的习惯。
4 8 2012 12. 我国计算围产儿死亡率的定义中哪个说法是正确的? (2.5分) A.分子是孕28周后所分娩的所有死胎,死产和7天内的新生儿死亡数。
B.分子是孕24周后所分娩的所有死胎,死产和7天内的新生儿死亡数。
C.分母为与分子同一时间段的产妇数。
D.分母为与分子同一时间段的活产数。
E.B和C
A.分子是孕28周后所分娩的所有死胎,死产和7天内的新生儿死亡数。
4 8 2012 13. 流行病学的研究范围是(2.5分) A.传染病
B.疾病和健康状况
C.传染病和地方病
D.传染病和非传染病
E.A和C
B.疾病和健康状况
4 8 2012 14. 在抽样调查中,下列哪种抽样方法的抽样误差最大? (2.5分) A.单纯随机抽样
B.系统抽样
C.分层抽样
D.整群抽样
E.A和B
D.整群抽样
4 8 2012 15. 在病例对照研究中,匹配是指(2.5分) A.在安排病例和对照组时,使两者的某些特征或变量相一致的方法
B.在安排病例和对照时,使两者的研究因素相一致的方法
C.在安排病例和对照时,使两者的所有特征或变量相一致的方法
D.病例组的研究因素的数量与对照组完全一致
E.B和D
A.在安排病例和对照组时,使两者的某些特征或变量相一致的方法
4 8 2012 16. 发病率指标来自(2.5分) A.对住院病人的调查
B.对门诊病人的调查
C.对社区人群的调查
D.对所有病人的调查
E.A和B
C.对社区人群的调查
4 8 2012 17. 队列研究的最大优点是(2.5分) A.对较多的人进行较长时间的随访
B.发生偏倚的机会少
C.较直接地验证因素与疾病的因果关系
D.研究的结果常能代表全人群
E.A和D
C.较直接地验证因素与疾病的因果关系
4 8 2012 18. 产时妇女保健的关键是(2.5分) A.要住院分娩
B.要新法接生
C.要做到防感染、防滞产、防产伤、防出血、防窒息
D.高危孕妇提前住院
E.以上均是
C.要做到防感染、防滞产、防产伤、防出血、防窒息
4 8 2012 19. 关于排卵性功能性出血的治疗,下列哪项是正确的(2.5分) A.排卵期出血可从月经第1天起服雌激素
B.排卵型月经过多用大量雌激素止血
C.内膜脱落不全用克罗米酚治疗
D.黄体功能不全可在体温升高第3天起用HCG治疗
E.黄体功能不全亦可给甲基睾丸素治疗
D.黄体功能不全可在体温升高第3天起用HCG治疗
4 8 2012 20. 对于功能失调性子宫出血,下列哪项是不恰当的(2.5分) A.功血可发生在任何年龄,但50%发生在绝经前期
B.功血是不伴有全身及内外生殖器官器质性病变的异常子宫出血
C.功血是由于调节生殖的神经内分泌机制失常引起的异常子宫出血
D.青春期功血多数有血液系统病变
E.更年期功血多数是无排卵型功血
D.青春期功血多数有血液系统病变
4 8 2012 21. 12岁少女,月经延长,2-3个月来潮一次,经量多导致贫血,现阴道流血已8日,量仍多。止血应采取的措施是: (2.5分) A.肌注黄体酮注射液
B.口服小剂量雌激素
C.口服大剂量雌激素
D.肌注丙酸睾酮
E.静脉滴注所宫素
C.口服大剂量雌激素
4 8 2012 22. 35岁,已生育2女孩,近半年来月经不调,8-12/26天,基础体温双相,月经第6天刮出宫内膜病理为:仍可见分泌性内膜,应考虑什么诊断? (2.5分) A.更年期月经紊乱
B.不全流产
C.黄体功能不足
D.黄体萎缩不全
E.无排卵型功血
D.黄体萎缩不全
4 8 2012 23. 女,53岁,近2-3年月经不调,表现为周期延长,经量增多且淋漓不净。此次停经3个月,阴道出血10余天,量多,给予诊刮止血,刮出物组织学检查为子宫内膜简单型增生过长。其诊断考虑为(2.5分) A.无排卵型功血
B.黄体功能不足
C.子宫内膜不规则脱落
D.子宫内膜炎
E.排卵型功血
A.无排卵型功血
4 8 2012 24. 题干同第23题,对此患者,最佳治疗方案是(2.5分) A.全子宫切除术
B.诊刮后应用强效孕激素
C.诊刮后抗感染治疗
D.子宫内膜电切术
E.诊刮后观察随访
B.诊刮后应用强效孕激素
4 8 2012 25. A 21-year-old diabetic athletic woman on a lowdose oral contraceptive comes to your clinic with irregular menses and galactorrhea. On examination, galactorrhea is confirmed, with fat globules seen microscopically. She currently takes metoclopramide (Reglan) for delayed gastric emptying. A random serum prolactin level is 65 ng/mL. Which of the following is most likely responsible for her hyperprolactinemia? (2.5分) A.metoclopramide
B.pregnancy
C.oral contraceptive
D.pituitary adenoma
E.exercise
A.metoclopramide
4 8 2012 26. A 25-year-old woman suffers a severe intrapartum hemorrhage. Which of the following symptoms is evidence of pituitary infarction? (2.5分) A.infrequent urination
B.diarrhea
C.easy bruisability
D.lactation failure
E.perspiration
D.lactation failure
4 8 2012 27. An 18-year-old patient has not experienced menarche. Examination shows normal breast development and absence of a uterus. Which of the following diagnostic tests is most useful in determining the etiology of the amenorrhea? (2.5分) A.serum FSH
B.serum estradiol
C.serum testosterone
D.MRI of the head
E.ovarian biopsy
C.serum testosterone
4 8 2012 28. A 45-year-old woman who has had two normal pregnancies 15 and 18 years ago presents with the complaint of amenorrhea for 7 month.She expresses the desire to become pregnant again.After exclusion of pregnancy,which of the following tests is next indicated in the evaluation of this patient's amenorrhea? (2.5分) A.Hysterosalpingogram
B.Endometrial biopsy
C.thyroid function tests
D.Testosterone and DHAS levels
E.LH and FSH levels
E.LH and FSH levels
4 8 2012 29. A 23-year-old G0 comes to the clinic because she is interested in becoming pregnant. She is in good health; however, she has not had any menses for the last two years. She had menarche at age 15, had normal periods until 3 years ago, when she started having periods irregularly every 3 months and then stopped 2 years ago. She has no history of pelvic infections or abnormal Pap smears. She exercises every day by running and has run 4 marathons in the last 3 years. She is 5'10" tall and weighs 105 pounds. Her examination including a pelvic exam is normal.Laboratory results show:Results Normal Values;TSH 10 mIU/ml 0.5-4.0 mIU/ml;Free T40.2 ng/dl 0.8-1.8 ng/dl;Prolactin 60 ng/ml <20 ng/ml;FSH 6 mIU/ml 5-25 mIU/ml;LH 4 mIU/ml 5-25 mIU/ml;BHCG 2 mIU/ml <5 mIU/ml;What is the most appropriate next step in the management of this patient? (2.5分) A.Check cortisol levels
B.Order a brain MRI
C.Obtain a pelvic ultrasound
D.Check testosterone levels
E.Check estrogen levels
E.Check estrogen levels
4 8 2012 30. 子宫颈真性糜烂是指 (2.5分) A.黏膜上皮坏死脱落
B.柱状上皮取代鳞状上皮
C.重度不典型增生
D.囊肿形成
E.鳞状上皮化生
A.黏膜上皮坏死脱落
4 8 2012 31. 子宫颈原位癌与不典型增生的鉴别主要在于原位癌 (2.5分) A.细胞排列极向消失
B.细胞具有异型性
C.累及上皮全层
D.细胞增生活跃.层数增多
E.增生细胞表层为不全角化层
C.累及上皮全层
4 8 2012 32. 子宫颈上皮内瘤变累及腺体是指 (2.5分) A.子宫颈腺体癌变
B.子宫颈异型细胞影响腺体分泌物排出
C.子宫颈鳞状上皮及腺上皮均癌变
D.异型细胞突破基底膜并侵及腺体
E.异型细胞沿基底膜伸入腺体内
E.异型细胞沿基底膜伸入腺体内
4 8 2012 33. 子宫颈癌的组织发生可来源于 (2.5分) A.子宫颈鳞状上皮
B.子宫颈管的黏膜柱状上皮
C.子宫颈的储备细胞
D.以上都可以
E.以上都不是
D.以上都可以
4 8 2012 34. A 34-year-old woman comes to you for a chief complaint of hirsutism. She states that this has been present since menarche, but has gotten worse in the past two years. Her menses have become more irregular, now every 28-45 days apart. She states that she quit smoking and gained approximately thirty pounds in the past three years. Her mother is obese, diabetic and has hirsutism. The patient has acanthosis nigricans present on the back of her neck. There is no hair seen on her chin, but she shaves every few days. She also has a significant amount hair present on the back of her hand and on her forearms. TSH, prolactin, 17-hydroxyprogesterone and DHEAS are normal. Testosterone is mildly elevated. Which of the following is the most likely etiology of her hirsutism? (2.5分) A.Polycystic ovarian syndrome
B.Ovarian neoplasm
C.Diabetes
D.Cushing's syndrome
E.Adrenal tumor
A.Polycystic ovarian syndrome
4 8 2012 35. A 32-year-old woman comes to the office due to the inability to conceive for last two years. She reports having been on oral contraceptives for 8 years prior to stopping them a year ago to attempt pregnancy. She had menarche at age 14 and had irregular cycles about every 3 months apart until started birth control pills, which made her cycles regular. In the last year, she has had about 5 cycles in total; her last menstrual period was 6 weeks ago. She is otherwise in good health and has not had any surgeries. She has no history of abnormal Pap smears or sexually transmitted diseases. Her only medication is a multivitamin. Her husband of 4 years is 35-years-old, and has a 10-year-old son from a previous marriage. She is 5'4" tall and weighs 165 pounds. On general appearance, she seems to be hirsute on the face and the abdomen. The rest of her exam is otherwise normal. In addition to weight loss and starting Metformin, what is the most appropriate treatment for this patient's infertility problem? (2.5分) A.Laparoscopy and ovarian biopsy
B.Ovulation induction agents such as Clomiphene Citrate
C.Intrauterine insemination
D.In vitro fertilization with ICSI (Intracytoplasmic sperm injection)
E.In vitro fertilization without ICSI
B.Ovulation induction agents such as Clomiphene Citrate
4 8 2012 36. A 26-year-old woman presents with hirsutism and irregular menses. Her mother, who is diabetic, had similar complaints prior to menopause. On physical exam, this patient is noted to have terminal hair on her chin and a gray-brown velvety discoloration on the back of her neck. This lesion is acanthosis nigricans. What is the most likely diagnosis in this patient?(2.5分) A.Hyperinsulinemia
B.Ovarian androgen excess
C.Late onset congenital adrenal hyperplasia
D.Adrenal tumor
E.Polycystic ovarian syndrome
A.Hyperinsulinemia
4 8 2012 37. A 24-year-old woman complains of bothersome hirsutism and skipping periods.She does not have evidence of voice changes,hair loss,or clitoromegaly.Pelvic examination does not reveal adnexal masses.serum DHEA-S,testosterone,and 17-hydroxyprogesterone levels are normal.LH/FSH ratio is 2:1.Which of the following is the most likely diagnosis? (2.5分) A.Polycystic ovarian syndrome
B.Familial hirsutism
C.Ovarian tumor
D.Adrenal tumor
E.Cushing's syndrome
A.Polycystic ovarian syndrome
4 8 2012 38. 哪位妇女最有可能已经绝经(2.5分) A.35岁的妇女,体重75kg,停经3个月
B.45岁妇女,最近服用减肥药,体重从原来的70kg减至50kg,2个月未来月经
C.50岁妇女,没来月经13个月
D.55岁妇女,停经11个月
E.21岁女性,从未来过月经
C.50岁妇女,没来月经13个月
4 8 2012 39. 宫颈早期浸润癌是指 (2.5分) A.癌穿过基底膜,深度在1mm以内
B.癌穿过基底膜,深度在2mm以内
C.癌穿过基底膜,深度在3mm以内
D.癌穿过基底膜,深度在4mm以内
E.癌穿过基底膜,深度在5mm以内
E.癌穿过基底膜,深度在5mm以内
4 8 2012 40. Hirsutism, ovarian androgen excess, and elevated serum LH levels are associated with which of the following? (2.5分) A.11-beta-hydroxylase deficiency
B.Cushing's syndrome
C.adrenal tumor
D.PCOS
E.arrhenoblastoma
D.PCOS
4 8 2013 1. 雌激素类药物的药理作用中,不包括(2.5分) A.促进女性性器官的发育和成熟,维持女性第二性征。
B.能明显促进蛋白质合成,减少蛋白质分解
C.使阴道上皮增生,浅表层细胞角化
D.增强子宫平滑肌对缩宫素的敏感性
E.较大剂量时可抑制下丘脑-垂体系统释放GnRH
B.能明显促进蛋白质合成,减少蛋白质分解
4 8 2013 2. 孕激素类药物可用于治疗 (2.5分) A.先兆流产
B.诱发流产
C.再生障碍性贫血
D.消耗性疾病等引起的虚弱
E.卵巢功能不全和闭经
A.先兆流产
4 8 2013 3. 绒毛膜癌与侵蚀性葡萄胎的区别是 (2.5分) A.间质的含量
B.有无血管
C.有无绒毛
D.有无转移
E.肿瘤细胞的异型性
C.有无绒毛
4 8 2013 4. 中年女性,一年前有流产史,现阴道流血不止,贫血外观,子宫体积增大。近来咳嗽、咯血。最可能的诊断是 (2.5分) A.肺癌
B.肺结核
C.子宫绒毛膜癌
D.葡萄胎
E.子宫内膜癌
C.子宫绒毛膜癌
4 8 2013 5. 恶性葡萄胎与良性葡萄胎的主要区别是 (2.5分) A.阴道转移结节
B.浸润肌层
C.细胞异型性
D.有绒毛结构
E.出血坏死
D.有绒毛结构
4 8 2013 6. 诊断早期宫颈癌最可靠的依据是 (2.5分) A.有接触性出血史
B.阴道镜检查
C.盆腔检查
D.宫颈细胞学检查
E.宫颈病理切片检查
E.宫颈病理切片检查
4 8 2013 7. 下列哪项不符合葡萄胎(2.5分) A.绒毛间质水肿,血管消失
B.绒毛滋养层上皮细胞明显增生
C.无胎动及胎心音
D.子宫体积比正常妊娠月份大
E.绒毛膜促性腺激素分泌减少
E.绒毛膜促性腺激素分泌减少
4 8 2013 8. 关注妇女保健的最重要意义? (2.5分) A.妇女从生理上体弱多病
B.妇女健康关系到子代健康;;妇女是脆弱人群(社会地位、经济、教育、就业、医疗);妇女在生理解剖结构上与男性不同,在青春期、围婚期、孕期、分娩期、产褥期、哺乳期、更年期有特殊的生理和病理问题。
C.妇女人口多
D.妇女社会地位低,大多数没有医疗保障
B.妇女健康关系到子代健康;;妇女是脆弱人群(社会地位、经济、教育、就业、医疗);妇女在生理解剖结构上与男性不同,在青春期、围婚期、孕期、分娩期、产褥期、哺乳期、更年期有特殊的生理和病理问题。
4 8 2013 9. 我国的孕产妇系统保健内容包括哪些内容 (2.5分) A.及早诊断妊娠
B.孕12周内进行早孕检查
C.孕期至少5次的产前检查
D.住院分娩,产后访视3次
E.以上都是
E.以上都是
4 8 2013 10. 目前我国开展婚检的主要目的是(2.5分) A.检查准备结婚的双方是否存在传染病。
B.检查准备结婚的双方是否患有严重的精神性疾病。
C.检查准备结婚的双方是否患有严重遗传性疾病
D.检查准备结婚的双方是否患有影响生育的严重疾病
E.以上都是
E.以上都是
4 8 2013 11. 女孩青春期出现的最早征兆是? (2.5分) A.月经来潮
B.乳房出现硬结
C.出现阴毛和腋毛
D.身高突增
B.乳房出现硬结
4 8 2013 12. 我国计算围产儿死亡率的定义中哪个说法是正确的? (2.5分) A.分子是孕28周后所分娩的所有死胎,死产和7天内的新生儿死亡数。
B.分子是孕24周后所分娩的所有死胎,死产和7天内的新生儿死亡数。
C.分母为与分子同一时间段的产妇数。
D.分母为与分子同一时间段的活产数。
A.分子是孕28周后所分娩的所有死胎,死产和7天内的新生儿死亡数。
4 8 2013 13. 流行病学的研究范围是(2.5分) A.传染病
B.疾病和健康状况
C.传染病和地方病
D.传染病和非传染病
B.疾病和健康状况
4 8 2013 14. 发病率指标来自(2.5分) A.对住院病人的调查
B.对门诊病人的调查
C.对社区人群的调查
D.对所有病人的调查
C.对社区人群的调查
4 8 2013 15. 在浙江省围产儿死亡抽样调查中,下列哪种说法是正确的? (2.5分) A.抽取全省各家医院的围产儿
B.抽取监测区(县)的所有助产单位的围产儿死亡病例
C.抽取在监测区县所有新生儿死亡病例
D.抽取在社区建卡的围产儿
B.抽取监测区(县)的所有助产单位的围产儿死亡病例
4 8 2013 16. 在病例对照研究中,匹配是指(2.5分) A.在安排病例和对照组时,使两者的某些特征或变量相一致的方法
B.在安排病例和对照时,使两者的研究因素相一致的方法
C.在安排病例和对照时,使两者的所有特征或变量相一致的方法
D.病例组的研究因素的数量与对照组完全一致
A.在安排病例和对照组时,使两者的某些特征或变量相一致的方法
4 8 2013 17. 队列研究的最大优点是(2.5分) A.对较多的人进行较长时间的随访
B.发生偏倚的机会少
C.较直接地验证因素与疾病的因果关系
D.研究的结果常能代表全人群
C.较直接地验证因素与疾病的因果关系
4 8 2013 18. A 44-year-old woman (gravida 5, para 5) comes in complaining that she has noticed a bulge protruding out of her vagina. Her other medical problems include hypertension treated with medication, diabetes mellitus, and alcoholism. She stands at work as a grocery clerk. She has a family history of genital prolapse. On examination,you notice a uterine prolapse, cystocele,and rectocele. Which of the following is her major risk factor for her pelvic support disorder? (2.5分) A.childbirth
B.hypertension
C.diabetes mellitus
D.positive family history
E.environmental factors-job
A.childbirth
4 8 2013 19. A 56-year-old woman complains that she is "sitting on a ball." She says constipation is a significant problem for her and that sometimes she needs to push stool out of her rectum by inserting a finger in the vagina and pressing on a bulge. On further examination, which of the following will you most likely find? (2.5分) A.cystocele
B.rectocele
C.enterocele
D.complete uterine prolapse
E.hemorrhoid
B.rectocele
4 8 2013 20. Kegel exercises were designed to do which of the following? (2.5分) A.strengthen the abdominal muscles after childbirth
B.increase the blood flow to the perineum to speed the healing of an episiotomy
C.improve the tone of the muscles surrounding the bladder base and proximal bladder neck
D.prevent denervation of pelvic muscles after childbirth
E.decrease the muscle atrophy associated with aging
C.improve the tone of the muscles surrounding the bladder base and proximal bladder neck
4 8 2013 21. A 52-year-old postmenopausal woman complains of urinary frequency, urgency, and urge incontinence. She is otherwise healthy. Which of the following is included in the behavioral treatment you recommend? (2.5分) A.relaxation techniques
B.anticholinergic medication
C.voiding every hour during the daytime
D.bladder retraining
E.incontinence pad testing
D.bladder retraining
4 8 2013 22. Gynecologic fistulas may be commonly related to which of the following? (2.5分) A.urologic surgery
B.total abdominal hysterectomy
C.general surgery
D.colorectal surgry
E.vascular surgry
B.total abdominal hysterectomy
4 8 2013 23. what is the most common cause of pathologic amenorrhea?(2.5分) A.Outflow obstruction
B.disruption of the hypothalamic-pituitary axis
C.asherman syndrome
D.kallman syndrome
C.asherman syndrome
4 8 2013 24. A 22-year-old woman with amenorrhea of 6 weeks' duration undergoes surgery for acute appendicitis. At the time of surgery a 3-cm semisolid left ovarian cyst is discovered. It is vascular and appears to contain a blood-filled central cavity. A serum pregnancy test is positive. Of the following, what is the most appropriate next step in this patient's management? (2.5分) A.ovarian cystectomy
B.ovarian wedge resection
C.oophorectomy
D.salpingo-oophorectomy
E.no additional therapy indicated
E.no additional therapy indicated
4 8 2013 25. An 18-year-old patient has not experienced menarche. Examination shows normal breast development and absence of a uterus. Which of the following diagnostic tests is most useful in determining the etiology of the amenorrhea? (2.5分) A.serum FSH
B.serum estradiol
C.serum testosterone
D.MRI of the head
E.ovarian biopsy
C.serum testosterone
4 8 2013 26. Luteectomy before 42 days' gestation is most likely to result in which of the following?(2.5分) A.prolonged (postdates) gestation
B.spontaneous miscarriage
C.reduction of basal body temperature (BBT)
D.masculinization of a female fetus
E.no demonstrable effect
B.spontaneous miscarriage
4 8 2013 27. A 28-year-old patient complains of amenorrhea after dilation and curettage (D&C) for postpartum bleeding. Of the following, which is the most likely diagnosis? (2.5分) A.gonadal dysgenesis
B.Sheehan's syndrome
C.Kallmann syndrome
D.Mayer-Rokitansky-Kiister-Hauser syndrome
E.Asherman's syndrome
E.Asherman's syndrome
4 8 2013 28. Hirsutism, ovarian androgen excess, and elevated serum LH levels are associated with which of the following? (2.5分) A.11-beta-hydroxylase deficiency
B.Cushing's syndrome
C.adrenal tumor
D.PCOS
E.Arrhenoblastoma
D.PCOS
4 8 2013 29. A 26-year-old woman presents with hirsutism and irregular menses. Her mother, who is diabetic, had similar complaints prior to menopause. On physical exam, this patient is noted to have terminal hair on her chin and a gray-brown velvety discoloration on the back of her neck. This lesion is acanthosis nigricans. What is the most likely diagnosis in this patient? (2.5分) A.Hyperinsulinemia
B.Ovarian androgen excess
C.Late onset congenital adrenal hyperplasia
D.Adrenal tumor
E.Polycystic ovarian syndrome
A.Hyperinsulinemia
4 8 2013 30. A 21-year-old woman comes to the office because of acne, irregular menses and hirsutism. She initially was evaluated 6 months ago. At that time, she was diagnosed with idiopathic hirsutism. She was started on oral contraceptive pills to improve her symptoms. Menstrual periods now occur every month, but her hirsutism has not significantly improved. Which of the following would be an appropriate treatment for hirsutism, in addition to the oral contraceptives?(2.5分) A.Spironolactone
B.Lupron
C.Danazol
D.Depo-Provera
E.Steroids
A.Spironolactone
4 8 2013 31. A 24-year-old woman complains of bothersome hirsutism and skipping periods.She does not have evidence of voice changes,hair loss,or clitoromegaly.Pelvic examination does not reveal adnexal masses.serum DHEA-S,testosterone,and 17-hydroxyprogesterone levels are normal.LH/FSH ratio is 2:1.Which of the following is the most likely diagnosis? (2.5分) A.Polycystic ovarian syndrome
B.Familial hirsutism
C.Ovarian tumor
D.Adrenal tumor
E.Cushing's syndrome
A.Polycystic ovarian syndrome
4 8 2013 32. A 26-year-old woman comes to the office due to irregular menses since menarche, worsening for the last 6 months. The patient has noted increasing hair growth on her chin and most recently hair growth on her chest, requiring that she shave periodically. No one in her family has hirsutism. On exam, you also notice acne on her chin, acanthosis nigricans and temporal balding. Her serum testosterone is elevated. You diagnose hyperthecosis. Which of the following might also be associated with this condition? (2.5分) A.Good response to oral contraceptive therapy
B.Insulin sensitivity
C.Atrophic changes of external genitalia
D.Deepening of the voice
E.Infertility easily treated with ovulation induction
D.Deepening of the voice
4 8 2013 33. 最常见的功血为(2.5分) A.黄体功能不足
B.子宫内膜脱落不全
C.排卵期出血
D.排卵型月经过多
E.无排卵性功血
E.无排卵性功血
4 8 2013 34. 关于排卵性功能性出血的治疗,下列哪项是正确的(2.5分) A.排卵期出血可从月经第1天起服雌激素
B.排卵型月经过多用大量雌激素止血
C.内膜脱落不全用克罗米酚治疗
D.黄体功能不全可在体温升高第3天起用HCG治疗
E.黄体功能不全亦可给甲基睾丸素治疗
D.黄体功能不全可在体温升高第3天起用HCG治疗
4 8 2013 35. 对于功能失调性子宫出血,下列哪项是不恰当的: (2.5分) A.功血可发生在任何年龄,但50%发生在绝经前期
B.功血是不伴有全身及内外生殖器官器质性病变的异常子宫出血
C.功血是由于调节生殖的神经内分泌机制失常引起的异常子宫出血
D.青春期功血多数有血液系统病变
E.更年期功血多数是无排卵型功血
D.青春期功血多数有血液系统病变
4 8 2013 36. 青春期功血大出血使用大剂量雌激素止血后,雌激素减量的原则(2.5分) A.止血后每天减1/2量至维持量
B.止血后每天减1/3量至维持量
C.止血后每2天减1/3量至维持量
D.止血后每2天减1/2量至维持量
E.止血后每3天减1/3量至维持量
E.止血后每3天减1/3量至维持量
4 8 2013 37. 12岁少女,月经延长,2-3个月来潮一次,经量多导致贫血,现阴道流血已8日,量仍多。止血应采取的措施是(2.5分) A.肌注黄体酮注射液
B.口服小剂量雌激素
C.口服大剂量雌激素
D.肌注丙酸睾酮
E.静脉滴注所宫素
C.口服大剂量雌激素
4 8 2013 38. 女,53岁,近2-3年月经不调,表现为周期延长,经量增多且淋漓不净。此次停经3个月,阴道出血10余天,量多,给予诊刮止血,刮出物组织学检查为子宫内膜简单型增生过长。其诊断考虑为(2.5分) A.无排卵型功血
B.黄体功能不足
C.子宫内膜不规则脱落
D.子宫内膜炎
E.排卵型功血
A.无排卵型功血
4 8 2013 39. 题干同第38题,对此患者,最佳治疗方案是(2.5分) A.全子宫切除术
B.诊刮后应用强效孕激素
C.诊刮后抗感染治疗
D.子宫内膜电切术
E.诊刮后观察随访
B.诊刮后应用强效孕激素
4 8 2013 40. 35岁,已生育2女孩,近半年来月经不调,8-12/26天,基础体温双相,月经第6天刮出宫内膜病理为:仍可见分泌性内膜,应考虑什么诊断? (2.5分) A.更年期月经紊乱
B.不全流产
C.黄体功能不足
D.黄体萎缩不全
E.无排卵型功血
D.黄体萎缩不全
4 8 2014 1. 最常见的异常子宫出血为(2.5分) A.黄体功能不足
B.子宫内膜脱落不全
C.排卵期出血
D.排卵型月经过多
E.无排卵性异常子宫出血
E.无排卵性异常子宫出血
4 8 2014 2. 一位60岁的妇女,孕3产2, 2年前感觉有肿物自阴道内向下掉出,休息后好转,近10天感到尿频、尿急和尿不尽,有时甚至需要用手上推肿物才能排尿。当你行妇科检查时,最有可能发现的异常疾病是什么? (2.5分) A.膀胱脱垂
B.直肠脱垂
C.子宫完全脱垂
D.尿路感染
E.阴道顶脱垂
A.膀胱脱垂
4 8 2014 3. 一位妇女56岁,因子宫颈原位癌,10天行子宫全切术,术后留置导尿3天,一直伴有低热,2天前患者诉说阴道内有少量液体流出,今天早晨起持续不断有液体自阴道流出。她最有可能发生了何种手术并发症? (2.5分) A.压力性尿失禁
B.尿瘘
C.尿路感染
D.阴道顶愈合不良
E.盆腔炎
B.尿瘘
4 8 2014 4. 绝经的根本原因是什么?(2.5分) A.卵巢功能的衰竭
B.手术切除了子宫
C.下丘脑-垂体-卵巢轴功能失调
D.手术切除一侧卵巢
E.垂体功能减退
A.卵巢功能的衰竭
4 8 2014 5. 下列哪项不属于HRT的适应证?(2.5分) A.心悸、潮热
B.骨质疏松,年龄小于60岁
C.生殖道萎缩
D.严重肝功能损害
E.月经紊乱伴睡眠障碍
D.严重肝功能损害
4 8 2014 6. 关注妇女保健的最重要意义? (2.5分) A.妇女从生理上体弱多病
B.妇女健康关系到子代健康;妇女是脆弱人群(社会地位、经济、教育、就业、医疗);妇女在生理解剖结构上与男性不同,在青春期、围婚期、孕期、分娩期、产褥期、哺乳期、更年期有特殊的生理和病理问题。
C.妇女人口多
D.妇女社会地位低,大多数没有医疗保障
B.妇女健康关系到子代健康;妇女是脆弱人群(社会地位、经济、教育、就业、医疗);妇女在生理解剖结构上与男性不同,在青春期、围婚期、孕期、分娩期、产褥期、哺乳期、更年期有特殊的生理和病理问题。
4 8 2014 7. 目前我国开展婚检的主要目的是(2.5分) A.检查准备结婚的双方是否存在传染病。
B.检查准备结婚的双方是否患有严重的精神性疾病。
C.检查准备结婚的双方是否患有严重遗传性疾病
D.检查准备结婚的双方是否患有影响生育的严重疾病
E.以上都是
E.以上都是
4 8 2014 8. 我国的孕产妇系统保健内容包括哪些内容(2.5分) A.及早诊断妊娠
B.孕12周内进行早孕检查
C.孕期至少5次的产前检查
D.住院分娩,产后访视3次
E.以上都是
E.以上都是
4 8 2014 9. 女孩青春期出现的最早征兆是? (2.5分) A.月经来潮
B.乳房出现硬结
C.出现阴毛和腋毛
D.身高突增
B.乳房出现硬结
4 8 2014 10. 孕产妇死亡率统计指标中哪项说法是不正确的? (2.5分) A.孕产妇死亡率的指标是反应该国家或地区经济医疗水平的指标。
B.孕产妇死亡指标计算通常以10万分娩量作为基数。
C.孕产妇死亡率指标计算:分子是当地某一时段的孕产妇死亡人数,分母是当地同一时段所有的孕产妇数。
D.孕产妇死亡率较高的地区死亡产科因素的死亡构成比较高,死亡率低的地区以非产科因素构成比较高。
C.孕产妇死亡率指标计算:分子是当地某一时段的孕产妇死亡人数,分母是当地同一时段所有的孕产妇数。
4 8 2014 11. 流行病学的研究范围是(2.5分) A.传染病
B.疾病和健康状况
C.传染病和地方病
D.传染病和非传染病
B.疾病和健康状况
4 8 2014 12. 发病率指标来自(2.5分) A.对住院病人的调查
B.对门诊病人的调查
C.对社区人群的调查
D.对所有病人的调查
C.对社区人群的调查
4 8 2014 13. 在浙江省围产儿死亡抽样调查中,下列哪种说法是正确的? (2.5分) A.抽取全省各家医院的围产儿
B.抽取监测区(县)的所有助产单位的围产儿死亡病例
C.抽取在监测区县所有新生儿死亡病例
D.抽取在社区建卡的围产儿
B.抽取监测区(县)的所有助产单位的围产儿死亡病例
4 8 2014 14. 在病例对照研究中,匹配是指: (2.5分) A.在安排病例和对照组时,使两者的某些特征或变量相一致的方法
B.在安排病例和对照时,使两者的研究因素相一致的方法
C.在安排病例和对照时,使两者的所有特征或变量相一致的方法
D.病例组的研究因素的数量与对照组完全一致
A.在安排病例和对照组时,使两者的某些特征或变量相一致的方法
4 8 2014 15. 在出生缺陷发病情况监测中,一般发生率的表述哪项是正确的(2.5分) A.总发生率以百分率表述,单病种以千分率表达
B.总发生率以千分率表述,单病种以万分率表达
C.总发生率和单病种发生率都以千分率表达
D.总发生率和单病种发生率都以万分率表达
B.总发生率以千分率表述,单病种以万分率表达
4 8 2014 16. 下列哪个不是异常子宫出血的病因(2.5分) A.子宫颈癌
B.子宫腺肌症
C.不排卵
D.错误使用口服避孕药
E.子宫内膜息肉
A.子宫颈癌
4 8 2014 17. 诊断性刮宫的描述错误的是(2.5分) A.疑诊黄体功能不足需在月经来潮前5-7天进行
B.月经第5-7天诊刮见分泌期内膜可诊断黄体萎缩不全
C.年龄>35岁、药物治疗无效的患者可进行诊刮
D.诊断性刮宫的重要目的是排除子宫内膜癌
E.青春期大量阴道出血首选诊刮止血
E.青春期大量阴道出血首选诊刮止血
4 8 2014 18. 35岁,已生育2女孩,近半年来月经不调,8-12/26天,基础体温双相,月经第6天刮出宫内膜病理为:仍可见分泌性内膜,应考虑什么诊断? (2.5分) A.更年期月经紊乱
B.不全流产
C.黄体功能不足
D.黄体萎缩不全
E.无排卵型异常子宫出血
D.黄体萎缩不全
4 8 2014 19. 绒毛膜癌与侵蚀性葡萄胎的区别是(2.5分) A.间质的含量
B.有无血管
C.有无绒毛
D.有无转移
E.肿瘤细胞的异型性
C.有无绒毛
4 8 2014 20. 宫颈早期浸润癌是指(2.5分) A.癌浸润深度不超过基底膜1mm
B.癌浸润深度不超过基底膜下5mm
C.癌浸润深度在基底膜下0.5~1cm
D.癌浸润深度在基底膜下1~3cm
E.癌浸润深度不超过基底膜下3cm
B.癌浸润深度不超过基底膜下5mm
4 8 2014 21. 中年女性,一年前有流产史,现阴道流血不止,贫血外观,子宫体积增大。近来咳嗽、咯血。最可能的诊断是(2.5分) A.肺癌
B.肺结核
C.子宫绒毛膜癌
D.葡萄胎
E.子宫内膜癌
C.子宫绒毛膜癌
4 8 2014 22. 青年女性,闭经3个月,阴道不规律出血,血块中夹有水泡。检查发现子宫体积大,阴道壁有暗紫色结节、出血、坏死。最大可能是(2.5分) A.宫外孕
B.葡萄状肉瘤
C.葡萄胎
D.恶性葡萄胎
E.绒毛膜癌
D.恶性葡萄胎
4 8 2014 23. A 21-year-old diabetic athletic woman on a lowdoseoral contraceptive comes to your clinic withirregular menses and galactorrhea. On examination,galactorrhea is confirmed, with fat globulesseen microscopically. She currently takesmetoclopramide (Reglan) for delayed gastricemptying. A random serum prolactin level is65 ng/mL. Which of the following is most likelyresponsible for her hyperprolactinemia? (2.5分) A.metoclopramide
B.pregnancy
C.oral contraceptive
D.pituitary adenoma
E.exercise
A.metoclopramide
4 8 2014 24. A 25-year-old woman suffers a severe intrapartumhemorrhage. Which of the followingsymptoms is evidence of pituitary infarction? (2.5分) A.infrequent urination
B.diarrhea
C.easybruisability
D.lactation failure
E.perspiration
D.lactation failure
4 8 2014 25. An 18-year-old patient has not experiencedmenarche. Examination shows normal breastdevelopment and absence of a uterus. Which ofthe following diagnostic tests is most useful indetermining the etiology of the amenorrhea? (2.5分) A.serum FSH
B.serum estradiol
C.serum testosterone
D.MRI of the head
E.ovarian biopsy
C.serum testosterone
4 8 2014 26. A 45-year-old woman who has had two normal pregnancies 15 and 18 years ago presents with the complaint of amenorrhea for 7 month.She expresses the desire to become pregnant again.After exclusion of pregnancy,which of the following tests is next indicated in the evaluation of this patient's amenorrhea(2.5分) A.Hysterosalpingogram
B.Endometrial biopsy
C.thyroid function tests
D.Testosterone and DHAS levels
E.LH and FSH levels
E.LH and FSH levels
4 8 2014 27. A 19-year-old woman and her boyfriend wishto use condoms as a barrier contraceptivemethod. This couple should be advised thatthe most common reason for failure (pregnancyor STD transmission) is which of the following? (2.5分) A.breakage
B.inconsistent use
C.leakage caused by spermicidal creams
D.use without concombinate use of aspermicide
E.spill of condom contents uponwithdrawal
B.inconsistent use
4 8 2014 28. A 23-year-old patient is considering contraceptivemethods but is devoutly religious and willnot accept a method that may "cause an abortion."The primary mechanism by which IUD prevent pregnancy is which of the following? (2.5分) A.creating chronic endometritis
B.preventing fertilization
C.inhibiting ovulation
D.altering tuba1 motility
E.destroying sperm
B.preventing fertilization
4 8 2014 29. Which of the following is the primary mechanismby which OCs prevent pregnancy?(2.5分) A.inhibiting serum follicle-stimulatinghormone (FSH) levels
B.inhibiting serum luteinizing hormone(LH) levels
C.inducing endometrial atrophy
D.inducing lymphocytic endometritis
E.increasing cervical mucus viscosity
A.inhibiting serum follicle-stimulatinghormone (FSH) levels
4 8 2014 30. Reducing the estrogen content of OCs results inan decreasing in the rate of which of the following?(2.5分) A.pregnancy
B.breakthrough bleeding (BTB)
C.thromboembolic complications
D.insulin resistance
E.premenstrual symptoms
C.thromboembolic complications
4 8 2014 31. The administration of mifepristone results in whichof the following?(2.5分) A.abortion when given in early pregnancy
B.delayed menses when given during themidluteal phase
C.menses when given during the follicularphase
D.resistance to prostaglandin inhibitors
E.induction of progesterone receptors inthe endometrium
A.abortion when given in early pregnancy
4 8 2014 32. Hirsutism, ovarian androgen excess, and elevated serum LH levels are associated with which of the following?(2.5分) A.11-beta-hydroxylase deficiency
B.Cushing's syndrome
C.adrenal tumor
D.PCOS
E.Arrhenoblastoma
D.PCOS
4 8 2014 33. An 18-year-old woman comes to your clinic with irregular cycles since menarche and mild hirsutism. She is not interested in pregnancy or contraception. Her serum TSH, prolactin, and dehydroepiandrosterone sulfate (DHEAS) levels are normal, with a slightly elevated serum testosterone level of 80 ng/dL. Which of the following is the most appropriate next step for this patient? (2.5分) A.oral contraceptive treatment
B.endometrial biopsy
C.GnRH stimulation test
D.clomiphene citrate
E.bromocriptine
A.oral contraceptive treatment
4 8 2014 34. A 26-year-old woman presents with hirsutism and irregular menses. Her mother, who is diabetic, had similar complaints prior to menopause. On physical exam, this patient is noted to have terminal hair on her chin and a gray-brown velvety discoloration on the back of her neck. This lesion is acanthosis nigricans. What is the most likely diagnosis in this patient? (2.5分) A.Hyperinsulinemia
B.Ovarian androgen excess
C.Late onset congenital adrenal hyperplasia
D.Adrenal tumor
E.Polycystic ovarian syndrome
A.Hyperinsulinemia
4 8 2014 35. A 26-year-old woman comes to the office due to irregular menses since menarche, worsening for the last 6 months. The patient has noted increasing hair growth on her chin and most recently hair growth on her chest, requiring that she shave periodically. No one in her family has hirsutism. On exam, you also notice acne on her chin, acanthosis nigricans and temporal balding. Her serum testosterone is elevated. You diagnose hyperthecosis. Which of the following might also be associated with this condition? (2.5分) A.Good response to oral contraceptive therapy
B.Insulin sensitivity
C.Atrophic changes of external genitalia
D.Deepening of the voice
E.Infertility easily treated with ovulation induction
D.Deepening of the voice
4 8 2014 36. A 44-year-old woman (gravida 5, para 5) comes in complaining that she has noticed a bulge protruding out of her vagina. Her other medical problems include hypertension treated with medication, diabetes mellitus, and alcoholism. She stands at work as a grocery clerk. She has a family history of genital prolapse. On examination,you notice a uterine prolapse, cystocele,and rectocele. Which of the following is her major risk factor for her pelvic support disorder? (2.5分) A.childbirth
B.hypertension
C.diabetes mellitus
D.positive family history
E.environmental factors-job
A.childbirth
4 8 2014 37. A 56-year-old woman complains that she is "sitting on a ball." She says constipation is a significant problem for her and that sometimes she needs to push stool out of her rectum by inserting a finger in the vagina and pressing on a bulge. On further examination, which of the following will you most likely find? (2.5分) A.cystocele
B.rectocele
C.enterocele
D.complete uterine prolapse
E.hemorrhoid
B.rectocele
4 8 2014 38. Kegel exercises were designed to do which of the following? (2.5分) A.strengthen the abdominal muscles after childbirth
B.increase the blood flow to the perineum to speed the healing of an episiotomy
C.improve the tone of the muscles surrounding the bladder base and proximal bladder neck
D.prevent denervation of pelvic muscles after childbirth
E.decrease the muscle atrophy associated with aging
C.improve the tone of the muscles surrounding the bladder base and proximal bladder neck
4 8 2014 39. A 52-year-old postmenopausal woman complains of urinary frequency, urgency, and urge incontinence. She is otherwise healthy. Which of the following is included in the behavioral treatment you recommend? (2.5分) A.relaxation techniques
B.anticholinergic medication
C.voiding every hour during the daytime
D.bladder retraining
E.incontinence pad testing
D.bladder retraining
4 8 2014 40. Gynecologic fistulas may be commonly related to which of the following? (2.5分) A.urologic surgery
B.total abdominal hysterectomy
C.general surgery
D.colorectalsurgry
E.vascularsurgry
B.total abdominal hysterectomy
4 8 2015 1. 关注妇女保健的最重要意义? (2.5分) A.妇女从生理上体弱多病
B.妇女健康关系到子代健康;;妇女是脆弱人群(社会地位、经济、教育、就业、医疗);妇女在生理解剖结构上与男性不同,在青春期、围婚期、孕期、分娩期、产褥期、哺乳期、更年期有特殊的生理和病理问题。
C.妇女人口多
D.妇女社会地位低,大多数没有医疗保障
B.妇女健康关系到子代健康;;妇女是脆弱人群(社会地位、经济、教育、就业、医疗);妇女在生理解剖结构上与男性不同,在青春期、围婚期、孕期、分娩期、产褥期、哺乳期、更年期有特殊的生理和病理问题。
4 8 2015 2. 我国的孕产妇系统保健内容包括哪些内容 (2.5分) A.及早诊断妊娠
B.孕12周内进行早孕检查
C.孕期至少5次的产前检查
D.住院分娩,产后访视3次
E.以上都是
E.以上都是
4 8 2015 3. 孕产妇死亡率统计指标中哪项说法是不正确的? (2.5分) A.孕产妇死亡率的指标是反应该国家或地区经济医疗水平的指标。
B.孕产妇死亡指标计算通常以10万分娩量作为基数。
C.孕产妇死亡率指标计算:分子是当地某一时段的孕产妇死亡人数,分母是当地同一时段所有的孕产妇数。
D.孕产妇死亡率较高的地区死亡产科因素的死亡构成比较高,死亡率低的地区以非产科因素构成比较高。
C.孕产妇死亡率指标计算:分子是当地某一时段的孕产妇死亡人数,分母是当地同一时段所有的孕产妇数。
4 8 2015 4. 关于流行病学,下列哪种说法正确 (2.5分) A.从个体的角度研究疾病和健康状况及其影响因素
B.只研究传染病的流行特征和防治措施
C.只研究慢性病的危险因素
D.研究人群中疾病和健康状况的分布及其影响因素
D.研究人群中疾病和健康状况的分布及其影响因素
4 8 2015 5. 在进行月经紊乱的发病情况调查时,最好的调查方法是 (2.5分) A.从医院就诊人群中调查
B.从某地区的全部人群中调查
C.抽取某个区域或不同区域不同年龄组的人群进行问卷调查
D.从社区健康档案中的登记进行统计
C.抽取某个区域或不同区域不同年龄组的人群进行问卷调查
4 8 2015 6. 在病例对照研究中,比值比(OR)的含义是(2.5分) A.病例组的发病率与对照组的发病率之比
B.病例组的暴露比值与对照组的暴露比值的比
C.对照组的暴露比值除以病例组的暴露比值
D.病例组的暴露比值与对照组的暴露比值之差
B.病例组的暴露比值与对照组的暴露比值的比
4 8 2015 7. 目前我国开展婚检的主要内容(2.5分) A.检查准备结婚的双方是否存在传染病。
B.检查准备结婚的双方是否患有严重的精神性疾病。
C.检查准备结婚的双方是否患有严重遗传性疾病
D.检查准备结婚的双方是否患有影响生育的严重疾病
E.以上都是
E.以上都是
4 8 2015 8. 在出生缺陷发病情况监测中,一般发生率的表述哪项是正确的(2.5分) A.总发生率以百分率表述, 单病种以千分率表达
B.总发生率以千分率表述, 单病种以万分率表达
C.总发生率和单病种发生率都以千分率表达
D.总发生率和单病种发生率都以万分率表达
B.总发生率以千分率表述, 单病种以万分率表达
4 8 2015 9. 绒毛膜癌与侵蚀性葡萄胎的区别是(2.5分) A.间质的含量
B.有无血管
C.肿瘤细胞的异型性
D.有无绒毛
E.有无转移
D.有无绒毛
4 8 2015 10. 青年女性,闭经3个月,阴道不规律出血,血块中夹有水泡。检查发现子宫体积大,阴道壁有暗紫色结节、出血、坏死。 最大可能是(2.5分) A.宫外孕
B.葡萄状肉瘤
C.葡萄胎
D.恶性葡萄胎
E.绒毛膜癌
D.恶性葡萄胎
4 8 2015 11. 宫颈早期浸润癌是指 (2.5分) A.癌浸润深度不超过基底膜下5mm
B.癌浸润深度超过基底膜5mm
C.癌浸润深度不超过基底膜下3cm
D.癌浸润深度超过基底膜1mm
E.癌浸润深度在基底膜下0。5~1cm
A.癌浸润深度不超过基底膜下5mm
4 8 2015 12. 诊断早期宫颈癌最可靠的依据是(2.5分) A.有接触性出血史
B.阴道镜检查
C.盆腔检查
D.宫颈细胞学检查
E.宫颈病理切片检查
E.宫颈病理切片检查
4 8 2015 13. 关于短效口服避孕药的避孕原理,正确的是(2.5分) A.宫颈黏液受雌激素影响,量多、黏稠度增加,不利于精子穿透
B.加速孕卵在输卵管内运行速度,使其与子宫内膜的发育同步
C.雌激素作用使子宫内膜发育不良,不利于受精卵着床
D.抑制排卵
E.影响下丘脑的LHRH 释放,促进FSH 和LH 的分泌
D.抑制排卵
4 8 2015 14. 检查宫内节育器是否存在最常用的方法是 (2.5分) A.盆腔X 线透视
B.子宫探针探触
C.盆腔B超检查
D.子宫碘油造影
E.宫腔镜检查
C.盆腔B超检查
4 8 2015 15. 女性,35岁,2-0-0-2,现停经85天,超声提示宫内活胎,与停经月份相符。拟终止妊娠,下列哪种方法正确 (2.5分) A.药物流产
B.负压吸引术
C.钳刮术
D.缩宫素静滴
E.依沙吖啶羊膜腔内注射引产
C.钳刮术
4 8 2015 16. 产后 3 个月的哺乳妇女,在行宫内节育器的放置术中,突然出现腹痛,最可能的诊断是(2.5分) A.羊水栓塞
B.盆腔炎症
C.节育器嵌顿
D.子宫穿孔
E.子宫收缩
D.子宫穿孔
4 8 2015 17. A 44-year-old woman (gravida 5, para 5) comes in complaining that she has noticed a bulge protruding out of her vagina. Her other medical problems include hypertension treated with medication, diabetes mellitus, and alcoholism. She stands at work as a grocery clerk. She has a family history of genital prolapse. On examination,you notice a uterine prolapse, cystocele ,and rectocele. Which of the following is her major risk factor for her pelvic support disorder? (2.5分) A.childbirth
B.hypertension
C.diabetes mellitus
D.positive family history
E.environmental factors-job
A.childbirth
4 8 2015 18. A 56-year-old woman complains that she is "sitting on a ball." She says constipation is a significant problem for her and that sometimes she needs to push stool out of her rectum by inserting a finger in the vagina and pressing on a bulge. On further examination, which of the following will you most likely find?(2.5分) A.cystocele
B.rectocele
C.enterocele
D.complete uterine prolapse
E.hemorrhoid
B.rectocele
4 8 2015 19. A 52-year-old postmenopausal woman complains of urinary frequency, urgency, and urge incontinence. She is otherwise healthy. Which of the following is included in the behavioral treatment you recommend? (2.5分) A.relaxation techniques
B.anticholinergic medication
C.voiding every hour during the daytime
D.bladder retraining
E.incontinence pad testing
D.bladder retraining
4 8 2015 20. 一位60岁的妇女,孕3产2, 2年前感觉有肿物自阴道内向下掉出,休息后好转,近10天感到尿频、尿急和尿不尽,有时甚至需要用手上推肿物才能排尿。当你行妇科检查时,最有可能发现的异常疾病是什么?(2.5分) A.膀胱脱垂
B.直肠脱垂
C.子宫完全脱垂
D.尿路感染
E.阴道顶脱垂
A.膀胱脱垂
4 8 2015 21. 一位妇女56岁,因子宫颈原位癌,10天行子宫全切术,术后留置导尿3天,一直伴有低热,2天前患者诉说阴道内有少量液体流出,今天早晨起持续不断有液体自阴道流出。她最有可能发生了何种手术并发症?(2.5分) A.压力性尿失禁
B.尿瘘
C.尿路感染
D.阴道顶愈合不良
E.盆腔炎
B.尿瘘
4 8 2015 22. 绝经的根本原因是什么?(2.5分) A.卵巢功能的衰竭
B.手术切除了子宫
C.下丘脑-垂体-卵巢轴功能失调
D.手术切除一侧卵巢
E.垂体功能减退
A.卵巢功能的衰竭
4 8 2015 23. 下列哪项不属于HRT的适应证?(2.5分) A.心悸、潮热
B.骨质疏松,年龄小于60岁
C.生殖道萎缩
D.严重肝功能损害
E.月经紊乱伴睡眠障碍
D.严重肝功能损害
4 8 2015 24. Gynecologic fistulas may be commonly related to which of the following? (2.5分) A.urologic surgery
B.total abdominal hysterectomy
C.general surgery
D.colorectal surgry
E.vascular surgry
B.total abdominal hysterectomy
4 8 2015 25. 最常见的异常子宫出血为 (2.5分) A.黄体功能不足
B.子宫内膜脱落不全
C.排卵期出血
D.排卵型月经过多
E.无排卵性异常子宫出血
E.无排卵性异常子宫出血
4 8 2015 26. 患者女,26岁。停经60天后阴道出血11天。检查:子宫正常大小,质软,宫颈黏液见典型羊齿叶状结晶。应考虑为 (2.5分) A.先兆流产
B.卵巢性闭经
C.异位妊娠
D.无排卵性异常子宫出血
E.黄体萎缩不全所致的出血
D.无排卵性异常子宫出血
4 8 2015 27. 诊断性刮宫的描述错误的是(2.5分) A.疑诊黄体功能不足需在月经来潮前5-7天进行
B.月经第5-7天诊刮见分泌期内膜可诊断黄体萎缩不全
C.年龄>35岁、药物治疗无效的患者可进行诊刮
D.诊断性刮宫的重要目的是排除子宫内膜癌
E.青春期大量阴道出血首选诊刮止血
E.青春期大量阴道出血首选诊刮止血
4 8 2015 28. 12岁少女,月经周期延长,2-3个月来潮一次,经量多导致贫血,现阴道流血已8日,量仍多。止血应采取的措施是(2.5分) A.肌注黄体酮注射液
B.口服小剂量雌激素
C.口服大剂量雌激素
D.肌注丙酸睾酮
E.静脉滴注缩宫素
C.口服大剂量雌激素
4 8 2015 29. Hirsutism, ovarian androgen excess, and elevated serum LH levels are associated with which of the following? (2.5分) A.11-beta-hydroxylase deficiency
B.Cushing's syndrome
C.adrenal tumor
D.PCOS
E.arrhenoblastoma
D.PCOS
4 8 2015 30. A 34-year-old woman comes to you for a chief complaint of hirsutism. She states that this has been present since menarche, but has gotten worse in the past two years. Her menses have become more irregular, now every 28-45 days apart. She states that she quit smoking and gained approximately thirty pounds in the past three years. Her mother is obese, diabetic and has hirsutism. The patient has acanthosis nigricans present on the back of her neck. There is no hair seen on her chin, but she shaves every few days. She also has a significant amount hair present on the back of her hand and on her forearms. TSH, prolactin, 17-hydroxyprogesterone and DHEAS are normal. Testosterone is mildly elevated. Which of the following is the most likely etiology of her hirsutism? (2.5分) A.Polycystic ovarian syndrome
B.Ovarian neoplasm
C.Diabetes
D.Cushing's syndrome
E.Adrenal tumor
A.Polycystic ovarian syndrome
4 8 2015 31. An 18-year-old woman comes to your clinic with irregular cycles since menarche and mild hirsutism. She is not interested in pregnancy or contraception. Her serum TSH, prolactin, and dehydroepiandrosterone sulfate (DHEAS) levels are normal, with a slightly elevated serum testosterone level of 80 ng/dL. Which of the following is the most appropriate next step for this patient? (2.5分) A.oral contraceptive treatment
B.endometrial biopsy
C.GnRH stimulation test
D.clomiphene citrate
E.bromocriptine
A.oral contraceptive treatment
4 8 2015 32. A 21-year-old woman comes to the office because of acne, irregular menses and hirsutism. She initially was evaluated 6 months ago. At that time, she was diagnosed with idiopathic hirsutism. She was started on oral contraceptive pills to improve her symptoms. Menstrual periods now occur every month, but her hirsutism has not significantly improved. Which of the following would be an appropriate treatment for hirsutism, in addition to the oral contraceptives? (2.5分) A.Spironolactone
B.Lupron
C.Danazol
D.Depo-Provera
E.Steroids
A.Spironolactone
4 8 2015 33. A 21-year-old diabetic athletic woman on a lowdose oral contraceptive comes to your clinic with irregular menses and galactorrhea. On examination, galactorrhea is confirmed, with fat globules seen microscopically. She currently takes metoclopramide (Reglan) for delayed gastric emptying. A random serum prolactin level is 65 ng/mL. Which of the following is most likely responsible for her hyperprolactinemia? (2.5分) A.metoclopramide
B.pregnancy
C.oral contraceptive
D.pituitary adenoma
E.exercis
A.metoclopramide
4 8 2015 34. A 25-year-old woman suffers a severe intrapartum hemorrhage. Which of the following symptoms is evidence of pituitary infarction? (2.5分) A.infrequent urination
B.diarrhea
C.easy bruisability
D.lactation failure
E.perspiration
D.lactation failure
4 8 2015 35. An 18-year-old patient has not experienced menarche. Examination shows normal breast development and absence of a uterus. Which of the following diagnostic tests is most useful in determining the etiology of the amenorrhea? (2.5分) A.serum FSH
B.serum estradiol
C.serum testosterone
D.MRI of the head
E.ovarian biopsy
C.serum testosterone
4 8 2015 36. Luteectomy before 42 days' gestation is most likely to result in which of the following? (2.5分) A.prolonged (postdates) gestation
B.spontaneous miscarriage
C.reduction of basal body temperature (BBT)
D.masculinization of a female fetus
E.no demonstrable effect
B.spontaneous miscarriage
4 8 2015 37. 雌激素类药物的药理作用中,不包括 (2.5分) A.促进女性性器官的发育和成熟,维持女性第二性征。
B.能明显促进蛋白质合成,减少蛋白质分解
C.使阴道上皮增生,浅表层细胞角化
D.增强子宫平滑肌对缩宫素的敏感性
E.较大剂量时可抑制下丘脑-垂体系统释放GnRH
B.能明显促进蛋白质合成,减少蛋白质分解
4 8 2015 38. 孕激素类药物可用于治疗 (2.5分) A.先兆流产
B.诱发流产
C.再生障碍性贫血
D.消耗性疾病等引起的虚弱
E.卵巢功能不全和闭经
A.先兆流产
4 8 2015 39. 对麦角新碱的叙述哪一项是错误的?(2.5分) A.对子宫兴奋作用强而持久
B.对子宫体和子宫颈的兴奋作用无明显差别
C.剂量稍大即引起子宫强直性收缩
D.适用于子宫出血及产后子宫复原
E.临产时的子宫对本药敏感性降低
E.临产时的子宫对本药敏感性降低
4 8 2015 40. 对缩宫素的叙述哪一项不正确?(2.5分) A.小剂量可加强子宫的节律性收缩
B.其收缩性质与正常分娩相似
C.大剂量引起子宫强直性收缩
D.大剂量适用于催产
E.小剂量适用于催产和引产
D.大剂量适用于催产